Volume 1 of Direct Tax by CA. Akash Sir

You might also like

Download as pdf or txt
Download as pdf or txt
You are on page 1of 346

A book on

INCOME TAX
(Volume - 1)
For CA/ CS/ CMA Intermediate MAY’ 24 and NOV’ 24 attempt

Previous Year 2023-24


Assessment Year 2024-25
Updated as per FINANCE ACT, 2023
Designed as per NEW Scheme of ICAI.

Connect with Akash Sir through


Instagram
Mobile Application
YouTube
WhatsApp

BY CA. AKASH BANSAL


( CA-CPT – AIR 7, CA-IPCC – AIR 14 and CA-Final – AIR 47)
TABLE OF
CONTENTS

BASICS OF INCOME TAX, COMPUTATION OF


1 TOTAL INCOME AND TAX LIABILITY 1

RESIDENTIAL STATUS AND SCOPE OF TOTAL INCOME


2 25

INCOME FROM HOUSE PROPERTY


3 71

INCOME UNDER HEAD CAPITAL GAINS


4 114

INCOME FROM OTHER SOURCES


5 204

PROFITS AND GAINS OF BUSINESS OR


6 PROFESSION (PGBP) 243
1
1. Basics of Income Tax, Computation of Total Income and Tax Liability

Basics of Income Tax, Computation of Total


Income and Tax Liability
TP: 1 What is Tax?

Tax is “cost of living in a society”. It is the amount collected by the Government to


meet the common welfare expenditure of the society.

There are 2 types of taxes:

1. Direct taxes: It is the tax levied on the income or wealth of the person, e.g.
Income Tax. The person who pays such tax to the Government cannot recover it
from others. It means that the burden of such tax cannot be shifted.

2. Indirect taxes: It is the tax levied on Goods or Services, e.g. Goods and Services
Tax (GST), Custom Duty, Excise Duty etc. The person who pays such tax to the
Government recovers the same from the customers/ consumers. It means that the
burden of such tax is transferred to the third person.

TP: 2 Who empowers the Government to levy taxes?

➢ As per Article 265 of the Constitution of India, “No tax shall be levied or
collected except by authority of law”. Accordingly, a law needs to be formed to
collect taxes.

➢ Seventh Schedule to Article 246 of the Constitution empowers the


Government to make law and it has 3 lists -
1. Union List: Parliament has exclusive power to make law on the matters stated
in Union list.
2. State List: State legislature has exclusive power to make law on the matters
stated in State list.
3. Concurrent List: Both Parliament and state legislature have power to make
law on the matters stated in Concurrent list.

➢ Entry 82 of Union List empowers Central Government to levy “Taxes on income


other than agricultural income’’

➢ Entry 46 of State List empowers State Government to levy “Taxes of Agricultural


Income”

1|Page #HakkसेCA by CA. Akash Sir


2
1. Basics of Income Tax, Computation of Total Income and Tax Liability

Note: No State Government has made any law to tax agricultural income even
though the State Government is empowered to do so.

TP: 3 What is Income Tax Law?

Income Tax Law is combination of various components relating to income tax which are:

1. Income Tax Act, 1961


➢ It extends to whole of India including Jamu & Kashmir
➢ It came into force on 1st April, 1962
➢ It contains section 1 to 298 and schedules I to XIV

2. The Finance Act


➢ Finance minister of India introduces the FINANCE BILL in the
Parliament’s budget session
➢ When bill is passed by both houses of the Parliament and gets accent of
the President, it becomes FINANCE ACT.
➢ Amendments are introduced in Income Tax Act through the Finance Act.
➢ First Schedule of the Finance Act (Finance Act, 2023 in our case) contains
4 Parts which has rates of taxes as follows:
o Part I: It specifies the rates of tax applicable for the current
Assessment Year i.e. A.Y. 2023-24 (relevant to P.Y. 2022-23)
o Part II: It specifies the rates at which tax is deductible at source for
the current Financial year i.e. F.Y. 2023-24
o Part III: It specifies the rate for calculating TDS deductible from
salaries and for calculating advance tax for F.Y. 2023-24 under old tax
regime.
o Part IV: It specifies the rules for computing net agricultural income.

3. Income Tax Rules, 1962


➢ Administration of the direct taxes is looked after by Central Board of
Direct Taxes (CBDT).
➢ CBDT has given with the powers to make rules for carrying out the
purposes of the Act.
➢ The rules formed by the CBDT is called as “Income tax Rules, 1962”
➢ Who is CBDT?
Govt. of India >> Finance Ministry >> Revenue Department >> CBDT

4. Circulars
➢ Issued by the CBDT
➢ Circulars deal with certain specific problems and clarify the doubts
regarding the scope and meaning of certain provisions
➢ Issued to guide department officers and/ or the Assessee

2|Page #HakkसेCA by CA. Akash Sir


3
1. Basics of Income Tax, Computation of Total Income and Tax Liability

➢ Circulars are binding on the department and not on the Assessee

5. Notifications
➢ Issued by the Central Government to give effect of the provisions of the
Act
➢ CBDT can also issue notifications to make/ amend rules
➢ Notifications are binding on both the department and the Assessee

6. Case Laws/ Legal decisions of Courts


➢ Case laws refer to the decisions given by the courts
➢ It is not possible for the Parliament (means Act) to provide all possible
issues that may arise in the implementation of the Act. Hence, judiciary hear
the disputes between the department and the Assessee and provide their
decisions
➢ The Hon’ble Supreme Court of India is the law of land and decisions
given by the Apex Court are applicable in Whole India as Deemed Law.
➢ Decisions given by various high courts are applicable in their respective
states.

Discussion on Section, Subsection, Clause, Sub-Clause, Proviso, Explanation etc. – To be


discussed in class.

TP: 4 Concept of Definitions

➢ Terms defined in the Act: Section 2 gives definition of various terms used in
the Act. Apart from section 2, there are various terms which are defined under
particular section where the term is being used. For example, the term
“perquisites” has been defined under section 17(2) of the Act under chapter
salary.

➢ Terms not defined under the Act: If any particular definition is not given in
the Act, reference can be made to the General Clauses Act or dictionaries.

TP: 5 Some important definitions

1. Person [Section 2(31)]: It includes,


a) Individual
b) Hindu Undivided Family (HUF)
c) Company
d) Firm
e) Association of Persons (AOP)
f) Body of Individual (BOI)
g) Local Authority

3|Page #HakkसेCA by CA. Akash Sir


4
1. Basics of Income Tax, Computation of Total Income and Tax Liability

h) Artificial Juridical Persons (AJP)

2. Hindu Undivided Family: HUF has not been defined in the Income tax Act. As
per Hindu Law, HUF is a family which consists of all male lineally descended
from a common ancestors and incudes their wives and daughters.

➢ Co-Parceners: A Hindu Coparcenary includes those persons who acquire an


interest in joint family property by birth. Earlier, only male descendants
were considered as coparceners. With effect from 06th September, 2005,
daughters have also been accorded coparcenary status. Please note that
other female members of the family, for example, wife or daughter in law
of a coparcener are not considered as coparceners even though they are
members of HUF.
➢ Under the Income Tax Act, Jain undivided families and Sikh undivided
families would also be assessed as a HUF.
➢ School of Hindu Law (i.e. types of HUF): a) Dayabaga School, b)
Mitakshara School

Dayabaga School of Hindu Law Mitakshara School of Hindu Law


Prevalent in West Bangal and Prevalent in rest of India
Assam
Nobody acquires the right, share One acquires the right to the
in the property by birth as long as family property by his birth and
the head of family is living. not by succession irrespective of
the fact that his elders are living.
.

3. Company [Section 2(17)]: Company means:


(1) any Indian company as defined in section 2(26) of Income tax Act,
(2) any foreign company
(3) any institution, association or body which is assessable or was assessed
as a company under Income tax Act 1922 or under present act
(4) any institution, association or body which is declared by the CBDT as
company

Classes of companies:
1) Domestic Company [Section 2(22A)]: It means
a) an Indian Company, which means
i) the company formed and registered under the companies act, and
ii) the registered office or the principal office of the company is in
India

4|Page #HakkसेCA by CA. Akash Sir


5
1. Basics of Income Tax, Computation of Total Income and Tax Liability

b) any other company which has made prescribed arrangements for the
declaration and payment of dividend (equity or preference) within
India out of the income taxable in India.
2) Foreign Company [Section 2(32A)]: It means a company which is not a
domestic company.

4. Firm [Section 2(23)]:


➢ It has same meaning as assigned under Indian Partnership Act, 1932.
➢ In addition, it includes the term “Limited Liability Partnership” as defined
under Limited Liability Partnership Act, 2008.
➢ Minor is also treated as partner for income tax purposes.

5. Association of Persons (AOP):


➢ When persons combine together for promotion of joint enterprises
without forming a partnership, they are assessable as an AOP
➢ Persons come together for a common purpose or action and their objective
is to produce income.
➢ It is not enough that the persons receive income jointly.
➢ For example, Mr. Ashok and Miss Aarti come together to carry out
construction of a road otherwise than as partnership firm.

6. Body of Individuals (BOI):


➢ Persons who merely receives income jointly and their interests are
indivisible.
➢ Income tax is payable by BOI as a separate entity and members shall not
be liable to pay tax individually.
➢ For example, mutual trade associations, members club etc.

7. Local authorities: It means a municipal committee, district board, body of


port commissioners or other authority legally entitled to or entrusted by the
Government with the control or management of a municipal or local fund.

8. Artificial Juridical Persons: It means any artificial person not falling under
any above category e.g. Bar Council, Universities etc.

9. India [Section 2(25A)]: The term India means:


(i) the territory of India as per Article 1 of the Constitution
(ii) its territorial waters, seabed and subsoil underlying such waters
(iii) continental shelf
(iv) exclusive economic zone or
(v) any other specified maritime zone and the air space above its territory and
territorial waters.

5|Page #HakkसेCA by CA. Akash Sir


6
1. Basics of Income Tax, Computation of Total Income and Tax Liability

TP: 6 Previous Year and Assessment Year

➢ Assessment Year [Section 2(9)]: This means a period of 12 months commencing


on 1st April every year. Generally, the income earned in Previous Year is taxable in
the Assessment Year. Income earned in Previous Year 2023-24 is taxable in the
Assessment Year 2024-25. It always has 12 months.
➢ Previous Year [Section 3]: It means the financial year immediately preceding
the assessment year. It may or may not have 12 months.

For example, Mr. Ashish started his CA firm on 01.09.2023, in this case,
Previous Year shall start from 01.09.2023 and shall end on 31.03.2024. However,
the assessment year shall be 2024-25 (12 months).

General rule
Income of previous year is assessed in the assessment year following the previous year

Exception to this rule


Cases where income of a previous year is assessed in the previous year itself

Shipping buisness of non-resident

Person leaving India

AOP/ BOI/ AJP formed for a particular event or purpose

Persons likley to transfer property to avoid tax

Discontinued business

TP: 7 Charge of Income tax (Charging section) [Section 4]

Section 4 of the Income tax Act is the charging section which provides that the tax
shall be levied at the
(i) rate prescribed under Annual Finance Act or Income tax Act, 1961 or both on
(ii) every person on
(iii) total income earned during previous year in
(iv) accordance with provisions of the Act.

Mann Ki Baat (simple language):

Simply speaking, section 4 is the most important section which is the backbone of the
Act and which provides that:
(i) Tax shall be levied at prescribed rates
(ii) Tax shall be levied on every person
(iii) Tax shall be charged at the total income earned during the previous year
6|Page #HakkसेCA by CA. Akash Sir
7
1. Basics of Income Tax, Computation of Total Income and Tax Liability

(iv) Total income shall be calculated in accordance with the provisions of the Act.

TP: 8 Concept of total income and tax liability

Steps involved in computing total income and tax liability are as under:

Step 1: Determination of residential status


A person can be Resident and Ordinarily Resident (ROR), Resident but not
Ordinarily Resident (NOR) or Non-Resident (NR). Computation of total income
(Scope) depends on the residential status of such person. (Discussed under
residential status chapter in detail.)

Step: 2 Classification and computation of income under 5 heads of income to arrive


at gross total income
Income under head salary xxxx
Income under head house property xxxx
Profits and Gains from business or profession xxxx
Income under head Capital gains xxxx
Income from other sources xxxx
Gross total income xxxx

For example:

Mr. Ramlal has income from the following sources for A.Y. 2024-25:
1. He is working for Adani Ltd. and has earned annual salary (computed) of
₹4,50,000
2. He owned one house property in Delhi and has renting income (computed) of
₹5,90,000 from that house property.
3. He also runs a small e-commerce business and has income (computed) of
₹2,30,000
4. He has sold his house property in Mumbai and has earned long term capital
gain (computed) of ₹4,00,000
5. He also has bank fixed deposit and has earned interest of ₹20,000 from
such FD
Calculate his gross total income for A.Y. 2024-25.

Solution:

Computation of gross total income of Mr. Ramlal for A.Y. 2024-25


Income under head salary 4,50,000
Income under head house property 5,90,000
Profits and Gains from business or profession 2,30,000
Income under head Capital gains 4,00,000
Income from other sources 20,000

7|Page #HakkसेCA by CA. Akash Sir


8
1. Basics of Income Tax, Computation of Total Income and Tax Liability

Gross total income 16,90,000

Step 3: Reduce the deduction under chapter VIA to arrive at the Total income
Gross total income xxxx
Less: Deductions under chapter VIA (xxxx)
Total income xxxx

For example:
In continuation of above example, assume that Mr. Ramlal also invested
₹50,000 in tax saving mutual fund which is eligible for deduction from gross
total income. Compute his total income under old tax regime for A.Y. 2024-25.

Solution:
Computation of total income of Mr. Ramlal for A.Y. 2024-25
Gross total income
16,90,000
Less: Deductions under chapter VIA
(50,000)
Total income
16,40,000

Important note:
Various deductions are prescribed under chapter VIA of the Income tax Act,
1961 which are to be considered while calculating income under each head and
total income. These deductions are available to the assessee only if he
opts for old tax regime.

New tax regime under section 115BAC (discussed later on) is the default tax
regime which provides for concessional rates of tax to Individual/ HUF/ AOP/
BOI/ AJP (discussed in later part of this chapter). Under new tax regime,
certain deductions are not allowed.

➢ Total income shall be rounded off u/s. 288A in multiple of ₹10.


For example, if
(i) Total income is ₹3,33,406 then it shall be rounded off to ₹3,33,410 and
(ii) Total income is ₹5,60,703 then it shall be rounded off to ₹5,60,700 and
(iii) Total income is ₹10,05,605 then it shall be rounded off to ₹10,05,610.

Mann Ki Baat (simple language):

Please note that there are 2 parallel tax regimes for A.Y. 2024-25. Under old tax
regime, deductions under chapter VIA are allowed to be deducted from Gross total

8|Page #HakkसेCA by CA. Akash Sir


9
1. Basics of Income Tax, Computation of Total Income and Tax Liability

income and under new tax regime (which is default tax regime), certain deductions are
not allowed. The assessee has option to choose any one which is more beneficial.

Step 4: Calculation of tax liability


Total income * tax rates xxxx
Add: Surcharge @ applicable rate (if applicable) xxxx
xxxx
Less: Rebate u/s. 87A (if applicable) xxxx
Tax liability before HEC xxxx
Add: Health and Education cess @ 4% xxxx
Gross tax liability/ Tax liability xxxx

Important note: Rates prescribed under section 115BAC (new tax regime) is
default tax rates (discussed in later part of this chapter).

Mann Ki Baat (simple language):

Since the Income tax Act has made new tax regime as default tax regime, we have to
solve answer as per new tax regime until it is specified in the question that the
Assessee has opted for old tax regime during the assessment year.

Step 5: Calculation of tax payable


Tax liability xxxx
Less: Tax deducted at source xxxx
Less: Tax collected at source xxxx
Less: Advance tax xxxx
Less: Self-assessment tax xxxx
Net tax payable xxxx

➢ Total tax payable shall be rounded off u/s 288B in multiple of ₹10.
For example, if
(i) Total tax payable is ₹3,33,406 then it shall be rounded off to ₹3,33,410 and
(ii) Total tax payable is ₹5,60,703 then it shall be rounded off to ₹5,60,700 and
(iii) Total tax payable is ₹10,05,605 then it shall be rounded off to ₹10,05,610.

TP: 9 Rates of tax, Surcharge and Cess [Prescribed under


Finance Act + Income tax Act, 1961]

1. Individual/ HUF/ AOP/ BOI/ Artificial Juridical person

a) Under old tax regime [under Finance Act]:

9|Page #HakkसेCA by CA. Akash Sir


10
1. Basics of Income Tax, Computation of Total Income and Tax Liability

A. Individual (Resident or Non-Resident)/ HUF/ AOP/ BOI/ Artificial


Juridical Person

Income Tax rate


First 2,50,000 Nil
Next 2,50,000 5%
Next 5,00,000 20%
Above 10,00,000 30%

For example:
Miss Manavi, aged 24 years, resident individual has total income of ₹8,50,000. Calculate
her total tax liability (before health and education cess) for A.Y. 2024-25 assuming
that she has opted out of the provisions of section 115BAC.

Solution:

Computation of tax liability (before HEC) in hands of Miss Manavi for A.Y. 2024-
25
First 2,50,000 Nil
Next 2,50,000 @ 5% 12,500
Next 3,50,000 @ 20% 70,000
Tax liability before HEC 82,500

➢ Surcharge (Tax on tax):

Income tax computed above shall be increased by the amount of surcharge calculated
as per below table:

Where total income is Surcharge Rate on tax


Lower than or equals to 50 Lakhs Nil
More than 50 Lakhs but less than 1 Crores 10%
More than 1 Crore but less than 2 Crores 15%
More than 2 Crores but less than 5 Crores 25%
More than 5 Crores 37%

For example:
Mrs. Anaya, aged 48 years, resident individual has total income of ₹2,28,50,000.
Calculate her total tax liability (before HEC) for A.Y. 2024-25 assuming that she has
opted out of the provisions of section 115BAC.

10 | P a g e #HakkसेCA by CA. Akash Sir


11
1. Basics of Income Tax, Computation of Total Income and Tax Liability

Solution:

Computation of tax liability (Before HEC) in hands of Mrs. Anaya for A.Y. 2024-
25
First 2,50,000 Nil
Next 2,50,000 @ 5% 12,500
Next 5,00,000 @ 20% 1,00,000
Next 2,18,50,000 @ 30% 65,55,000
Tax liability before surcharge 66,67,500
Add: Surcharge @ 25% 16,66,875
Tax liability before HEC 83,34,375

➢ Health and Education Cess:

Additional 4% cess is leviable on Tax and Surcharge by the name of Health and
Education Cess which is basically for development of Health and Education sector only.

For example:
Mr. Kashish, aged 49 years, resident individual has total income of ₹5,30,50,000.
Calculate his total tax liability for A.Y. 2024-25 assuming that he has opted out the
provisions of section 115BAC.

Solution:

Computation of tax liability in hands of Mr. Kashish for A.Y. 2024-25


First 2,50,000 Nil
Next 2,50,000 @ 5% 12,500
Next 5,00,000 @ 20% 1,00,000
Next 5,20,50,000 @ 30% 1,56,15,000
Tax liability before surcharge 1,57,27,500
Add: Surcharge @ 37% 58,19,175
Tax liability before HEC 2,15,46,675
Add: Health and education cess @ 4% 8,61,867
Tax liability 2,24,08,542
Rounded off u/s. 288B 2,24,08,540

HW Question: 1

Calculate tax liability assuming that the assessee has opted out of default tax
regime in following cases-
(i) Mr. Ajit has total income of ₹8,00,000
(ii) Mr. Anmol has total income of ₹10,00,005

11 | P a g e #HakkसेCA by CA. Akash Sir


12
1. Basics of Income Tax, Computation of Total Income and Tax Liability

(iii) Mr. Ajay has total income of ₹12,00,000


(iv) Mr. Rohan has total income of ₹14,00,004
(v) Mr. Soham has total income of ₹23,00,000
(vi) Miss Anamika has total income of ₹54,00,000
(vii) Mrs. Manavi has total income of ₹1,30,00,008
(viii) Mr. Akash has total income of ₹2,23,00,000
(ix) Miss Kalpana has total income of ₹5,40,00,000
It is assumed that the age of above individuals is below 60 years during P.Y. 2023-24.

B. Individuals who are Resident Senior Citizen (i.e. their age is 60 years
or more but less than 80 years)

Income Tax rate


First 3,00,000 Nil
Next 2,00,000 5%
Next 5,00,000 20%
Above 10,00,000 30%

➢ Surcharge: Same as above


➢ Health and education cess: 4%

HW Question: 2

Calculate tax liability assuming that the assessee has opted out of default tax
regime in following cases -
(i) Mr. Ajit, a resident senior citizen has total income of ₹8,00,000
(ii) Mr. Anmol, a resident of age 45 years has total income of ₹10,00,005
(iii) Mr. Ajay, a non-resident senior citizen has total income of ₹12,00,000
(iv) Mr. Rohan, a resident of age 61 years has total income of ₹14,00,004
(v) Mr. Soham, a resident senior citizen has total income of ₹23,00,000
(vi) Miss Anamika, a non-resident senior citizen has total income of ₹54,00,000
(vii) Mrs. Manavi, a resident of age 25 years has total income of ₹1,30,00,008
(viii) Mr. Akash, a resident of age 16 years has total income of ₹2,23,00,000
(ix) Miss Kalpana, a resident senior citizen has total income of ₹5,40,00,000

C. Individuals who are Resident Super Senior Citizen (i.e. their age is 80
years or more)

Income Tax rate


First 5,00,000 Nil
Next 5,00,000 20%
Above 10,00,000 30%

12 | P a g e #HakkसेCA by CA. Akash Sir


13
1. Basics of Income Tax, Computation of Total Income and Tax Liability

➢ Surcharge: Same as above


➢ Health and education cess: 4%

HW Question: 3

Calculate tax liability assuming that the assessee has opted out of default tax
regime in following cases -
(i) Mr. Ajit, a resident super senior citizen has total income of ₹8,00,000
(ii) Mr. Anmol, a resident of age 95 years has total income of ₹1,20,00,000
(iii) Mr. Ajay, a non-resident super senior citizen has total income of ₹12,00,000

Important note:

Circular No. 28/2016, dated 27.07.2016 – Clarification regarding attaining


prescribed age of 60 years/ 80 years on 31st March itself, in case of senior/
very senior citizens whose date of birth falls on 1st April.

As per the above circular, if any person has born on 01.04.1964, then for the purpose
of calculation of age during the previous year, it shall be considered that the person
has attain the age of 60 years on 31.03.2024 even if he celebrates his birth day on
01.04.2024. Therefore, he shall be considered as Senior Citizen during the P.Y.
2023-24.
Similarly, if any person has born on 01.04.1944, then for the purpose of calculation of
age during the previous year, it shall be considered that the person has attain the
age of 80 years on 31.03.2024 even if he celebrates his birth day on 01.04.2024.
Therefore, he shall be considered as Super Senior Citizen during the P.Y. 2023-24.

b) Under new tax regime [Section 115BAC]:

Individual/ HUF/ AOP/ BOI and AJP can pay tax at concessional rates under the
default tax regime under section 115BAC of the Act. However, the Assessee has to
forego certain exemptions and deductions under this tax regime.

All Individual (Resident or Non-Resident/ Normal or Senior Citizen)/ HUF/


AOP/ BOI/ Artificial Juridical Person

Income Tax rate


First 3,00,000 Nil
Next 3,00,000 5%
Next 3,00,000 10%
Next 3,00,000 15%
Next 3,00,000 20%

13 | P a g e #HakkसेCA by CA. Akash Sir


14
1. Basics of Income Tax, Computation of Total Income and Tax Liability

Above 15,00,000 30%

Important points to remember for exam:

i. List of deductions/ exemptions not allowable [Section 115BAC(2)]:

# Section Exemption/ Deduction


1. 10(5) Leave travel concession
2. 10(13A) House rent allowance
3. 10(14) Special allowances
4. 10(17) Daily allowance or constituency allowance of MPs/ MLAs
5. 10(32) Exemption is respect of income of minor child included in
income of parents
6. 10AA Tax holidays for units established in SEZ
7. 16 (i) Entertainment allowance
(ii) Professional tax
8. 24(b) Interest on loan in respect of self-occupied property
9. 32(1)(iia) Additional depreciation
10. 35 Donation made for scientific research
11. 35AD Investment linked tax incentives for specified businesses
12. 80C to Deductions under chapter VIA (other than 80CCD(2),
80U 80CCH(2) and 80JJAA)

ii. Certain losses are not allowed to be set-off:

(i) Carried forward losses or unabsorbed depreciation for earlier years are not
allowed to be set-off during current year if such losses or depreciation is
attributable to any deduction above (explained above, which is not allowed under
new tax regime).

(ii) Set-off of loss under head house property with any other head of income is
not allowed.

iii. Depreciation is restricted to 40% where rate of depreciation is more than


40%.

iv. Exemption or deductions for allowances or perquisites provided under any


other act is not allowed.

v. Unabsorbed additional depreciation shall be added back in WDV as on


01.04.2023:

14 | P a g e #HakkसेCA by CA. Akash Sir


15
1. Basics of Income Tax, Computation of Total Income and Tax Liability

Since unabsorbed depreciation of earlier years attributable to additional


depreciation is not allowed to be set-off against any head of income, the amount
of unabsorbed additional depreciation shall be added back while calculating WDV
as on 01.04.2023 if the Assessee has opted for section 115BAC.

For example:
Mr. X who carries on business of manufacturing of steel. He has unabsorbed
depreciation of ₹5,00,000 as on 01.04.2023, which includes amount of
₹2,80,000 attributable to additional depreciation for P.Y. 2022-23 in respect of
block of plant and machinery. If he pays tax under default tax regime under
section 115BAC for P.Y. 2023-24, the amount of ₹2,80,000 shall not be eligible
to be set-off against current year income and no further deduction of such
depreciation shall be allowed in any subsequent year. Accordingly, the WDV of
the block as on 01.04.2023 has to be increased by the said amount of ₹2,80,000,
not allowed to be set-off.

vi. Time limit for exercising the option to shift out of the default tax regime:

Assessee having no income from business or profession:

The assessee has option to opts out of new tax regime and such option shall be
exercised along with the return of income on or before due date of return filing.
Moreover, he shall have option to opt for new tax regime in one year and old tax
regime in next year and vice-versa.

Assessee having income from business or profession:

The assessee has option to opts out of new tax regime and such option shall be
exercised on or before the due date specified under section 139(1) for
furnishing the return of income. Moreover, once the option is exercised, it would
apply to subsequent years.

The person who has opted out of new tax regime shall have option to shift back
to new tax regime only once in lifetime. Thereafter, the person shall never be
eligible to exercise this option again until he ceases to have business/ profession
income.

vii. Provisions of Alternate Minimum Tax (AMT) shall not be attracted.

15 | P a g e #HakkसेCA by CA. Akash Sir


16
1. Basics of Income Tax, Computation of Total Income and Tax Liability

For Example:
Mr. Avinash has a total income of ₹16,00,000 for P.Y. 2023-24, comprising of income
from house property and interest on fixed deposits. Compute his tax liability for A.Y.
2024-25 under the default tax regime under section 115BAC.

Solution:

Computation of tax liability of Mr. Avinash for A.Y. 2024-25


First ₹3,00,000 Nil
Next ₹3,00,001 – ₹6,00,000 @5% of ₹3,00,000 15,000
Next ₹6,00,001 – ₹9,00,000 @10% of ₹3,00,000 30,000
Next ₹9,00,001 - ₹12,00,000 @15% of ₹3,00,000 45,000
Next ₹12,00,001 – ₹15,00,000 @30% of ₹3,00,000 60,000
Balance i.e., ₹16,00,000 (-) ₹15,00,000 @30% of ₹1,00,000 30,000
1,80,000
Add: Health and Education cess @4% 7,200
1,87,200

➢ Surcharge: Same as under old tax regime


➢ Health and education cess: 4%

2. Firm/ Limited liability partnership (LLP)

Flat tax rate: 30%


Surcharge: Where total income is more than 1 Crore – 12%
Health and education cess: 4%

3. Local Authority

Flat tax rate: 30%


Surcharge: Where total income is more than 1 Crore – 12%
Health and education cess: 4%

4. Co-operative society

a) Normal tax rates:

Income Tax rate


First 10,000 10%
Next 10,000 20%
Above 20,000 30%

16 | P a g e #HakkसेCA by CA. Akash Sir


17
1. Basics of Income Tax, Computation of Total Income and Tax Liability

➢ Surcharge rate:

Where total income is Surcharge Rate on tax


Lower than or equals to 1 Crore Nil
More than 1 Crores but less than 10 Crores 7%
More than 10 Crores 12%

➢ Health and education cess: 4%

b) Special tax rates for Resident Manufacturing co-operative society


[Section 115BAE]:

Tax shall be 15% of income derived from or incidental to manufacturing or production


of an article or thing.

Condition: Society should be set-up and registered on or after 01.04.2023 and


commences manufacture of article or thing before 31.03.2024

➢ Surcharge rate: 10% without any threshold limit


➢ Health and education cess: 4%

c) Special tax rate for other Resident co-operative society [Section


115BAD]:

Tax shall be 22% of total income

➢ Surcharge rate: 10% without any threshold limit


➢ Health and education cess: 4%

Other important condition under section 115BAE and 115BAD:

i. List of deductions/ exemptions not allowable:

# Section Exemption/ Deduction


1. 10AA Tax holidays for units established in SEZ
2. 33AB Tea/ coffee/ rubber development account
3. 33ABA Site restoration fund
4. 32(1)(iia) Additional depreciation
5. 35 Donation made for scientific research
6. 35AD Investment linked tax incentives for specified businesses
7. 35CCC Agricultural extension project

17 | P a g e #HakkसेCA by CA. Akash Sir


18
1. Basics of Income Tax, Computation of Total Income and Tax Liability

8. 80C to 80U Deductions under chapter VIA (other than 80JJAA)

ii. Certain losses are not allowed to be set-off:

Carried forward losses or unabsorbed depreciation for earlier years are not
allowed to be set-off during current year if such losses or depreciation is
attributable to any deduction above.

iii. Provisions of Alternate minimum tax (AMT) shall not be attracted.

5. Domestic Company

a) Normal tax rates:

Turnover/ Gross receipt for P.Y. 21-22 Tax rate


Less than or equals to 400 Crores 25%
Other 30%

➢ Surcharge:

Where total income is Surcharge Rate on tax


Lower than or equals to 1 Crore Nil
More than 1 Crores but less than 10 7%
Crores
More than 10 Crores 12%

➢ Health and education cess: 4%

b) Special tax rates for domestic Manufacturing company [Section 115BAB]:

Tax shall be 15% of income derived from or incidental to manufacturing or production


of an article or thing.

Condition: Company should be set-up and registered on or after 01.10.2019 and


commences manufacture of article or thing before 31.03.2024

➢ Surcharge rate: 10% without any threshold limit


➢ Health and education cess: 4%

c) Special tax rate for other domestic company [Section 115BAA]:

Tax shall be 22% of total income

18 | P a g e #HakkसेCA by CA. Akash Sir


19
1. Basics of Income Tax, Computation of Total Income and Tax Liability

➢ Surcharge rate: 10% without any threshold limit


➢ Health and education cess: 4%

Other important condition under section 115BAB and 115BAA:

i. List of deductions/ exemptions not allowable:

# Section Exemption/ Deduction


1. 10AA Tax holidays for units established in SEZ
2. 33AB Tea/ coffee/ rubber development account
3. 33ABA Site restoration fund
4. 32(1)(iia) Additional depreciation
5. 35 Donation made for scientific research
6. 35AD Investment linked tax incentives for specified businesses
7. 35CCC Agricultural extension project
8. 35CCD Expenditure for skill development
9. 80C to 80U Deductions under chapter VIA (other than 80JJAA and 80M)

ii. Certain losses are not allowed to be set-off:

Carried forward losses or unabsorbed depreciation for earlier years are not
allowed to be set-off during current year if such losses or depreciation is
attributable to any deduction above.

6. Other than domestic company

Flat tax rate: 40%

➢ Surcharge:

Where total income is Surcharge Rate on tax


Lower than or equals to 1 Crore Nil
More than 1 Crores but less than 10 Crores 2%
More than 10 Crores 5%

➢ Health and education cess: 4%

HW Question: 4

Calculate tax liability in following cases under normal tax rates:


a) XYZ firm has business income of ₹6,00,000

19 | P a g e #HakkसेCA by CA. Akash Sir


20
1. Basics of Income Tax, Computation of Total Income and Tax Liability

b) Adani Pvt. Ltd., a domestic company, has business income of ₹2,04,20,000


c) Mukesh Inc, an American company has taxable business income of ₹12,50,00,000
d) Akash Pvt. Ltd., a domestic company having turnover of ₹390 Crores during P.Y.
2021-22, has business income of ₹1,45,00,000 during A.Y. 2024-25

TP: 10 Concept of Marginal Relief on Surcharge

Where incremental income is less than the increase in tax (together with surcharge)
due to charge of surcharge at a higher rate, then Marginal relief is allowed to the
assessee up to the amount of difference in incremental income and incremental tax.

For example:
Mr. J S Bansal, aged 68 years, non-resident individual has total income of
₹2,03,10,000. Calculate his total tax liability for A.Y. 2024-25 assuming that he has
not opted for the provisions of section 115BAC.

Solution:

Computation of tax liability in hands of Mr. J S Bansal for A.Y. 2024-25


First 2,50,000 Nil
Next 2,50,000 @ 5% 12,500
Next 5,00,000 @ 20% 1,00,000
Next 1,93,10,000 @ 30% 57,93,000
Tax liability before surcharge 59,05,500
Add: Surcharge @ 25% 14,76,375
(A) Tax liability including surcharge on ₹2,03,10,000 73,81,875
(B) Tax liability (including surcharge) on ₹2,00,00,000 66,84,375
(C) Excess tax (A-B) 6,97,500
(D) Income over and above 2,00,00,000 3,10,000
(E) Marginal relief (C-D) 3,87,500
(F) Tax liability before HEC (A-E) 69,94,375
Add: Health and education cess @ 4% 2,79,775
Tax liability 72,74,150

Steps to solve Marginal relief problems:

Step A: Calculate tax plus surcharge on net total income (actual)


Step B: Calculate tax plus surcharge on total income without considering
incremental income
Step C: Deduct Step B from Step A i.e. Step A - Step B
Step D: Write down additional income over and above nearest surcharge slab
income
Step E: Marginal Relief = Deduct Step D from Step C i.e. Step C – Step D
Step F: Deduct Marginal relief from Step A

20 | P a g e #HakkसेCA by CA. Akash Sir


21
1. Basics of Income Tax, Computation of Total Income and Tax Liability

Step G: Add Health and education cess @4%

Mann Ki Baat (simple language):

Marginal relief is provided in order to remove the burden of additional tax over and
above the additional income. This excess tax becomes payable due to change in slab
rate of surcharge.

Important Note: Marginal relief is provided on tax liability before health and
education cess.

HW Question: 5

Calculate tax liability in following cases under old tax regime-


(i) Mr. Ajit has total income of ₹51,00,000
(ii) Mr. Anmol has total income of ₹1,01,00,005
(iii) Mr. Ajay has total income of ₹2,02,00,000
(iv) Mr. Rohan has total income of ₹5,05,00,004
It is assumed that the age of above individuals is below 60 years during P.Y. 2023-24.

HW Question: 6

Mr. Bablu has total income of ₹2,05,00,000 comprising of Income from house property
and bank interest. Calculate tax liability of Mr. Bablu assuming that:
a) His age is 45 years and he is resident
b) His age is 65 years and he is non-resident
c) His age is 85 years and he is resident
Assuming that Mr. Babu has not opted for the provisions of section 115BAC.

TP: 11 Rebate under section 87A

➢ The person should be RESIDENT Individual.

Where assessee opts for default tax regime under section 115BAC
➢ Total income shall not exceed ₹7,00,000
➢ Rebate equals to
a) Tax payable on total income or
b) ₹25,000
Whichever is lower.
➢ Rebate shall be provided on tax before health and education cess.

21 | P a g e #HakkसेCA by CA. Akash Sir


22
1. Basics of Income Tax, Computation of Total Income and Tax Liability

For example:
Mr. Ram aged 96 years and a resident in India, has a total income of ₹6,50,000,
comprising his salary income and interest on bank fixed deposit. Compute his tax
liability for A.Y. 2024-25 under default tax regime under section 115BAC.

Solution:

Computation of tax liability of Mr. Ram for A.Y. 2024-25


Tax on total income of ₹6,50,000 20,000
Less: Rebate u/s. 87A (Lower of tax payable or ₹25,000) 20,000
Tax liability Nil

➢ If total income is more than ₹7,00,000 and tax payable on such income is more
than the total income in excess of ₹7,00,000, then rebate equals to
Step 1: Calculate tax liability before HEC on total income
Step 2: Calculate total income in excess of ₹7,00,000
Step 3: Step 1-Step 2
➢ Rebate shall be provided on tax before health and education cess.

For Example:
Mr. Shyam aged 68 years and a resident in India, has a total income of ₹7,15,000,
comprising his salary income and interest on bank fixed deposit. Compute his tax
liability for A.Y. 2024-25 under default tax regime under section 115BAC.

Solution:

Computation of tax liability of Mr. Shyam for A.Y. 2024-25


Tax on total income of ₹7,15,000 26,500
Less: Rebate u/s. 87A 11,500
[Working note:
Step 1: Tax on total income of ₹7,15,000 26,500
Step 2: Total income in excess of ₹7,00,000 15,000
Step 3: Rebate = Step 1 (-) Step 2 11,500]
15,000
Add: HEC @ 4% 600
Tax Liability 15,600

Where assessee has opted out of default tax regime under section 115BAC
➢ Total income shall not exceed ₹5,00,000
➢ Rebate equals to
a) Tax payable on total income or
b) ₹12,500
Whichever is lower.
➢ Rebate shall be provided on tax before health and education cess.

22 | P a g e #HakkसेCA by CA. Akash Sir


23
1. Basics of Income Tax, Computation of Total Income and Tax Liability

For Example:
Mr. Rohan is a resident senior citizen and has income of ₹4,50,000 from grocery
store. Calculate his tax liability for A.Y. 2024-25 under old tax regime.

Solution:

Computation of tax liability of Mr. Rohan for A.Y. 2024-25


First 3,00,000 Nil
Next 1,50,000 @ 5% 7,500
7,500
Less: Rebate u/s. 87A 7,500
Tax liability 0

Important note on both old and new tax regime:


Rebate shall not be allowed on tax payable on long term capital gains taxable under
section 112A.

Mann Ki Baat (simple language):

We have studies slab rate in case of individual and we know that where the total income
of the assessee is upto ₹5,00,000, he shall pay tax (before HEC) of ₹12,500 under old
tax regime. Now, in order to reduce burden of tax on small tax payer, the government
has introduced section 87A which provides that in case of resident individual having
total income upto ₹5,00,000, a rebate of upto ₹12,500 shall be provided from tax
liability.
Please note that if the total income is even slightly higher than ₹5,00,000 (let’s say
5,00,010), then the individual has to pay tax on total income without any rebate under
old tax regime.
Similarly, where the total income of the assessee is upto ₹7,00,000, he shall pay tax
(before HEC) of ₹25,000 under new tax regime. Now, in order to reduce burden of tax
on small tax payer, the government has introduced section 87A which provides that in
case of resident individual having total income upto ₹7,00,000, a rebate of upto
₹25,000 shall be provided from tax liability.
Please note that if the total income is even slightly higher than ₹7,00,000 (let’s say
7,10,000), then the individual shall be allowed a rebate of differential tax burden under
new tax regime.

HW Question: 7

Calculate tax liability for the following for A.Y. 2024-25 in following cases:
a) Mr. Rajkumar, resident individual, aged 46 years having total income of
₹4,99,000

23 | P a g e #HakkसेCA by CA. Akash Sir


24
1. Basics of Income Tax, Computation of Total Income and Tax Liability

b) Mr. Shyam, a non-resident individual, aged 59 years having total income of


₹3,00,000
c) Mr. Hanuman, a resident senior citizen having total income of ₹5,50,000
d) Mr. Vibhishan, a resident super senior citizen having total income of ₹5,10,000
e) Mrs. Sita, a resident individual, aged 61 years having total income of ₹4,50,000
f) Miss Riya, a non-resident individual, aged 62 years having total income of
₹6,50,000
g) Miss Kanika, a resident individual, aged 82 years having total income of
₹7,12,000
Case 1: Assuming that they have not opted for the provisions of section 115BAC.
Case 2: Assuming that they have opted for the provisions of section 115BAC.

24 | P a g e #HakkसेCA by CA. Akash Sir


25
2. Residential Status and Scope of Total Income

Residential Status [Section 6]


Residential status means relationship of a person with the country or territory in
which he resides. Scope of total income means the nature of income which are to be
included in “total income” of a person and accordingly that person is required to pay
tax.

Mann Ki Baat (simple language):

First question that comes in mind is why residential status is important to study for
income tax purposes? Just imagine a situation that the Government is levying taxes on
every person for the income earned by them without actually determining whether he is
a Resident in India or not. Residential status of a person actually helps the government
in determining whether a person is required to pay tax or not and consequently which
incomes can be taxed in his hand i.e. his scope of total income. Below diagram
summarises the discussion we will have in this chapter with respect to residential
status.

Next question is that do we determine residential status of a person for each


previous year? Obviously yes. Because a person who is resident in one year can become
non-resident in another year. Let’s study residential status.

How to determine residential status of a person (i.e. Individual, HUF, Company


etc.)

Just think whether it is simple to say that every citizen of India is resident in India or
every company incorporated outside India is non-resident in India? Answer to this
question should be “No”, simply because citizenship has nothing to do with taxing
income of a person and every company which is incorporated outside India does not
mean that it has no income generated from India and thus not liable to tax.

25 | P a g e #HakkसेCA by CA. Akash Sir


26
2. Residential Status and Scope of Total Income

So, there must be some formula (criteria) to determine the residential status of a
person and we are going to study that in coming part of this chapter.

TP: 1 Residential status of an Individual [Section 6(1)]

Under section 6(1), an individual is said to be resident in India in any previous year, if
he satisfies any one of the following two conditions:
(i) He stays in India for 182 days or more during the relevant previous year;
(ii) He stays in India for 60 days or more during relevant previous year and also for
365 days or more during 4 years preceding the relevant previous year.

If the individual does not satisfy any one of the above two conditions, he is a non-
resident.

For example:

# Number of days stayed in Number of days stayed in Residential


India during P.Y. 2023-24 India during preceding 4 PYs Status
1. 200 0 Resident
2. 181 365 Resident
3. 59 400 Non-Resident
4. 170 360 Non-Resident
5. 60 365 Resident
6. 183 400 Resident
7. 50 100 Non-Resident

Mann Ki Baat (simple language):

Above conditions to determine residential status of individual are simple. Now, consider
a situation wherein a person stays in water, say around 10 nautical miles into the sea
from Indian coastline for almost 200 days due to nature of his work. He might be
working for an oil extraction company. Will you say, he stays in India during those 200
days? Reconsider that he stays in water, say around 100 nautical miles into the sea
from Indian coastline for 365 days. Will you say, he stays in India during that year?
You must be thinking that this answer is not that simple. It is very easy to conclude
basis our gut feelings that answer to first situation is “Yes” and answer to second
situation is “No” but act never leaves an answer on gut feelings of a reader.

26 | P a g e #HakkसेCA by CA. Akash Sir


27
2. Residential Status and Scope of Total Income

TP: 2 Some other important points

1. The term “stay in India” includes stay in the territorial waters of India (i.e. 12
nautical miles into the sea from the Indian coastline). Even the stay in a ship or boat
in the territorial waters of India shall be considered to be stay in India. (1 nautical
mile = 1.1515 miles = 1.852 Kms).
2. It is not necessary that the period of stay must be continuous nor is it essential
that the stay should be at the usual place of residence, business or employment of
the individual.
3. For the purpose of counting the number of days stayed in India, both the date of
departure as well as the date of arrival are considered to be in India.[ Important]

Illustration: 1

Determine residential status in following cases:

# Name Stay in India during Stay in India during Status


P.Y. 2023-24 past 4 PYs
1. Ravish 183 40
2. Lavish 181 370
3. Kavish 65 380
4. Rabish 59 400
5. Vashish 82 364

Solution:

As per section 6(1), an individual is said to be resident in India in any previous year, if
he satisfies any one of the following two conditions:
(i) He stays in India for 182 days or more during the relevant previous year;
(ii) He stays in India for 60 days or more during relevant previous year and also for
365 days or more during 4 years preceding the relevant previous year.
If the individual does not satisfy any one of the above two conditions, he is a non-
resident.

Accordingly, residential status shall be as follow:

Ravish – Resident, Lavish – Resident, Kavish – Resident, Rabish – Non-Resident, Vashish


– Non-Resident. (Write full explanation in exam which means write section, ten number
of days stay in India, then conclusion)

27 | P a g e #HakkसेCA by CA. Akash Sir


28
2. Residential Status and Scope of Total Income

HW Question: 1

Determine residential status of individual in following cases. Write your answer along
with relevant provision and explanation (for practice purpose):

# Name Stay in India during Stay in India Status


P.Y. 2023-24 during past 4 PYs
1. Ramu 364 0
2. Samu 300 5
3. Meenu 165 65
4. Sheenu 182 64
5. Monu 125 370
6. Kanu 25 400

HW Question: 2

Mr. Mohan has a flat in Delhi where he normally resides while he visits India. On
02.07.2023, he visited India and stays in Hotel in Mumbai for some personal purpose.
Determine his residential status for A.Y. 2024-25. He claims that he shall be treated
as non-resident since he has not stayed at usual place of residence. Comment.

TP: 3 Explanation to section 6(1)

As per explanation to section 6(1), certain individuals will be considered to be resident


in India only if they stay in India for 182 days or more during the relevant previous
year (We call it as “Special Category” for our reading purpose. Do not use this word in
exams) and such individuals are:

1. Individual + Citizen of India + Left India + For employment outside India

Any individual who is a citizen of India and has left India for taking up any business
or profession or employment outside India. For example, Mr. Vimal is a citizen of
India and has left India on 11.08.2023 for taking up an employment in USA, in this
case he will be covered in the special category and his status shall be non-resident
since his total stay is less than 182 days during P.Y. 2023-24.
If any such person is employed in India and he has been transferred outside India,
he will also be covered in the special category. For example, Mr. Bimal is employed in
ICICI bank in India and he has been transferred to the London branch, in this case
he will also be covered in the special category.

Important note: If any person has business or profession in India and he is going
out of India in connection with business or profession, he will not be covered in
special category.

28 | P a g e #HakkसेCA by CA. Akash Sir


29
2. Residential Status and Scope of Total Income

Illustration: 2

Miss Monika, an Indian citizen, leaves India on 22.09.2023 for the first time, to work
as an officer of a company in France. Determine her residential status for the A.Y.
2024-25.

Solution:

As per section 6(1), an individual is said to be resident in India in any previous year, if
he satisfies any one of the following two conditions:
(i) He stays in India for 182 days or more during the relevant previous year;
(ii) He stays in India for 60 days or more during relevant previous year and also for
365 days or more during 4 years preceding the relevant previous year.
If the individual does not satisfy any one of the above two conditions, he is a non-
resident.

Also, as per explanation to section 6(1), any individual who is a citizen of India and has
left India for taking up any business or profession or employment outside India will be
considered to be resident only if they stay in India for 182 days or more during the
relevant previous year.

During the previous year 2023-24, Miss Monika, an Indian citizen, stayed in India for
175 days (i.e. 30 + 31+ 30 + 31 + 31 + 22 days). Therefore, she does not satisfy the
minimum criteria of 182 days. Also, since she is an Indian citizen leaving India for the
purposes of employment, the second condition under section 6(1) is not applicable to
her. Therefore, Miss Monika is a non-resident in India for the A.Y. 2024-25.

HW Question: 3

Mr. Hansraj, an Indian Citizen, has business in India and has left India on 29.06.2023
to USA for a business deal. He came back to India on 05.04.2024. His total stay in
India during past 4 PYs is 380 days. Determine his residential status for A.Y. 2024-25.

2. Individual + Citizen of India/ Person of Indian Origin + Visit India +


Employment outside India

Any individual who is a citizen of India or is a person of Indian origin* and is having
business or profession or employment outside India comes on a visit to India during
relevant previous year. For example, Mr. Akshay is a citizen of India and is settled
as a doctor in USA and has come to India on a visit for 181 days, he will be covered in
the special category and his status shall be non-resident.

29 | P a g e #HakkसेCA by CA. Akash Sir


30
2. Residential Status and Scope of Total Income

*A person is said to be of Indian origin if he or either of his parents or either of his


grandparents (including parents of mother) were born in undivided India.
For example, Mr. Rohan has taken birth in USA and is a citizen of USA but his
grandfather has taken birth in India in 1942, in this case Mr. Rohan will be
considered to be a person of Indian origin.

Illustration: 3

Mr. Alok and Mrs. Alok are settled outside India for the purpose of employment and
they came to India on 15.10.2023 on a visit for 7 months. Both of them are Indian
citizens. In the earlier years they were in India as follows:
Year Mr. Alok Mrs. Alok
2022 – 2023 235 Days 365 Days
2021 – 2022 330 Days 30 Days
2020 – 2021 Nil 28 Days
2019 – 2020 118 Days 120 Days
Find out the residential status of Mr. Alok and Mrs. Alok for the assessment year
2024-25.

Solution:

As per section 6(1), an individual is said to be resident in India in any previous year, if
he satisfies any one of the following two conditions:
(i) He stays in India for 182 days or more during the relevant previous year;
(ii) He stays in India for 60 days or more during relevant previous year and also for
365 days or more during 4 years preceding the relevant previous year.
If the individual does not satisfy any one of the above two conditions, he is a non-
resident.

Also, as per explanation to section 6(1), any individual who is a citizen of India or is a
person of Indian origin and is having business or profession or employment outside
India comes on a visit to India during relevant previous year will be considered to be
resident only if they stay in India for 182 days or more during the relevant previous
year.

Stay of Mr. Alok in India during previous year 2023-24 is 169 Days {17 + 30 + 31 + 31 +
29 + 31} and stay of Mrs. Alok in India during previous year 2023-24 is 169 Days {17 +
30 + 31 + 31 + 29 + 31}. Since they are employed outside Indian and has come to visit
India during P.Y. 2023-24, they will be resident only if their stay in India in relevant
previous year is 182 days or more, hence they are non–resident.

30 | P a g e #HakkसेCA by CA. Akash Sir


31
2. Residential Status and Scope of Total Income

3. Individual + Citizen of India + Left India as crew member of Indian Ship

Any individual who is a citizen of India and has left India as a member of crew of
an Indian ship. The time period mentioned in Continuous Discharge Certificate
(CDC)** shall be considered to be the period of stay outside India and remaining
time period shall be considered to be stay in India.

** As per Rule 126 of Income Tax Rules, 1962, in case of an individual, being citizen
of India and crew member of an Indian ship, the period of stay in India shall not
include the period commencing from the date entered into the CDC in respect of
joining the ship and period ending on the date entered into CDC in respect of signing
off the ship.

Illustration: 4 [NOV – 2002 (3 Marks)]

In the year P.Y. 2023-2024, a sailor has remained on ship as a crew member of an
Indian Ship as follows:
(1) Outside the territorial waters of India for 184 days.
(2) Inside the territorial waters of India for 182 days.
Is he considered to be resident or not for the Assessment Year 2024-25. Comment.

Solution:

As per section 6(1), an individual is said to be resident in India in any previous year, if
he satisfies any one of the following two conditions:
(i) He stays in India for 182 days or more during the relevant previous year;
(ii) He stays in India for 60 days or more during relevant previous year and also for
365 days or more during 4 years preceding the relevant previous year.

Also, as per explanation to section 6(1), any individual who is a citizen of India and has
left India as a member of crew of an Indian ship will be considered to be resident only
if they stay in India for 182 days or more during the relevant previous year.

Since, he left India as a crew member of Indian ship, he will be considered to be


resident if he stays in India for 182 days or more. In the given case, period of stay in
India is 182 days. Therefore, the sailor is treated as a resident in India.

Illustration: 5

Mr. Ramesh is an Indian citizen and a member of the crew of a Singapore bound Indian
ship engaged in international traffic departing from Mumbai port on 6th June, 2023.
From the following details for the P.Y. 2023-24, determine the residential status of
Mr. Ramesh for A.Y. 2024-25, assuming that his stay in India in the last 4 previous

31 | P a g e #HakkसेCA by CA. Akash Sir


32
2. Residential Status and Scope of Total Income

years (preceding to P.Y. 2023-24) is 400 days and last seven previous years (preceding
to P.Y. 2023-24) is 750 days:
Particulars Date
Date entered into the Continuous Discharge Certificate in
respect of joining the ship by Mr. Ramesh 6th June, 2023
Date entered into the Continuous Discharge Certificate in
respect of signing off the ship by Mr. Ramesh 9th December, 2023

Solution:

As per section 6(1), an individual is said to be resident in India in any previous year, if
he satisfies any one of the following two conditions:
(i) He stays in India for 182 days or more during the relevant previous year;
(ii) He stays in India for 60 days or more during relevant previous year and also for
365 days or more during 4 years preceding the relevant previous year.

Also, as per explanation to section 6(1), any individual who is a citizen of India and has
left India as a member of crew of an Indian ship will be considered to be resident only
if they stay in India for 182 days or more during the relevant previous year. Moreover,
the time period mentioned in Continuous Discharge Certificate (CDC) shall be
considered to be the period of stay outside India and remaining time period shall be
considered to be stay in India.

Hence, the period beginning from 6th June, 2023 and ending on 9th December, 2023,
being the dates entered into the Continuous Discharge Certificate in respect of joining
the ship and signing off from the ship by Mr. Ramesh, has to be excluded for computing
the period of his stay in India. Accordingly, 187 days [25+31+31+30+31+30+9] have to
be excluded from the period of his stay in India. Consequently, Mr. Ramesh’s period of
stay in India during the P.Y. 2023-24 would be 179 days [i.e., 366 days – 187 days].

Since his period of stay in India during the P.Y. 2023-24 is less than 182 days, he is a
non-resident for A.Y. 2024-25.

Note - Since the residential status of Mr. Ramesh is “non-resident” for A.Y. 2024-25
consequent to his number of days of stay in P.Y. 2023-24 being less than 182 days, his
period of stay in the earlier previous years become irrelevant.

HW Question: 4 [RTP Nov 2020]

You are required to determine the residential status of Mr. Kamal, a citizen of India,
for the previous year 2023-24.

32 | P a g e #HakkसेCA by CA. Akash Sir


33
2. Residential Status and Scope of Total Income

Mr. Kamal is a member of crew of a Singapore bound Indian ship, carrying passengers in
the international waters, which left Kochi port in Kerala, on 16th August, 2023.
Following details are made available to you for the previous year 2023-24:
Particulars Date
Date entered into the Continuous Discharge Certificate in
respect of joining the ship by Mr. Kamal 16th August, 2023
Date entered into the Continuous Discharge Certificate in
respect of signing off the ship by Mr. Kamal 21st January, 2024

In June, 2023, he had gone out of India to Dubai on a private tour for a continuous
period of 27 days. During the last four years preceding the previous year 2023-24, he
was present in India for 425 days. During the last seven previous years preceding the
previous year 2023-24, he was present in India for 830 days.

Mann Ki Baat (simple language):

Basis above discussion, you have understood that the above categories of individuals
need to comply with only 1st condition of section 6(1) i.e. stay in India for 182 days or
more, to determine his residential status. This benefit is given to individuals to
discriminate them from other persons based on their foreign relationships and
employment.
Till now, we have learned to calculate residential status of an Individual basis number
of days he stays in India. Now, let’s make it more complicated. Let’s mix income criteria
with number of days.

TP: 4 Exception to above exception point 2

Individual + Citizen of India/ Person of Indian Origin + Visit India + Employment


outside India + Indian income>15 lakhs during PY

In case of an individual who is “a citizen of India or is a person of Indian origin and


is having business or profession or employment outside India comes on a visit to
India during relevant previous year” having income other than income from foreign
sources*** exceeding ₹15 lakhs during the previous year will be treated as resident
in India if:
(i) He stays in India for 182 days or more during the relevant previous year;
(ii) He stays in India for 120 days or more during relevant previous year and also
for 365 days or more during 4 years preceding the relevant previous year.
Further, as per section 6(6)(c), he will be Resident but not ordinarily resident (NOR)
if his stay is less than 182 days during relevant previous year (Covered in later part
of the chapter).

33 | P a g e #HakkसेCA by CA. Akash Sir


34
2. Residential Status and Scope of Total Income

***Income which accrues or arises outside India (except income from a business
controlled from or profession setup in India) and which is not deemed to accrue or
arise in India.

Mann Ki Baat (simple language):

If you read exception 2 and exception to exception 2 together, then, you will notice
that where the individual is earning less than or equals 15 lakhs in India, only 1st
condition to section 6(1) needs to be checked and where the individual is earning more
than 15 lakhs, one additional condition needs to be checked to determine residential
status. In means that residential status is not only dependent upon stay but also
depends upon income of individual.

Illustration: 6

Mr. Alok and Mrs. Alok are settled outside India for the purpose of employment and
they came to India on 15.10.2023 on a visit for 7 months. Both of them are Indian
citizens. In the earlier years they were in India as follows:
Year Mr. Alok Mrs. Alok
2022 – 2023 235 Days 365 Days
2021 – 2022 330 Days 30 Days
2020 – 2021 Nil 28 Days
2019 – 2020 118 Days 120 Days
Find out the residential status of Mr. Alok and Mrs. Alok for the assessment year
2024-25 assuming that Mr. Alok and Mrs. Alok has total income from India of
₹16,00,000 and 14,00,000 respectively during P.Y. 2023-24.

Solution:

As per section 6(1), an individual is said to be resident in India in any previous year, if
he satisfies any one of the following two conditions:
(i) He stays in India for 182 days or more during the relevant previous year;
(ii) He stays in India for 60 days or more during relevant previous year and also for
365 days or more during 4 years preceding the relevant previous year.
If the individual does not satisfy any one of the above two conditions, he is a non-
resident.

Also, as per explanation to section 6(1), any individual who is a citizen of India or is a
person of Indian origin and is having business or profession or employment outside
India comes on a visit to India during relevant previous year will be considered to be
resident only if they stay in India for 182 days or more during the relevant previous
year.

34 | P a g e #HakkसेCA by CA. Akash Sir


35
2. Residential Status and Scope of Total Income

Also, in case of an individual who is “a citizen of India or is a person of Indian origin and
is having business or profession or employment outside India comes on a visit India
during relevant previous year” having income other than income from foreign sources
exceeding ₹15 lakhs during the previous year will be treated as resident in India if:
(i) He stays in India for 182 days or more during the relevant previous year; or
(ii) He stays in India for 120 days or more during relevant previous year and also for
365 days or more during 4 years preceding the relevant previous year.

Further as per section 6(6)(c), he will be NOR if his stay is less than 182 days during
relevant previous year

Stay of Mr. Alok in India during previous year 2023-24 is 169 Days {17 + 30 + 31 + 31 +
29 + 31} and stay of Mrs. Alok in India during previous year 2023-24 is 169 Days {17 +
30 + 31 + 31 + 29 + 31}.

Therefore, Mr. Alok having Indian income of ₹16,00,000 would be treated as NOR
during A.Y. 2024-25 and Mrs. Alok is non–resident.

HW Question: 5

Mr. Kushal, an Indian Citizen is employed outside India and he visited India on
01.11.2023 and left India on 30.05.2024. In the earlier years he was in India as follows:
Year Days
2022 – 2023 235 Days
2021 – 2022 330 Days
2020 – 2021 Nil
2019 – 2020 118 Days
Find out the residential status of Mr. Kushal for the assessment year 2024-25
assuming that:
a) His total income from India is ₹12,00,000 for P.Y. 2023-24
b) His total income from India is ₹17,00,000 for P.Y. 2023-24

TP: 5 Deemed Resident [Section 6(1A)]

An individual being an Indian citizen, having total income, other than income from
foreign sources***, exceeding ₹15 lakhs during the previous year, would be deemed
to be resident in India in that previous year, if he is not liable to pay tax in any other
country or territory by reason of his domicile or residence or any other criteria of
similar nature. However, this provision will not apply in case of an individual who is a
resident in India in the previous year as per section 6(1).

35 | P a g e #HakkसेCA by CA. Akash Sir


36
2. Residential Status and Scope of Total Income

Further as per section 6(6)(d), he will be NOR (Covered in later part of the chapter).

***Income which accrues or arises outside India (except income from a business
controlled from or profession setup in India) and which is not deemed to accrue or
arise in India.

Mann Ki Baat (simple language):

This is new section introduced by the Government in Finance Act, 2020. If you analyse
this carefully, you will know that all those Indian citizens who have income exceeding
₹15 lakhs from India and are not liable to tax in other country have been treated as
deemed resident in India (NOR) during that previous year. This has increased the scope
of their income being taxed in India because such individuals even if do not stay in
India for a single day would be deemed to be Resident (NOR) in India in that previous
year.

TP: 6 Resident and ordinarily resident (ROR) and Not-ordinarily


resident (NOR) [Section 6(6)]

As per section 6(6)(a) of the Act, an individual who is resident in India shall be
considered to be NOR if he has complied with at least one of the conditions given
below:
(i) If such individual has during the 7 previous years preceding the relevant previous
year been in India for a period of 729 days or less, or;
(ii) If such individual has been non-resident in India in 9 years out of 10 previous
years preceding the relevant previous year.

If he has not complied with even a single condition, he will be considered to be ROR.

For example:

Mr. Bean stays in India for 360 days during the P.Y. 2023-24. His total stay during 7
PYs preceding the current PY is 720 days. In this case, he shall be NOR in India.

Further, let’s suppose that his total stay in India during 7 PYs preceding current PY is
750 days, however, he has been NR during 9 PYs preceding the current PY. In this case
as well, he shall be NOR in India.

Further, let’s suppose that his total stay in India during 7 PYs preceding the current PY
is 800 days and he has been Resident in India for 2 years out of 10 PYs preceding
current PY, then he shall be ROR in India.

36 | P a g e #HakkसेCA by CA. Akash Sir


37
2. Residential Status and Scope of Total Income

Mann Ki Baat (simple language):

You might be thinking that whether the ROR and NOR are categories of a Resident?
You are right. Now a very simple question comes, why we need these categories? Let’s
think about this. Let’s think if you want to differentiate between residential status of
a resident individual who stays in India for longer period and a resident individual who
barely stays in India. Answer to this question should be “Yes” and this is why the
Government has included concept of ROR and NOR in Income tax act. You will
understand this more when you will have clarity of types of income which are taxed for
both the individuals (discussed in later part of this chapter).

Illustration: 7

Mr. James Bond an American citizen has come to India for the first time on
10.07.2019, as an employee of a multinational company. The particulars of his arrival
and departure are as given below:
Date of Arrival Date of Departure
10.07.2019 07.08.2020
07.10.2020 27.11.2021
01.03.2022 01.02.2023
10.05.2023 30.03.2024
Not yet returned
Determine his residential status for previous year 2019-20 to 2023-24 assuming that
the provisions for A.Y. 2024-25 were applicable during prior years as well.

Solution:

As per section 6(1), an individual is said to be resident in India in any previous year, if
he satisfies any one of the following two conditions:
(i) He stays in India for 182 days or more during the relevant previous year;
(ii) He stays in India for 60 days or more during relevant previous year and also for
365 days or more during 4 years preceding the relevant previous year.
If the individual does not satisfy any one of the above two conditions, he is a non-
resident.

As per section 6(6)(a) of the Act, an individual who is resident in India shall be
considered to be NOR if he has complied with at least one of the conditions given
below:
(i) If such individual has during the 7 previous years preceding the relevant previous
year been in India for a period of 729 days or less, or;
(ii) If such individual has been non-resident in India in atleast 9 years out of 10
previous years preceding the relevant previous year.

37 | P a g e #HakkसेCA by CA. Akash Sir


38
2. Residential Status and Scope of Total Income

Previous Year 2019-20:


Days of stay in India are 266.
Since his stay in India is more than 182 days during the P.Y. 2019-20, hence he is
resident and his stay during 7 years is Nil hence, he is NOR.

Previous Year 2020-21:


Days of stay in India are 305.
Since his stay in India is more than 182 days during P.Y. 2020-21, hence he is resident
and his stay during 7 years is 266 days hence, he is NOR.

Previous Year 2021-22:


Days of stay in India are 272.
Since his stay in India is more than 182 days during P.Y. 2021-22, hence he is resident
and his stay during 7 years is 571 days hence, he is NOR.

Previous Year 2022-23:


Days of stay in India are 307.
Since his stay in India is more than 182 days during P.Y. 2022-23, hence he is resident
and his stay during 7 years is more than 729 days hence he is not able to comply with
first condition of section 6(6)(a) and he is not able to comply second condition also i.e.
non-resident in at-least 9 years out of 10 years preceding the relevant previous year.
Hence, he is ROR.

Previous Year 2023-24:


Days of stay in India are 326.
Since his stay in India is more than 182 days during P.Y. 2023-24, hence he is resident
and his stay during 7 years is more than 729 days hence he is not able to comply with
first condition of section 6(6)(a) and he is not able to comply second condition of
section 6(6)(a) also i.e. non-resident in atleast 9 years out of 10 years preceding the
relevant previous year. Hence, he is ROR.

HW Question: 6

Mr. Shiva an American citizen has come to India for the first time on 01.07.2020 as an
executive of a multinational company. His employer has allowed him to visit USA every
year and for this purpose he will be leaving India every year on 1st November and shall
come back on 31st December, besides that he has visited Hong Kong on several
occasions in connection with the official work, because he is looking after the
employer’s operations in Hong Kong also, with details as under:

38 | P a g e #HakkसेCA by CA. Akash Sir


39
2. Residential Status and Scope of Total Income

Date of leaving India Date of arriving in India


10.09.2020 30.09.2020
07.02.2021 08.05.2021
11.07.2021 21.10.2021
10.02.2022 23.07.2022
11.02.2023 12.06.2023
01.02.2024 10.04.2024
Determine his residential status for the previous years 2020-21 to 2023-24 assuming
that the provisions for A.Y. 2024-25 were applicable during prior years as well.

HW Question: 7

Mr. Dey, a non-resident, residing in US since 1990, came back to India on 1.4.2022 for
permanent settlement. What will be his residential status for assessment year 2024-
25?

TP: 7 Residential Status of HUF [Section 6(2)]

As per section 6(2), an HUF would be resident in India if the control and management
of its affairs is situated wholly or partly in India. If the control and management of
the affairs is situated wholly outside India it will be considered to be non-resident.

The expression ‘control and management’ refers to the central control and
management and not to the carrying on of day-to-day business by servants,
employees or agents.

Resident and ordinarily resident (ROR) and Not-ordinarily resident (NOR)

If a HUF is resident, as per section 6(6)(b), it will be considered to be NOR if its


Karta has complied with at least one of the conditions given below:
(i) If the karta is in India during the 7 previous years preceding the relevant
previous year for a period of 729 days or less, or
(ii) If the karta is non-resident in India in any 9 out of the 10 previous years
preceding the relevant previous year.
If karta has not complied with even a single condition, HUF shall be ROR.

Mann Ki Baat (simple language):

From the above, please notice that it is very simple to determine residential status of a
HUF in theoretical life. However, you can imagine that it is equally difficult in practical
world simply because this needs in depth analysis of facts to determine control and
management.

39 | P a g e #HakkसेCA by CA. Akash Sir


40
2. Residential Status and Scope of Total Income

Please note from above that the residential status of HUF (ROR or NOR) is linked with
residential status of Karta. If Karta has complied with any of the conditions mentioned
above, then HUF is NOR otherwise ROR.

Now, most of you must be thinking that if Karta is ROR in a previous year, then HUF is
also ROR and if Karta is NOR in a previous year, then HUF is also NOR. This is not
correct.
Let’s take an example, An HUF is a resident in India during P.Y. 2023-24. Mr. Ram,
Karta of HUF, has stayed in India for 59 days during P.Y. 2023-24 and his total stay in
India during 7 preceding PYs is 730 days and he has been resident in India during past
2 years. Now, applying section 6(6)(a), Mr. Ram is NR in India during P.Y. 2023-24,
however, applying section 6(6)(b), HUF is ROR in India during P.Y. 2023-24.

Now, one will say that if Karta is resident in India during PY, then residential status of
Karta (ROR or NOR) will determine the residential status of HUF. This is not correct.
Let’s take another example, An HUF is wholly managed and controlled from outside
India during the P.Y. 2023-24. Mr. Ram, Karta of HUF, is ROR in India during the P.Y.
2023-24 as per provisions of section 6(1) read with section 6(6)(a). Now, applying the
provision of section 6(2), HUF is NR in India.

Let’s conclude the discussion by saying that, there are many examples where residential
status of HUF will be same as of Karta. However, we cannot generalise this statement

40 | P a g e #HakkसेCA by CA. Akash Sir


41
2. Residential Status and Scope of Total Income

and therefore, keep your mind open and apply provisions instead of taking a shorter
path.

Illustration: 8

One HUF is controlled and managed from India. The Karta of the Hindu Undivided
Family comes to India every year for minimum 60 days and maximum 91 days.
Determine residential status of the Hindu Undivided Family and also that of the Karta
for the assessment year 2024-25.

Solution:

Hindu Undivided Family is resident since it is controlled and managed from India. Since,
the total stay of the Karta during seven years can be maximum 637 days, hence, Hindu
Undivided Family shall be considered to be resident but not ordinarily resident. Total
stay of Karta during PY is maximum 91 days and during 4 preceding PYs is 364 days
therefore, in his individual capacity is non-resident because he cannot comply with even
one of the two conditions given under section 6(1).

HW Question: 8

One Hindu Undivided Family is being managed partly from Mumbai and partly from
Nepal. Mr. Sam (a foreign citizen), Karta of Hindu Undivided Family, comes on a visit to
India every year since 1982 in month of April for 105 days. Determine residential
status of the Hindu Undivided Family and also that of the Karta in his individual
capacity for the assessment year 2024-25.

HW Question: 9 [JUNE – 2009 (2 Marks)]

State with reason, whether the following statements are True or False:
Mr. Ram, Karta of HUF, claims that the HUF is non-resident as the business of HUF is
transacted from UK and all the policy decisions are taken there.

TP: 8 Residential Status of a Company [Section 6(3)]

As per section 6(3), an Indian company is always resident in India even if its control
and management is outside India or its business is outside India.
A foreign company shall be resident in India if its place of effective management,
at any time in that year, is in India.

41 | P a g e #HakkसेCA by CA. Akash Sir


42
2. Residential Status and Scope of Total Income

“Place of effective management (POEM)” means a place where key management and
commercial decisions that are necessary for the conduct of the business of an entity
as a whole are, in substance made [explanation to section 6(3)].

NOTE – The guidelines issued by CBDT for determination of POEM of a foreign


company and transition mechanism for a company which is incorporated outside
India, which has not been assessed to tax in India earlier and has become resident in
India for the first time due to application of POEM, has been provided in Chapter
Xll-BC. The same will be dealt with at the final level.

Mann Ki Baat (simple language):

Now, let’s apply the similar concept of HUF on company. One may definitely conclude
that if control and management of company is situated wholly outside India, then
company is a non-resident otherwise resident. However, the act has been amended few
years back to change the concept for companies and new concept of Place of Effective
Management (POEM) was introduced. There is a separate very lengthy discussion on
POEM which is part of CA Final.

Illustration: 9

POPULAR Inc., a Swedish company headquartered at Stockholm, not having a


permanent establishment in India, has set up a liaison office in Mumbai in April, 2023 in
compliance with RBI guidelines to look after its day-to-day business operations in India,
spread awareness about the company’s products and explore further opportunities. The
liaison office takes decisions relating to day-to-day routine operations and performs

42 | P a g e #HakkसेCA by CA. Akash Sir


43
2. Residential Status and Scope of Total Income

support functions that are preparatory and auxiliary in nature. The significant
management and commercial decisions are, however, in substance made by the Board of
Directors at Sweden. Determine the residential status of POPULAR Inc. for A.Y. 2024-
25.

Solution:

As per Section 6(3), a company would be resident in India in any previous year, if-
(i) it is an Indian company; or
(ii) its place of effective management, in that year, is in India.

In this case, POPULAR Inc. is a foreign company. Therefore, it would be resident in


India for P.Y. 2023-24 only if its place of effective management, in that year, is in
India.

Explanation to section 6(3) defines “place of effective management” to mean a place


where key management and commercial decisions that are necessary for the conduct of
the business of an entity as a whole are, in substance made.

In the case of POPULAR Inc., its place of effective management for P.Y. 2023-24 is
not in India, since the significant management and commercial decisions are, in
substance, made by the Board of Directors outside India in Sweden. POPULAR Inc. has
only a liaison office in India through which it looks after its routine day to day business
operations in India. The place where decisions relating to day-to-day routine operations
are taken and support functions that are preparatory or auxiliary in nature are
performed are not relevant in determining the place of effective management.

Hence, POPULAR Inc., being a foreign company is a non-resident for A.Y. 2024-25,
since its place of effective management is outside India in the P.Y. 2023-24.

HW Question: 10

Adani Ltd. an Indian company has most of its business outside India. Determine its
residential status.

HW Question: 11

Rock Infrastructure International Ltd. is incorporated in Mauritius and its place of


effective management is in Mauritius. Determine its residential status for the
assessment year 2024-25.

43 | P a g e #HakkसेCA by CA. Akash Sir


44
2. Residential Status and Scope of Total Income

HW Question: 12

Best Ltd., a foreign company and it carries on majority of its operations and decision
making activities from Calcutta and Assam but some part of operational activities and
few decisions are being taken from the place at which registered office of Best Ltd. is
located, i.e. Dhaka. Determine its residential status for the assessment year 2024-25.

TP: 9 Residential Status of others such as firms, AOP, BOI,


local authorities and artificial juridical person:

These persons would be resident in India if the control and management of its
affairs is situated wholly or partly in India. Where the control and management of
the affairs is situated wholly outside India, they would become non-residents.

44 | P a g e #HakkसेCA by CA. Akash Sir


45
2. Residential Status and Scope of Total Income

Scope of total income [Section 5]


TP: 10 What is scope of total income?

It means the nature of income which are to be included in “total income” of a person
based on his residential status (as determined in residential status chapter) and
accordingly that person is required to pay tax on such income as per the provisions of
Income tax law. Under the scope of total income, we will study about the types of
income which are to be included in the total income of ROR, NOR and NR.

Mann Ki Baat (simple language):

We have understood the various provisions for various persons to determine their
residential status. Now, question comes that why it is important to determine? How
residential status plays an important role in computation of total income of a person?
Why have we studied residential status? All these questions have the same answer and
that is “to determine the scope of total income of a person”.

TP: 11 Scope of total income in hands of ROR [Section 5(1)]

As per section 5(1), total income of ROR would include-


(i) income accruing/ arising in India during the PY;
(ii) income deemed to accrue/ arise in India during the PY;
(iii) income received in India during the PY even if accruing/arising outside India;
(iv) income deemed to be received in India during the PY even if accruing/arising
outside India;
(v) income accruing/ arising outside India even if it is not received or brought into
India during the PY.
In simpler terms, ROR has to include his world income in his total income in India.

For example, Mr. Anubhav is ROR in India during P.Y. 2023-24 and has earned income
from let-out of house property in Canada. In this case, even if his income in accrued/
arised outside India, it shall be included in his total income in India.

TP: 12 Scope of total income in hands of NOR [Section 5(1)]

As per section 5(1), total income of NOR would include-


(i) income accruing/ arising in India during the PY;
(ii) income deemed to accrue/ arise in India during the PY;
(iii) income received in India during the PY even if accruing/arising outside India;

45 | P a g e #HakkसेCA by CA. Akash Sir


46
2. Residential Status and Scope of Total Income

(iv) income deemed to be received in India during the PY even if accruing/arising


outside India;
(v) income accruing/ arising outside India and received outside India but only from a
business controlled from India or from a profession which was set up in India*.

* Profession set up in India means that it was originally setup in India and
subsequently there was an expansion outside India. For example, CA. Akash started
his profession in Rajasthan and subsequently he opened his branch outside India, it
will be called profession setup in India.

For example, Mr. Zakir, a comic, is a professional who started his profession from
India and now he has setup his profession in USA as well. He has earned ₹10,00,000
from the show in USA. He is NOR in India during the PY. In this case, even if income is
accrued/ arised outside India, it shall be included in the total income in India.

TP: 13 Scope of total income in hands of NR [Section 5(2)]

As per section 5(2), total income of NR would include,


(i) income accruing/ arising in India during the PY;
(ii) income deemed to accrue/ arise in India during the PY;
(iii) income received in India during the PY even if accruing/arising outside India.
(iv) income deemed to be received in India during the PY even if accruing/arising
outside India.

For example, Mr. Bhuwan, an Indian based comic, is NR in India during the P.Y. 2023-
24. He has earned ₹20,00,000 from a show in USA which he deposited in USA bank
account. In this case, his total income in India shall not include above income since
income is accrued outside India.

Mann Ki Baat (simple language):

From above, we understand that certain incomes which are includible in the total
income of ROR are not includible in total income of NOR or NR. This is why we have
studied residential status first. Now, based on the discussion above, we have to
understand the meaning of each technical term which has been used in section 5(1) and
5(2) for better understanding of the provisions.

Clarification [Circular no. 13/2017 dated 11.04.2017 and 17/2017 dated


26.04.2017]: Salary income received by NR seafarers, for services rendered
outside India as a crew member on foreign bound ship (Indian Flag or Foreign Flag),
into NRE account maintained with an Indian Bank shall be included in his total income.

46 | P a g e #HakkसेCA by CA. Akash Sir


47
2. Residential Status and Scope of Total Income

TP: 14 What is meaning of “income accruing/ arising in India”?

Accrue means “Right to receive Income”. For example, salary become due on 31stday
of the month.
Due means “Right to enforce payment”. For example, salary for the work done will
accrue throughout the month.
Receipt means “Actual receipt of Income”. For example, actual credit of salary on
the 1st day of next month.

TP: 15 What is the meaning of “income deemed to accrue/ arise


in India”? [Section 9(1)]

As per section 9(1), certain types of income are deemed to accrue or arise in India
even though they may actually accrue or arise outside India.

In simple words, income deemed to accrue in India includes those specific income
which actually arise outside India but are deemed to be arising in India so that
Indian government can tax those income in India.

Mann Ki Baat (simple language):

The one who is reading it for the first time might think that how it is possible that a
certain income which is accrued outside India (means which is generated from an asset
outside India) can be said to be accrued in India. This concept is called deeming
provisions where prima facie it appears that income is not accrued in India however,
the Government has made it as “deemed to be accrue/ arise in India” to tax such
income in India in hands of NOR and NR. Let’s understand the meaning in detail.

TP: 16 Business Connection/ Assets in India [Section 9(1)(i)]

Any income accruing or arising outside India whether directly or indirectly


(A) through or from any business connection in India,
(B) through or from any property in India,
(C) through or from any asset or source of income in India or
(D) through the transfer of a capital asset situated in India,
would be deemed to accrue or arise in India.

Now, let’s understand the concept one by one.

47 | P a g e #HakkसेCA by CA. Akash Sir


48
2. Residential Status and Scope of Total Income

A) Through or from any business connection in India

Q1. What is business connection?

Ans. (i) ‘Business connection’ shall include any business activity carried out
through a person acting on behalf of the non-resident.
(ii) Significant economic presence of a non-resident in India shall also constitute
business connection in India.

Q2. What are the conditions when we can say that a person is acting on
behalf of Non-resident?

Ans. (i) must have an authority, which is habitually exercised in India, to


conclude contracts or plays principal role in concluding contract on behalf of the
non-resident, or
(ii) if he does not have authority, he habitually maintains in India a stock of
goods or merchandise from which he regularly delivers goods or merchandise on
behalf of the non-resident, or
(iii) habitually secures orders in India, mainly or wholly for the non-resident.

Q3. What if agent secures orders for another non-resident (other than
assessee)? Shall it be considered as business connection in India of other
NR?

Ans. There will be business connection in India of other NR only if,


(i) such other non-resident controls the non-resident or
(ii) such other non-resident is controlled by the non-resident or
(iii) such other non-resident is subject to same control as that of non-
resident.

48 | P a g e #HakkसेCA by CA. Akash Sir


49
2. Residential Status and Scope of Total Income

Q4. What if business is carried out in India through independent agents?

Ans. No business connection in India if assessee carries on business activity


through a broker, general commission agent or any other agent having an
independent status, if such an agent is acting in the ordinary course of his
business.

Q5. What is meaning of significant economic presence?

Ans. Significant economic presence means

Nature of transaction Conditions


(a) in respect of any goods, Aggregate of payments arising
services or property carried out from such transaction or
by a non-resident with any person transactions during the previous
in India including provision of year should exceed ₹2 crores.
download of data or software in
India
(b) systematic and continuous The number of users should be
soliciting of business activities or atleast 3 lakhs.
engaging in interaction with users
in India

Q6. What are the transactions which are not treated as business connection
in India?

Ans.
(i) Purchase of goods in India for export,
(ii) Collection of news and views in India for transmission out of India
(iii) Shooting of cinematograph films in India for display outside India
(iv) Activities confined to display of rough diamonds in Special Notified Zones

B) & C) Income from property, asset or source of income in India

Any income which arises from any property (movable, immovable, tangible and
intangible property), asset or source would be deemed to accrue or arise in India. For
example, (i) Rent paid outside India for the use of the machinery situated in India.
(ii) Deposits with an Indian company for which interest is received outside India.

D) Income through transfer of a capital asset situated in India

Capital gains arising through the transfer of a capital asset situated in India would
be deemed to accrue or arise in India. For example, Mr. John of USA has
transferred a house property situated in Delhi to a resident of Germany for

49 | P a g e #HakkसेCA by CA. Akash Sir


50
2. Residential Status and Scope of Total Income

$1,50,000. In this case, capital gain arising on transfer of house property would be
deemed to accrue in India.

Note: Declaration of dividend by a foreign company (derive their value substantially


from assets situated in India) outside India does not have the effect of transfer of
any underlying assets located in India.

Illustration: 10

Mr. Ashish, an American Citizen has following incomes during the P.Y. 2023-24:

a) Capital gains on transfer of shares of Indian companies, sold in USA


and gains were received there 2,00,000
b) Income from property in Paris, received there 1,00,000
Calculate total income in his hand for A.Y. 2024-25 assuming that he is NR.

Solution:

Computation of Gross total income for A.Y. 2024-25:

a) Capital gains on transfer of shares of Indian companies, sold in USA


and gains were received there 2,00,000
b) Income from property in Paris, received there -
Gross total income 2,00,000

TP: 17 Salaries earned in India [Section 9(1)(ii)]

Income from salaries earned in India: Income, which falls under the head
“Salaries”, is deemed to accrue or arise in India, if it is earned in India. Salary
payable for service rendered in India would be treated as earned in India.

Further, any income under the head “Salaries” payable for rest period or leave period
which is preceded and succeeded by services rendered in India, and forms part of
the service contract of employment, shall be regarded as income earned in India.

For example, Mr. John who is working for Uniliver Limited in UK comes to India on an
assignment to work for Hindustan Uniliver Limited for 3 months. He received salary for
3 months in UK bank account. In this case, his salary for 3 months would be deemed to
accrue in India.

50 | P a g e #HakkसेCA by CA. Akash Sir


51
2. Residential Status and Scope of Total Income

TP: 18 Salaries by Government [Section 9(1)(iii)]

Income from salaries payable by the Government for services rendered outside
India: Income from ‘Salaries’ which is payable by the Government to a citizen of
India for services rendered outside India would be deemed to accrue or arise in
India.

However, allowances and perquisites paid or allowed outside India by the Government
to an Indian citizen for services rendered outside India is exempt, by virtue of
section 10(7).

For example, Mr. Lokesh, an Indian Citizen, is working in Aeronautical division for
Central Government of India. Indian Government has sent him to USA for job
purpose. He has received ₹20,00,000 as basic salary and ₹10,00,000 as allowance and
perquisites for services rendered outside India. In this case, ₹20,00,000 would be
deemed to accrue/ arise in India.

Illustration: 11

Mr. Manmohan, an Indian citizen aged 45 years, a government employee serving in the
Ministry of External Affairs, left India for the first time on 31.03.2023 due to his
transfer to High Commission of Singapore. He did not visit India any time during the
previous year 2023-24. He has received the following income for the Financial Year
2023-24:

S.No. Particulars ₹
(i) Salary (Computed) 5,00,000
(ii) Foreign Allowance [not included in (i) above] 4,00,000
(iii) Interest on fixed deposit from bank in India 1,00,000
(iv) Income from agriculture in Nepal 2,00,000
(v) Income from house property in Nepal 2,50,000
Compute his Gross Total Income for Assessment Year 2024-25.

Solution:

As per section 6(1), Mr. Manmohan is a non-resident for the A.Y. 2024-25, since he was
not present in India at any time during the previous year 2023-24.

As per section 5(2), a non-resident is chargeable to tax in India only in respect of


following incomes:
(i) Income received or deemed to be received in India; and
(ii) Income accruing or arising or deemed to accrue or arise in India.

51 | P a g e #HakkसेCA by CA. Akash Sir


52
2. Residential Status and Scope of Total Income

In view of the above provisions, income from agriculture in Nepal and income from
house property in Nepal would not be chargeable to tax in the hands of Manmohan,
assuming that the same were received in Nepal. Income from ‘Salaries’ payable by the
Government to a citizen of India for services rendered outside India is deemed to
accrue or arise in India as per section 9(1)(iii). Hence, such income is taxable in the
hands of Mr. Manmohan, even though he is a non-resident.

However, allowances or perquisites paid or allowed as such outside India by the


Government to a citizen of India for rendering service outside India is exempt under
section 10(7). Hence, foreign allowance of ₹4,00,000 is exempt under section 10(7) in
the hands of Mr. Manmohan.

Gross Total Income of Mr. Manmohan for A.Y. 2024-25


Particulars ₹
Salaries (computed) 5,00,000
Income from other sources (Interest on fixed deposit in India) 1,00,000
Gross Total Income 6,00,000

TP: 19 Dividend by Indian Company [Section 9(1)(iv)]

Dividend paid by an Indian company outside India: Dividends paid by an Indian


company outside India is deemed to be accrue or arise in India and would be taxable
in the hands of shareholders.

For example, Ajanta Limited, an Indian company has paid dividend outside India
during P.Y. 2023-24 to its all shareholders. In this case, dividend income would be
deemed to accrue/ arise in India.

Illustration: 12

Miss Rupali has received following dividends from various companies:

a) Dividend from Indian companies received in USA 4,50,000


b) Dividend from Apple Inc. (half received in India) 5,00,000

Calculate her gross total income from A.Y. 2024-25 assuming that she is (i) ROR (ii)
NOR (iii) NR.

Solution:

Computation of the Gross Total Income:

52 | P a g e #HakkसेCA by CA. Akash Sir


53
2. Residential Status and Scope of Total Income

Particulars ROR NOR NR


a) Dividend from Indian companies received
in USA 4,50,000 4,50,000 4,50,000
b) Dividend from Apple Inc. (half received
in India) 5,00,000 2,50,000 2,50,000
Gross total income 9,50,000 7,00,000 7,00,000

TP: 20 Interest [Section 9(1)(v)]

Interest: Interest shall be deemed to accrue/ arise in India if it is payable by:

Payable by

Government A resident in India A non-resident in India


of India
If money borrowed and used for
Exception the purpose of business or
profession carried on in India
If money borrowed and used for the
purpose of business or profession
carried outside India

If money borrowed and used for


making income from any source
outside India

Illustration: 13 [MAY 2006]

Mr. Ram, left for USA on 01.05.2023. He has not visited India thereafter. Mr. Ram
borrows money from his friend Mr. Laxman, who left India one week before Mr. Ram's
departure, to the extent of ₹10 lakhs and buys shares in Ram Ltd., an Indian company.
Discuss the taxability of the interest charged @ 10% in Mr. Laxman's hands where the
same has been received in New York.

Solution:

Stay of Mr. Ram and Mr. Laxman during the previous year 2023-24 is less than 60 days
hence both of them are non-residents as per section 6(1).

As per section 9, if any non-resident has taken loan from outside India and the loan was
utilized in India in any source other than business or profession, interest received by

53 | P a g e #HakkसेCA by CA. Akash Sir


54
2. Residential Status and Scope of Total Income

the person who has given the loan shall not be considered to be accruing/arising in
India and is not taxable in India. In the given case, loan amount was invested in the
shares of an Indian company hence interest received by Mr. Laxman shall not be
considered to be income accruing/arising in India.

TP: 21 Royalty [Section 9(1)(vi)]

Royalty: Royalty shall be deemed to accrue/ arise in India if it is payable by:

Payable by

Government of A resident in India A non-resident in India


India
If royalty services are utilised for
Exception the purpose of business or
profession carried on in India
If royalty services are utilised for
the purpose of business or
profession carried outside India If royalty services are utilised for
making income from any source in
If royalty services are utilised for India
making income from any source
outside India

Note:

1. Consideration for use or right to use of computer software is Royalty.


2. Consideration in respect of any right, property or information is Royalty whether or
not possession or control of such right, property or information is with payer etc.

54 | P a g e #HakkसेCA by CA. Akash Sir


55
2. Residential Status and Scope of Total Income

TP: 22 Fees for technical services [Section [9(1)(vii)]

Fees for technical services: Fees for technical services shall be deemed to accrue/
arise in India if it is payable by:

Payable by

Government of A resident in India A non-resident in India


India
Exception If fees for technical services
services are utilised for the
purpose of business or
If fees for technical services profession carried on in India
services are utilised for the
purpose of business or
profession carried outside India If fees for technical services
services are utilised for making
income from any source in India
If fees for technical services services
are utilised for making income from
any source outside India

Illustration: 14 [MAY 2011]

Miss Vivitha paid a sum of $5000 to Mr. Kulasekhara, a management consultant


practicing in Colombo, specializing in project financing. The payment was made in
Colombo. Mr. Kulasekhara is a non-resident. The consultancy related to a project in
India with possible Ceylonese collaboration. Is this payment chargeable to tax in India
in the hands of Mr. Kulasekhara, since the services were used in India?

Solution:

As per section 9, if any non-resident has provided any patent right or any managerial,
technical services and such patent right etc was used in India, in such cases any royalty
or fee received by non-resident shall be considered to be income accruing/arising in
India and shall be taxable and it do not matter that the non-resident do not have
residence or place of business or business connection in India i.e. there is no territorial
nexus or non-resident has not rendered services in India. In the instant case, since the
services were utilized in India, the payment received by Mr. Kulasekhara, a non-
resident, in Colombo is chargeable to tax in his hands in India, as it is deemed to accrue
or arise in India.

55 | P a g e #HakkसेCA by CA. Akash Sir


56
2. Residential Status and Scope of Total Income

Illustration: 15 [NOV – 2020 EXAM (NEW COURSE)]

Mr. Mark, a non-resident and citizen of Japan entered into following transactions
during the previous year ended 31.03.2024. Examine the tax implications in the hands
of Mr. Mark for the Assessment Year 2024 - 25 as per Income Tax Act, 1961. (Give
brief reasoning)
(1) Interest received from Mr. John, a non-resident, outside India (The borrowed fund
is used by Mr. John for investing in Indian company's debt fund for earning interest)
(2) Received ₹10 lakhs in Japan from a business enterprise in India for granting license
for computer software (not hardware specific).
(3) He is also engaged in the business of running news agency and earned income of ₹10
lakhs from collection of news and views in India for transmission outside India.
(4) He entered into an agreement with SKK & Co., a partnership firm for transfer of
technical documents and design and for providing services relating thereto, to set up a
Denim Jeans manufacturing plant, in Surat (India). He charged ₹10 lakhs for these
services from SKK & Co.

Solution:

(1) As per section 9, if loan has been taken by a non-resident, interest income shall be
accruing/ arising in India only if loan amount has been utilised in India in business/
profession but if loan amount is utilised in any other source in India or it has been used
outside India, interest income shall be accruing / arising abroad. In the given case, loan
amount is used for investing in Indian company debt fund for earning interest and not
for business purpose hence interest income shall not be considered to be accruing
arising from India and shall not be taxable in India.

(2) As per section 9, if any income is accruing and arising in India relating to royalty or
technical fees etc., it will be taxable in India even if the person receiving income is non-
resident and even if such non-resident does not have any Territorial Nexus with India
i.e., such non-resident does not have a residence or place of business or business
connection in India and also the non-resident has not rendered services in India. In the
given case, income received for granting licence for computer software shall be deemed
to be income accruing arising in India and shall be taxable in India.

(3) As per section 9, if any non-resident has the business of running a news agency or
of publishing newspapers, magazines or journals etc. outside India, no income shall be
deemed to accrue or arise in India to him from activities which are confined to the
collection of news and views in India for transmission out of India. In the given case,
income is from transmission outside India hence income shall not be deemed to accrue
arise in India and shall not be taxable in India.

56 | P a g e #HakkसेCA by CA. Akash Sir


57
2. Residential Status and Scope of Total Income

(4) As per section 9, income by way of fees for technical services payable by a person
who is a non-resident, where the fees are payable in respect of services utilised in a
business or profession carried on by such person in India or for the purposes of making
or earning any income from any source in India. In the given case, services utilized in a
business in India hence income shall be accruing arising from India and same shall be
taxable in India.

HW Question: 13 [NOV 2009]

Determine the taxability of income of US based company Mr. Been Ltd., in India on
entering following transactions during the financial year 2023-24:
(i) ₹5 lacs received from an Indian domestic company for providing technical know-how
in India.
(ii) ₹6 lacs from an Indian firm for conducting the feasibility study for the new project
in Finland.
(iii) ₹4 lacs from a non-resident for use of patent for a business in India.
(iv) ₹8 lacs from a non-resident Indian for use of know-how for a business in Singapore.
(v) ₹10 lacs for supply of manuals and designs for the business to be established in
Singapore.
Explain the rate of tax applicable on taxable income for US based company, Mr. Been
Ltd., in India.

TP: 23 Gift to NR [Section 9(1)(viii)]

Any sum of money paid by a resident Indian to a non-corporate non-resident


or foreign company (on or after 05.07.2019) or NOR (on or after 01.04.2023):
Any sum of money paid without consideration, by an Indian resident person to a non-
resident or foreign company or NOR would be deemed to accrue or arise in India if
the same is chargeable to tax under section 56(2)(x) i.e., if the aggregate of such
sum received by a non-corporate non-resident or foreign company exceeds ₹50,000.

Note: Above provision is not applicable in respect of property transferred outside


India without consideration or for inadequate consideration.

57 | P a g e #HakkसेCA by CA. Akash Sir


58
2. Residential Status and Scope of Total Income

TP: 24 What is meaning of “Income received in India”?

The receipt of income refers to only the first occasion when the recipient gets the
money under his control.
For example, Mr. Mohan is working in a UK based company. He received his salary
directly in his Indian bank account. In this case, income shall be considered as
received in India.

When once an amount is received as income, remittance or transmission of that


amount from one place or person to another does not constitute receipt of income in
the hands of the subsequent recipient or at the place of subsequent receipt. For
example, Mr. Mohan in above example has received his salary in UK Bank account and
then remit such money in Indian bank account. In this case, income shall not be
considered as received in India.

58 | P a g e #HakkसेCA by CA. Akash Sir


59
2. Residential Status and Scope of Total Income

TP: 25 What is meaning of “Income deemed to be received in


India”? [Section 7]

59 | P a g e #HakkसेCA by CA. Akash Sir


60
2. Residential Status and Scope of Total Income

Comprehensive question

Question: 1 [Residential Status]

Brett Lee, an Australian cricket player visits India for 100 days in every financial year.
This has been his practice for the past 10 financial years.
(a) Find out his residential status for the assessment year 2024-25.
(b) Would your answer change if the above facts relate to Srinath, an Indian citizen
who resides in Australia and represents the Australian cricket team?
(c) What would be your answer if Srinath had visited India for 120 days instead of 100
days every year, including P.Y. 2023-24?

Question: 2 [Residential Status] [May 1998]

Mr. Madhav, an American citizen, is appointed by a multi-national company to its branch


in Mumbai in 2020. Mr. Madhav has never been to India before this appointment. He
arrives in Delhi on 15th April, 2020 and joins the Mumbai office on 20th April, 2020.
His wife and children join him in India on 20th October, 2020. The company allotted
him a leased residence for purposes of his stay. This residence is occupied by him from
the beginning of October, 2020.
On 10th February, 2021, he is transferred by his employer, on deputation basis, to be
the Regional Chief of his employer’s operations in South East Asia having headquarters
in Hongkong. He leaves Mumbai on 11th February and arrives in Hongkong on 12th
February, 2021. Mr. Madhav leaves behind his wife and children in India till 14th
August, 2022, when they leave along with him for Hongkong. Mr. Madhav had come to
India earlier on 15th June, 2022, on two months’ leave. The members of the family
occupied the residence till date of departure to Hongkong.
At the end of the period of deputation, Mr. Madhav is reposted to India and joins the
Mumbai office of his employer as Chief of Indian operations on 2nd February, 2024.
In what residential status Mr. Madhav will be assessable, for the various years, to
income tax in India?

Question: 3 [Scope of total income]

Mr. Usman has income as under:


1. He has income from a business in Germany amounting to ₹3,00,000 and half of it was
received in India.
2. He has interest income of ₹1,00,000 from UK Development Bond and entire interest
income was credited to a bank account in UK. Subsequently, the amount was
transferred in India.
3. He has a business in Bombay and entire income of ₹3,00,000 was received in UK.

60 | P a g e #HakkसेCA by CA. Akash Sir


61
2. Residential Status and Scope of Total Income

4. He has one house property in Ghaziabad and income of ₹5,00,000 was received in UK.
5. He has received salary income of ₹5,00,000 (computed) in India and half of the
services were rendered in UK and half in India.
(Presume all the above incomes are computed incomes)
Compute his income presuming that he is NOR, NR and ROR.

Question: 4 [Scope of total income]

Mr. Amir had following income during the previous year ended 31st March, 2024:
(1) Salary received in India for three months (being computed income) 25,000
(2) Income from house property in India 18,000
(3) Interest on savings bank deposit in SBI, in India 4,000
(4) Amount brought into India out of the past-untaxed profits earned
in Germany 20,500
(5) Income from business in Bangladesh, being controlled from India 12,542
You are required to compute his gross total income for the assessment year 2024-25,
if he is a
(a) resident and ordinarily resident;
(b) not ordinarily resident; and
(c) non-resident.
Presume all the above income is computed income.

Question: 5 [Scope of total income]

Mr. Rahman earns the following income during the financial year 2023-24:
(1) Income from house property in London, received in India 60,000
(2) Profits from business in Japan and managed from there
(received in Japan) 9,00,000
(3) Profits from business in Kenya, controlled from India, Profits
received in Kenya 3,00,000
(4) Profits from business in Delhi, managed from Japan 7,00,000
(5) Capital gains on transfer of shares of Indian companies, sold in
USA and gains were received there 2,00,000
(6) Pension from former employer in India, received in Japan 50,000
(7) Profits from business in Pakistan, deposited in bank there 20,000
(8) Profit on sale of asset in India but received in London 8,000
(9) Past untaxed profits of UK business of 2020-21 brought into India
in 2023-24 90,000
(10) Interest on Government securities accrued in India but received in
Paris 80,000
(11) Interest on USA Government securities, received in India 20,000
(12) Salary earned in Bombay, but received in UK 60,000

61 | P a g e #HakkसेCA by CA. Akash Sir


62
2. Residential Status and Scope of Total Income

(13) Income from property in Paris, received there 1,00,000


(Presume all the above incomes are computed incomes)
Determine the gross total income of Mr. Rahman if he is (i) resident and ordinarily
resident, resident but not ordinarily resident, non-resident in India during the financial
year 2022-23.

Question: 6 [Scope of total income]

Ramrahim Pvt. Ltd., an Indian company has an income of ₹30 lakhs from a business in
India. This company has a business income of ₹12 lakhs from outside India. Out of
which 7 lakhs were received in India and balance outside India.
Compute total income of the Indian company for the assessment year 2024-25.

Question: 7 [Residential status and scope of total income]

Mr. John, a foreign citizen (not being a person of Indian origin) came to India for the
first time on 2nd December, 2023 for a visit of 210 days. Mr. John had the following
income during the previous year ended 31st March, 2024:
(1) Salary (computed) received in India for three months 1,00,000
(2) Income from house property in London (received there) 2,75,200
(3) Amount brought into India out of the past-untaxed profits earned
in Germany 80,000
(4) Income from agriculture in Sri Lanka, received and invested there 12,300
(5) Income from business in Nepal, being controlled from India 35,000
(6) Income from house property in USA received in USA (₹76,000 is used in
Canada for meeting the educational expenses of Mr. John’s daughter and
₹10,000 is later on remitted in India) 86,000
You are required to compute his total income for the assessment year 2024-25.

Question: 8 [Scope of total income]

Mr. Rohan earns the following incomes during the financial year 2023-24.
(1) Profits from a business in Japan, controlled from India,
(half of the profits received in India) 40,000
(2) Income from property in Bombay, received in UK 70,000
(3) Income from a property in USA, received there but subsequently
remitted to India 2,00,000
(4) Income from property in USA, received there (₹50,000 remitted
in India) 80,000
(5) Salary received in India for services rendered in USA 50,000
(6) Income from profession in Paris, which was set up in India, received
in Paris 80,000

62 | P a g e #HakkसेCA by CA. Akash Sir


63
2. Residential Status and Scope of Total Income

(7) Interest from deposit with an Indian company, received in Japan 9,000
(8) Income from profession in Bombay received in Paris 30,000
(9) Profits of business in Iran, deposited in a bank there, business controlled
from India (out of ₹4,00,000, ₹1,00,000 is remitted in India) 4,00,000
(10) Interest on German development bonds, half of which is received in
India 10,000
(11) Income from property in Canada, one-fifth is received in India 50,000
(Presume all the above incomes are computed income i.e. all the exemptions and
deductions have already been allowed)
Determine the gross total income of Mr. Rohan if he is (i) resident and ordinarily
resident, (ii) resident but not ordinarily resident, (iii) non-resident in India during the
financial year 2023-24.

Question: 9 [Residential status, scope of total income, salary, gift, Agri Income]

Mr. Mohan is a citizen of India and is employed in Sarla Limited and getting salary
₹1,00,000 p.m. and he was transferred out of India on 01.09.2023 and he left India for
first time from 01.09.2023 and he visited India from 26.01.2024 to 15.02.2024 and
salary for January 2024 was received in India and at the time of departure he received
3 gifts ₹20,000 from 3 friends each and also a phone of ₹70,000. He has agricultural
income in India ₹4,00,000.
Compute his tax liability for assessment year 2024-25.

Question: 10 [Residential status, scope of total income, salary and exemption]

Mrs. Mehta is a citizen of India and is employed in Raj Ltd. in India and is getting
salary of ₹60,000 p.m. and she was transferred out of India w.e.f. 01.09.2023 and for
this purpose she left India on 01.09.2023 for the first time and she visited India from
27.12.2023 to 07.01.2024 and her salary for the month of Dec’ 2023 was received in
India. Employer and employee both have contributed @ 13% (each) of salary to the
recognized provident fund and during the year interest of ₹50,000 was credited to the
recognized provident fund @ 10% p.a.
Compute her total income and tax liability in India for assessment year 2024-25.
(b) Presume she was transferred w.e.f. 01.11.2023 and she left India on 01.11.2023 for
the first time.

Question: 11 [Scope of total income]

Determine the scope of the following incomes in the hands of a resident and ordinarily
resident, resident but not ordinarily resident, and non-resident for the A. Y. 2024-25 –

63 | P a g e #HakkसेCA by CA. Akash Sir


64
2. Residential Status and Scope of Total Income

Particulars Amount (₹)


(1) Interest on UK Development Bonds, 50% of interest received in 10,000
India
(2) Income from a business in Chennai (50% is received in India) 20,000
(3) Profits on sale of shares of an Indian company received in London 20,000
(4) Profits on sale of plant at Germany 50% of profits are received in
India 40,000
(5) Income earned from business in Germany which is controlled from
Delhi (₹40,000 is received in India) 70,000
(6) Profits from a business in Delhi but managed entirely from London 15,000
(7) Income from property in London deposited in an Indian Bank at
London, brought to India 50,000
(8) Interest for debentures in an Indian company received in London. 12,000
(9) Fees for technical services rendered in India but received in London
(10) Profits from a business in Bombay managed from London 8,000
(11) Pension for services rendered in India but received in Burma 26,000
(12) Income from property situated in Pakistan received there 4,000
(13) Past foreign untaxed income brought to India during the previous 16,000
year
(14) Income from agricultural land in Nepal received there and then 5,000
brought to India
(15) Income from profession in Kenya which was set up in India, 18,000
received there but spent in India
(16) Gift received on the occasion of his wedding 5,000
(17) Interest on savings bank deposit in State Bank of India 20,000
(18) Income from a business in Russia, controlled from Russia 10,000
(19) Agricultural income from a land in Rajasthan 20,000
15,000

Question: 12 [Scope of total income]

Calculate taxable income of an individual on the basis of the following information, for
the assessment year 2024-25, if he is:
(a) Ordinarily Resident
(b) Not Ordinarily Resident; and
(c) Non-Resident

(i) Profit from business in Japan received in India. 10,000


(ii) Income from agriculture in Pakistan – it is all spent on the education
of children there 5,000
(iii) Income accrued in India but received in England 10,000
(iv) Income from house property in Pakistan deposited in a bank there 2,000
(v) Profits of business in America deposited in a bank there. This business
is controlled from India 50,000

64 | P a g e #HakkसेCA by CA. Akash Sir


65
2. Residential Status and Scope of Total Income

(vi) Profits earned from business in Meerut 12,000


(vii) Past untaxed foreign income brought into India during the previous
year 10,000
(Presume that all the incomes are computed incomes)

Question: 13 [scope of total income]

Mr. Ram earns the following income during the previous year 2023-24.
Compute his gross total income for assessment year 2024-25 if he is
(i) resident and ordinarily resident.
(ii) resident but not ordinarily resident.
(iii) non-resident.
(1) Income from agricultural land in Bhutan received there and
remitted to India later on 40,000
(2) Pension for service rendered in India, but received in Paris 15,000
(3) Past untaxed profits of 2022-23 brought into India in 2023-24 50,000
(4) Profits from business in Paris, deposited in bank there 1,00,000
(5) Profits from business in Canada, controlled from India, profits received
there 1,75,000
(6) Interest on saving bank deposit in Punjab National Bank, in India 20,000
(7) Capital gain on sale of a house in Delhi, amount received in Paris 2,00,000

Question: 14 [Scope of total income]

Mr. Rahim earns the following income during the previous year 2023-24. Compute his
Gross total income for assessment year 2024-25 if he is
(i) resident and ordinarily resident.
(ii) resident but not ordinarily resident.
(iii) non-resident.
(1) Profit on sale of machinery in India, but received in Japan 1,20,000
(2) Profits from business in Bombay, managed from Japan 2,25,000
(3) Profits from business in Japan, managed from there, received there 1,45,000
(4) Income from house property in India 1,50,000
(5) Income from property in Japan and received there 1,50,000
(6) Income from agriculture in Japan being invested there 75,000
(7) Fees for technical services rendered in India but received in Japan 65,000
(8) Interest on Government securities accrued in India but received in
Japan 80,000
(9) Interest on Japan Government securities, received in India 40,000
(Presume that all the incomes are computed incomes)

65 | P a g e #HakkसेCA by CA. Akash Sir


66
2. Residential Status and Scope of Total Income

Question: 15 [Scope of total income]

Mr. Soham earns the following incomes during the financial year 2023-24.
(1) Profits from a business in Japan, controlled from India, half of the
profits received in India 60,000
(2) Income from agriculture in Nepal, brought to India 10,000
(3) Income u/h house property in Bombay, received in UK 1,70,000
(4) Income u/h house property in USA, received there but subsequently
remitted to India 2,20,000
(5) Income u/h house property in USA, received there (₹50,000 remitted
in India) 1,00,000
(6) Salary received in India for services rendered in USA 60,000
(7) Income from profession in Paris, which was set up in India, received in
Paris 90,000
(8) Interest from deposit with an Indian company, received in Japan 19,000
(9) Income from profession in Bombay received in Paris 39,000
(10) Profits of business in Iran, deposited in a bank there, business
controlled from India (out of ₹4,80,000, ₹1,00,000 is remitted in India) 4,80,000
(11) Interest on German development bonds, half of which is received in
India 12,000
(12) Income under the head house property in Canada, one-fifth is received
in India 50,000
(Presume all the above incomes are computed income i.e. all the exemptions and
deductions have already been allowed)
Determine the gross total income of Mr. Soham if he is
(i) resident and ordinarily resident,
(ii) resident but not ordinarily resident,
(iii) non-resident in India during the financial year 2023-24.

Question: 16 [Residential status, Scope of total income, Salary, Other sources,


Gift, Capital Gain]

Mr. Mohan is a citizen of India and is employed in Lenskart Ltd and is getting a salary
of ₹60,000 p.m. He purchased one building in India on 1st May, 2023 for ₹10,00,000
and its market value is ₹22,00,000 and value for the purpose of charging stamp duty is
₹13,00,000. He purchased gold for ₹8,00,000 and its market value is ₹11,00,000. He
was transferred out of India w.e.f. 1st Sept, 2023 and he left India on 1st Sept, 2023.
One of his friends gifted him one colour TV on this occasion, market value ₹1,00,000.
He has gone out of India in earlier years also.
P.Y. 2022-23 100 days
P.Y. 2021-22 200 days

66 | P a g e #HakkसेCA by CA. Akash Sir


67
2. Residential Status and Scope of Total Income

He visited India from 01.02.2024 to 14.02.2024 and salary for January, 2024 was
received in India.
He has purchased one house property in USA in December 2023 and sold in March
2024 and there was short term capital gain of ₹6,00,000 and the amount was received
in USA. Compute his tax liability for the A.Y. 2024-25 under normal provisions of the
Act.

Question: 17 [Residential status, scope of total income, Gift]

Mrs. X is employed in ABC Ltd in India and she is an American citizen and is getting a
salary of ₹2,00,000 p.m.
She received gift of one painting in India from her friend on 01.07.2023 and its market
value is ₹49,000 and she also received gift in cash of ₹49,000 from the same friend
and gift of immovable property with value for the purpose of charging stamp duty is
₹51,000 from the same friend.
She purchased UK Development bond and interest equivalent of ₹2,00,000 was
received in USA.
She visited USA for 182 days during P.Y. 2023-24.
In the earlier year her stay in India was:
P.Y. 2022-23 110 days
P.Y. 2021-22 120 days
P.Y. 2020-21 300 days
P.Y. 2019-20 182 days
P.Y. 2018-19 185 days
P.Y. 2017-18 200 days
P.Y. 2016-17 300 days
Compute her tax liability in India for the A.Y. 2024-25 under normal provisions of the
Act.

Question: 18 [Scope of total income] [May 2019 (NEW COURSE)]

The following are the income of Shri Subhash Chandra, a citizen of India for the
previous year 2023-24:
(i) Income from business in India ₹2,00,000. The business is controlled from London
and ₹60,000 were remitted to London.
(ii) Profits from business earned in Japan ₹70,000 of which ₹20,000 were received in
India. This business is controlled from India.
(iii) Untaxed income of ₹1,30,000 for the year 2020-21 of a business in England which
was brought in India on 3rd March, 2024.
(iv) Royalty of ₹4,00,000 received from Shri Ramesh, a resident for technical service
provided to run a business outside India.
(v) Agricultural income ₹90,000 in Bhutan.

67 | P a g e #HakkसेCA by CA. Akash Sir


68
2. Residential Status and Scope of Total Income

(vi) Income of ₹73,000 from house property in Dubai, which was deposited in bank at
Dubai. Compute Gross total income of Shri Subhash Chandra for the A.Y. 2024-25, if
he is –
(1) A resident and Ordinarily Resident, and
(2) A resident and Not Ordinarily Resident

Question: 19 [Scope of total income] [May 2019 (OLD COURSE)]

Mr. Bachhan has provided the following details of his income for the year ended
31.03.2024.
(1) Short term capital gains on sale of shares in Indian company
received in Japan. 85,000
(2) Rent from property in Bangladesh deposited in a bank at Dhaka,
later on, remitted to India through approved banking channels. 96,000
Compute his total income for the Assessment Year 2024-25 in case of he is:
(i) Resident and ordinarily resident;
(ii) Resident but not ordinarily resident; or
(iii) Non-resident

Question: 20 [Residential status, Scope of total income] [Nov 2018 (Old Course)]

Mr. Surya, an Indian citizen, travelled frequently out of India for his business trip as
well as for his outings. He left India from Mumbai airport on 15th May 2023 as
stamped in the passport. He has been in India for less than 365 days during the 4
years immediately preceding the previous year and has not been in India for at least 60
days in the previous year.
Determine:
(A) Residential status of Mr. Surya and
(B) Total income for the assessment year 2024-25 from the following information:
(1) Short term capital gain on the sale of shares of Trena India Ltd. a listed Indian
company amounting to ₹35,000. The sale proceeds were credited to his Swiss bank
account.
(2) Interest on fixed deposit with State Bank of India (Mumbai) amounting to
₹8,000 was credited to his saving account.

Question: 21 [Scope of total income] [May 2018 (NEW COURSE)]

Compute the Gross Total Income in the hands of an individual, if he is


(a) a resident and ordinary resident; and
(b) a non – resident
for the A.Y. 2024-25

68 | P a g e #HakkसेCA by CA. Akash Sir


69
2. Residential Status and Scope of Total Income

S. No. Particulars Amount


(i) Interest from German Derivatives Bonds (1/3 received in 21,000
India)
(ii) Income from agriculture land situated in Malaysia, remitted 51,000
to India
(iii) Income earned from business in Dubai, controlled from 75,000
India (₹20,000 received in India)
(iv) Profit from business in Mumbai, controlled from Australia 1,75,000
(v) Interest received from Mr. Ashok (NRI) on loan provided 35,000
to him for business in India
(vi) Profit from business in Canada controlled from Mumbai 60,000
(60% of profits deposited in a bank in Canada and 40%
remitted to India)
(vii) Amount received from an NRI for the use of know-how for 8,00,000
his business in Singapore
(viii) Past years untaxed foreign income brought to India 50,000

Question: 22 [Residential status, Scope of total income] [NOV 2014]

Mrs. X and Mrs. Y are sisters and they earned the following income during the Financial
Year 2023-24. Mrs. X is settled in Malaysia since 2018 and visits India for a month
every year. Mrs. Y is settled in Indore since her marriage in 2018. Compute the total
income of Mrs. X and Mrs. Y for the assessment year 2024-25:

S. No. Particulars Mrs. X Mrs. Y


(i) Income from Profession in Malaysia, (set up in 15,000 -
India) received there
(ii) Profit from business in Delhi, but managed 40,000 -
directly from Malaysia
(iii) Rent (computed) from property in Malaysia 1,20,000 -
deposited in a Bank at Malaysia, later on
remitted to India through approved banking
channels.
(iv) Cash gift received from a friend on Mrs. Y’s - 51,000
50th birthday
(v) Agricultural income from land in Maharashtra 7,500 4,000
(vi) Past foreign untaxed income brought to India 5,000 -
(vii) Fees for technical services rendered in India 25,000 -
received in Malaysia
(viii) Income from a business in Pune (Mrs. X 12,000 15,000
receives 50% of the income in India)
(ix) Interest on debentures in an Indian company 18,500 14,000
(Mrs. X received the same in Malaysia)

69 | P a g e #HakkसेCA by CA. Akash Sir


70
2. Residential Status and Scope of Total Income

(x) Short-term capital gain on sale of shares of an 15,000 25,500


Indian company
(xi) Interest on Fixed Deposit with SBI in India 12,000 8,000

Question: 23 [Residential status, Scope of total income] [RTP Nov 2022]

Mr. Dhanush, an Indian citizen aged 35 years, worked in ABC Ltd. in Mumbai. He got a
job offer from XYZ Inc., USA on 01.06.2022. He left India for the first time on
31.07.2022 and joined XYZ Inc. on 08.08.2022. During the P.Y. 2023-24, Mr. Dhanush
visited India from 25.05.2023 to 22.09.2023. He has received the following income for
the previous year 2023-24 –
Particulars Amount
Salary from XYZ Inc., USA received in USA 7,00,000
Dividend from Indian companies 5,50,000
Agricultural income from land situated in Punjab 55,000
Rent received/receivable from house property in Lucknow 4,00,000
Profits from a profession in USA, which was set up in India,
received there 6,00,000
Determine the residential status of Mr. Dhanush and compute his total income for the
A.Y. 2024-25.

70 | P a g e #HakkसेCA by CA. Akash Sir


71
3. Income from house property

Income from House Property [Section 22


to 27]
TP: 1 Charging section [section 22]

The annual value of any property comprising of buildings or lands appurtenant


thereto of which the assessee is the owner is chargeable to tax under the head
“Income from house property”.
Exception:
i) House property used by assessee in his own business. (Depreciation allowed
under head PGBP)
ii) Assessee engaged in the business of renting out of house property.
(Income chargeable to tax under head PGBP)

Mann Ki Baat (simple language):

Where an assessee derives any income from renting out of house property being
buildings or land appurtenant thereto, such income (net annual value) would be
changeable to tax under head house property provided that the assessee is the owner
of house property.

Now, to understand the concept better, we should understand the meaning of “Buildings
or land appurtenant thereto” & “Owner”.

TP: 2 Meaning of Building or land appurtenant thereto

(a) Buildings include not only residential buildings, but also factory buildings,
offices, shops, godowns and other commercial premises.
(b) Land appurtenant means land connected with the building like garden, garage etc.

Mann Ki Baat (simple language):

Form above we understand that buildings not only include residential buildings but also
includes commercial buildings. For example, Mr. Rohan rents out shop for ₹10,000 p.m.,
then such rental income would be chargeable to tax under head house property.

TP: 3 Meaning of Owner

(a) Owner is the person who is entitled to receive income from the property in his
own right.

71 | P a g e #HakkसेCA by CA. Akash Sir


72
3. Income from house property

(b) The requirement of registration of the sale deed is not warranted.


(c) Ownership includes both free-hold and lease-hold rights.
(d) Ownership includes deemed ownership.
(e) The person who owns the building need not also be the owner of the land upon
which it stands.
(f) The assessee must be the owner of the house property during the previous
year. It is not material whether he is the owner in the assessment year.
(g) If the title of the ownership of the property is under dispute in a court of
law, the decision as to who will be the owner chargeable to income tax under
section 22 will be of the Income-tax department till the court gives its
decision to the suit filed in respect of such property.

Mann Ki Baat (simple language):

From above we can see that owner does not only include the person who is legal owner
of the property but also includes the person who enjoys the right in the property. DO
NOT LEARN FOR EXAMS.

TP: 4 Determination of Net Annual Value (NAV) [Section 23]

Net annual value [NAV] = Gross annual value [GAV] (-) Municipal tax paid

For example: Mr. Vijendra has let-out house property @15,000 p.m. for 12 months
and has made payment of municipal taxes of ₹30,000, then, the NAV would be
₹1,50,000.

Mann Ki Baat (simple language):

There are various cases wherein the method of determination of Gross annual value
differs. We are going to study those cases later in this chapter (TP 7).

TP: 5 Property taxes (Municipal taxes)

Property taxes (a.k.a. Municipal tax, House tax, Land revenue) are allowable as
deduction from the GAV provided that it should be borne by the assessee (owner);
and it should be actually paid during the previous year.

Note: In case of property situated outside India, taxes levied by local authority of
the country in which the property is situated is deductible.

72 | P a g e #HakkसेCA by CA. Akash Sir


73
3. Income from house property

Mann Ki Baat (simple language):

Deduction of municipal tax is only allowed if payment is made by the owner himself. If
property taxes for a particular previous year are not paid during that year, no
deduction shall be allowed from GAV for that year.

However, if in any subsequent year, the arrears are paid, then, the amount so paid is
allowed as deduction in computation of income from house property for that subsequent
year.

For example:

Case Gross annual Municipal Remarks Net annual


value tax value
(i) 10,00,000 2,00,000 MT relates to P.Y. 2023-24 which 8,00,000
has been paid during the year
(ii) 12,00,000 4,00,000 MT relates to P.Y. 2023-24 which 12,00,000
has not been paid during the year
(iii) 15,00,000 3,00,000 MT relates to P.Y. 2023-24. Out 14,00,000
of total tax, tenant has paid tax of
₹2,00,000 and balance ₹1,00,000
is paid by owner
(iv) 8,00,000 4,00,000 MT relates to P.Y. 2022-23. It has 4,00,000
been paid during the P.Y. 2023-24
(v) 13,00,000 3,00,000 MT paid outside India for the 10,00,000
property situated in America

Illustration: 1 [ICAI Mat]

Mr. John, a British national, is a resident and ordinarily resident (ROR) in India during
the P.Y. 2023-24. He owns a house in London, which he has let out at £10,000 p.m. The
municipal taxes paid to the Municipal Corporation of London is £8,000 during the P.Y.
2023-24. The value of one £ in Indian rupee to be taken at ₹95. Compute John’s Net
annual value of the property for the A.Y. 2024-25.

Solution:

For the P.Y. 2023-24, Mr. John, a British national, is resident and ordinarily resident in
India. Therefore, as per section 5(1) of the Act, income received by him by way of rent
of the house property located in London is to be included in the total income in India.

Computation of Net Annual Value of the property of Mr. John for A.Y. 2024-25

Gross Annual Value (£ 10,000 × 12 × 95) 1,14,00,000


73 | P a g e #HakkसेCA by CA. Akash Sir
74
3. Income from house property

Less: Municipal taxes paid (£ 8,000 × 95) 7,60,000


Net Annual Value (NAV) 1,06,40,000

Note: Municipal taxes paid in London is be to allowed as deduction from the gross
annual value.

TP: 6 Deductions from net annual value [Section 24]

There are two deductions allowed from net annual value. These are –
i) Standard deduction of 30% of NAV u/s. 24(a); and
ii) Interest on borrowed capital u/s. 24(b)

TP: 6A Statutory Deduction or Standard Deduction [Section


24(a)]

Under section 24(a) of the Act, every assessee shall be allowed a notional
expenditure equals to 30% of the net annual value of the house property in lieu of
various expenditures incurred by him. Actual expenditure incurred by the assessee
shall not be taken into consideration.

Deduction in respect of self-occupied or unoccupied property shall be NIL.

Mann Ki Baat (simple language):

From above, we understand that actual expenditure like electricity, water charges,
maintenance expense etc. shall not be allowed as deduction. However, a notional
expense of 30% of NAV shall be allowed as deduction. For example: Net annual value
of one house is ₹3,00,000 and actual expenditure incurred on repairs are ₹75,000,
deduction allowed under section 24(a) shall be ₹90,000 i.e. 30% of NAV. Ignore actual
expenditure.

TP: 6B Interest on borrowed capital [Section 24(b)]

If any assessee has taken a loan or advance for purchase/ construction/ renovation/
addition/ alteration/ substitution or repair etc. of the house property, interest on
such loan shall be allowed to be deducted under section 24(b) from NAV. Such
interest is allowed on due basis i.e. actual payment of interest is not mandatory for
claiming deduction. Only simple interest is allowed i.e. interest on interest is not
allowed. The assessee can take any number of loans.

Note: If loan is taken from outside India, then as per section 25, interest is allowed
but the person making payment of interest should deduct tax at source or the person
receiving interest should have an agent in India.

74 | P a g e #HakkसेCA by CA. Akash Sir


75
3. Income from house property

Mann Ki Baat (simple language):

Let’s simplify the above concept. If a person has taken any loan for house property and
has some income on let-out of house property, then it is logical to allow deduction of
interest on loan from such income.

Now, let’s assume that Mr. Akash has taken loan on 01.04.2020 for construction of
house property and made payment of interest during P.Y. 2020-21, P.Y. 2021-22 and
P.Y. 2022-23 to the bank. During P.Y. 2023-24, construction was completed and house
was let-out. Now, think that how can we take deduction of interest for P.Y. 20-21, 21-
22 and 22-23 since there was no income earned during those years? Let’s read concept
further.

Current period interest: Interest for the year for which income is being computed
shall be allowed in the same year and shall be called current period interest.

Prior period interest: Interest for the period prior to the year in which the house
was purchased or constructed shall be called prior period interest and such interest
shall be allowed in 5 annual equal instalments starting from the year in which the
house was purchased or constructed.

Now, to understand the concept of prior period, let’s take some examples.

Start of End of Prior period 1st Current period


construction construction
01.05.2020 01.04.2023 01.05.2020 to 31.03.2023 01.04.2023 to 31.03.2024
01.05.2020 01.05.2023 01.05.2020 to 31.03.2023 01.04.2023 to 31.03.2024
01.06.2020 01.12.2023 01.06.2020 to 31.03.2023 01.04.2023 to 31.03.2024
01.04.2020 31.03.2024 01.04.2020 to 31.03.2023 01.04.2023 to 31.03.2024
01.09.2018 31.10.2022 01.09.2018 to 31.03.2022 01.04.2022 to 31.03.2023

For example: If Mr. Bablu had taken a loan of ₹5,00,000 for construction of property
on 01.10.2022 and interest is payable @ 10% p.a. and the construction was completed on
30.06.2023, in this case interest allowed under section 24(b) shall be:
Interest for the year (01.04.2023 to 31.03.2024) = 10% of ₹5,00,000 = ₹50,000
Prior period interest = 10% of ₹5,00,000 for 6 months (from 01.10.2022 to 31.03.2023)
= ₹25,000

Illustration: 2

Mr. Sahil took a loan of ₹5,00,000 on 01.10.2020 @ 10% p.a. for construction of house
which was completed on 31.03.2024. Compute interest on capital borrowed allowable as
deduction for the previous year 2023-24.

75 | P a g e #HakkसेCA by CA. Akash Sir


76
3. Income from house property

Solution:

Computation of interest allowable under section 24(b)

Prior period interest (from 01.10.2020 to 31.03.2023)


5,00,000 x 10% x 30/12 = ₹1,25,000
Instalment = ₹1,25,000/5 = ₹25,000
Current year interest (from 01.04.2023 to 31.03.2024)
5,00,000 x 10% = ₹50,000
Total Interest = ₹25,000 + ₹50,000 = ₹75,000

HW Question: 1

Mr. Sahid has taken a loan of ₹10,00,000 from SBI on 01.04.2021 @ 12% p.a. for
construction of one house which was completed on 01.07.2023 and was let out at a rent
of ₹30,000 per month. He paid municipal taxes ₹40,000. He has taken loan of
₹10,00,000 from PNB on 01.10.2023 @ 10% p.a. to repay the original loan. Compute
interest allowable u/s. 24(b) for A.Y. 2024-25.

HW Question: 2

Mr. Amjad has taken a loan of ₹10,00,000 from SBI on 01.04.2020 @ 10% p.a. for
construction of one house. The Assessee has taken a loan of ₹6,00,000 from PNB on
01.10.2022 @ 12% p.a. to repay loan to SBI. House was completed on 01.07.2023.
Compute interest allowable u/s. 24(b) for A.Y. 2024-25.

Illustration: 3

Mr. Ayush purchased a house property on 01.05.2019 for ₹15,00,000. He took a loan of
₹12,00,000 @ 10% p.a. to purchase the same. Loan is to be repaid in 24 equal monthly
instalments starting from 01.04.2022. Calculate interest allowable u/s. 24(b) for A.Y.
2024-25.

Solution:

Computation of interest on capital borrowed u/s 24(b)

Current period Interest (from 01.04.2023 to 31.03.2024)


(5,50,000 x 10% x 1/12) + (5,00,000 x 10% x 1/12) + (4,50,000 x 10% x 1/12) +
(4,00,000 x 10% x 1/12) + (3,50,000 x 10% x 1/12) + (3,00,000 x 10% x 1/12) +
(2,50,000 x 10% x 1/12) + (2,00,000 x 10% x 1/12) + (1,50,000 x 10% x 1/12) +
(1,00,000 x 10% x 1/12) + (50,000 x 10% x 1/12) = ₹27,500
OR (5,50,000+0)/2 x 10% = ₹27,500
Total Interest = ₹27,500

76 | P a g e #HakkसेCA by CA. Akash Sir


77
3. Income from house property

HW Question: 3

Mr. Dinesh started the construction of the house on 01.04.2016 with the borrowed
capital of ₹22,00,000 @ 10% p.a. He completed the house on 31.03.2018. Repayment of
loan has been made in 11 annual equal instalments starting from 31.03.2019. Calculate
interest allowable u/s. 24(b) for A.Y. 2024-25.

Illustration: 4 [MAY – 2007 (6 Marks)] (Residential status, Scope of total income,


house property, IFOS)

Miss Charlie, an American national, got married to Mr. Radhey of India in USA on
02.03.2023 and came to India for the first time on 16.03.2023. She left for USA on
23.09.2023. She returned to India again on 27.03.2024. While in India, she had
purchased a show room in Mumbai on 22.04.2023, which was leased out to a company on
a rent of ₹25,000 p.m. from 01.05.2023. She had taken loan from a bank for purchase
of this show room on which bank had charged interest of ₹97,500 up to 31.03.2024.
Determine her residential status and compute the total income chargeable to tax along
with the amount of tax payable on such income for the Assessment Year 2024-25.

Solution:

As per section 6(1) of the Act, an individual is considered to be resident in India if he


stays in India for 182 days or more or he stays in India for 60 days or more during the
relevant previous year and also for 365 days or more during 4 years preceding the
relevant previous year.

Her stay in India during the previous year 2023-24 and in the preceding four years is
as under:
P.Y. 2023-24:
01.04.2023 to 23.09.2023 - 176 days and
27.03.2024 to 31.03.2024 - 5 days.
Total 181 days.
Four preceding previous years:
P.Y. 2022- 2023 [16.03.2023 to 31.03.2023] - 16 days
P.Y. 2021- 2022 [01.04.2021 to 31.03.2022] - Nil
P.Y. 2020- 2021 [01.04.2020 to 31.03.2021] - Nil
P.Y. 2019- 2020 [01.04.2019 to 31.03.2020] - Nil
Total 16 days

Since Miss Charlie is not able to comply with any of the conditions mentioned above,
she is non-resident during previous year 2023-24.

77 | P a g e #HakkसेCA by CA. Akash Sir


78
3. Income from house property

Computation of total income of Miss. Charlie for the A.Y. 2024-25

Income from house property


Gross Annual Value [25,000 x 11] 2,75,000
Less: Municipal taxes Nil
Net Annual Value 2,75,000
Less: Standard deduction 30% of NAV u/s. 24(a) (82,500)
Less: Interest on loan u/s. 24(b) (97,500)
Income under the head House Property 95,000

Total Income 95,000

Computation of tax payable by Miss. Charlie for the A.Y. 2024-25

Tax on total income of ₹95,000 @ slab rate Nil


Add: HEC @ 4% Nil
Tax Liability Nil

Notes:
1. Actual rent received has been taken as the gross annual value in the absence of other
information (i.e. Municipal value, fair rental value and standard rent) in the question.

TP: 7 ‘Computation of GAV’ and ‘Treatment of Interest’ in


various cases [IMPORTANT + +]

Case 1: GAV where the property is let out throughout the year [Section
23(1)(a)/(b)]

Where the property is let out for the whole year, then the GAV shall be calculated
as below:
(a) Fair Rent for the year;
(b) Municipal Value for the year;
(c) Higher of (a) or (b)
(d) Standard Rent for the year;
(e) Expected Rent = Lower of (c) or (d);
(f) Rent received/ receivable for the year
GAV shall be higher of (e) or (f)

Notes:
1. Municipal value is the value determined by the municipal authorities for levying
municipal taxes on house property.
2. Fair rent means rent for similar property in the same locality would fetch.
3. The standard rent (SR) is fixed by the Rent Control Act.

78 | P a g e #HakkसेCA by CA. Akash Sir


79
3. Income from house property

For example: Compute GAV of each house from the information given below:

Particulars House 1 House 2 House 3 House 4 House 5


Municipal value 80000 55000 65000 24000 80000
Fair rent 90000 60000 65000 25000 75000
Standard rent N.A. 75000 58000 N.A. 78000
Actual rent received/receivable 72000 72000 60000 30000 72000

Solution:

Particulars House 1 House 2 House 3 House 4 House 5


(a) Municipal value 80,000 55,000 65,000 24,000 80,000
(b) Fair rent 90000 60000 65000 25000 75000
(c) higher of (a) and 90,000 60000 65000 25000 80000
(b)
(d) standard rent N.A. 75000 58000 N.A. 78000
(e) Expected rent 90000 60000 58000 25000 78000
[lower of (c) and (d)]
(f) Actual rent 72000 72000 60000 30000 72000
received/receivable
GAV [higher of (e) 90000 72000 60000 30000 78000
and (f)]

Illustration: 5

Mr. Babu has one house property which is let out @ ₹80,000 p.m. Fair rent ₹90,000
p.m., Municipal Valuation ₹70,000 p.m., Standard Rent ₹81,000 p.m. Compute the GAV
of the house property.

Solution:

Computation of GAV for A.Y. 2024-25

(a) Fair Rent (90,000 x 12) 10,80,000


(b) Municipal Value (70,000 x 12) 8,40,000
(c) Higher of (a) or (b) 10,80,000
(d) Standard Rent (81,000 x 12) 9,72,000
(e) Expected Rent {Lower of c or d} 9,72,000
(f) Rent received /receivable (80,000 x 12) 9,60,000
GAV shall be higher of (e) or (f) 9,72,000

79 | P a g e #HakkसेCA by CA. Akash Sir


80
3. Income from house property

HW Question: 4

Mrs. Babu has let out one house property @ ₹62,000 p.m., municipal valuation ₹72,000
p.m., fair rent ₹90,000 p.m., standard rent ₹1,00,000 p.m. Compute GAV of the House
Property.

HW Question: 5 [ICAI Mat]

Mr. Jagmal owns five houses in Chennai, all of which are let-out. Compute the GAV of
each house from the information given below –

Particulars House I House II House III House IV House V


Municipal Value 80,000 55,000 65,000 24,000 75,000
Fair Rent 90,000 60,000 65,000 25,000 80,000
Standard Rent N.A. 75,000 58,000 N.A. 78,000
Actual rent
Received 72,000 72,000 60,000 30,000 72,000

➢ Treatment of interest u/s. 24(b): Interest allowed in normal manner as


calculated in TP 6 without any ceiling limit.

Mann Ki Baat (Simple Language):

The first thing that you should note from above is that notional income is taxable under
head income from house property. This is to curb the opportunities of tax evasion by
showing lower rent income.

Illustration: 6

Mr. Ramesh has taken a loan of ₹15,00,000 on 01.07.2019 from State Bank of India @
12% p.a. for construction of one house which was completed on 01.04.2022 and was let
out @ ₹90,000 p.m. w.e.f. 01.04.2022 and fair rent is ₹1,00,000 p.m. and the assessee
has paid municipal tax of ₹30,000 in P.Y. 2023-24 and the assessee has repaid the loan
amount in annual instalment of ₹1,00,000 starting from 01.01.2022. Compute his income
tax liability for the assessment year 2024-25 assuming that he has not opted for new
tax regime u/s. 115BAC.

Solution:

Computation of income under the head House Property and tax liability for A.Y.
2024-25

Gross Annual Value (W. N. #1) 12,00,000

80 | P a g e #HakkसेCA by CA. Akash Sir


81
3. Income from house property

Less: Municipal Tax (30,000)


Net Annual Value 11,70,000
Less: 30% of NAV u/s 24(a) (3,51,000)
Less: Interest on capital borrowed u/s 24(b) (W. N. #2) (2,51,400)
Income under the head house property 5,67,600

Working Note: 1
(a) Fair Rent (1,00,000 x 12) 12,00,000
(b) Expected Rent 12,00,000
(c) Rent received /receivable (90,000 x 12) 10,80,000
GAV 12,00,000

Working Note: 2
Current period Interest (from 01.04.2023 to 31.03.2024)
(13,00,000 x 12% x 9/12) + (12,00,000 x 12% x 3 /12) = 1,53,000
Prior period interest (from 01.07.2019 to 31.03.2022)
15,00,000 x 12% x 30/12 = 4,50,000
14,00,000 x 12% x 3/12 = 42,000
Instalment = 4,92,000 / 5 = 98,400
Total Interest = ₹1,53,000 + ₹98,400 = ₹2,51,400

Gross Total Income 5,67,600


Less: Deduction u/s 80C (1,00,000)
Total Income 4,67,600

Computation of Tax Liability


Tax on normal income ₹4,67,600 at slab rate 10,880
Less: Rebate u/s 87A (10,880)
Tax Liability Nil

HW Question: 6

Mr. Ashok has one house property which is let out @ ₹80,000 p.m. Fair rent ₹90,000
p.m., Municipal Valuation ₹70,000 p.m., Standard Rent ₹81,000 p.m. Municipal tax paid
₹60,000 and interest paid on loan for construction of house property is ₹ 50,000.
Compute his Income Tax Liability for A.Y 2024-25 assuming that he has opted for old
tax regime.

HW Question: 7

Mrs. Verma has let out one House property @ ₹62,000 p.m., Municipal Valuation
₹72,000 p.m., Fair Rent ₹90,000 p.m., Standard Rent ₹1,00,000 p.m., Municipal Tax paid
₹40,000 and Interest on loan taken for construction ₹60,000. She has completed the
age of 60 years on 01.04.2025. Compute Income Tax Liability for the A.Y. 2024-25
assuming that he has opted for old tax regime.

81 | P a g e #HakkसेCA by CA. Akash Sir


82
3. Income from house property

HW Question: 8

Mr. Rajeev has taken a loan of ₹10,00,000 from SBI on 01/04/2020 @ 10% p.a. for
construction of one house. The Assessee has taken a loan of 6,00,000 from PNB on
01/10/2022 @ 12% p.a. to repay loan to SBI. House was completed on 01/07/2023 and
was let out at a rent of ₹1,00,000 per month, paid municipal taxes ₹10,000. Compute his
income and tax liability for Assessment year 2024-25 assuming that he has not opted
for default tax regime u/s. 115BAC.

Case 2: GAV where let-out house property is vacant for part of the year
[Section 23(1)(c)] [IMPORTANT]:

Where let out property is vacant for part of the year and owing to vacancy, the
actual rent is lower than the expected rent, then the actual rent received or
receivable will be the GAV of the property.
In simple language, where house is vacant for part of the year and actual rent is
lower than expected rent only because of vacancy, then actual rent would be GAV.
However, if actual rent calculated for 12 months is lower than expected rent, then
expected rent would be GAV.

- If actual rent receivable (for whole year) >= Expected rent, then GAV =
Actual rent (for let out period)
- If actual rent receivable (for whole year) < Expected rent, then GAV =
Expected rent.

For example: Suppose that the expected rent is ₹2,40,000 and rent received/
receivable is ₹15,000 p.m. and there is vacancy for 5 months, then actual rent
received is ₹1,05,000. However, if there was no vacancy, rent would have been
₹1,80,000 for whole year which is still less than expected rent. Therefore, expected
rent i.e. ₹2,40,000 would be GAV.

Now, let’s suppose that the expected rent is ₹2,40,000 and rent received/ receivable
is ₹21,000 p.m. and there is vacancy for 5 months, then actual rent received is
₹1,47,000. However, if there was no vacancy, rent would have been ₹2,52,000 for
whole year which is higher than the expected rent. Therefore, actual rent i.e.
₹1,47,000 would be GAV.

Illustration: 7

Compute gross annual value in the following cases for the assessment year 2024-25:

Particulars House 1 House 2 House 3 House 4


Fair Rent (p.m.) 9,000 13,000 16,000 12,000

82 | P a g e #HakkसेCA by CA. Akash Sir


83
3. Income from house property

Municipal Valuation 10,000 9,000 18,000 9,000


(p.m.)
Standard Rent (p.m.) 12,000 11,000 16,000 7,000
Rent received/ 7,000 11,500 16,000 20,000
receivable (p.m.)
Vacancy 1 month 1 month 2 months 2 months

Solution:

Computation of GAV for A.Y. 2024-25

Particulars House 1 House 2 House 3 House 4


(a) Municipal value p.a. 1,20,000 1,08,000 2,16,000 1,08,000
(b) Fair rent p.a. 1,08,000 1,56,000 1,92,000 1,44,000
(c) higher of (a) and (b) 1,20,000 1,56,000 2,16,000 1,44,000
(d) standard rent p.a. 1,44,000 1,32,000 1,92,000 84,000
(e) Expected rent [lower 1,20,000 1,32,000 1,92,000 84,000
of (c) and (d)]
(f) Actual rent 77,000 1,26,500 1,60,000 2,00,000
received/receivable p.a.
Actual rent if there is 84,000 1,38,000 1,92,000 2,40,000
no vacancy (I)
Is Actual rent in (I) No Yes Yes Yes
more than or equals to
expected rent in (e) ??
GAV 1,20,000 1,26,500 1,60,000 2,00,000

➢ Treatment of interest u/s. 24(b): Interest allowed in normal manner without


any ceiling limit as calculated in TP: 6.

Mann Ki Baat (Simple Language):

Everything in the question will remain same as case (1), just check if actual rent would
have been greater than expected rent if there is no vacancy. If answer is YES, then
treat actual rent as GAV and if answer is NO, then treat expected rent as GAV.

Illustration: 8

Mr. Vakil has taken a loan of ₹15,00,000 on 01.07.2019 from State Bank of India @
12% p.a. for construction of one house which was completed on 01.05.2023 and was let
out @ ₹90,000 p.m. w.e.f. 01.07.2023 and fair rent is ₹1,25,000 p.m. and the assessee
has paid municipal tax of ₹30,000 in P.Y. 2023-24 and the assessee has repaid the loan
amount in annual instalment of ₹1,00,000 starting from 01.01.2022. Compute his income

83 | P a g e #HakkसेCA by CA. Akash Sir


84
3. Income from house property

tax liability for the assessment year 2024-25 assuming that he has not opted for
section 115BAC.

Solution:

Computation of income under the head House Property for A.Y. 2024-25

Gross Annual Value (W.N. #1) 13,75,000


Less: Municipal Tax (30,000)
Net Annual Value 13,45,000
Less: 30% of NAV u/s 24(a) (4,03,500)
Less: Interest on capital borrowed u/s 24(b) (W.N. #2) (2,84,400)
Income under the head house property 6,57,100

Gross Total Income 6,57,100


Less: Deduction u/s 80C (1,00,000)
Total Income 5,57,100

Computation of Tax Liability


Tax on normal income ₹5,57,100 at slab rate 23,920
Add: HEC @ 4% 957
Tax Liability 24,877
Rounded off u/s 288B 24,880

Working Note: 1
(a) Fair Rent (1,25,000 x 11) 13,75,000
(b) Expected Rent 13,75,000
(c) Rent received /receivable (90,000 x 9) 8,10,000
If there was no vacancy, in that case rent received/receivable would have been
₹9,90,000 and it was still less than expected rent, therefore GAV shall be expected
rent i.e. ₹13,75,000

Working Note: 2
Current period Interest (from 01.04.2023 to 31.03.2024)
(13,00,000 x 12% x 9/12) + (12,00,000 x 12% x 3/12) = ₹1,53,000
Prior period interest (from 01.07.2019 to 31.03.2023)
15,00,000 x 12% x 30/12 = 4,50,000
14,00,000 x 12% x 12/12 = 1,68,000
13,00,000 x 12% x 3/12 = 39,000
Instalment = 6,57,000 / 5 = ₹1,31,400
Total Interest = ₹1,53,000 + ₹1,31,400 = ₹2,84,400

84 | P a g e #HakkसेCA by CA. Akash Sir


85
3. Income from house property

HW Question: 9

Mr. Kavish has let out one house at a rent of ₹50,000 p.m. Fair rent ₹55,000 p.m.
Municipal Valuation ₹52,000 p.m., standard rent ₹60,000 p.m. The house remains vacant
for 3 months. The assessee paid municipal tax ₹30,000. Interest on loan u/s. 24(b) is
₹20,000. Compute Income u/h house property for A.Y. 2024-25.
(b) Presume it is let out at a rent of ₹60,000 P.m.
(c) Presume it is let out at a rent of ₹55,000 P.m.
(d) Presume it is let out at a rent of ₹1,00,000 P.m.

Case: 3 GAV in case of self-occupied property or unoccupied property


[Section 23(2)]:

Where the property is self-occupied for own residence or unoccupied throughout the
previous year, it’s Net Annual Value will be NIL, provided no other benefit is derived
by the owner from Such property.

“Unoccupied property”- a property which cannot be occupied by the owner by reason


of his employment, business or profession at a different place and he Resides at such
other place in a building not belonging to him.

➢ Treatment of interest u/s. 24(b) [Only under old tax regime]: Interest
allowed upto ₹30,000.
However, if all the following conditions are satisfied, the assessee can claim a
maximum deduction of ₹2,00,000:
a) Loan taken on or after 01.04.1999
b) Loan taken for purchase or construction of house property
c) House property purchased or constructed withing 5 years from the end of the
year in which loan was taken
d) A certificate has been obtained from the lender certifying the amount of
interest.

Note: Maximum loss from one or more self-occupied house properties cannot
exceed ₹30,000 or ₹2,00,000 as the case may be.

Important Note: Deduction of interest is not allowed where the Assessee has
opted for default tax regime under section 115BAC.

85 | P a g e #HakkसेCA by CA. Akash Sir


86
3. Income from house property

For example: Calculate Income under head house property under old tax regime

Particulars House 1 House 2 House 3 House 4


Let-out Self- Partly vacant Un-occupied
occupied and partly let-
out
Fair Rent (p.m.) 9,000 13,000 16,000 12,000
Municipal Valuation 10,000 9,000 18,000 9,000
(p.m.)
Standard Rent (p.m.) 12,000 11,000 16,000 7,000
Rent received/ 7,000 11,500 16,000 20,000
receivable (p.m.)
Vacancy NA NA 2 months NA
Municipal tax paid by 15,000 20,000 25,000 10,000
owner
Interest on capital 15,000 1,50,000 20,000 60,000
borrowed (not paid)

Solution:

Calculation of income from house property for A.Y. 2024-25

Particulars House 1 House 2 House 3 House 4


(a) Municipal value p.a. 1,20,000 2,16,000
(b) Fair rent p.a. 1,08,000 1,92,000
(c) higher of (a) and (b) 1,20,000 2,16,000
(d) standard rent p.a. 1,44,000 1,92,000
(e) Expected rent [lower 1,20,000 1,92,000
of (c) and (d)]
(f) Actual rent 84,000 1,60,000
received/receivable p.a.
Actual rent if there is no NA 1,92,000
vacancy (I)
Is Actual rent in (I) more NA Yes
than or equals to
expected rent in (e) ??
GAV 1,20,000 0 1,60,000 0
Less: Municipal tax 15,000 0 25,000 0
NAV 1,05,000 0 1,35,000 0
Less: Deduction u/s. 31,500 0 40,500 0
24(a)
Less: Deduction u/s. 15,000 1,50,000 20,000 60,000
24(b)

86 | P a g e #HakkसेCA by CA. Akash Sir


87
3. Income from house property

Income from house 58,500 (1,50,000) 74,500 (60,000)


property

Income under head house property-


House 1 58,500
House 3 74,500
House 2 and 4 (Loss can not exceed 2,00,000) (2,00,000)
Total income from house property (67,000)

Mann Ki Baat (Simple Language):

This case is very simple. Just focus on interest u/s. 24(b). Just remember that
maximum interest allowed as deduction is ₹2,00,000 in case of construction or
purchase (combined all self-occupied or un-occupied properties). Also, remember that
maximum interest deduction of ₹30,000 is allowed in case of loan taken for other than
construction and purchase.

Illustration: 9

Miss Kajal Maheshwari occupied two flats for her residential purpose, particulars of
which are as follows:
Particulars Flat I (in ₹) Flat II (in ₹)
Municipal Valuation p.m. 90,000 45,000
Fair Rent p.m. 1,20,000 40,000
Standard Rent p.m. 80,000 Not available
Municipal taxes paid 10% of municipal valuation 10% of municipal valuation
Fire insurance paid 1,000 600
Interest payable on capital borrowed
for renovation of flat 40,000 Nil

Income of Miss Kajal from her proprietary business, Kajal Warehousing Corporation is
₹6,50,000. Determine the taxable income and tax liability for the assessment year
2024-25 under old provisions of the Act. You are informed that Miss Kajal could not
occupy flat for 2 months commencing from 01.12.2023 and that she has attained the
age of 60 on 23.08.2023.

(ii) Determine the taxable income and tax liability for the assessment year 2024-25
under default provisions of the Act under section 115BAC.

Solution: (i)

As per the regular provisions of Act, in case of self-occupied property, the aggregate
amount of interest shall not exceed ₹30,000/ ₹2,00,000 as the case may be.

87 | P a g e #HakkसेCA by CA. Akash Sir


88
3. Income from house property

Flat I Flat II
Net annual value 0 0
Less: Interest u/s. 24(b) 40,000 0
Income from house property (30,000) 0
[Loss cannot exceed ₹30,000 in case
where loan is taken for renovation]
Total income from house property (30,000)

Computation of Gross Total Income


Income under the head House Property (30,000.00)
Income under the head Business/ Profession 6,50,000.00
Gross Total Income 6,20,000.00
Less: Deduction u/s. 80C to 80U Nil
Total Income 6,20,000.00

Computation of Tax Liability


Tax on ₹6,20,000 at slab rate 34,000.00
Add: HEC @ 4% 1,360.00
Tax Liability 35,360.00

(ii) Under default tax regime, interest on self-occupied property under section 24(b) is
not allowed to be deducted while calculating income from house property. Therefore,
income from house property shall be Nil.

Computation of Gross Total Income


Income under the head House Property Nil
Income under the head Business/ Profession 6,50,000.00
Gross Total Income 6,50,000.00
Total Income 6,50,000.00

Computation of Tax Liability


Tax on ₹6,50,000 at slab rate 20,000.00
Less: Rebate u/s. 87A 20,000.00
Tax Liability Nil

HW Question: 10

(a) Mr. Ompal has taken a loan of ₹5,00,000 on 01.10.1999 @ 10% p.a. for construction
of a house which was completed on 01.10.2022 and the house remained self-occupied
throughout the previous year 2023-24. The assessee has income under the head PGBP
₹4,00,000. Compute tax liability for assessment year 2024-25 under old tax regime.

(b) Presume in above question, the loan was taken on 01.10.2019. The assessee has
submitted a certificate confirming the amount of interest.

88 | P a g e #HakkसेCA by CA. Akash Sir


89
3. Income from house property

HW Question: 11

Mrs. Ritvik has taken a loan on 01.11.2019 from PNB @ 10% p.a. of ₹10,00,000 for
purchase of one house which was purchased on 01.01.2020 and was self-occupied and
municipal taxes paid in previous year 2023-24 is ₹30,000. She has repaid the loan
amount in annual installments of ₹50,000 starting from 01.01.2021. The house was
vacant for 1 month in previous year 2023-24. She has submitted a certificate
confirming the amount of interest. She has short term capital gains under section 111A
₹10,00,000. Compute his total income for assessment year 2024-25 under old tax
regime and new tax regime.

Case: 4 Where a house property is let-out for part of the year and self-
occupied for part of the year [Section 23(3)]:

If a single unit of a property is self-occupied for part of the year and let-out for the
remaining part of the year, then the expected rent for the whole year shall be taken
into account for determining the GAV.

The expected rent for the whole year shall be compared with the actual rent for the
let-out period and whichever is higher shall be adopted as the GAV. However,
municipal tax for the whole year is allowed deduction provided it is paid by the owner
during the previous year.

In other words, where the property is let out for the for part of the year and self-
occupied for part of the year, then the GAV shall be calculated in the same manner as
in case (1) as below:
(a) Fair Rent for the year;
(b) Municipal Value for the year;
(c) Higher of (a) or (b)
(d) Standard Rent for the year;
(e) Expected Rent {Lower of c or d};
(f) Rent received/ receivable for the period of let-out
GAV shall be higher of (e) or (f)

For example: Miss Muskan constructed one house in 2021 and it is let out for 4
months and self-occupied for 8 months during previous year 2023-24. Municipal
valuation of the house is ₹40,000 p.m. and fair rent ₹30,000 p.m. Standard rent of the
house is ₹38,000 p.m. It was let out @ ₹32,000 p.m. Municipal tax of ₹50,000 paid for
whole year. In this case NAV shall be computed as below:

89 | P a g e #HakkसेCA by CA. Akash Sir


90
3. Income from house property

(a) Fair Rent (30,000 x 12) 3,60,000


(b) Municipal Valuation (40,000 x 12) 4,80,000
(c) Higher of (a) or (b) 4,80,000
(d) Standard Rent (38,000 x 12) 4,56,000
(e) Expected Rent {Lower of (c) or (d)} 4,56,000
(f) Rent Received/ Receivable (32,000 x 4) 1,28,000
Gross Annual Value 4,56,000
Less: Municipal tax 50,000
Net Annual Value 4,06,000

➢ Treatment of interest u/s. 24(b): Interest allowed in normal manner


without any ceiling limit.

Mann Ki Baat (Simple Language):

Where the house is let-out and self-occupied during the year, then just treat house as
let-out and do not think about self-occupation.

Illustration: 10

Mr. Lokesh constructed one house in 2022 and it is let out for 4 months and self-
occupied for 6 months and vacant for 2 months during previous year 2023-24. Municipal
valuation of the house is ₹40,000 p.m. and fair rent ₹30,000 p.m. Standard rent of the
house is ₹38,000 p.m. It was let out @ ₹32,000 p.m. Compute the GAV of the house.

Solution:

Calculation of GAV for A.Y. 2024-25

(a) Fair Rent (30,000 x 12) 3,60,000


(b) Municipal Valuation (40,000 x 12) 4,80,000
(c) Higher of (a) or (b) 4,80,000
(d) Standard Rent (38,000 x 12) 4,56,000
(e) Expected Rent {Lower of (c) or (d)} 4,56,000
(f) Rent Received/ Receivable (32,000 x 4) 1,28,000
If there was no vacancy, in that case rent received/receivable would have been
₹1,92,000 (₹32,000*6) and it was still less than expected rent, therefore GAV shall be
expected rent.

90 | P a g e #HakkसेCA by CA. Akash Sir


91
3. Income from house property

Gross Annual Value 4,56,000

HW Question: 12

Mrs. Sharma has one house property at Indira Nagar in Bangalore. She stays with her
family in the house. The rent of similar property in the neighbourhood is ₹25,000 p.m.
The municipal valuation is ₹23,000 p.m. Municipal taxes paid is ₹8,000. The loan of
₹20,00,000 was taken on 01.01.2017 from SBI Housing Finance Ltd. The construction
was completed on 30.11.2019. The accumulated interest up to 31.03.2019 is ₹3,00,000.
During the previous year 2023-24, Mrs. Sharma paid ₹ 1,88,000 which included
₹1,44,000 as interest. Compute Mrs. Sharma’s income from house property for A.Y.
2024-25 assuming that she has not opted for default tax regime. All the conditions for
higher deduction of interest in case of self-occupied property is satisfied.

Case: 5 In case of deemed to be let out property [Section 23(4)]

Two self-occupied house properties: Where the assessee owns more than two
properties for self-occupation, then the income from any two properties, at the
option of the assessee, shall be computed under the self-occupied property category
and their annual value will be NIL.

Other self-occupied house property: The other self-occupied/ unoccupied


properties shall be treated as “deemed let out” properties. In case of deemed let-
out property, the ER shall be taken as the GAV. This option can be changed year
after year in a manner beneficial to the assessee. Municipal taxes actually paid by
the owner during the Previous Year, can be claimed as deduction.

➢ Treatment of interest u/s. 24(b) (under old tax regime): Interest allowed
in normal manner without any ceiling limit on deemed let-out property.
Interest on self-occupied house property shall be subject to ceiling limit of
₹30,000/ ₹2,00,000 as discussed in case (3) above.

Important Note: Deduction of interest on self-occupied house property is not


allowed where the Assessee has opted for default tax regime under section
115BAC.

Illustration: 11

Mr. Sumit has 3 houses which are self-occupied and the details of these houses is as
under:

Particulars House I House II House III


Fair Rent p.a. 11,00,000 12,00,000 11,50,000
Municipal Valuation p.a. 11,24,000 11,78,000 11,25,000

91 | P a g e #HakkसेCA by CA. Akash Sir


92
3. Income from house property

Standard Rent p.a. 13,00,000 12,50,000 11,40,000


Municipal Taxes Paid by Owner 1,00,000 80,000 90,000
Interest on capital borrowed on 3,20,000 2,90,000 1,90,000
01.04.2018 and all the necessary
conditions are complied with to avail
higher amount of interest.
Repair Charges 10,000 3,000 8,000
Date of Completion of house 01.10.2020 01.10.2020 01.10.2020

Calculate income under head house property assuming that


(i) the Assessee has opted to shift out of provisions of section 115BAC.
(ii) the Assessee has opted for section 115BAC.

Solution: (i) Under old tax regime

Calculation of Income for each house property considering all as deemed let-out:

Particulars House I House II House III


(a) Fair Rent p.a. 11,00,000 12,00,000 11,50,000
(b) Municipal Valuation p.a. 11,24,000 11,78,000 11,25,000
(c) Higher of (a) or (b) 11,24,000 12,00,000 11,50,000
(d) Standard Rent p.a. 13,00,000 12,50,000 11,40,000
(e) Expected rent (lower of (c) or 11,24,000 12,00,000 11,40,000
(d))
Gross annual value 11,24,000 12,00,000 11,40,000
Less: Municipal Taxes Paid 1,00,000 80,000 90,000
Net annual value 10,24,000 11,20,000 10,50,000
Less: Standard deduction @30% of 7,16,800 7,84,000 7,35,000
NAV u/s. 24(a)
Less: Interest on capital borrowed 3,20,000 2,90,000 1,90,000
u/s. 24(b)
Income from house property 3,96,800 4,94,000 5,45,000

Calculation of income under various options:

Particulars Option I Option II Option III


House I & II House II & House I &
are self- III are self- III are
occupied and occupied and self-occupied
house III is house I is and house II
deemed let- deemed let- is deemed
out out let-out
House property I (2,00,000) 3,96,800 (2,00,000)
House property II (2,00,000) (2,00,000) 4,94,000

92 | P a g e #HakkसेCA by CA. Akash Sir


93
3. Income from house property

House property III 5,45,000 (1,90,000) (1,90,000)


Total loss from self-occupied (2,00,000) (2,00,000) (2,00,000)
property cannot exceed ₹2,00,000
Income from house property 3,45,000 1,96,800 2,94,000

Second Option is the best Income under the head House Property is ₹1,96,800.

(ii) Under default tax regime

Calculation of Income for each house property considering all as deemed let-out:

Particulars House I House II House III


(a) Fair Rent p.a. 11,00,000 12,00,000 11,50,000
(b) Municipal Valuation p.a. 11,24,000 11,78,000 11,25,000
(c) Higher of (a) or (b) 11,24,000 12,00,000 11,50,000
(d) Standard Rent p.a. 13,00,000 12,50,000 11,40,000
(e) Expected rent (lower of (c) or 11,24,000 12,00,000 11,40,000
(d))
Gross annual value 11,24,000 12,00,000 11,40,000
Less: Municipal Taxes Paid 1,00,000 80,000 90,000
Net annual value 10,24,000 11,20,000 10,50,000
Less: Standard deduction @30% of 7,16,800 7,84,000 7,35,000
NAV u/s. 24(a)
Less: Interest on capital borrowed 3,20,000 2,90,000 1,90,000
u/s. 24(b)
Income from house property 3,96,800 4,94,000 5,45,000

Calculation of income under various options:

Particulars Option I Option II Option III


House I & II House II & House I &
are self- III are self- III are
occupied and occupied and self-occupied
house III is house I is and house II
deemed let- deemed let- is deemed
out out let-out
House property I - 3,96,800 -
House property II - - 4,94,000
House property III 5,45,000 - -
Income from house property 5,45,000 3,96,800 4,94,000

Second Option is the best Income under the head House Property is ₹3,96,800.

93 | P a g e #HakkसेCA by CA. Akash Sir


94
3. Income from house property

Mann Ki Baat (Simple Language):

From above, we understand that the government has made it very clear that the
assessee cannot take benefit of self-occupied house property for more than 2 house
properties. Assessee has to offer income on more than 2 self-occupied house
properties. It is to curb the opportunities of tax evasion by showing all properties as
self occupied.

Case: 6 In case of a house property held as stock-in-trade [Section 23(5)]

In some cases, property consisting of any buildings or lands appurtenant thereto may
be held as stock-in-trade (i.e. in case of builders), and the whole or any part of the
property may not be let out during the whole or any part of the previous year. In
such cases, the annual value of such property or part of the property shall be Nil.

This benefit would be available for the period up to two years from the end of the
financial year in which certificate of completion (CC) of construction of the property
is obtained from the competent authority.

Mann Ki Baat (Simple Language):

Where a builder could not sell the house property for a period of two years from end
of financial year in which CC was received and property is vacant, the builder has to
consider the property as deemed let out and has to pay income tax of notional rent of
vacant property.

Case: 7 In case of a house property, a portion let out and a portion self-
occupied

Income from any portion or part of a property which is let out shall be computed
separately under the “let out property” category and the other portion or part which
is self-occupied shall be computed under the “self-occupied property” category.

Municipal valuation/ fair rent/ standard rent and municipal tax, if not given
separately, shall be apportioned between the let-out portion and self-occupied
portion on reasonable basis.

Illustration: 12 [ICAI Mat]

Mrs. Manavi owns a house in Madras. During the previous year 2023-24, 2/3rd portion
of the house was self-occupied and 1/3rd portion was let out for residential purposes
at a rent of ₹8,000 p.m. Municipal value of the property is ₹3,00,000 p.a., fair rent is
₹2,70,000 p.a. and standard rent is ₹3,30,000 p.a. He paid municipal taxes @ 10% of

94 | P a g e #HakkसेCA by CA. Akash Sir


95
3. Income from house property

municipal value during the year. A loan of ₹25,00,000 was taken by him during the year
2019 for acquiring the property. Interest on loan paid during the previous year 2023-
24 was ₹1,20,000. Compute her income from house property for the A.Y. 2024-25
under old tax regime. All the conditions for higher deduction of interest in case of
self-occupied property is satisfied.

Solution:

There are two units of the house. Unit I with 2/3rd area is used by Mrs. Manavi for
self-occupation throughout the year and no benefit is derived from that unit, hence it
will be treated as self-occupied and its annual value will be nil.
Unit 2 with 1/3rd area is let-out through-out the previous year and its annual value has
to be determined as per section 23(1).

Computation of income from house property of Mrs. Manavi for A.Y. 2024-25

Unit I (2/3rd area – self-occupied)


Annual Value Nil
Less: Deduction under section 24(b) 2/3rd of ₹1,20,000 (80,000)
Loss from Unit I (self-occupied) (80,000)

Unit II (1/3rd area – let out)


Gross Annual Value 1,00,000
Working Note:
(a) Fair rent 90,000
(b) Municipal valuation 1,00,000
(c) Higher of (a) or (b) 1,00,000
(d) Standard rent 1,10,000
(e) Expected rent {Lower of (c) or (d)} 1,00,000
(f) Rent received/ receivable (8,000 x 12) 96,000
GAV = Expected rent 1,00,000
Less: Municipal Taxes (10,000)
Net Annual Value 90,000
Less: 30% of NAV u/s 24(a) (27,000)
Less: Interest on capital borrowed u/s 24(b) (40,000)
Income under the head House Property 23,000
Net Loss from house property (57,000)

TP: 8 Treatment of Unrealised Rent [Explanation below section


23(1)/ Rule 4]

Unrealised rent means such rent which is irrecoverable and is considered to be bad
and therefore actual rent received/ receivable should not include unrealised rent.

95 | P a g e #HakkसेCA by CA. Akash Sir


96
3. Income from house property

However, the following conditions prescribed in Rule 4 should be satisfied:-


(a) tenancy is bona fide;
(b) the defaulting tenant has vacated, or steps have been taken to compel him to
vacate the property;
(c) the defaulting tenant is not in occupation of any other property of the assessee;
(d) the assessee has taken all reasonable steps to institute legal proceedings for the
recovery of the unpaid rent or satisfies the Assessing Officer that legal
proceedings would be useless.

Now, in the above cases, expected rent shall be calculated for full period and
unrealised rent shall be excluded from actual rent received/ receivable. GAV shall be
higher of expected rent or actual rent. No special treatment is to be made like
vacancy case.

For example: Mr. Pawan has let out one house ₹50,000 p.m., fair rent ₹45,000 p.m.,
municipal valuation ₹40,000 p.m. standard rent ₹70,000 p.m. and there was
unrealized rent for 3 months, in this case GAV of the house shall be higher of
expected rent (45,000 x 12)= 5,40,000 and rent received /receivable (50,000 x 9)=
4,50,000 and therefore, GAV shall be 5,40,000.

Mann Ki Baat (Simple Language):

Just treat unrealised rent as a deduction from actual rent received/ receivable and
apply the knowledge from case (1) to (7) studied above. No special treatment shall be
given to unrealised rent.

Illustration: 13

Compute gross annual value in the following cases for the assessment year 2024-25:

Particulars Situation 1 Situation 2 Situation 3 Situation 4


Fair Rent (p.m.) 11,000 13,000 16,000 14,000
Municipal Valuation (p.m.) 12,000 11,000 18,000 9,000
Standard Rent (p.m.) 13,000 12,000 17,000 8,000
Rent received/ receivable
(p.m.) 8,000 12,500 17,000 21,000
Vacancy - 2 months 3 months 1 month
Unrealised rent 1 month - 1 month 3 months

Solution:

Situation 1:
Computation of Gross Annual Value
(a) Fair Rent 1,32,000 (11,000 x 12)
(b) Municipal Valuation 1,44,000 (12,000 x 12)

96 | P a g e #HakkसेCA by CA. Akash Sir


97
3. Income from house property

(c) Higher of (a) or (b) 1,44,000


(d) Standard Rent 1,56,000 (13,000 x 12)
(e) Expected Rent {Lower of (c) or (d)} 1,44,000
(f) Rent Received/Receivable 88,000 (8,000 x 11)
GAV = Higher of (e) or (f) 1,44,000
Gross Annual Value 1,44,000

Situation 2:
Computation of Gross Annual Value
(a) Fair Rent 1,56,000 (13,000 x 12)
(b) Municipal Valuation 1,32,000 (11,000 x 12)
(c) Higher of (a) or (b) 1,56,000
(d) Standard Rent 1,44,000 (12,000 x 12)
(e) Expected Rent {Lower of (c) or (d)} 1,44,000
(f) Rent Received/Receivable 1,25,000 (12,500 x 10)
In this case, if there was no vacancy, rent received/receivable would have been
₹12,500 x 12 = ₹1,50,000 hence rent received/receivable is lower in this case due to
vacancy, therefore GAV shall be the rent received/receivable. Gross Annual Value
1,25,000

Situation 3:
Computation of Gross Annual Value
(a) Fair Rent 1,92,000 (16,000 x 12)
(b) Municipal Valuation 2,16,000 (18,000 x 12)
(c) Higher of (a) or (b) 2,16,000
(d) Standard Rent 2,04,000 (17,000 x 12)
(e) Expected Rent {Lower of (c) or (d)} 2,04,000
(f) Rent Received/Receivable 1,36,000 (17,000 x 8)
If there was no vacancy, in that case rent received/receivable would have been
₹17,000 x 11= ₹1,87,000 and It was still less than expected rent, therefore GAV shall
be expected rent. Gross Annual Value 2,04,000

Situation 4:
Computation of Gross Annual Value
(a) Fair Rent 1,68,000 (14,000 x 12)
(b) Municipal Valuation 1,08,000 (9,000 x 12)
(c) Higher of (a) or (b) 1,68,000
(d) Standard Rent 96,000 (8,000 x 12)
(e) Expected Rent {Lower of (c) or (d)} 96,000
(f) Rent Received/Receivable 1,68,000 (21,000 x 8)
In this case, rent R/R is higher than the expected rent, GAV shall be Rent R/R Gross
Annual Value 1,68,000

97 | P a g e #HakkसेCA by CA. Akash Sir


98
3. Income from house property

Illustration: 14 [IFHP, IFOS] [ICAI Mat]

Mr. Ramesh has a property whose municipal valuation is ₹2,50,000 p.a. The fair rent is
₹2,00,000 p.a. and the standard rent fixed by the Rent Control Act is ₹2,10,000 p.a.
The property was let out for a rent of ₹20,000 p.m. However, the tenant vacated the
property on 31.01.2024. Unrealised rent was ₹20,000 and all conditions prescribed by
Rule 4 are satisfied. He paid municipal taxes @ 8% of municipal valuation. Interest on
borrowed capital was ₹1,65,000 for the year. He has casual income ₹3,00,000. Compute
his tax liability for A.Y. 2024-25 under regular provisions of the Act.
(a) Presume he is resident and his date of birth 01.04.1964
(b) Presume he is non-resident and his date of birth 01.04.1964

Solution:

Computation of income from house property and total income of Mr. Ramesh for
A.Y. 2024-25

Gross Annual Value 1,80,000.00


Working Note:
(a) Fair Rent 2,00,000
(b) Municipal Valuation 2,50,000
(c) Higher of (a) or (b) 2,50,000
(d) Standard Rent 2,10,000
(e) Expected Rent {Lower of (c) or (d)} 2,10,000
(f) Rent received /receivable (20,000 x 9) 1,80,000
If there was no vacancy, in that case rent received receivable would have been (20,000
x 11) ₹2,20,000 which is exceeding expected rent hence GAV shall be rent received
/receivable i.e. ₹1,80,000

Less: Municipal Tax (8% of ₹2,50,000) (20,000.00)


Net Annual Value 1,60,000.00
Less: 30% of NAV u/s 24(a) (48,000.00)
Less: Interest on capital borrowed u/s 24(b) (1,65,000.00)
Loss under the head House Property (53,000.00)
Note: Loss shall be carried forward and as per section 58(4), loss shall not be adjusted
from casual income

Income under head other sources


Casual income 3,00,000.00
Net total income 3,00,000.00

(a) He is a resident:

98 | P a g e #HakkसेCA by CA. Akash Sir


99
3. Income from house property

Income Tax ₹3,00,000 x 30% 90,000.00


Less: Rebate u/s 87A (12,500.00)
Tax before health & education cess 77,500.00
Add: HEC @ 4% 3,100.00
Tax Liability 80,600.00

(b) He is a non-resident

Income Tax ₹3,00,000 x 30% 90,000.00


Add: HEC @ 4% 3,600.00
Tax Liability 93,600.00
Note: Rebate under section 87A is not allowed to non-resident.

HW Question: 13 [ICAI Mat]

Mrs. Lokesh owns a house property at Adyar in Chennai. The municipal value of the
property is ₹5,00,000, fair rent is ₹4,20,000 and standard rent is ₹4,80,000. The
property was let-out for ₹50,000 p.m. up to December 2023. Thereafter, the tenant
vacated the property and Mrs. Lokesh used the house for self-occupation. Rent for the
months of November and December 2023 could not be realised in spite of the owner’s
efforts. All the conditions prescribed under Rule 4 are satisfied. She paid municipal
taxes @ 12% during the year. She had paid interest of ₹25,000 during the year for
amount borrowed for repairs for the house property. Compute Income u/h House
Property for A.Y. 2024-25.

HW Question: 14

Mr. Shyam has let out one house property to Mr. Laxman @ ₹ 80,000 p.m. Fair rent
₹90,000 p.m. Municipal valuation ₹80,000 p.m. and Standard rent of the house ₹76,000
p.m. The house remained vacant for 2 months and there was unrealised rent for 3
months. Mr. Shyam has paid municipal tax of ₹60,000 and interest on loan for
construction of house property is ₹69,000. Compute his Income Tax Liability for A. Y.
2024-25 under regular provisions of the Act and default tax regime.

TP: 9 Recovery of unrealised rent [Section 25A]

If any assessee has recovered unrealized rent in subsequent years, rent so


recovered shall be considered to be income of the assessee under the head house
property and it does not matter whether the assessee has any house property in his
name in that year or not.

99 | P a g e #HakkसेCA by CA. Akash Sir


100
3. Income from house property

If assessee has received any interest, it will be considered to be income of the


assessee under the head other sources. If assessee has incurred any expenses on
legal proceedings, it will not be allowed to be deducted.

Deduction: A sum equal to 30% of the unrealised rent shall be allowed as


deduction.

Illustration: 15

Mr. Ashok has let out one house at ₹70,000 per month, fair rent ₹80,000 per month,
municipal valuation ₹60,000 per month, standard rent ₹ 65,000 per month. Municipal
tax paid ₹40,000, Interest u/s 24 (b) ₹50,000. Assessee has recovered unrealized
rent of ₹60,000 plus interest ₹7,000. He has incurred legal expenses ₹12,000. Compute
his Income and Tax Liability A. Y. 2024-25 under old tax regime.

Solution:

Computation of income under the head house property for A.Y. 2024-25

Gross Annual Value (W.N. #1) 8,40,000.00


Less: Municipal Tax (40,000.00)
Net Annual Value 8,00,000.00
Less: 30% of NAV u/s 24(a) (2,40,000.00)
Less: Interest on capital borrowed u/s 24(b) (50,000.00)
Income under the head House Property 5,10,000.00
Add: Recovery of Unrealised rent u/s 25A 60,000.00
Less: Deduction @ 30% (18,000.00) 42,000.00
Income under the head House Property 5,52,000.00

Working Note 1:
(a) Fair Rent (80,000 x 12) 9,60,000
(b) Municipal Valuation (60,000 x 12) 7,20,000
(c) Higher of (a) or (b) 9,60,000
(d) Standard Rent (65,000 x 12) 7,80,000
(e) Expected Rent {Lower of (c) or (d)} 7,80,000
(f) Rent received /receivable (70,000 x 12) 8,40,000
(g) Higher of (e) or (f) shall be GAV 8,40,000

Income under the head other sources


Interest from unrealized rent 7,000.00

Gross Total Income 5,59,000.00


Less: Deduction u/s 80C to 80U NIL
Total Income 5,59,000.00

100 | P a g e #HakkसेCA by CA. Akash Sir


101
3. Income from house property

Computation of Tax Liability


Tax on ₹5,59,000 at slab rate 24,300.00
Add: HEC @ 4% 972.00
Tax Liability 25,272.00
Rounded off u/s 288B 25,270.00

TP: 10 Taxability in case of arrears of rent received


subsequently [section 25A]

Arrear of rent means increase in rent of house property from retrospective date i.e.
from back date. Such amount of rent received in arrears from a tenant shall be
deemed to be the income from house property in the financial year in which such rent
is received or realised, and shall be included in the total income of the assessee
under the head “Income from house property”, whether the assessee is the owner of
the property or not in that financial year.

Deduction: Section 25A(2) provides a deduction of 30% of arrears of rent.

Illustration: 16

Mr. Somesh has let out his house to State Bank @ ₹20,000 p.m. The bank has increased
the rent on 01.07.2023 to ₹27,000 p.m. retrospectively w.e.f. 01.11.2022. The assessee
has paid municipal taxes of ₹7,000 during the previous year 2023-24. Compute income
under the head House Property for assessment year 2024-25.

Solution:

Computation of income under the head House Property for A.Y. 2024-25

Gross Annual Value (27,000 x 12) 3,24,000


Less: Municipal Taxes (7,000)
Net Annual Value 3,17,000
Less: 30% of NAV u/s 24(a) (95,100)
Less: Interest on capital borrowed u/s 24(b) Nil
2,21,900
Add: Arrears of rent (Sec 25A) (7,000 x 5) 35,000
Less: 30% of ₹35,000 (10,500) 24,500
Income under the House Property 2,46,400

HW Question: 15

Mr. Mukesh owns a house property which is let out. During the previous year ending
31.03.2024 he receives (i) arrears of rent of ₹30,000 and (ii) unrealised rent of
₹20,000. You are requested to

101 | P a g e #HakkसेCA by CA. Akash Sir


102
3. Income from house property

(a) state, how they should be dealt with as per the provisions of the Act, and
(b) compute the income chargeable under the head “Income from house property”.

HW Question: 16 [ICAI Study Mat]

Mr. Binod sold his residential house property in March 2023. In June 2023, he
recovered rent of ₹10,000 from Mr. Gaurav, to whom he had let out his house for two
years from April 2017 to March 2019. He could not realise two month rent of ₹20,000
from him and to that extent his actual rent was reduced while computing income from
house property for A. Y. 2019-20. Further, he had let out his property from April, 2019
to February, 2023 to Mr. Satish. In April, 2021, he had increased the rent from
₹12,000 to ₹15,000 per month and the same was a subject matter of dispute. In
September 2023, the matter was finally settled and Mr. Binod received ₹69,000 as
arrears of rent for the period April 2021 to February, 2023. Would the recovery of
unrealised rent and arrears of rent be taxable in the hands of Mr. Binod, and if so in
which year?

TP: 11 Treatment of income from co-owned property [Section


26]

Where property is owned by two or more persons, whose shares are definite and
ascertainable, then the income from such property cannot be taxed as income of an
AOP.

Co-owned property [Section 26]


Self –occupied property Let –out property
a) The annual value of the property of a) The income from such property shall be
each co-owner will be Nil computed as if the property is owned by
b) Each co-owner shall be entitled to a one owner and thereafter the income so
deduction of ₹30,000/ ₹2,00,000, as the computed shall be apportioned amongst
case may be, on account of interest on each co-owner as per their specific share.
borrowed capital where the Assessee
chooses to shift out of new tax regime.
c) However, if the co-owner owns another
self-occupied/ unoccupied property, the
aggregate interest from the co-owned
property and the other self-occupied
property cannot exceed ₹30,000/
₹2,00,000, as the case may be.

Note: As mentioned above, no interest


deduction in respect of self-occupied

102 | P a g e #HakkसेCA by CA. Akash Sir


103
3. Income from house property

property would be allowable to the co-


owners under the default tax regime.

Illustration: 17 [ICAI Mat]

Mr. Ankit is a co-owner of a house property along-with his brother.


Municipal value of the Property 1,60,000
Fair Rent 1,50,000
Standard Rent under the Rent Control Act 1,70,000
Rent received 15,000 p.m.
The loan for the construction of this property is jointly taken and the interest charged
by the bank is ₹25,000 out of which ₹21,000 have been paid. Interest on the unpaid
interest is ₹450. To repay this loan, Mr. Ankit and his brother have taken a fresh loan
and interest charged on this loan is ₹5,000.
The Municipal taxes of ₹5,100 have been paid by the tenant.
Mr. Ankit has 50% share in the house property.
Mr. Ankit has income from other sources ₹2,60,000.
Compute the total income chargeable in the hands of Mr. Ankit.

Solution:

Computation of income from house property of Mr. Ankit for A.Y. 2024-25

Gross Annual Value 1,80,000


Working Note:
(a) Municipal value of property 1,60,000
(b) Fair rent 1,50,000
(c) Higher of (a) and (b) 1,60,000
(d) Standard rent 1,70,000
(e) Annual Letting Value / Expected Rent [lower of (c) and (d)] 1,60,000
(f) Actual rent [15,000 x 12] 1,80,000
(g) Gross Annual Value [higher of (e) and (f)] 1,80,000
Less: Municipal taxes – paid by the tenant, hence not deductible Nil
Net Annual Value (NAV) 1,80,000
Less: Standard deduction 30% of NAV u/s 24(a) (54,000)
Less: Interest on housing loan u/s 24(b)-
- Interest on loan taken from bank (25,000)
- Interest on fresh loan to repay old loan for this property (5,000)
Income under the head house property 96,000
50% share taxable in the hands of Mr. Ankit 48,000
Income under the head House Property 48,000

103 | P a g e #HakkसेCA by CA. Akash Sir


104
3. Income from house property

Income under the head Other Sources 2,60,000


Gross Total Income 3,08,000
Less: Deduction u/s 80C to 80U Nil
Total Income 3,08,000

HW Question: 17 [ICAI Mat]

Two brothers Anshul and Vimal are the co-owners of a house property with equal share.
The property was constructed during the financial year 1998-1999. The property
consists of 8 identical units and is situated at Cochin.
During the financial year 2023-24, each co-owner occupied 1 unit for residence and the
balance of 6 units were let out at a rent of ₹12,000 per month per unit. The municipal
value of the house property is ₹9,00,000 and the municipal taxes are 20% of municipal
value, which were paid during the year. The other expenses were as follows:
(i) Repairs ₹40,000
(ii) Insurance premium (paid) ₹15,000
(iii) Interest payable on loan taken for construction of house ₹3,00,000
One of the let-out units remained vacant for four months during the year. Anshul could
not occupy his unit for six months as he was transferred to Chennai. He does not own
any other house.
The other income of Mr. Anshul and Mr. Vimal are ₹2,90,000 and ₹1,80,000
respectively, for the financial year 2023-24.
Compute the income under the head ‘Income from House Property’ and the total income
of two brothers for the assessment year 2024-25 if they opt to shift out of default
tax regime.
Also, show the computation of income under this head, if they both exercised the
option under section 115BAC.

TP: 12 Sub-Letting of House Property

Sub-Letting covers a situation where a tenant (who has taken a house property on rent)
himself gives the same property on rent to another person.

Income in hands of original Income in hands of tenant who has sublet the
owner property
Income of the original owner Income of such person is generally computed u/h
would be computed u/h ‘House ‘income from other sources’. All expenses incurred
Property’ to earn such income are allowed as deduction.

104 | P a g e #HakkसेCA by CA. Akash Sir


105
3. Income from house property

TP: 13 Treatment of Composite Rent

Where a house property is let out by a person along with various facilities like
generator, security, air-conditioner, furniture, etc and a combined rent is charged by
the tenant towards the use of the house property and the facilities, such combined
rent is commonly known as composite rent.

Case i) Rent of house property & rent of other facilities is separable


• Rent of house property is taxable u/h house property.
• Rent of other facilities is taxable u/h PGBP or other sources.

Case ii) Rent of house property & rent of other facilities is not separable
• Rent is taxable u/h PGBP if assessee is engaged in the business of renting (e.g.
Hotels, PGs, auditoriums, etc)
• Rent is taxable u/h other sources if assessee is not engaged in the business of
such renting.

Illustration: 18

Mr. Kamlesh has let out one house along with generator facility and has charged a sum
of ₹40,000 p.m. as rent, out of which ₹3,000 p.m. is attributable to the generator. He
has paid ₹2,300 and the tenant has paid ₹900 towards municipal taxes. The interest on
the capital borrowed for construction of the house is ₹7,000. Mr. Kamlesh has paid
repair charge of the generator ₹3,400, fuel charges ₹5,600 and operator’s salary ₹300
p.m. Compute the total income of Mr. Kamlesh for assessment year 2024-25.

Solution:

Computation of income under the head House Property for A.Y. 2024-25

Gross Annual Value (37,000 x 12) 4,44,000.00


Less: Municipal Taxes (2,300.00)
Net Annual Value 4,41,700.00
Less: 30% of NAV u/s 24(a) (1,32,510.00)
Less: Interest on capital borrowed u/s 24(b) (7,000.00)
Income under the head House Property 3,02,190.00

Computation of income under the head Other Sources


Income from generator (3,000 x 12) 36,000.00
Less: Repair charges (3,400.00)
Less: Fuel charges (5,600.00)
Less: Operator Salary (300 x 12) (3,600.00)

105 | P a g e #HakkसेCA by CA. Akash Sir


106
3. Income from house property

Income under the head Other Sources 23,400.00

Computation of Total Income


Income under the head House Property 3,02,190.00
Income under the head Other Sources 23,400.00
Gross Total Income 3,25,590.00
Less: Deduction u/s 80C to 80U Nil
Total Income 3,25,590.00

TP: 14 Deemed Ownership [Section 27]

As per section 27, the following persons, though not legal owners of a property, are
deemed to be the owners for the purposes of section 22 to 26.

(i) Transfer to a spouse [Section 27(i)] – In case of transfer of house property by


an individual to his or her spouse otherwise than for adequate consideration, the
transferor is deemed to be the owner of the transferred property.
Exception – In case of transfer to spouse in connection with an agreement to live
apart, the transferor will not be deemed to be the owner. The transferee will be the
owner of the house property.

(ii) Transfer to a minor child [Section 27(i)] – In case of transfer of house


property by an individual to his or her minor child otherwise than for adequate
consideration, the transferor would be deemed to be owner of the house property
transferred.
Exception – In case of transfer to a minor married daughter, the transferor is not
deemed to be the owner.

Note - Where cash is transferred to spouse/minor child and the transferee acquires
property out of such cash, then, the transferor shall not be treated as deemed
owner of the property. However, clubbing provisions will be attracted.

(iii) Holder of an impartible estate [Section 27(ii)] – The impartible estate is a


property which is not legally divisible. The holder of an impartible estate shall be
deemed to be the individual owner of all properties comprised in the estate.
After enactment of the Hindu Succession Act, 1956, all the properties comprised in
an impartible estate by custom is to be assessed in the status of a HUF. However,
section 27(ii) will continue to be applicable in relation to impartible estates by grant
or covenant.

(iv) Member of a co-operative society etc. [Section 27(iii)] – A member of a co-


operative society, company or other association of persons to whom a building or part
thereof is allotted or leased under a House Building Scheme of a society/ company/
association, shall be deemed to be owner of that building or part thereof allotted to

106 | P a g e #HakkसेCA by CA. Akash Sir


107
3. Income from house property

him although the co-operative society/ company/ association is the legal owner of
that building.

(v) Person in possession of a property [Section 27(iiia)] – A person who is allowed


to take or retain the possession of any building or part thereof in part performance
of a contract of the nature referred to in section 53A of the Transfer of Property
Act shall be the deemed owner of that house property. This would include cases
where the –
(1) possession of property has been handed over to the buyer
(2) sale consideration has been paid or promised to be paid to the seller by the buyer
(3) sale deed has not been executed in favour of the buyer, although certain other
documents like power of attorney/ agreement to sell/ will etc. have been executed.

In all the above cases, the buyer would be deemed to be the owner of the property
although it is not registered in his name.

(vi) Person having right in a property for a period not less than 12 years
[Section 27(iiib)] – A person who acquires any rights in or with respect to any
building or part thereof, by virtue of any transaction as is referred to in section
269UA(f) i.e. transfer by way of lease for not less than 12 years, shall be deemed to
be the owner of that building or part thereof.
Exception – In case the person acquiring any rights by way of lease from month to
month or for a period not exceeding one year, such person will not be deemed to be
the owner.

TP: 15 Owner in case of Disputed Ownership [Section 22]

• An owner of a house property is liable to pay income tax in respect of income


earned by him from such house property.
• If the title of ownership is under dispute in a court of law, the decision about
who is the real owner lies with the court.
• In case where the decision of the court is pending, the power to decide who is
the owner of the house property in the meanwhile lies with the Income Tax
Department. Generally, the beneficial owner of such house property is
considered to be the owner and thus liable to pay tax in respect of income from
such property.

107 | P a g e #HakkसेCA by CA. Akash Sir


108
3. Income from house property

Comprehensive Questions
Question: 1 [IFHP]

Mr. Vijendra has constructed one house on 01.09.2023 and it was let out @ ₹1,25,000
p.m. and municipal taxes paid are ₹35,000. The house was constructed after taking a
loan from outside India and interest allowed under section 24(b) is ₹2,10,000, but the
assessee has not deducted tax at source. Compute Assessee’s tax liability for
assessment year 2024-25.

(b) Presume in the above question, the person who has given the loan has one agent in
India as per section 163. Compute tax liability for the assessment year 2024-25.

Question: 2 [IFHP] [ICAI Mat]

Mrs. Verma, a citizen of the U.S.A., is a resident and ordinarily resident in India during
the financial year 2023-24. She owns a house property at Los Angeles, U.S.A., which is
used as her residence. The annual value of the house is $20,000. The value of one USD
($) may be taken as ₹ 65. She took ownership and possession of a flat in Chennai on
1.7.2023, which is used for self-occupation, while she is in India. The flat was used by
her for 7 months only during the year ended 31.3.2024. The municipal valuation is
₹32,000 p.m. and the fair rent is ₹4,20,000 p.a. She paid the following to Corporation
of Chennai:
Property Tax ₹16,200
Sewerage Tax ₹1,800
She had taken a loan from Standard Chartered Bank in June, 2021 for purchasing this
flat. Interest on loan was as under:
Period prior to 1.4.2023 ₹49,200
1.4.2023 to 30.6.2023 ₹50,800
1.7.2023 to 31.3.2024 ₹1,31,300
Certificate confirming the amount of Interest has been deposited. She had a house
property in Bangalore, which was sold in March, 2019. In respect of this house, she
received arrears of rent of ₹60,000 in March, 2024. This amount has not been charged
to tax earlier. Compute the income chargeable from house property of Mrs. Verma for
the assessment year 2024-25 assuming that she has exercised the option of shifting
out of the default tax regime u/s. 115BAC(1A).

Would your answer change if she pays tax under the default tax regime under section
115BAC?

Question: 3 [IFHP]

Mr. Vidhit has let out one house property @ ₹70,000 per month and there is unrealised
Rent of 2 months and there is vacancy of 3 month. Fair rent ₹60,000 per month,

108 | P a g e #HakkसेCA by CA. Akash Sir


109
3. Income from house property

municipal valuation ₹55,000 per month and standard rent ₹80,000 per month. Municipal
tax paid ₹62,000. Interest on loan for construction of the house property is
₹75,000.The assessee has unrealised Rent of ₹2,00,000 in P.Y. 2021-22 and he has
recovered ₹1,50,000 in P.Y. 2023-24 and interest of ₹18,000 and he has incurred
₹11,000 as legal expense. Compute his tax liability for assessment year 2024-25
assuming that he shifts out of default tax regime.

Question: 4 [IFHP]

Mr. Rajeev has let out one house @ ₹45,000 p.m., but this house was vacated on
01.11.2023. The house was self-occupied w.e.f. 01.01.2024. Fair rent of this house is
₹50,000 p.m., municipal valuation is ₹47,000 p.m. and standard rent is ₹48,000 p.m. The
assessee has paid municipal taxes @ 10% of municipal valuation. Interest on capital
borrowed is ₹42,000. Land revenue paid by the assessee is ₹11,000 and ground rent
paid by him is ₹3,000. The assessee has taken a loan for payment of municipal tax and
interest paid on loan is ₹500. Compute his income under the head house property and
tax liability for assessment year 2024-25 assuming that he opts for default tax
regime.

Question: 5 [IFHP]

Mr. Raju and Mr. Ramu constructed their houses on a piece of land purchased by them
at New Delhi. The built-up area of each house was 1,000 sq. ft. ground floor and an
equal area at the first floor. Mr. Raju started construction of the house on 01.04.2022
and completed it on 31.03.2023. Mr. Raju occupied the entire house on 01.04.2023. Mr.
Raju has availed a housing loan of ₹25 lakhs @ 12% p.a. on 01.04.2022 and has also
submitted a certificate from the lender certifying the amount of interest. Mr. Ramu
started construction on 01.04.2022 and completed it on 30.06.2023. Mr. Ramu occupied
the ground floor on 01.07.2023 and let out the first floor for a rent of ₹20,000 per
month. However, the tenant vacated the house on 31.12.2023 and Mr. Ramu occupied
the entire house during the period 01.01.2024 to 31.03.2024. Mr. Ramu has availed a
housing loan of ₹15 lakhs @ 10% p.a. on 01.07.2022 and has also submitted a certificate
from the lender certifying the amount of interest. Following are the other information:
(i) Fair rental value of each unit 1,20,000 Per annum (Ground floor / first floor)
(ii) Municipal value of each unit 92,000 Per annum (Ground floor / first floor)
(iii) Municipal taxes paid by Raju - 10,000, Ramu - 10,000
(iv) Repair and maintenance charges paid by Raju - 30,000, Ramu - 32,000
No repayment was made by either of them till 31.03.2024. Compute income from house
property for Mr. Raju and Mr. Ramu for the previous year 2023-24 (assessment year
2024-25) under regular provisions of the Act.

109 | P a g e #HakkसेCA by CA. Akash Sir


110
3. Income from house property

Question: 6 [IFHP]

Mr. Saurabh has a house property situated in Mumbai which has two units. Unit I has a
floor area of 70% whereas the unit II has a floor area of 30%. Both the units were
self-occupied by the assessee. As the assessee was allowed a rent free accommodation
by his employer w.e.f. 01.04.2023, he vacated both of the units and let out unit I at a
rent of ₹13,000 p.m. and unit II for ₹5,000 p.m. Unit I remained vacant for 1½ months
whereas unit II was vacant for one month. Other particulars of the house property are
as under:
Municipal Valuation 1,55,000
Fair Rent 1,75,000
Standard Rent 1,65,000
Municipal taxes paid 35,000
Ground rent due 15,000
Compute income from house property for the assessment year 2024-25.

Question: 7 [IFHP] NOV – 2015 Question 6(a). (8 Marks)

Mr. Anand constructed a shopping complex. He had taken a loan of ₹25 lakhs for
construction of the said property on 01.08.2021 from SBI @ 10% for 5 years. The
construction was completed on 30.06.2022. Rental income received from shopping
complex ₹30,000 per month let out for the whole year. Municipal Taxes paid for
shopping complex ₹8,000. Arrears of rent received from shopping complex ₹1,20,000.
Interest paid on loan taken from SBI for purchase of house for use as own residence
for the period 2023- 2024 of ₹3 lakhs. The loan was taken after 01.04.1999 and house
was purchased within 5 years from the end of the year in which loan was taken and
assessee has submitted certificate certifying the amount of interest. You are required
to compute Income from House property of Mr. Anand for A.Y. 2024-2025 as per
Income Tax Act, 1961 under regular provisions of the Act.

Question: 8 [IFHP, Capital Gain]

Mr. Mukesh constructed one house in 2022 and it is let out for 4 months and self-
occupied for 6 months and vacant for 2 months during previous year 2023-24. Municipal
valuation of the house is ₹40,000 p.m. and fair rent ₹30,000 p.m. Standard rent of the
house is ₹38,000 p.m. It was let out @ ₹32,000 p.m. Municipal tax levied is ₹6,000 out
of which ₹2,000 was paid by the tenant and ₹2,000 by the assessee and balance ₹2,000
yet to be paid. Interest on the capital borrowed for construction of the house is
₹30,000. Long Term Capital Gains is ₹2,10,000 Compute his income and tax Liability for
the assessment year 2024-25 under regular provisions of the Act.

110 | P a g e #HakkसेCA by CA. Akash Sir


111
3. Income from house property

Question: 9 [IFHP, PGBP]

Mr. Umesh has one big house. 25% of it is being used by the assessee in his own
business/ profession and 50% of the house is let out @ ₹10,000 p.m. However, it
remained vacant for one month and there is unrealised rent for 1½ month. Remaining
25% is self-occupied throughout the year. Fair rent of the entire house is ₹25,000
p.m., municipal valuation ₹22,000 p.m. and municipal tax paid is ₹22,000. Insurance
premium paid is ₹6,000, repair charges ₹8,000, land revenue paid ₹4,000, ground rent
is ₹3,000 and depreciation of the house is ₹12,000. Assessee’s income under the head
business/profession before charging expenditure relating to house property is
₹2,00,000. Compute his total income and tax liability for assessment year 2024-25
under regular provisions of the Act.

Question: 10 [IFHP, PGBP] JULY – 2021 (NEW COURSE) Question 3(a) (6 Marks)

Mr. Ramlal constructed a big house (construction completed in Previous Year 2013-
2014) with 3 independent units. Unit-1 (50% of floor area) is let out for residential
purpose at monthly rent of ₹15,000. A sum of 3,000 could not be collected from the
tenant and a notice to vacate the unit was given to the tenant. No other property of
Mr. Ramlal is occupied by the tenant. Unit- 1 remains vacant for 2 months when it is not
put to any use. Unit – 2 (25% of the floor area) is used by Mr. Ramlal for the purpose
of his business, while Unit – 3 (the remaining 25%) is utilized for the purpose of his
residence. Other particulars of the house are as follows:
Municipal valuation - ₹ 1,88,000,
fair rent - ₹ 2,48,000,
Standard rent under the Rent Control Act - ₹ 2,28,000,
Municipal taxes - ₹ 20,000,
repairs - ₹ 5,000,
Interest on capital borrowed for the construction of the property - ₹ 60,000,
Ground rent – 6,000 and
Fire the insurance premium paid - ₹60,000.
Income of Ramlal from the business is ₹ 1,40,000 (without debiting house rent and
other incidental expenditure).
Determine the taxable income of Mr. Ramlal for the assessment year 2024-25 if he
does not opt to be taxed under section 115BAC.

Question: 11 [ICAI Mat]

Mr. Avinas owns two house properties one at Bomaby, wherein his family resides and
the other at Delhi, which is unoccupied. He lives in Chandigarh for his employment
purposes in a rented house. For acquisition of house property at Bombay, he has taken a
loan of Rs 30 lakh @ 10% p.a. on 01.04.2022. He has not repaid any amount so far. In
respect of house property at Delhi, he has taken a loan of Rs. 5 lakhs @11% p.a. on

111 | P a g e #HakkसेCA by CA. Akash Sir


112
3. Income from house property

1.10.2022 towards repairs. Compute the deduction which would be available to him
under section 24(b) for A. Y. 2024-25 in respect of interest payable on such loan if he
exercises the option of shifting out of the default tax regime provided under section
115BAC(1A).

Question: 12 [ICAI Mat]

Shiv has three houses all of which are self–occupied. The particulars of the houses for
the P. Y. 2023-24 are as under-

Particulars House I House II House III


Municipal valuation p. a. Rs. 3,00,000 Rs. 3,60,000 Rs. 3,30,000
Fair rent p.a. Rs. 3,75,000 Rs. 2,75,000 Rs. 3,80,000
Standard rent p.a Rs. 3,50,000 Rs. 3,70,000 Rs. 3,75,000
Date of completion /purchase 31.3.2000 31.3.2002 01.4.2016
Municipal taxes paid during the year 12% 8% 6%
Interest on money borrowed for
repair of property during the
current year Rs. 55,000
Interest for current year on money
borrowed in April 2016 for purchase
of property Rs. 1,75,000

Compute Shiv’s income from house property for A. Y. 2024-25 and suggest which
houses should be opted by Shiv to be assessed as self-occupied so that his tax liability
is minimum.

Question: 13 [ICAI Mat]

Ms. Anjali co-owns a residential house property in Calcutta along with her sister Ms.
Charu, where her sister’s family resides. Both of them have equal share in the property
and the same is used by them for self- occupation. Interest is payable in respect of
loan of Rs. 50,00,000 @ 10% taken on 01.04.2022 for acquisition of such property. In
addition, Ms. Anjali owns a flat in Pune in which she and her parents reside. She has
taken a loan of Rs. 3,00,000 @ 12% on 01.10.2022 for repairs of this flat. Compute the
deduction which would be available to Ms. Anjali and Ms. Charu under section 24(b) for
A. Y. 2024-25, if both exercise the option of shifting out of the default tax regime
provided under section 115BAC(1A).

Question: 14 [ICAI Mat]

Mr. Vikas owns a house property whose Municipal Value, Fair Rent and Standard Rent
are Rs. 96,000, Rs. 1,26,000 and Rs. 1,08,000 (per annum), respectively. During the F. Y.

112 | P a g e #HakkसेCA by CA. Akash Sir


113
3. Income from house property

2023-24, one–third of the portion of the house was let out for residential purpose at a
monthly rent of Rs. 5,000. The remaining two–third portion was self-occupied by him.
Municipal tax @ 11% of municipal value was paid during the year.
The construction of the house began in June, 2016 and was completed on 31.05.2019.
Vikas took a loan of Rs. 1,00,000 on 01.07.2016 for the construction of building. He paid
interest on loan @12% per annum and every month such interest was paid.
Compute income from house property of Mr. Vikas for the A. Y. 2024-25 if he has
exercised the option of shifting out of the default tax regime provided under section
115BAC(1A).

113 | P a g e #HakkसेCA by CA. Akash Sir


114
4. Income under head Capital Gains

Income under head Capital Gains [Section


45 to 55]
TP: 1 Charging section [section 45]

Section 45 provides that any profits or gains arising from the Transfer of a Capital
Asset effected in the previous year will be chargeable to income tax under the head
‘Capital Gains’. Such capital gains will be deemed to be the income of the Previous
Year in which the transfer took place.

Mann Ki Baat (simple language):

Please note that the entire chapter is based on this charging section which includes 3
important concepts. First is the definition of capital asset, second is the definition of
transfer and third is year of taxability. Now, let’s understand 3 concepts one by one.

TP: 2 Meaning of Capital Asset [Section 2(14)]

According to section 2(14), a capital asset means –

(a) property of any kind held by an assessee, whether or not connected with his
business or profession;
(b) any securities held by a Foreign Institutional Investor (FII);
(c) any unit linked insurance policy (ULIP) issued on or after 01.02.2021, to which
exemption under section 10(10D) does not apply on account of aggregate premium
payable exceeding ₹2,50,000 for any of the previous years during the term of such
policy.

However, it does NOT include—

(i) Stock-in trade: Any stock-in-trade [other than securities referred to in (b)
above], consumable stores or raw materials held for the purpose of the business or
profession of the assessee.
For example: Cars held by a person having business of trading of cars will not be
treated as capital asset since car is stock in trade for that person.

(ii) Personal effects: Personal effects, that is to say, movable property (including
wearing apparel and furniture) held for personal use by the assessee or any member
of his family dependent on him.
For example: Car held by a person for his personal use is NOT a capital asset.

114 | P a g e #HakkसेCA by CA. Akash Sir


115
4. Income under head Capital Gains

EXCLUSIONS from above (means considered as capital asset): (a)


jewellery; (b) archaeological collections; (c) drawings; (d) paintings; (e)
sculptures; or (f) any work of art.
For example: Jewellery held by a person shall be capital asset even if it is
held for personal purpose.

(iii) Rural agricultural land in INDIA i.e., agricultural land in India which is not situated
in urban area.

Urban area shall mean


(a) agricultural land situated in any area within the jurisdiction of a municipality or
cantonment board having population of 10,000 or more,
(b) agricultural land situated in any area within such distance, measured aerially, in
relation to the range of population as shown hereunder –

Population according to the last preceding Shortest aerial distance from


census of which the relevant figures have the local distance of a
been published before the first day of the municipality or cantonment
preceding year referred to in item (a)
i >10,000 but <1,00,000 ≤ 2 kms
ii >1,00,000 but <10,00,000 ≤ 6 kms
iii >10,00,000 ≤ 8 kms

For example:

City Population Land situated at arial Rural or Capital Asset or


distance from city Urban? not?
Delhi >10,00,000 7 km Urban Capital Asset
Chennai >10,00,000 9 km Rural Not a capital asset
Sikar >1,00,000 but < 1 km Urban Capital Asset
10,00,000
Singhana > 10,000 but < 3 km Rural Not a capital asset
1,00,000
Khetri < 10,000 0 km Rural Not a capital asset
Chirawa >10,000 but < 2 km Urban Capital asset
1,00,000

Note: The capital gains arising from the transfer of such urban agricultural lands
would not be treated as agricultural income for the purpose of exemption under
section 10(1). Hence, such gains would be subject to tax under section 45.

115 | P a g e #HakkसेCA by CA. Akash Sir


116
4. Income under head Capital Gains

(iv) Specified Gold Bonds: 6½% Gold Bonds, 1977, or 7% Gold Bonds, 1980, or
National Defence Gold Bonds, 1980, issued by the Central Government;

(v) Special Bearer Bonds, 1991 issued by the Central Government;

(vi) Gold Deposit Bonds issued under the Gold Deposit Scheme, 1999 or deposit
certificates issued under the Gold Monetisation Scheme, 2015 and Gold Monetisation
Scheme, 2018 notified by the Central Government.

Mann Ki Baat (Simple Language):

From the above discussion, it is clear that not every property is a capital asset. Now
you have to just remember that stock in trade, personal movable effects (other than
few assets including jewellery) and rural agriculture land are not capital assets and
therefore, no capital gain shall arise on their transfer.

Illustration: 1

Mr. Anshul, aged 40 years sold an agricultural land for ₹56 lakhs on 14.11.2023 acquired
at a cost of ₹49 lakhs on 23.10.2022 situated at 8 kms from the jurisdiction of
municipality having population of 4,00,000 and also sold another agricultural land for

116 | P a g e #HakkसेCA by CA. Akash Sir


117
4. Income under head Capital Gains

₹63 lakhs on 22.01.2024 acquired at a cost of ₹49 lakhs on 15.03.2022 situated at 1


kms from the jurisdiction of municipality having population of 11,000. Whether the
agricultural land of Mr. Anshul will be considered as capital asset in the assessment
year 2024-25?

Solution:

As per the definition of capital asset, a land shall be considered to be urban land if it is
situated within 2 kms from the limits of urban area having population more than 10,000
but not exceeding 1,00,000 and within 6 kms from the limits of urban area having
population more than 1,00,000 but not exceeding 10,00,000. In the given case, first
agricultural land is situated after 6 kms having population of 4,00,000 hence such land
is considered as rural agricultural land and second land is situated within 2 kms from
the municipality having population of 11,000 hence such land is considered as
agricultural land in urban area and considered as capital asset.

HW Question: 1

Discuss whether the following are capital assets or not:


(i) Household furniture
(ii) Personal Motor car
(iii) Residential house
(iv) Urban land
(v) Agricultural land in rural area in India
(vi) Agricultural land in rural area in Nepal
(vii) Stock in trade
(viii) Gold ornaments
(ix) Music system for personal use
(x) Music system in business use
(xi) Motor car in business use
(xii) Plant and machinery in business use
(xiii) Silver utensils for personal use
(xiv) Precious stones in personal use

TP: 3 Short-term Capital Asset [Section 2(42A)]

As per section 2(42A), Short term capital asset means a capital asset held by the
assessee for not more than “36 months” immediately preceding to date of its
transfer.

117 | P a g e #HakkसेCA by CA. Akash Sir


118
4. Income under head Capital Gains

Replace “36 months” with “12 months” Replace “36 months” with “24 months”
1. A security (other than a unit) listed in 1. A share of a company (not being a
a recognized stock exchange (other share listed in a recognised stock
than market linked debentures and exchange in India)
units of specified mutual fund)
2. A unit of an equity-oriented fund 2. An immovable property being land or
building or both
3. Unit of a Union Trust of India
4. A Zero Coupon Bond

For example:

Nature of capital asset Period of holding Short term or long term?


Machinery 25 months Short term
Shares of Reliance Ltd. 13 months Long term
Shares of Bansal Pvt. Ltd. 18 months Short term
Shares of Bansal Pvt. Ltd. 26 months Long term
Furniture 48 months Long term
Debentures of Adani Ltd. 19 months Long term
Debentures of Bansal Pvt. Ltd. 28 months Short term
Debt oriented mutual funds 25 months Short term
Equity oriented mutual funds 13 months Long term
Building 25 months Long term

118 | P a g e #HakkसेCA by CA. Akash Sir


119
4. Income under head Capital Gains

Mann Ki Baat (simple language):

From above, please note that there are 3 limits (36, 24 and 12 months) to classify any
asset under short term capital asset. You have to remember assets which comes under
24 months and 12 months category and rest all assets will fall under 36 months.

TP: 4 Long-term Capital Asset [Section2(29A)]

As per section 2(29A), long term capital asset means capital asset which is not a
short-term capital asset.

Mann Ki Baat (simple language):

You need not to remember anything from perspective of long-term capital asset. All
those assets which do not satisfy the definition of section 2(42A) will be considered as
long-term capital assets.

TP: 5 Meaning of Transfer [section 2(47)]

The Act contains an inclusive definition of the term ‘transfer’. Accordingly, transfer
in relation to a capital asset includer the following type of transactions─
i. The sale, exchange or relinquishment of the asset; or
ii. The extinguishment of any right therein; or
iii. The compulsory acquisition thereof under any law; or
iv. The owner of a capital asset may convert the same into the stock-in-trade of
a business carried on by him. Such conversion is treated as transfer, or
v. The maturity or redemption of a zero-coupon bond; or
vi. Part-time performance of the contract: Sometimes, possession of an
immovable property is given in consideration of part- performance of a
contract.
vii. Lastly, there are certain types of transactions which have the efffect of
transferring or enabling the enjoyment of an immovable property.

Example: Miss Asha enters into an agreement for the sale of her house. The
purchaser gives the entire sale consideration to Miss Asha. Miss Asha hands over
complete rights of possession to the purchaser since she has realised the enire sale
consideration, however, sales deed is not yet executed. Under Inncome-tax Act, 1961
the above transaction is considered as transfer.

119 | P a g e #HakkसेCA by CA. Akash Sir


120
4. Income under head Capital Gains

Mann Ki Baat (simple language):

From reading of above definition, it is very clear that the meaning of transfer is very
vide in capital gain chapter. For exam purpose, we can learn that everything which is
given in the question is transfer and do not pay attention of the above definition.

TP: 6 Rates of tax

1. Short term capital gain taxable under section 111A

➢ Short term capital gain arising on transfer of -


o Listed equity shares (means period of holding (POH) <= 12 months)
o Units of equity oriented mutual funds (means POH <= 12 months)
o Units of business trust (means POH <= 36 months)
Provided Securities transaction tax (STT) has been paid on transfer of above
assets.

➢ Rate of Tax – Flat 15%

➢ Benefit of unexhausted basic exemption limit is available to RESIDENT. It


means that if normal income is less than basic exemption limit (i.e. 2.5 lakhs, 3
lakhs, 5 lakhs), then difference is reduced from short term capital gains. The
benefit is NOT available to NON-RESIDENT.

For example, if a resident individual (aged 40 years) has normal income of


₹1,80,000 and STCG u/s. 111A of ₹4,00,000, then, in this case, the
unexhausted basic exemption limit of ₹70,000 (2,50,000-1,80,000) shall be
allowed to deducted from STCG and remaining STCG of ₹3,30,000 shall be
taxed @15%.

➢ Deductions under chapter VIA are NOT available from STCG taxable u/s.
111A.

➢ Rebate u/s. 87A is allowed.

Illustration: 2

Mr. Omkar (aged 42 years) has incomes as given below:


• Income under the head salary - 35,000
• Income under the head house property - 45,000
• Income under the head business/profession - 30,000
120 | P a g e #HakkसेCA by CA. Akash Sir
121
4. Income under head Capital Gains

• Short term capital gains u/s 111A - 7,00,000


• Other income - 3,000
Compute his tax liability for the assessment year 2024-25 assuming that he shifts out
of default tax regime.

Solution:

Computation of Total Income

Income under the head salary 2,35,000


Income under the head house property 45,000
Income under the head business/profession 30,000
Short term capital gains u/s 111A 7,00,000
Other income 3,000
Gross Total Income/ Total Income 10,13,000

Computation of Tax Liability

Tax on Short term capital gains ₹7,00,000 @ 15% u/s 111A 1,05,000
Tax on normal income 3,13,000 @ slab rate 3,150
Tax before health & education cess 1,08,150
Add: HEC @ 4% 4,326
Tax Liability 1,12,476
Rounded off u/s. 288B 1,12,480

(b) Presume he is resident and is aged 62 years.

Computation of Tax Liability

Tax on Short term capital gains ₹7,00,000 @ 15% u/s 111A 1,05,000
Tax on normal income 3,13,000 @ slab rate 650
Tax before health & education cess 1,05,650
Add: HEC @ 4% 4,226
Tax Liability 1,09,876
Rounded off u/s. 288B 1,09,880

(c) Presume he is resident and is aged 82 years.

Computation of Tax Liability

Tax on Short term capital gains (₹7,00,000-₹1,87,000) @ 15% u/s 111A 76,950
Tax on normal income 3,13,000 @ slab rate Nil

121 | P a g e #HakkसेCA by CA. Akash Sir


122
4. Income under head Capital Gains

Tax before health & education cess 76,950


Add: HEC @ 4% 3,078
Tax Liability 80,028
Rounded off u/s. 288B 80,030

HW Question: 2

Compute tax liability for the assessment year 2024-25 in the following situations under
normal provisions of the Act:
(i) Mr. Rohan (aged 45 years) is resident in India and has income under the head house
property of ₹40,000 and income under the head salary of ₹30,000 and short term
capital gains from sale of listed equity shares of ₹4,80,000 (STT Paid).
(ii) Presume in the above situation the assessee is Mrs. Rohan.
(iii) Presume in the above situation the assessee is Mrs. Rohan and she is aged about 70
years.
(iv) Presume in the above situation the assessee is Mr. Rohan and he is aged about 70
years.
(v) Presume in the above situation the assessee is Mrs. Rohan and she is aged about 85
years.
(vi) Presume in the above situation the assessee is Mr. Rohan and he is aged about 85
years.

Mann Ki Baat (simple language):

From above, one should notice the the capital assets which are short term for a period
of 12 months are different from short term capital assets taxable under section 111A.
For example, units of buisness trust is short term for a period of 36 months and the
tax rate on such short term capital gains is 15%. Similarly, units of UTI is short term
for a period of 12 months however, the tax rate for such short term capital gain is not
15%.

Learn both the definition of short term capital asset and section 111A separatly
without mixing it.

2. Other short term capital gains

➢ Short term capital gain arising on transfer of other capital assets e.g. –
o Land, building (means POH <= 24 months)
o Unlisted shares (means POH <= 24 months)
o Debt oriented mutual fund (means POH <= 36 months)
o Units of unit trust (means POH <= 12 months)
o Listed debentures, bonds etc. (means POH <= 12 months)

122 | P a g e #HakkसेCA by CA. Akash Sir


123
4. Income under head Capital Gains

o Unlisted debentues, bonds etc. (means POH <= 36 months)


o Market linked debentures and units of specified mutual fund (POH does
not matter)
o Etc.

➢ Rate of tax – Normal rates (no specific section)

➢ There is no question of unexhausted basic exemption limit since STCG is itself


treated as normal income.

➢ Deductions under chpater VIA are available from STCG since it is normal
income.

➢ Rebate u/s. 87A is allowed.

Illustration: 3

Compute tax liability in following cases under normal provisions of the Act:

(i) Mr. Harsha (resident aged 23 years) having PGBP income of ₹4,00,000, Salary of
₹5,00,000, Short term capital gain on sale of unlisted shares of ₹5,00,000, Short term
capital gain of sale of equity oriented mutual fund (STT paid) of ₹8,00,000. [Ans:
3,66,600]

(ii) Mr. Druv (resident aged 62 years) having Salary of ₹12,00,000, STCG u/s. 111A of
₹4,00,000, STCG of ₹3,00,000 and other income of ₹2,00,000 [Ans: 3,95,200]

(iii) Miss Dhawani (non-resident aged 82 years) having Salary of ₹1,00,000, STCG of
₹1,00,000 and STCG u/s. 111A of ₹2,90,000. [Ans: 45,240]

(iv) Mr. Ritik (resident aged 82 years) having PGBP income of ₹3,00,000, STCG of
₹50,000 and STCG u/s. 111A of ₹4,00,000. [Ans: 39,000]

(v) Mr. Kulkarni (resident aged 19 years) having PGBP of ₹20,00,000, STCG u/s. 111A of
₹25,00,000 and STCG (normal) of ₹15,00,000. [Ans: 14,15,700]

Mann Ki Baat (simple language):

From above, we can conclude that there is no specific tax rate for other short term
capital gains and it is included in normal income and taxed at slab rate in normal way.
Also remember that definition of short term capital asset and short term capital gain

123 | P a g e #HakkसेCA by CA. Akash Sir


124
4. Income under head Capital Gains

tax rates are different. For example, listed debentures are short term for a period of
12 months however, short term capital gain on transfer is taxed at slab rate.

3. Long term capital gains taxable under section 112A

➢ Long term capital gain arising on transfer of –


o Listed equity shares (means POH > 12 months)
(Provided that STT is paid at the time of purchase and sale)
o Units of equity oriented mutual fund (means POH > 12 months)
(Provided that STT is paid at the time of transfer)
o Units of business trust (means POH > 36 months)
(Provided that STT is paid at the time of transfer)

➢ Rate of Tax

First 1,00,000 Nil


Above 1,00,000 10%

➢ Benefit of unexhausted basic exemption limit is available to RESIDENT. It


means that if normal income is less than basic exemption limit (i.e. 2.5 lakhs, 3
lakhs, 5 lakhs), then difference is reduced from long term capital gains. The
benefit is NOT available to NON-RESIDENT.

➢ Benfit of indexation is NOT available.

➢ Deductions under chpater VIA are NOT available from LTCG taxable u/s.
112A.

➢ Rebate u/s. 87A is NOT allowed.

Illustration: 4

Compute tax liability in following cases assuming that the assessee shifts out of
default tax regime:

(i) Mr. Vikram (resident aged 23 years) having PGBP income of ₹4,00,000, Salary of
₹5,00,000, Short term capital gain on sale of unlisted shares of ₹5,00,000, Short term
capital gain of sale of equity oriented mutual fund (STT Paid) of ₹8,00,000 and long
term capital gain u/s. 112A of ₹7,00,000. [Ans: 4,29,000]

124 | P a g e #HakkसेCA by CA. Akash Sir


125
4. Income under head Capital Gains

(ii) Mr. Aman (resident aged 62 years) having Salary of ₹12,00,000, STCG u/s. 111A of
₹4,00,000, STCG of ₹3,00,000, long term capital gain on sale of equity oriented mutual
fund of ₹3,00,000 and other income of ₹2,00,000. [Ans: 4,16,000]

(iii) Miss Shazia (non-resident aged 82 years) having Salary of ₹1,00,000, STCG of
₹1,00,000, LTCG u/s. 112A of ₹5,00,000 and STCG u/s. 111A of ₹2,90,000. [Ans:
86,840]

(iv) Mr. Sambhav (resident aged 82 years) having PGBP income of ₹3,00,000, STCG of
₹50,000, LTCG u/s. 112A of ₹50,000 and STCG u/s. 111A of ₹4,00,000. [Ans: 39,000]

(v) Mr. Jeet (resident aged 19 years) having PGBP of ₹20,00,000, STCG u/s. 111A of
₹25,00,000, LTCG u/s. 112A of 10,00,000 and STCG (normal) of ₹15,00,000. [Ans:
15,18,660]

(vi) Mr. Ranvijay (aged 59 years) has LTCG u/s. 112A of ₹50,50,000. [Ans: 5,35,600]

(vii) Mr. Kohli (aged 35 years) has LTCG u/s. 112A of ₹51,00,000. [Ans: 5,43,400]

HW Question: 3

Compute tax liability in following cases assuming that the assessee shifts out of
the default tax regime:
Case (i) Mr. Raman has income under the head house property of ₹5,00,000 and LTCG
u/s. 112A of ₹1,00,000.
Case (ii) Mr. Raman has LTCG 112A ₹50,50,000.
Case (iii) Mr. Raman has LTCG 112A ₹51,00,000.

Mann Ki Baat (simple language):

From above, we can notice that this section is very similar to section 111A. However,
one of the difference between these section is that under section 111A, a listed share
is covered even if STT is not paid at the time of purchase, however, under section
112A, STT has to be paid both at the time of purchase and transfer.

4. Long term capital gains taxable under section 112

➢ Long term capital gain arising on transfer of other capital assets e.g. –
o Land, building (means POH > 24 months)
o Unlisted shares (means POH > 24 months)
o Debt oriented mutual fund (means POH > 36 months)
o Units of unit trust (means POH > 12 months)

125 | P a g e #HakkसेCA by CA. Akash Sir


126
4. Income under head Capital Gains

o Listed debentures, bonds etc (means POH > 12 months)


o Unlisted debentues, bonds etc (means POH > 36 months)
o Etc.

➢ Rate of tax

Assessee Type of Asset Rate of tax


Resident Individual/ HUF/ Any LTCG 20% (with indexation)
Domestic Company/
Others
Non resident Individual/ Unlisted securities or 10% (without
HUF/ Foreign Company Shares of pvt ltd indexation)
Other LTCG 20% (with indexation)
Any person Any listed security 10% (without
(other than unit) or indexation)
Zero coupon bond

➢ Benefit of unexhausted basic exemption limit is available to RESIDENT. It


means that if normal income is less than basic exemption limit (2.5 lakhs, 3
lakhs, 5 lakhs), then difference is reduced from long term capital gains. The
benefit is NOT available to NON-RESIDENT.

➢ Deductions under chpater VIA are NOT available from LTCG taxable u/s. 112.

➢ Rebate u/s. 87A is allowed.

Illustration: 5

Compute tax liability in the following cases assuming that he shifts out of the
default tax regime:
(i) Mr. Abhishek a resident has long term capital gains ₹3,50,000.
(ii) Mr. Abhishek a resident has short term capital gains u/s 111A ₹3,50,000.
(iii) Mr. Abhishek a non-resident has long term capital gains ₹3,50,000.
(iv) Mr. Abhishek a non-resident has short term capital gains u/s 111A ₹3,50,000.
(v) Mr. Abhishek a non-resident aged 61 years has long term capital gains ₹3,50,000.
(vi) Mr. Abhishek a non-resident aged 61 years has short term capital gains u/s 111A
₹3,50,000.

Solution:

(i) Computation of Tax Liability


Tax on Long term capital gains ₹1,00,000

126 | P a g e #HakkसेCA by CA. Akash Sir


127
4. Income under head Capital Gains

(₹3,50,000 – ₹2,50,000)@ 20% u/s 112 20,000


Less: Rebate u/s 87A (12,500)
Tax before health & education cess 7,500
Add: HEC @ 4% 300
Tax Liability 7,800

(ii) Computation of Tax Liability


Tax on Short term capital gains ₹1,00,000
(₹3,50,000 – ₹2,50,000)@ 15% u/s 111A 15,000
Less: Rebate u/s 87A (12,500)
Tax before health & education cess 2,500
Add: HEC @ 4% 100
Tax Liability 2,600

(iii) Computation of Tax Liability


Tax on Long term capital gains ₹3,50,000 @ 20% u/s 112 70,000
Add: HEC @ 4% 2,800
Tax Liability 72,800

(iv) Computation of Tax Liability


Tax on Short term capital gains ₹3,50,000 @ 15% u/s 111A 52,500
Add: HEC @ 4% 2,100
Tax Liability 54,600

(v) Computation of Tax Liability


Tax on Long term capital gains ₹3,50,000 @ 20% u/s 112 70,000
Add: HEC @ 4% 2,800
Tax Liability 72,800

(vi) Computation of Tax Liability


Tax on Short term capital gains ₹3,50,000 @ 15% u/s 111A 52,500
Add: HEC @ 4% 2,100
Tax Liability 54,600

Illustration: 6

Compute tax liability of Mr. Kishan when he has following incomes under normal
provisions of the Act:
a) Income from business in chennai of ₹5,50,000
b) STCG on sale of land in Delhi of ₹8,00,000
c) STCG on sale of listed euiqty shares (STT paid at the time of transfer) of
₹6,00,000

127 | P a g e #HakkसेCA by CA. Akash Sir


128
4. Income under head Capital Gains

d) LTCG on sale of listed equity shares (STT paid only at the time of transfer) of
₹4,00,000
e) LTCG on sale of unlisted equity shares of ₹2,00,000
f) Total deductions under chapter VIA is ₹6,00,000
[Ans: ₹2,83,400]

HW Question: 4

Compute tax liability for the assessment year 2024-25 in the following situations
under old tax regime:
(i) Mr. Shyam is resident in India and has income under the head house property
₹40,000 and income under the head salary ₹30,000 and long term capital gains
₹4,80,000.
(ii) Presume in the above situation the assessee is Mrs. Shyam and she is aged about 70
years.
(iii) Presume in the above situation the assessee is Mrs. Shyam and she is aged about
85 years.
(iv) Presume in the above situation (iii), the assessee is non-resident in India.

Mann Ki Baat (simple language):

After reading entire TP 6 i.e. rates of tax, we understand that the whole capital gain
tax structure is divided into 4 parts STCG, STCG 111A, LTCG 112 and LTCG 112A. We
just need to learn STCG 111A and LTCG 112A, and the rest is understood.

TP: 7 Concept of enhanced surcharge

Enhanced surcharge of 25% & 37% have been withdrawan on tax payable in respect
of (i) Dividend Income, (ii) Capital Gains u/s. 111A, (iii) Capital Gains u/s. 112 and (iv)
Capital Gains u/s. 112A where total income of the Individual/ HUF/ AOP/ BOI/ AJP
exceeds 2 crores or 5 crores.

Mann Ki Baat (simple language):

In simple words, where total income includes dividend income or capital gains u/s. 111A,
112 or 112A and income exceeds 2 crores/ 5 crores, then surcharge rate of 25% and
37% shall NOT be applicable on above income. This means that if a person has earned
above incomes, then he shall not be liable to pay surcharge at higher rate of 25%/ 37%
on such income and the maximum rate of surcharge will be 15%. This provision has come
to reduce the burden of tax on persons who has capital gains/ dividend income.

128 | P a g e #HakkसेCA by CA. Akash Sir


129
4. Income under head Capital Gains

Refer the below table to understand the concept in a better way:

# Income Surcharge on tax on


Dividend income/ Other
capital gains u/s. Income
111A, 112 and 112A
1 Total Income (including dividend, capital gains Nil Nil
u/s. 111A, 112 and 112A) does not exceed
₹50,00,000
2. Total Income (including dividend, capital gains 10% 10%
u/s. 111A, 112 and 112A) exceeds ₹50,00,000 but
does not exceed ₹1,00,00,000
3. Total Income (including dividend, capital gains 15% 15%
u/s. 111A, 112 and 112A) exceeds ₹1,00,00,000
but does not exceed ₹2,00,00,000
4. Total Income (excluding dividend, capital gains 15% 25%
u/s. 111A, 112 and 112A) exceeds ₹2,00,00,000
but does not exceed ₹5,00,00,000
5. Total Income (excluding dividend, capital gains 15% 37%
u/s. 111A, 112 and 112A) exceeds ₹5,00,00,000
6. Total Income (including dividend, capital gains 15% 15%
u/s. 111A, 112 and 112A) exceeds ₹2,00,00,000
and case does not fall in above cases

Illustration: 7

Compute tax liability in following cases under old tax regime:

Case (i) Mr. Rohan has PGBP income of ₹20,00,000 and LTCG u/s. 112 of ₹25,00,000.
[Ans: 9,49,000]
Case (ii) Mr. Rohan has Salary income (computed) of ₹30,00,000, STCG u/s. 111A of
₹40,00,000 and LTCG u/s. 112A of ₹30,00,000. [Ans: 18,33,260]
Case (iii) Mr. Rohan has income under head House Property of ₹50,00,000, STCG of
₹40,00,000 and LTCG u/s. 112 of ₹60,00,000. [Ans: 44,40,150]
Case (iv) Mr. Rohan has other income of ₹1,50,00,000, PGBP income of ₹60,00,000 and
LTCG u/s. 112A of ₹30,00,000. [Ans: 82,93,090]

129 | P a g e #HakkसेCA by CA. Akash Sir


130
4. Income under head Capital Gains

Case (v) Mr. Rohan has PGBP income of ₹6,00,00,000 and STCG u/s. 111A of
₹10,00,000. [Ans: 2,55,58,650]
Case (vi) Mr. Rohan has Salary income of ₹1,40,00,000 and LTCG u/s. 112 of
₹70,00,000. [Ans: 64,73,350]

Illustration: 8

Mr. Rajpal has income as given below:


Income under the head PGBP 5,05,00,000
LTCG 112A 1,01,00,000
STCG 111A 1,00,00,000
Compute his tax liability for A.Y. 2024-25 assuming that he shifts out of default tax
regime.

Solution:

Computation of Total Income and Tax Liability of Mr. Rajpal


Income under the head PGBP 5,05,00,000
LTCG 112A 1,01,00,000
STCG 111A 1,00,00,000
Gross Total Income 7,06,00,000
Less: Deductions u/s 80C to 80U Nil
Total Income 7,06,00,000

Computation of Tax Liability


Tax on LTCG 100,00,000 (101,00,000 - 1,00,000) @ 10% u/s 112A 10,00,000
Tax on STCG 100,00,000 @ 15% u/s. 111A 15,00,000
Tax on 5,05,00,000 at slab rate 1,49,62,500
Add: Surcharge on PGBP Income (1,49,62,500) X 37% 55,36,125
Add: Surcharge @ 15% on 25,00,000 3,75,000
Less: Marginal Relief (14,83,000)
Working Note:
Tax + surcharge @ 37% on income of 5,05,00,000 204,98,625
Tax + surcharge @25% on income of 5,00,00,000 (185,15,625)
Increase in tax 19,83,000
Increase in income 5,00,000
Marginal Relief (₹19,83,000 – ₹5,00,000) 14,83,000
Tax Before cess 2,18,90,625
Add: Health and education cess @ 4% 8,75,625
Tax Liability 2,27,66,250

130 | P a g e #HakkसेCA by CA. Akash Sir


131
4. Income under head Capital Gains

Illustration: 9

Mr. Soham has income as given below:

Income under the head PGBP 3,00,00,000


LTCG 112A 51,00,000
STCG 111A 50,00,000
Dividend from domestic company 1,00,00,000
Compute his tax liability for A.Y. 2024-25 under normal provisions of the Act.

Solution:

Computation of Total Income and Tax Liability of Mr. Soham

Income under the head PGBP 3,00,00,000


LTCG 112A 51,00,000
STCG 111A 50,00,000
Dividend from domestic company 1,00,00,000
Gross Total Income 5,01,00,000
Less: Deductions u/s 80C to 80U Nil
Total Income 5,01,00,000

Option 1: Taking Dividend tax at slab rate for surcharge

Computation of Tax Liability

Tax on LTCG 50,00,000 (51,00,000-1,00,000) @ 10% u/s 112A 5,00,000


Tax on STCG 50,00,000 @15% u/s 111A 7,50,000
Tax on 4,00,00,000 at slab rate 1,18,12,500
Tax Before Surcharge 1,30,62,500
Add: Surcharge @ 15% on 12,50,000 1,87,500
Add: Surcharge on Dividend Income (slab basis) (28,12,500) X 15% 4,21,875
Add: Surcharge on PGBP Income
(3,00,00,000 x 30% = 90,00,000) X 25% 22,50,000
Tax Before cess 1,59,21,875
Add: Health and education cess @ 4% 6,36,875
Tax Liability 1,65,58,750

Option 2: Taking Dividend tax @ 30% for surcharge and PGBP income on slab rate

Computation of Tax Liability

131 | P a g e #HakkसेCA by CA. Akash Sir


132
4. Income under head Capital Gains

Tax on LTCG 50,00,000 (51,00,000-1,00,000) @ 10% u/s 112A 5,00,000


Tax on STCG 50,00,000 @15% u/s 111A 7,50,000
Tax on 4,00,00,000 at slab rate 118,12,500
Tax Before Surcharge 1,30,62,500
Add: Surcharge @ 15% on 12,50,000 1,87,500
Add: Surcharge on Dividend Income
(100,00,000 x 30% = 30,00,000) X 15% 4,50,000
Add: Surcharge on PGBP Income at slab (88,12,500) X 25% 22,03,125
Tax Before cess 1,59,03,125
Add: Health and education cess @ 4% 6,36,125
Tax Liability 1,65,39,250

Option 3: Taking Dividend tax on average basis for surcharge

Computation of Tax Liability

Tax on LTCG 50,00,000 (51,00,000-1,00,000) @ 10% u/s 112A 5,00,000


Tax on STCG 50,00,000 @15% u/s 111A 7,50,000
Tax on 4,00,00,000 at slab rate 1,18,12,500
Tax Before Surcharge 1,30,62,500
Add: Surcharge @ 15% on 12,50,000 1,87,500
Add: Surcharge on Dividend Income
(1,18,12,500/4,00,00,000 x 1,00,00,000 = 29,53,125) X 15% 4,42,968
Add: Surcharge on PGBP Income
(1,18,12,500/4,00,00,000 x 3,00,00,000 = 88,59,375) X 25% 22,14,843
Tax Before cess 159,07,811
Add: Health and education cess @ 4% 6,36,312
Tax Liability 1,65,44,123
Rounded off u/s 288B 1,65,44,120

HW Question: 5

Compute tax liability in following cases under old tax regime:


Case (i) Mr. Vijay has PGBP income of ₹2,00,00,000 and LTCG u/s. 112 of ₹25,00,000.
Case (ii) Mr. Vijay has Salary income (computed) of ₹3,00,000, STCG u/s. 111A of
₹40,00,000 and LTCG u/s. 112A of ₹30,00,000.
Case (iii) Mr. Vijay has income under head House Property of ₹5,00,00,000, STCG of
₹40,00,000 and LTCG u/s. 112 of ₹60,00,000.
Case (iv) Mr. Vijay has other income of ₹15,00,000, PGBP income of ₹60,00,000 and
LTCG u/s. 112A of ₹30,00,000.
Case (v) Mr. Vijay has PGBP income of ₹60,00,000 and STCG u/s. 111A of ₹1,00,00,000.

132 | P a g e #HakkसेCA by CA. Akash Sir


133
4. Income under head Capital Gains

Case (vi) Mr. Vijay has Salary income (computed) of ₹1,40,00,000 and LTCG u/s. 112 of
₹7,00,00,000.

HW Question: 6

Mr. Sanjay has income from business ₹203 lakhs and short term capital gain under
section 111A ₹30 lakhs and long term capital gains under section 112A ₹41 lakhs.
Calculate his tax liability under old tax regime.

HW Question: 7

Mr. Vikrant has income from business of ₹501 lakhs and short term capital gain under
section 111A of ₹30 lakhs and long term capital gains under section 112A of ₹41 lakhs.
Calculate his tax liability under old tax regime.

HW Question: 8

Mr. Vikas has income from business of ₹550 lakhs and short term capital gain under
section 111A of ₹30 lakhs and long term capital gains under section 112A of ₹41 lakhs.
Compute his tax liability under old tax regime.

HW Question: 9

Mr. Govind has income as given below:


Income under the head PGBP 2,02,00,000
Dividend Income 1,00,00,000

Compute his tax liability for A.Y. 2024-25.

***************************************************************************
Discussion on Full value of consideration: Covered in class
***************************************************************************

TP: 8 Computation of capitan gain [Section 48]

Mann Ki Baat (simple language):

Till now, we have studied about charging section, Type of capital assets, rates of taxes
in various cases, Enhanced surcharge etc., but we have not calculated capital gain. Now,
our focus in on computation of capital gain under various circumstances. Let’s start.

133 | P a g e #HakkसेCA by CA. Akash Sir


134
4. Income under head Capital Gains

1. Computation of Short term capital gain:

Full value of consideration xxxx


(-) Expenses incurred on transfer (excluding STT) (xxxx)
(-) Cost of acquisition (xxxx)
(-) Cost of improvement (xxxx)
Short term capital gain XXXX

For example: Mr. Mohan purchased one house on 01-07-2021 of ₹12,00,000 and
constructed its first floor on 01-07-2022 by incurring ₹4,00,000 and sold the house
on 01-05-2023 for ₹68,50,000. Compute Income and Tax Liability under normal
provisions of the Act.

Solution:

Computation of total income & tax liability in case of Mr. Mohan for A.Y.2024-
25

Period of holding of the house (01-07-2021 to 01-05-2023) - 22 months


Capital Asset – Short term capital asset
Rate of tax – Slab rate

Full Value of Consideration 68,50,000


Less:
- Cost of Acquisition (12,00,000)
- Cost of Improvement (4,00,000)
- Transfer Expenses Nil
Short Term Capital Gain 52,50,000
Gross Total Income/ Total Income 52,50,000

Computation of Tax Liability

Tax on STCG ₹52,50,000 at slab rate 13,87,500


Add: Surcharge @ 10% 1,38,750
Tax before health & education cess 15,26,250
Add: HEC @ 4% 61,050
Tax liability 15,87,300

2. Computation of long term capital gain (without indexation):

Full value of consideration xxxx


(-) Expenses incurred on transfer (excluding STT) (xxxx)

134 | P a g e #HakkसेCA by CA. Akash Sir


135
4. Income under head Capital Gains

(-) Cost of acquisition (xxxx)


(-) Cost of improvement (xxxx)
Long term capital gain XXXX

For example: Mr. Mohan purchased listed equity shares on 01.07.2021 for
₹12,00,000 (STT Paid) and sold the same on 01.05.2023 for ₹68,50,000 (STT Paid).
Compute Income and Tax Liability under normal provisions of the Act.

Solution:

Computation of total income & tax liability in case of Mr. Mohan for A.Y.2024-
25

Period of holding of the listed equity shares (01-07-2021 to 01-05-2023) - 22


months
Capital Asset – Long term capital asset
Rate of tax – Upto 1,00,000 – Nil, Balance @ 10%

Full Value of Consideration 68,50,000


Less:
- Cost of Acquisition (12,00,000)
- Cost of Improvement 0
- Transfer Expenses 0
Long Term Capital Gain 56,50,000
Gross Total Income/ Total Income 56,50,000

Computation of Tax Liability

Tax on LTCG of (₹56,50,000 – 1,00,000 – 2,50,000) at 10% 5,30,000


Add: Surcharge @ 10% 53,000
Tax before health & education cess 5,83,000
Add: HEC @ 4% 23,320
Tax liability 6,06,320

3. Computation of long term capital gain (with indexation):

Full value of consideration xxxx


(-) Expenses incurred on transfer (excluding STT) (xxxx)
(-) INDEXED Cost of acquisition (COA) (xxxx)

( COA x Cost inflation index of year in which asset is transferred )


Cost inflation index of year in which asset was first held

135 | P a g e #HakkसेCA by CA. Akash Sir


136
4. Income under head Capital Gains

(-) INDEXED Cost of improvement (COI) (xxxx)

( COI x Cost inflation index of year in which asset is transferred )


Cost inflation index of year in which improvement was made

Long term capital gain XXXX

For example: Mr. Manit purchased one house 01.07.2018 for ₹10,00,000 and
constructed its first floor on 01.07.2019 by incurring ₹6,00,000 and sold the house
on 01.05.2023 for ₹70,00,000. Compute Income and Tax Liability under normal
provisions of the Act.

Solution:

Computation of tax liability in case of Mr. Manit for A.Y. 2024-25

Period of holding of the house (01-07-2018 to 01-05-2023) - 58 months


Capital Asset – Long term capital asset
Rate of tax – 20% with indexation

Full Value of Consideration 70,00,000


Less: Indexed cost of acquisition
= 10,00,000 / Index of 18-19 x Index of 23-24
= 10,00,000 / 280 x 348 (12,42,857)
Less: Indexed cost of improvement
Cost of constructing first floor
= 6,00,000 / Index of 19-20 x Index of 23-24
= 6,00,000 / 289 x 348 (7,22,491)
Long Term Capital Gain 50,34,652
Gross Total Income/ Total Income 50,34,652
Rounded off u/s 288A 50,34,650

Computation of Tax Liability


Tax on LTCG ₹47,84,650 (₹50,34,650 – ₹2,50,000) @ 20% 9,56,930
Add: Surcharge @ 10% 95,693
Tax before health & education cess 10,52,623
Add: HEC @ 4% 42,105
Tax liability 10,94,728
Rounded off u/s 288B 10,94,730

136 | P a g e #HakkसेCA by CA. Akash Sir


137
4. Income under head Capital Gains

Important Note: The cost of acquisition of the asset or the cost of improvement
thereto would not include the deductions claimed on interest u/s. 24(b) or
deduction under section 80EE and 80EEA with respect to interest on home loan.

Cost Inflation Index (CII) table (Do not learn for exam)

Financial year Cost Inflation Index Financial year Cost Inflation Index
2001 - 2002 100 2012 – 2013 200
2002 - 2003 105 2013 – 2014 220
2003 - 2004 109 2014 – 2015 240
2004 - 2005 113 2015 – 2016 254
2005 - 2006 117 2016 – 2017 264
2006 - 2007 122 2017 – 2018 272
2007 – 2008 129 2018 – 2019 280
2008 - 2009 137 2019 – 2020 289
2009 - 2010 148 2020 – 2021 301
2010 - 2011 167 2021 – 2022 317
2011 - 2012 184 2022 – 2023 331
2023 - 2024 348

Illustration: 10

Calculate income under head capital gain and tax liability in following cases under
normal tax provisions:

Case (i) Mr. Sahil, resident aged 68 years, has purchased one house property on
01.04.2011 for ₹10,00,000. He sold the property for ₹40,00,000 on 02.02.2024. [Ans:
21,08,700, 3,76,210]

Case (ii) Mr. Manoj, resident aged 45 years, has purchased 1,00,000 shares of a listed
company (STT paid) @ ₹30 each on 01.04.2022 and sold 50,000 shares on 03.04.2023
for ₹40 each and 50,000 shares on 04.05.2023 for ₹25 each (STT paid). [Ans:
2,50,000, 0]

Case (iii) Mr. Ashok, non-resident aged 80 years, has purchased 50,000 shares of
unlisted company @ ₹100 each on 01.06.2022 and sold 40,000 shares on 01.02.2024 for
₹120 each. [Ans: 8,00,000, 75,400]

Case (iv) Mr. Ritik, resident aged 20 years, purchased unlisted equity oriented mutual
funds on 01.05.2022 for ₹50,00,000 and sold such units on 01.06.2023 for ₹75,00,000.
[Ans: 22,43,200, 4,14,650]

137 | P a g e #HakkसेCA by CA. Akash Sir


138
4. Income under head Capital Gains

HW Question: 10

Mr. Rohit purchased one house on 01.07.2005 for ₹3,50,000. He constructed its first
floor on 01.10.2014 by incurring ₹4,00,000 and constructed its second floor on
01.10.2015 by incurring ₹6,00,000 and third floor on 01.10.2017 by incurring ₹7,00,000.
Finally, sold the building on 01.01.2024 for ₹1,20,00,000 and selling expenses were 2%
of the sale price. He has deposited ₹1,00,000 in NSC. Compute tax liability of the
assessee for the assessment year 2024-25 assuming that he shifts out of default tax
regime under section 115BAC.

Important Note: The cost of acquisition of the asset or the cost of improvement
thereto shall not include the deductions claimed on the amount of interest under
section 24(b) or under the provisions of Chapter VIA.

TP: 9 Cost of acquisition where asset is acquired before


01.04.2001

Mann Ki Baat (simple language):

If we notice CII table, we should note that the base year for the indexation is F.Y.
2001-02. Now, question arises that what if the asset is acquired before 01.04.2001?
How to consider the cost of acquisition of such asset since the indexation for earlier
years is not available. In order to overcome such practical dificulty, following provision
has been inserted.

Where the asset is acquired before 01.04.2001, cost of acquisition shall be


higher of following:

a) Actual cost of acquisition


b) Fair market value as on 01.04.2001

However, in case of immovable property being land or building, cost of acquisition


shall be higher of:

a) Actual cost of acquisition


b) lower of (i) Fair market value as on 01.04.2001
(ii) Stamp duty value as on 01.04.2001

Important note: Cost of improvement incurred before 01.04.2001 shall not be


considered since it is already part of fair market value as on 01.04.2001 which is
consider in above formula.

138 | P a g e #HakkसेCA by CA. Akash Sir


139
4. Income under head Capital Gains

For example: Mr. Dilip purchased one house on 01.07.1998 for ₹2,00,000 and
incurred ₹3,00,000 on its improvement on 01.10.2000 and its fair market value as on
01.04.2001 is ₹7,00,000 and stamp duty value is 9,00,000, in this case if the asset is
sold, its cost of acquisition shall be taken to be ₹7,00,000 and index of 2001-02 shall
be applied. But if stamp duty value is 5,00,000, cost of acquisition shall be taken to
be 5,00,000 but if stamp duty value is 1,00,000, cost of acquisition shall be 2,00,000.

Illustration: 11

Compute cost of acquisition as on 01.04.2001 in following cases:

# Nature Date & Date & Fair market Stamp duty Cost of
of Purchase Improvement value as on value as on acquisition
property price cost 01.04.2001 01.04.2001
1. Shares 01.04.1995 NA 12,00,000 NA
of XYZ 10,00,000
Ltd.
2. Land 01.04.1980 01.05.1999 45,00,000 40,00,000
30,50,000 10,00,000
3. Building 01.05.1970 01.04.2002 70,00,000 75,00,000
40,00,000 10,50,000
4. Villa 01.06.1997 01.03.2000 40,00,000 45,00,000
50,60,000 4,00,000
5. Bonus 01.04.1999 NA 3,50,000 NA
shares Nil

Illustration: 12

Compute capital gains and tax liability of Mr. Laxman for the assessment year
2024-25 under normal provisions of the Act:
Asset Gold Land Residential
House
Date of purchase 01.07.1992 01.04.1995 01.07.1997
Cost price 4,00,000 6,00,000 8,00,000
Cost of improvement 1,00,000 2,00,000 4,00,000
Year of improvement 1999-2000 2000-01 2005-06
Fair market value on
01.04.2001 30,00,000 60,00,000 5,00,000
Date of Sale 01.02.2024 01.03.2024 01.01.2024
Full value of consideration 2,00,00,000 3,00,00,000 4,00,00,000

139 | P a g e #HakkसेCA by CA. Akash Sir


140
4. Income under head Capital Gains

Solution:

Computation of total income in case of Mr. Laxman

Gold
Full value of consideration 2,00,00,000
Less: Indexed cost of acquisition
= 30,00,000 / Index of 01-02 x Index of 23-24
= 30,00,000 / 100 x 348 (1,04,40,000)
Long term capital gain 95,60,000

Land
Full value of consideration 3,00,00,000
Less: Indexed cost of acquisition
= 60,00,000 / Index of 01-02 x Index of 23-24
= 60,00,000 / 100 x 348 (2,08,80,000)
Long term capital gain 91,20,000

Residential House
Full value of consideration 4,00,00,000
Less: Indexed cost of acquisition
= 8,00,000 / Index of 01-02 x Index of 23-24
= 8,00,000 / 100 x 348 (27,84,000)
Less: Indexed cost of improvement
= 4,00,000 / Index of 05-06 x Index of 23-24
= 4,00,000 / 117 x 348 (11,89,744)
Long term capital gains 3,60,26,256
Income under head capital gains 5,47,06,256

Gross total income/ Total income 5,47,06,256


Rounded off u/s. 288B 5,47,06,260

Computation of Tax Liability

Tax on LTCG ₹5,44,56,260 (₹5,47,06,260 – ₹2,50,000) @ 20% 1,08,91,252


Add: Surcharge @ 15% 16,33,688
Tax before HEC 1,25,24,940
Add: HEC @ 4% 5,00,998
Tax liability 1,30,25,938
Rounded off u/s. 288B 1,30,25,940

140 | P a g e #HakkसेCA by CA. Akash Sir


141
4. Income under head Capital Gains

HW Question: 11

Mr. Aman acquired a residential house in January, 1999 for ₹2,00,000 and its market
value on 01.04.2001 is ₹1,80,000 and he constructed its 1st floor in September’ 2007
by incurring ₹3,00,000 and constructed second floor in October’ 2011 by incurring
₹4,00,000 and constructed its third floor in February’ 2013 by incurring ₹5,00,000 and
sold the house on 01.01.2024 for ₹1,00,00,000 and paid brokerage @ 1% and he
invested ₹20,000 in equity shares of infrastructure development company notified
under section 80C. Compute his tax liability for assessment year 2024-25 under normal
provisions of the Act.

HW Question: 12

Compute capital gains and tax liability of Mr. Anmol in the following Independent
situations for the assessment year 2024-25 under normal provisions of the Act:
Asset Gold Land Residential house
Date of purchase 01.07.1998 01.04.1996 01.07.1993
Cost price 3,00,000 5,00,000 7,00,000
Cost of improvement 20,000 1,00,000 3,00,000
Year of improvement 1999-00 2000-01 2016-17
Fair market value on 01.04.2001 35,00,000 45,00,000 55,00,000
Date of Sale 01.01.2024 01.01.2024 01.01.2024
Full value of consideration 150,00,000 320,00,000 400,00,000

TP: 10 Cost of shares/ securities in certain cases

# Type of shares/ securities Cost of acquisition


1. Original shares/ securities Actual amount paid for acquiring
shares/ securities
2. Right entitlement Zero
3. Rights shares/ securities (if acquired Actual amount paid for acquiring
by original right holder) shares/ securities
4. Rights shares/ securities (if acquired Actual amount paid for acquiring
by person who purchased rights) shares/ securities + Renouncement cost
5. Bonus shares (acquired on or after Zero
01.04.2001)
6. Bonus shares (acquired before FMV as on 01.04.2001
01.04.2001)

1. Sale of Original shares: Cost of acquisition of original share is the actual


purchase price provided that the shares are purchased after 01.04.2001.

141 | P a g e #HakkसेCA by CA. Akash Sir


142
4. Income under head Capital Gains

2. Sale of Right entitlement: Right entitlement means receipt of right from the
company to subscribe for the shares before the subscription is open for general
public. It is given to the existing share holder as per Companies Act 2013. The cost
of acquisition of right entitlement is ZERO since no cost is involved. Also, period of
holding shall be from the date of right announcement till the date of transfer.

3. Sale of Right shares (acquired by original right holder itself): Cost of


acquisition of right share is the actual purchase price provided that the shares are
purchased after 01.04.2001.

4. Sale of Right shares (if acquired by person who purchased rights from
original holder): It means that the original holder of the rights can renounce the
rights to any third person for a price if he does not want to subscribe to the shares
himself. In this case, the cost of acquisition to the subsequent right holder shall be
the actual amount paid for acquiring shares/ securities and the renouncement cost
paid to original right holder.

Illustration: 13

Mr. Ameesh holds 500 shares of Aabra Ka Dabra Ltd. which were allotted to him on
22.04.2002 @ ₹30 per share. On 22.07.2023, Aabra Ka Dabra Ltd. made right issue to
the existing shareholders at the rate of one share for every five shares held @ ₹20
per share. Mr. Ameesh instead of exercising his rights to obtain right shares, has
exercised his right of renouncement by renouncing the said right entitlement in favour
of Mr. Nameesh @ ₹13 per right share entitlement on 04.08.2023.
(a) Determine the nature and amount of capital gain, if any, taxable in the hands of Mr.
Ameesh.
(b) What will be the cost of acquisition of shares purchased by Mr. Nameesh?

Solution:

Computation of Capital Gains in the hands of Mr. Ameesh

Full value of consideration 1,300


(100 x 13)
Less: Cost of acquisition Nil
Short Term Capital Gain 1,300

Cost of acquisition of shares purchased by Mr. Nameeh = ₹33 x 100 = ₹3,300

142 | P a g e #HakkसेCA by CA. Akash Sir


143
4. Income under head Capital Gains

HW Question: 13

Mr. Soham is a shareholder of Somani Ltd. holding 1,000 shares of the face value of
₹10 each. The company made a right issue in the ratio of 1:1 on 01.01.2024 at a premium
of ₹50 per share. He renounced it in favour of Mr. Mohan at a price of ₹10 per share.
What is the capital gain chargeable in the hands of Mr. Soham? What will be the cost
of the shares in the hands of Mr. Mohan?

5. Sale of Bonus shares (acquired on or after 01.04.2001): Bonus shares are


issued out of free reserves and does not involve any capital inflow for the company.
Also, no cost is involved in hands of shareholders. Therefore, the cost of acquisition
of bonus shares shall be Zero. Also, period of holding shall be from the date of bonus
share allotment till the date of transfer.

6. Sale of Bonus shares (acquired before 01.04.2001): As per TP discussed


above, where any property is acquired before 01.04.2001, cost of acquisition shall be
higher of actual cost or FMV as on 01.04.2001. Therefore, cost of acquisition of
bonus shares acquired before 01.04.2001 shall be FMV as on 01.04.2001.

Illustration: 14

Mr. Pawan purchased 100 equity shares in Mambani Ltd. on 01.10.1996 @ ₹10 per share.
The company has issued 100 bonus shares on 01.10.1998 and market value of the shares
on 01.04.2001 was ₹7 per share. The company has again issued 100 bonus shares on
01.10.2013.
The company has offered 100 right shares on 01.04.2023 @ ₹140 per share though the
market value is ₹250 per share. Mr. Pawan purchased half of the shares and remaining
half were renounced by him in favour of his friend Mr. Jagmal. He has charged ₹20 per
share from Mr. Jagmal for renouncing the right.
All the shares were sold by Mr. Pawan and Mr. Jagmal @ ₹300 per share on 01.01.2024.
Mr. Pawan has income under the head house property ₹2,20,000 and has invested
₹1,00,000 in NSC.
Mr. Jagmal has income under the head house property ₹3,50,000 and has invested
₹30,000 in NSC.
Compute tax liability of Mr. Pawan and Mr. Jagmal (No STT was paid) if they opts out
of default has regime under section 115BAC.

Solution:

Computation of Capital Gains of Mr. Pawan

Original Shares

143 | P a g e #HakkसेCA by CA. Akash Sir


144
4. Income under head Capital Gains

Full value of consideration (100 x 300) 30,000


Less: Indexed Cost of Acquisition (100 x 10)/100 x 348 (3,480)
Long Term Capital Gain u/s. 112 26,520

1st Bonus Shares


Full value of consideration (100 x 300) 30,000
Less: Indexed Cost of Acquisition (100 x 7)/100 x 348 (2,436)
Long Term Capital Gain u/s. 112 27,564

2nd Bonus Shares


Full value of consideration (100 x 300) 30,000
Less: Indexed Cost of Acquisition Nil
Long Term Capital Gain u/s. 112 30,000

Right Shares
Full value of consideration (50 x 300) 15,000
Less: Cost of Acquisition (50 x 140) (7,000)
Short Term Capital Gain 8,000

Renouncing of right to purchase shares


Full value of consideration 1,000
Less: Cost of acquisition Nil
Short Term Capital Gain 1,000

Computation of Total Income

Income under the head House Property 2,20,000


Income under the head Capital Gains
Long Term Capital Gain u/s. 112 84,084
Short Term Capital Gain 9,000
Gross Total Income 3,13,084
Less: Deduction u/s 80C (1,00,000)
Total Income 2,13,084
Rounded off u/s 288A 2,13,080

Computation of Tax Liability in case of Mr. Pawan

Tax on ₹1,29,000 at slab rate Nil


Tax on (84,084 – 84,084) @ 20% u/s 112 Nil
Tax Liability Nil

Computation of Capital Gains of Mr. Jagmal

144 | P a g e #HakkसेCA by CA. Akash Sir


145
4. Income under head Capital Gains

Full value of consideration 15,000


Less: Cost of acquisition (50 x 160) (8,000)
Short Term Capital Gain 7,000

Computation of Total Income

Income under the head House Property 3,50,000


Income under the head Capital Gains 7,000
Gross Total Income 3,57,000
Less: Deduction u/s 80C (30,000)
Total Income 3,27,000

Computation of Tax Liability

Tax on ₹3,27,000 at slab rate 3,850


Less: Rebate u/s 87A (3,850)
Tax Liability Nil

TP: 11 Cost of acquisition where long term capital u/s. 112A are
acquired before 01.02.2018

Mann Ki Baat (simple language):

Before we study this point, we need to know a little background story of this. Before
Finance act 2018, long term capital gain on assets mentioned u/s. 112A were exempt
from income tax as per section 10(38). In the Finance Act 2018, the Government
introduced capital gain tax on the same u/s. 112A and taxed long term capital gain
@10%. However, since such capital gain tax was introduced on 31.01.2018, capital gain
earned prior to such date could not taxed and therefore, cost of acquisition is defined
as below.

Where the tranfer of the asset acquired before 01.02.2018 is to be taxed


under section 112A (i.e. equity shares or units of equity oriented mutual funds
or units of business trust), cost of acquisition shall be higher of following:

a) Actual cost of acquisition


b) Lower of (i) Fair market value as on 31.01.2018
(ii) Full value of consideration

145 | P a g e #HakkसेCA by CA. Akash Sir


146
4. Income under head Capital Gains

For example:

Calculate cost of acquisition in following cases:

Case (i) Mr. Raman acquired listed equity shares (STT paid) on 01.02.2013 for
₹30,000 and sold these shares on 01.01.2024 for ₹50,000 (STT paid). FMV as on
31.01.2018 was ₹45,000. [Ans: 45,000]

Case (ii) Mr. Namam acquired listed equity shares (STT Paid) on 01.10.2017 for
₹2,50,000 and sold these shares on 09.10.2023 for ₹2,20,000 (STT paid). FMV as on
31.01.2018 was ₹2,30,000. [Ans: 2,50,000]

Case (iii) Mr. Aman acquired equity oriented mutual funds (STT not paid) on
01.04.2015 for ₹3,00,000 and sold these units on 10.10.2023 for ₹2,80,000 (STT
Paid). FMV as on 31.01.2018 was ₹3,40,000. [Ans: 3,00,000]

Case (iv) Mr. Sanam acquired equity oriented mutual funds (STT not paid) on
01.04.2015 for ₹4,00,000 and sold these units on 10.10.2023 for ₹5,80,000 (STT
Paid). FMV as on 31.01.2018 was ₹6,40,000. [Ans: 5,80,000]

Illustration: 15

Mr. Ashish purchased 100 equity shares in ABC Ltd. on 01.10.1995 @ ₹10 per share. The
company has issued 100 bonus shares on 01.10.1998 and market value of the shares on
01.04.2001 was ₹7 per share. The company has again issued 100 bonus shares on
01.10.2013.
The company has offered 100 right shares on 01.04.2023 @ ₹140 per share though the
market value is ₹250 per share. Mr. Ashish purchased half of the shares and remaining
half were renounced by him in favour of his friend Mr. Manish. He has charged ₹20 per
share from Mr. Manish for renouncing the right.
All the shares were sold by Mr. Ashish and Mr. Manish @ ₹300 per share on 01.01.2024
and securities transaction tax has been paid. (market value on 31-01-2018 is ₹200 per
share)
Mr. Ashish has income under the head house property ₹2,20,000 and has invested
₹1,00,000 in NSC.
Mr. Manish has income under the head house property ₹3,50,000 and has invested
₹30,000 in NSC.
Compute tax liability of Mr. Ashish and Mr. Manish under old tax regime.

Solution:

Computation of Capital Gains of Mr. Ashish

146 | P a g e #HakkसेCA by CA. Akash Sir


147
4. Income under head Capital Gains

Original Shares
Full value of consideration (100 x 300) 30,000
Less: Cost of Acquisition (20,000)
Higher of
(i) COA = 100 x 10 = 1,000
(ii) lower of
(a) FMV as on 31-01-2018 = 100 x 200 = 20,000
(b) sale value = 100 x 300 = 30,000
Long Term Capital Gain u/s 112A 10,000

1st Bonus Shares


Full value of consideration (100 x 300) 30,000
Less: Cost of Acquisition (20,000)
Higher of
(i) COA (7 x 100) = 700
(ii) lower of
(a) FMV as on 31-01-2018 = 100 x 200 = 20,000
(b) sale value = 100 x 300 = 30,000
Long Term Capital Gain u/s 112A 10,000

2nd Bonus Shares


Full value of consideration (100 x 300) 30,000
Less: Cost of Acquisition (20,000)
Higher of
(i) COA = Nil
(ii) lower of
(a) FMV as on 31-01-2018 = 100 x 200 = 20,000
(b) sale value = 100 x 300 = 30,000
Long Term Capital Gain u/s 112A 10,000

Right Shares
Full value of consideration (50 x 300) 15,000
Less: Cost of Acquisition (50 x 140) (7,000)
Short Term Capital Gain u/s 111A 8,000

Renouncing of right to purchase shares


Full value of consideration 1,000
Less: Cost of acquisition Nil
Short Term Capital Gain 1,000

Computation of Total Income

147 | P a g e #HakkसेCA by CA. Akash Sir


148
4. Income under head Capital Gains

Income under the head House Property 2,20,000


Income under the head Capital Gains
Long term capital gains 112A 30,000
Short term capital gains 111A 8,000
Short term capital gains 1,000
Income under the head Capital Gains 39,000
Gross Total Income 2,59,000
Less: Deduction u/s 80C (1,00,000)
Total Income 1,59,000

Computation of Tax Liability


Tax on normal income ₹1,21,000 at slab rate Nil
Tax on (30,000 – 30,000) @ 10% u/s 112A Nil
Tax on (8,000 – 8,000) @ 15% u/s 111A Nil
Tax Liability Nil

Computation of Capital Gains of Mr. Manish


Full value of consideration 15,000
Less: Cost of acquisition (50 x 160) (8,000)
Short Term Capital Gain u/s 111A 7,000

Computation of Total Income


Income under the head House Property 3,50,000
Income under the head Capital Gains 7,000
Gross Total Income 3,57,000
Less: Deduction u/s 80C (30,000)
Total Income 3,27,000

Computation of Tax Liability


Tax on ₹3,20,000 at slab rate 3,500
Tax on ₹7,000 @ 15% u/s 111A 1,050
Less: Rebate u/s 87A (4,550)
Tax Liability Nil

HW Question: 14

Mr. Sunil purchased 100 equity shares in ABC Ltd. (listed) on 01.10.1996 @ ₹10 per
share. The company had issued 100 bonus shares on 01.10.2000 and market value of the
share as on 01.04.2001 is ₹8 per share.
Company has again issued 100 bonus shares on 01.10.2006.

148 | P a g e #HakkसेCA by CA. Akash Sir


149
4. Income under head Capital Gains

The company has further offered 100 right shares on 01.05.2023 @ ₹150 per share
and Mr. Sunil has purchased half of the shares and balance half was renounced in
favour of Mr. Jeet by charging ₹5 per share.
Mr. Sunil and Mr. Jeet both have transferred all the shares on 01.01.2024 @ ₹200 per
share and securities transaction tax has been paid. Market value as on 31-01-2018 ₹100
per share.
Mr. Sunil has income under the head business/profession ₹20,00,000 and he has
invested ₹70,000 in public provident fund.
Mr. Jeet has income under the head business/profession ₹10,00,000 and he has
invested ₹50,000 in public provident fund.
Compute tax liability of Mr. Sunil and Mr. Jeet assuming that he shifts out of default
tax regime under section 115BAC.

HW Question: 15

Mrs. Kamla purchases 1,000 equity shares in Adani Ltd. at a cost of ₹ 15 per share
(brokerage 1%) in January 1998. She gets 100 bonus shares in August 2000. She again
gets 1100 bonus shares by virtue of her holding on February 2005. Fair market value of
the shares of Adani Ltd. On April 1, 2001 is ₹25. In January 2024, she transfers all
her shares @ ₹ 120 per share (brokerage 2%). (market value on 31-01-2018 is ₹70 per
share).
Compute the capital gains taxable in the hands of Mrs. Kamla for the A.Y. 2024-25
assuming:
(a) Adani Ltd. is an unlisted company and securities transaction tax was not applicable
at the time of sale.
(b) Adani Ltd. is a listed company and the shares are sold in a recognised stock
exchange and securities transaction tax was paid at the time of sale.

HW Question: 16 [Capital Gains, IFOS, set-off of losses]

Mr. Arvind holding 1000 shares of Logistic Ltd acquired on 01.07.2022 for ₹600 per
share, sold 500 shares to Mr. Sunil, on 01.05.2023 for ₹550 per share. Logistic Ltd.
declared dividend @ ₹65 per share on 20.07.2023, being the record date for
declaration of dividend. Mr. Sunil sold 300 equity shares at ₹475 per share on
28.09.2023 and the balance 200 equity shares at ₹450 per share on 28.10.2023. Apart
from above mentioned information, Mr. Sunil was having only long-term capital gains
from sale of unlisted shares of ₹50,000. Compute his total income for A.Y. 2024-25.

149 | P a g e #HakkसेCA by CA. Akash Sir


150
4. Income under head Capital Gains

TP: 12 Cost of acquisition in case Intangible Assets

➢ Acquired from previous owner:

In case of goodwill of a business or profession is acquired from previous owner


(under gift, will, sccession, etc. [Refer section 47]) in respect of which
depreciation has been claimed under section 32(1) in any previous year upto P.Y.
2019-20, then COA shall be amount of purchase price for such previous owner as
reduced by total amount of depreciation claimed. In other cases, COA shall be
purchase price in hands of such previous onwer.

➢ Self generted intangible assets:

In case of self-generated assets, namely, goodwill of a business or profession or


any other intangible asset or a trademark or brand name associated with a
business or profession or a right to manufacture, produce or process any article or
thing, or right to carry on business or profession, tenancy rights, stage carriage
permits or loom hours, or any other right, the cost of acquisition will be taken to
be NIL.

TP: 13 Transactions not regarded as Transfer [Section 47]

1) Any distribution of capital asset to HUF members on total or partial partition


of HUF

Distribution of capital asset by HUF to Members on partition of HUF is not regarded


as transfer and hence no capital gain shall arise in hands of HUF.

Period of Holding POH of HUF + POH of Member


Cost of Acquisition COA of HUF
Cost of Improvement COI of HUF + COI of Member

For example: Pankaj HUF has been partitioned and has distributed its capital asset
to Mr. Pankaj, Mrs. Pankaj and their child. In this case, no capital gain shall arise
even if capital asset is transferred from HUF to members since the transaction is
not regarded as Transfer u/s. 47.

Illustration: 16

Mr. Ramit & sons, HUF, purchased a land for ₹40,000 in 2001-02. In 2005-06, a
partition takes place when Mr. Rohit, a coparcener, is allotted this plot valued at

150 | P a g e #HakkसेCA by CA. Akash Sir


151
4. Income under head Capital Gains

₹2,00,000. In 2006-07, he had incurred expenses of ₹1,85,000 towards fencing of the


plot. Mr. Rohit sells this plot of land for ₹15,00,000 in 2023-24 after incurring
expenses of ₹20,000. You are required to compute the capital gain for the A.Y. 2024-
25.

Solution:

Computation of taxable capital gains for the A.Y. 2024-25


Sale consideration 15,00,000
Less: Indexed cost of acquisition
= 40,000 / Index of 01-02 x Index of 23-24
= ₹40,000 /100 x 348 (1,39,200)
Less: Indexed cost of improvement
= 1,85,000 / Index of 06-07 x Index of 23-24
= ₹1,85,000 /122 × 348 (5,27,705)
Less: Expenses incurred for transfer (20,000)
Long term capital gains 8,13,095

2) Transfer of capital asset as Gift, under Will or under Inheritance

Asset transferred as gift or under will or under inheritance is not regarded as


transfer and hence no capital gain shall be computed.

Period of Holding POH of Previous owner + POH of Transferee


Cost of Acquisition COA of Previous owner (Transferor)
Cost of Improvement COI of Previous owner + COI of Transferee

For example: Mr. Jeet gifted jewellary of ₹20,00,000 to his girlfriend on the
ocassion of one week anniversary. In this case, no capital gain/ loss shall arise since
the transaction is not regarded as transfer.

For example: Mr. Sharma transferred his 50% of property to his son under will
worth ₹50,00,00,000. In this case, no capital gain shall arise in hands of Mr. Sharma
even if assets are actually transferred to son.

Illustration: 17

Mr. Jatin purchased one house on 01.10.1998 for ₹2,00,000 and incurred ₹1,00,000 on
its improvement on 01.10.1999. Its fair market value on 01.04.2001 is ₹4,50,000.
Mr. Jatin expired on 01.05.2006 and the house was inherited by his son Mr. Jenil and
value for the purpose of charging stamp duty was ₹10,00,000.
Mr. Jenil has sold the house on 01.11.2023 for ₹72,00,000.

151 | P a g e #HakkसेCA by CA. Akash Sir


152
4. Income under head Capital Gains

Compute tax liability of Mr. Jenil for the assessment year 2024-25 under old tax
regime.

Solution:

Computation of Capital Gains and total income in case of Mr. Jenil


Full value of consideration 72,00,000
Less: Indexed cost of acquisition
= 4,50,000 / 100 x 348 (15,66,000)
Long Term Capital Gain 56,34,000
Income under the head Capital Gain 56,34,000
Gross Total Income 56,34,000
Less: Deduction u/s 80C to 80U Nil
Total Income 56,34,000

Computation of Tax Liability


Tax on ₹54,60,500 (₹56,34,000 – ₹2,50,000) @ 20% 10,76,800
Add: Surcharge @ 10% 1,07,680
Tax before health & education cess 11,84,480
Add: HEC @ 4% 47,379
Tax Liability 12,31,859
Rounded off u/s 288B 12,31,860

3) Transfer of capital asset by Holding company to 100% Indian Subsidiary


company or by Subsidiary company to 100% Indian Holding company.

Transfer of capital asset by holding company to subsidiary company or by subsidiary


company to holding company is NOT regarded as transfer provided that the company
receiving the capital asset is Indian Company and is 100% holding/ subsidiary.

Period of Holding POH of Previous owner + POH of Transferee


Cost of Acquisition COA of Previous owner (Transferor)
Cost of Improvement COI of Previous owner + COI of Transferee

Note: If the transferee has considered such asset as stock in trade, then it shall be
considered as transfer in hands of transferor.

For example: Reliance Limited (holding company) transferred its Thane office
building to Reliance Jio Infocomm Limited (100% Indian subsidiary company) of
₹50,00,000 having FMV of ₹70,00,000. In this case, no capital gain shall be payable
by Reliance Limited. Cost of acquisition for Reliance Jio Infocomm Limited shall be
COA of Reliance Limited.

152 | P a g e #HakkसेCA by CA. Akash Sir


153
4. Income under head Capital Gains

4) Transfer of any capital asset by the amalgamating company to the Indian


amalgamated company

Transfer of capital asset by amalgamating company to amalgamated company is NOT


regarded as transfer provided that the company receiving the capital asset is an
Indian Company.

Period of Holding POH of amalgamating company + POH of amalgamted


company
Cost of Acquisition COA of amalgamating company
Cost of Improvement COI of amalgamating company + COI of amalgamted
company

5) Transfer of any capital asset by demerged company to the resulting Indian


company

Transfer of capital asset by demerged company to resulting company is NOT


regarded as transfer provided that the company receiving the capital asset is an
Indian Company.

Period of Holding POH of demerged company + POH of resulting company


Cost of Acquisition COA of demerged company
Cost of Improvement COI of demerged company + COI of resulting company

6) Transfer (surrender) of shares by a shareholder in a scheme of an


amalgamation provided that the amalgamated company is an Indian company

Surrender of shares of amalgamating company by the shareholder in a scheme of an


amalgamation (provided that the amalgamated company is an Indian company) shall
not be considered as Transfer.

Period of Holding POH of amalgamating co. + POH of amalgamated co.


Cost of Acquisition COA of shares in amalgamating company
Cost of Improvement NA

For example: Mr. Ramesh purchased 2000 shares in JSW Paints Ltd. on 01.07.2023
@ ₹10 per share and JSW Paints Ltd. was amalgamated with JSW Ltd. on 01.12.2023.
Mr. Ramesh transferred (surrenderd) his shares in JSW Paints Ltd. and received
1000 shares in JSW Ltd. and market value is ₹50 per share, in this case no capital
gains

153 | P a g e #HakkसेCA by CA. Akash Sir


154
4. Income under head Capital Gains

shall be computed but if Mr. Ramesh has sold the shares, capital gains shall be
computed and cost will be ₹20,000.

Illustration: 18

Manish held 2000 shares in a company ABC Ltd. an Indian company. This company
amalgamated with another Indian company XYZ Ltd. during the previous year ending
31.03.2024. Under the scheme of amalgamation, Manish was alloted 1000 shares in a
new company. The market value of shares alloted is higher by ₹50,000 than the value
of holding in ABC Ltd. The assessing officer proposes to treat the transaction as an
exchange and to tax ₹50,000 as capital gain. Is he justified?

Solution:

As per provisions of the act, surrender of shares of amalgamating company by the


shareholder in a scheme of an amalgamation (provided that the amalgamated company is
an Indian company) shall not be considered as Transfer. Therefore, in the given case,
assessing officer is not justified to tax such capital gain.

7) Transfer (sureender) of shares of demerged company by the shareholder in


exchange of shares of resulting company

Surrender of shares of demerged company by the shareholder in exchange of shares


of resulting company shall not be considered as Transfer.

Period of Holding POH of demerged co. + POH of resulting co.


Cost of Acquisition COA of shares in demerged company
Cost of Improvement NA

For example: Mr. Ramesh purchased 2000 shares in JSW Ltd. on 01.07.2023 @ ₹10
per share and JSW Ltd. was demerged and formed JSW Paints Ltd. and JSW Steels
Ltd. on 01.12.2023. Mr. Ramesh transferred (surrenderd) his shares in JSW Ltd. and
received 1000 shares in JSW Paints Ltd. and JSW Steels Ltd. each and market value
is ₹50 per share and ₹40 per share respectively, in this case no capital gains shall be
computed at the time of surrender of shares of JSW Ltd. but if Mr. Ramesh has sold
the shares of resulting companies, capital gains shall be computed and cost will be
₹20,000 (in total).

154 | P a g e #HakkसेCA by CA. Akash Sir


155
4. Income under head Capital Gains

8) Transfer on conversion of bond or debenture etc into shares or debenture of


that company / preference shares into equity shares

Transfer on account of conversion of bonds, debentures etc into shares or debenture


of the same company is not treated as transfer.

Period of Holding POH of original bonds or debentures + POH of converted


shares or debentures
Cost of Acquisition COA of original bonds or debentures
Cost of Improvement NA

For example: Mr. Ramit purchased 1000 debentures @ ₹100 each of Ram Ltd. Ram
Ltd. has converted the above debentures and issued 10,000 shares of ₹10 each (FMV
– ₹12 each) to Mr. Ramit. In this case, no capital gain is payable on transfer of
debentures and also cost of acquisition of shares will be ₹1,00,000 (1,000 * 100).

Illustration: 18

Mr. Druv purchased 100 debentures in ABC Ltd. on 01.10.2004 @ ₹300 per debenture
and subsequently the company has converted the debentures into shares on 01.10.2017
and for each debenture 3 shares were issued and market value of the shares on the
date of conversion was ₹250 per share and market value as on 31-01-2018 is ₹300 and
subsequently assessee has sold all these shares on 01.04.2023 @ ₹500 per share and
has paid brokerage @ 1% of the sale price. Compute capital gains in the hands of Mr.
Druv in the following cases:
(a) STT not paid
(b) STT paid

Solution:

(a) STT not paid


Computation of Capital Gains
Full value of consideration (300 x 500) 1,50,000
Less: Indexed Cost of acquisition
= (100 x 300) / Index of 04-05 x Index of 23-24
= 30,000 / 113 x 348 (92,389)
Less: Brokerage @ 1% on 1,50,000 (1,500)
Long Term Capital Gain 56,111

(b) STT Paid


Computation of Capital Gains
Full value of consideration (300 x 500) 1,50,000

155 | P a g e #HakkसेCA by CA. Akash Sir


156
4. Income under head Capital Gains

Less: Cost of acquisition (90,000)


Higher of
(i) COA = 100 debentures x 300 = 30,000
(ii) lower of
(a) FMV as on 31-01-2018 = 300 shares x 300 = 90,000
(b) sale value = 300 shares x 500 = 1,50,000
Less: Brokerage @ 1% on 1,50,000 (1,500)
Long Term Capital Gain u/s 112A 58,500

9) Transfer of rupee denominated bond outside India by a non-resident to


another non-resident

10) Transfer of Government security outside India by a non-resident to another


non-resident

11) Redemption of sovereign gold bonds by an Individual

12) Transfer of following capital asset to Government, University, National


Museum, National Art Gallary, National Archives or any other public Museum
etc.:
a) work of art
b) archaeological, scientific or art collection
c) book
d) manuscript
e) drawing
f) painting
g) photograph or
h) print

13) Transfer of capital asset under reverse mortgage:

In this case, the assessee (senior citizen) transfers the house property to the bank
under an agreement to pay him loan amount in monthly/ fortnightly/ semi annually/
annually till his death/ maturity.
After the death of the assessee or on maturity of loan, property will be transferred
to the bank and bank will sell the same and recover the installment along with
interest from sale proceeds. Excess amount (if any) will be handed over to the legal
assignee.

In this case:

156 | P a g e #HakkसेCA by CA. Akash Sir


157
4. Income under head Capital Gains

a) Transfer of asset by the assessee to the bank will not be treated as Transfer u/s.
47. Therefore, there shall not be any capital gain tax on transfer.
b) Income received by way of installment is exempt from tax u/s. 10(43).
c) Bank has to calculate and pay capital gain tax on sale of property on behalf of the
assessee after his death.

Illustration: 19

Mrs. Monika an individual, aged 68 years, mortgaged her Residential Property,


purchased for ₹3 lakhs on 01.10.2002, with a bank, under a notified reverse mortgage
scheme and was sanctioned a loan of ₹20 lakh. As per the said scheme she was
receiving the loan amount in equal monthly installments of ₹30 thousand per month
from the bank. Mrs. Mahalakshmi was not able to repay the loan on maturity and in lieu
of settlement of the loan surrenders the residential property to the bank. Bank sold
the property for ₹25 lakhs on 22-02-2024. She had no other income during the year.

Discuss the Tax consequences and compute tax for the Assessment Year 2024-25
under old tax regime.

Solution:

Any transfer of a capital asset in a transaction of reverse mortgage under a scheme


made and notified by the Central Government shall not be considered as a “transfer”
for the purpose of capital gain as per section 47. Accordingly, the mortgaging of
residential house with bank by Mrs. Monica will not be regarded as a transfer for levy
of capital gains. Therefore, no capital gain will be attracted on such mortgage.

Any amount received by the senior citizen as a loan, either in lump sum or in
installments, in a transaction of reverse mortgage is exempt from income-tax under
section 10(43). Therefore, the monthly installment of ₹30,000 received by Mrs. Monica
is exempt from income-tax under section 10(43).

However, capital gains tax liability would be attracted in the P.Y. 2023-24 when the
bank sells the mortgaged property for the purposes of recovering the loan.

Computation of Total income in the hands of Mrs. Monica for A.Y. 2024-25

Long-term Capital gains [Since the residential house property was held by Mrs.
Mahalakshmi for more than 24 months immediately preceding the date of its transfer]

Full Value of Consideration 25,00,000


Less: Indexed Cost of Acquisition (3,00,000/105 x 348) (9,94,286)

157 | P a g e #HakkसेCA by CA. Akash Sir


158
4. Income under head Capital Gains

Long term Capital Gains 15,05,714


Gross Total Income 15,05,714
Less: Deduction u/s 80C to 80U Nil
Total Income (rounded off u/s. 288A) 15,05,710

Computation of Tax Liability

Tax on LTCG u/s 112 on 12,05,710 (15,05,710-3,00,000) @ 20% 2,41,142


Add: HEC @ 4% 9,646
Tax Liability 2,50,788
Tax Liability (Rounded off u/s 288B) 2,50,790

HW Question: 17

Mr. Jatin (62 years old), pledged his residential house to a bank under a notified
reverse mortgage scheme. He was getting loan from bank in monthly instalments. Mr.
Jatin did not repay the loan on maturity and hence gave possession of the house to the
bank to discharge his loan. How will the treatment of long-term capital gain be on such
reverse mortgage transaction?

HW Question: 18

Mr. Jatin received ₹15,00,000 on 23.01.2024 on transfer of his residential building in a


transaction of reverse mortgage under a scheme notified by the Central Government.
The building was acquired in March 2004 for ₹8,00,000.

Is the amount received on reverse mortgage chargeable to tax in the hands of Mr.
Jatin under the head ‘Capital Gains’?

TP: 14 Exception to year of chargeability

In general, liability to pay capital gain tax arises in the year in which transfer took
place. However, these are some exceptions to it:

1) Insurance Receipt [Section 45(1A)]:

Where any person receives money or other asset from insurer on account of:
- Flood, typhoon, cyclone, earthquake, hurricane, or any other convulsion of nature,
- Riots or civil disturbance,
- Accidental fire or explosion or,
- Because of action by any enemy or action taken in combating an enemy (whether
with or without declaration or war), then,

158 | P a g e #HakkसेCA by CA. Akash Sir


159
4. Income under head Capital Gains

any profit or gain arising from receipt of such money or other assets shall be
chargeable to income-tax under the head “capital Gain” and shall be deemed to be
the income of such person for the previous year in which such money or other
asset was received.

Transaction Year of Full value of Cost of Period of


taxability consideration acquisition holding
Insurance Money or other FMV of asset WDV of asset Upto the
receipt assets received received and/ or or purchase date of
money or both price accident

Example:

Mr. Ramashankar has purchased one plant and machinery on 01.04.2022 with written
down value ₹25,00,000 the asset is destroyed on 06.06.2022 due to natural calamity
and he has received insurance claim of ₹30,00,000 on 30.11.2023, in this case there
will be short term capital gain of ₹5,00,000 chargeable to tax in the P.Y. 2023-
24.

2) Conversion of a capital asset as stock-in-trade [Section 45(2)]

A person who is the owner of a capital asset may convert the same or treat it as
stock-in-trade of the business carried on by him. As noted above, the above
transaction is a transfer.

As per section 45(2), the profits or gains arising from the above conversion or
treatment will be chargeable to income-tax as his income of the previous year in
which such stock-in-trade is sold or otherwise transferred by him.

Transaction Year of Full value of Cost of Period of


taxability consideration acquisition holding
Conversion of Stock is sold FMV of asset on Purchase Upto the
capital asset into the date of value of asset date of
stock in trade transfer conversion

Note – Both Capital Gain and Business income are chargeable to tax in the year
in which stock-in-trade is proportionately sold or otherwise transferred.

159 | P a g e #HakkसेCA by CA. Akash Sir


160
4. Income under head Capital Gains

Illustration: 20

Mr. Anshuman is the owner of a jewellery of ₹3,00,000 purchased on 01.06.2019. On


01.04.2022, he starts a business of purchase and sale of jewellery. He treats the above
jewellery as part of the stock-in-trade of his new business on 01.05.2022. FMV of the
jewellery on 01.05.2022 was ₹5,00,000. He sells the same on 31.03.2024 for ₹6,00,000
and gets a profit of ₹1,00,000. Discuss the tax implication in his hands under the head
“Capital Gains” and “PGBP”.

Solution:

In this case, capital gain shall be computed during P.Y. 2022-23 (year of conversion) as
follows:
FVC (Being FMV on date of transfer): 5,00,000
Cost of Acquisition (purchase price): 3,00,000
Short term capital gain (POH is 35 months): 2,00,000

Above capital gain shall be taxable in P.Y. 2023-24 i.e. year of sell of stock.

Computation of income from business:


Sale value 6,00,000
Purchase value 5,00,000
Profit 1,00,000

Illustration: 21

Mr. Anshu is the owner of a car of ₹3,00,000 purchased on 01.06.2019. On 01.04.2022,


he starts a business of purchase and sale of car. He treats the above car as part of the
stock-in-trade of his new business on 01.05.2022. FMV of the car on 01.05.2022 was
₹5,00,000. He sells the same on 31.03.2024 for ₹6,00,000 and gets a profit of
₹1,00,000. Discuss the tax implication in his hands under the head “Capital Gains”.

Solution:

Since car is not a capital asset in hands of Mr. Anshu, no capital gain shall arise on
conversion of the same as stock in trade.

Note: Recall the definition of “Capital assets”, it specifically excludes personal movable
effects from its definition and therefore, no capital gain is calculated in above case.

160 | P a g e #HakkसेCA by CA. Akash Sir


161
4. Income under head Capital Gains

3) Compensation on compulsory acquisition [Section 45(5)]

If any capital asset has been acquired compulsorily by the Government or other
similar agency, capital gains shall be computed in the year in which the asset was
acquired but capital gains so computed shall be taxable in the year in which the
compensation or the part of compensation is first received.

For example: Suppose that the government has compulsorily acquired the land in P.Y.
2019-20, then capital gain shall be computed in the same year. However, if amount of
compensation is first received in P.Y. 2023-24, then capital gain calculated in P.Y.
2019-20 shall be taxable in P.Y. 2023-24 only.

Enhanced Compensation:

If the compensation is enhanced by the Court, Tribunal etc., such enhanced


compensation shall be the capital gains of the year in which the enhanced
compensation is received.

The cost of acquisition and the cost of improvement shall be taken to be Nil.

For example: Suppose in above example, the person approached court with the plea
to enhance the amount of compensation and court ordered enhanced compensation in
P.Y. 2024-25 which is actually paid by the government in P.Y. 2026-27, then such
enhanced compensation shall be taxable during P.Y. 2026-27.
Further, the amount of compensation received in pursuance of an interim order of
the Court, Tribunal or other authority shall be deemed to be income chargeable
under the head ‘Capital gains’ in the previous year in which the final order of such
Court, Tribunal or other authority is made.

For example: Suppose in above example, the person approached court with the plea
to enhance the amount of compensation and court passed an interim order to
enhanced compensation in P.Y. 2024-25 which is actually paid by the government in
P.Y. 2026-27. Subsequently, court passed the final order in P.Y. 2027-28, in this case
such enhanced compensation shall be taxable during P.Y. 2027-28.

Reduction of enhanced compensation:

Where capital gain has been charged on the compensation received and subsequently
such compensation is reduced by any Court, Tribunal or any authority, the assessed
capital gain shall be recomputed and such re-computation shall be done by way of
rectification of return under section 155.

161 | P a g e #HakkसेCA by CA. Akash Sir


162
4. Income under head Capital Gains

Death of the transferor:

It is possible that the transferor may die before he receives the enhanced
compensation. In that case, the enhanced compensation will be chargeable to tax in
the hands of the person who receives the same.

Transaction Year of Full value of Cost of Period of


taxability consideration acquisition holding
Compulsory Money first Money received Purchase value + Upto the date
acquisition received improvement cost of acquisition
(if any)
Enhanced Money Money received NIL Upto the date
compensation received of acquisition
Interim order Final order Money received NIL Upto the date
of acquisition

Illustration: 22

Mrs. Bansal purchased one house on 01.07.2021 for ₹4,00,000 and the house was
acquired by the Government on 01.08.2022 and compensation fixed is ₹6,00,000 and
half of the amount was paid by the Government on 01.04.2023 and balance half on
01.04.2024. She also received enhanced compensation of ₹2,00,000 on 01.06.2024 and
₹1,00,000 on 01.07.2025. Compute capital gains of Mrs. Bansal for various assessment
years assuming that income tax provisions are same for all assessment years.

Solution:

Computation of income under the head Capital Gains Capital gain shall be computed in
the year in which the asset was acquired by the Government i.e. in the previous year
2021-22 and shall be taxed in the year in which the first payment has been received by
the assessee i.e. in the previous year 2023-24

Full value of consideration 6,00,000


Less: cost of acquisition 4,00,000
Short term capital gain (POH is 13 months) 2,00,000

Enhanced compensation shall be taxed in the year of receipt and shall be taxable as
“short term” or “long term” based on original tax. Therefore, enhanced compensation of
₹2,00,000 shall be taxable as short-term capital gain in the P.Y. 2024-25 and
₹1,00,000 as short-term capital gain in the P.Y. 2025-26.

162 | P a g e #HakkसेCA by CA. Akash Sir


163
4. Income under head Capital Gains

HW Question: 19

Mr. Suyash purchased one house on 01.10.2003 for ₹5,00,000 and this house was
acquired compulsorily by the Government on 01.07.2015. Compensation fixed by the
Government was ₹55,00,000. Government has paid half of the amount on 01.10.2023
and balance half on 01.10.2024.
The assessee was not satisfied with the compensation and he has filed an appeal in the
High Court. The High Court has given decision on 31.03.2026 directing the Government
to pay additional compensation of ₹5,00,000 and the Government has paid ₹3,00,000 on
10.04.2026 and balance ₹2,00,000 on 10.04.2027.
Compute capital gains for the various years and tax liability for assessment year 2024-
25 under old tax regime.

4) Specified agreement for development of a project [Section 45(5A)]

Sometimes it may happen that the assessee transfers his capital asset being land/
building or both to the developer and agrees to allow other person (developer) to
develop a real estate project on such land. In this case, the assessee does not
receive any amount in the year of transfer. However, he receives rights in property/
money or both after completion of project.

Considering the above situation, it is provided in the act that:

➢ In case of the assessee being Individual or HUF


➢ Who enters into a specified agreement for developments of project
➢ The capital gain arising from such transfer shall be chargeable to capital gain tax
in previous year in which certificate of completion for the whole or part of the
project is issued by competent authority.

Transaction Year of Full value of Cost of Period of


taxability consideration acquisition holding
Specified Certificate of Stamp duty Purchase value Upto the date
Agreement completion value of new + improvement of transfer to
issued building + Money cost (if any) developer
received

For example: Ms. Nishita has entered into an agreement with M/s Oppo Build
Limited on 25.04.2018 in which she agrees to allow the company to develop a shopping
mall on land owned by her in New Delhi. She purchased such land on 05.05.2010 in
₹15,00,000. In consideration, M/s Oppo Build Limited will provide 20% share in
shopping mall to Ms. Nishita. The certificate of completion of shopping mall was
issued by competent authority as on 26.12.2023. On such date, Stamp duty value of

163 | P a g e #HakkसेCA by CA. Akash Sir


164
4. Income under head Capital Gains

shopping mall was ₹4,14,00,000.

Q. 1 What is the year of transfer?


Ans. PY 2018-19

Q. 2 What will the year of taxability?


Ans. PY 2023-24

Q. 3 What will be the Full value of consideration?


Ans. ₹82,80,000 being 20% of SDV

Q. 4 What will be the cost of acquisition?


Ans. ₹15,00,000

Non-applicability of above provision:

If the assessee transfers his right in property to other person before completion of
projects, then capital gain shall be deemed to arise in the Previous year in which
rights in property are transferred.

Transaction Year of Full value of Cost of Period of


taxability consideration acquisition holding
Transfer of Year of Sale value of Purchase value Upto the date
rights in transfer of rights + improvement of transfer to
property rights cost (if any) rights

For example: Ms. Nishita has entered into an agreement with M/s Oppo Build
Limited on 25.04.2018 in which she agrees to allow the company to develop a shopping
mall on land owned by her in New Delhi. She purchased such land on 05.05.2010 in
₹15,00,000. In consideration, M/s Oppo Build Limited will provide 20% share in
shopping mall to Ms. Nishita. The certificate of completion of shopping mall was
issued by competent authority as on 26.12.2023. On such date, Stamp duty value of
shopping mall was ₹4,14,00,000. Ms. Nishita has transferred his right in property on
01.05.2022 to Mr. Akash for ₹1,00,00,000.

Q. 1 What is the year of transfer?


Ans. PY 2018-19

Q. 2 What will the year of taxability?


Ans. PY 2022-23

Q. 3 What will be the Full value of consideration?

164 | P a g e #HakkसेCA by CA. Akash Sir


165
4. Income under head Capital Gains

Ans. ₹1,00,00,000

Q. 4 What will be the cost of acquisition?


Ans. ₹15,00,000

Illustration: 23 [IFHP, Capital gain, Deductions, set-off] V. Good

Ms. Nehal has entered into an agreement with M/s DLF Build Limited on 25.05.2018 in
which she agrees to allow such Company to develop a residential property on land owned
by her in New Delhi. She purchased such land on 05.04.2010 in ₹15,00,000. In
consideration, M/s DLF Build Limited will provide 20% share in residential property to
Nehal. The certificate of completion of residential property was issued by authority as
on 26.11.2023. On such date, Stamp duty value of residential property was
₹4,14,00,000. Subsequently on 18.02.2024, she sold her 15% share in residential
property to Mr. Keshav in consideration of ₹65,00,000.

She has also purchased a house on 09.05.2023 in consideration of ₹46,00,000 and


occupied for own residence. SBI has sanctioned a loan of ₹35,50,000 (being 80% of
stamp duty value) at the interest rate of 12% per annum on 01.05.2023 and
disbursement was made on 01.06.2023. She does not own any other residential house on
the date of sanction of loan. Principal amount of ₹1,30,000 was paid during the financial
year 2023-24.

Compute total income of Ms. Nehal for the assessment year 2024-25 assuming that she
has not opted provisions under section 115BAC.

Solution:

Computation of total income of Ms. Nehal for the A.Y. 2024 -25

Income from house property [Self-occupied]


Net Annual Value Nil
Less: Interest on housing loan of ₹3,55,000 [₹35,50,000 x 12% x 10/12
months] restricted to ₹2,00,000/- 2,00,000
(2,00,000)

Long-term capital gains on transfer of land under specified agreement

Since Ms. Nehal transferred her share in the project after issue of completion
certificate, capital gains on transfer of land handed over to developer under specified
agreement in the P.Y. 2018-19 would be taxable in the previous year 2023-24, being the

165 | P a g e #HakkसेCA by CA. Akash Sir


166
4. Income under head Capital Gains

year in which certificate of completion is issued as per section 45(5A). Accordingly,


capital gain arising in respect of land would be-
Full value of consideration, being 20% share in shopping mall
[Stamp duty value on the date of issue of completion certificate
(₹4,14,00,000 x 20%)] 82,80,000
Less: Indexed of cost of acquisition [₹15,00,000 x 280/167] 25,14,970
Long-term capital gain 57,65,030
Less: Deduction under section 54F
Deduction in respect of amount invested for purchase of a residential
house acquired within one year prior to date of transfer would be allowable
proportionately, since amount invested is less than the net consideration.
Accordingly, deduction would be ₹32,02,794
(₹57,65,030 x ₹46,00,000 / ₹82,80,000) (32,02,794)
Long-term capital gains 25,62,236
Less: Set-off of loss from house property
[It is beneficial to set-off loss from house property against long-term
capital gains, since in case of Ms. Nehal total income comprises of LTCG
taxable @ 20% and STCG taxable at normal slab rates; and she can claim
deduction of ₹2,80,000 under Chapter VI-A against STCG of ₹2,90,000.
Moreover, the remaining STCG would also not be taxable since it would be
below the basic exemption limit] (2,00,000)
23,62,236

Short-term capital gains

Sale of 15% share in residential property


[short-term capital asset, since held for not more than 24 months]
Full value of consideration 65,00,000
Less: Cost of acquisition, being the full value of consideration taxable on
transfer of land [₹4,14,00,000 x 15%] 62,10,000
Short-term capital gains 2,90,000
Gross Total Income 26,52,236
Less: Deductions under Chapter VI-A (allowable against short-term
capital gains of ₹ 2,90,000)
Deduction under section 80C – Repayment of principal amount
Of housing loan 1,30,000
Deduction under section 80EEA – Ms. Mishika would be eligible
for deduction of interest on housing loan
(₹3,55,000 - ₹2,00,000 = ₹1,55,000) to the extent
of ₹1,50,000, since stamp duty value of the house does not
exceed ₹45,00,000 [being ₹44,37,500 (₹35,50,000 x 100/80)]
and she does not own any other residential house on the date of

166 | P a g e #HakkसेCA by CA. Akash Sir


167
4. Income under head Capital Gains

sanction of loan. 1,50,000


(2,80,000)
Total Income 23,72,236
Total Income (rounded off) 23,72,240

TP: 15 Buyback of shares or specified securities [Section 46A]

When a company calls back its shares from the shareholders and cancels those
shares, it is called buyback of shares. It is a situation wherein company reduces its
share capital by paying of shareholder. Tax treatment is as below:

1. Buyback of shares/ Employee stock option (ESOP) being shares issued by


Domestic company (Listed or not):

In hands of the company: Additional income tax @ 23.296% (20% + 12%


surcharge + 4% HEC) [Section 115QA]

In hands of shareholder: Exempt u/s. 10(34A)

2. Buyback of shares issued by other than domestic company or buyback of


Employee stock option (ESOP) not being shares issued by any company:

In hands of the company: No tax payable

In hands of shareholder: Capital gain = Full value of consideration (FVC) – Cost


of acquisition (COA) in normal manner as calculated u/s. 48.

Taxation provisions in respect of buyback

(1) (2) (3) (4)


Taxability in the Buyback of Buyback of shares Buyback of
hands of shares by by a company, specified securities
domestic other than a by any company
companies domestic company
Company Subject to Not subject to tax Not subject to tax
additional income- in the hands of the in the hands of the
tax @ 23.296%. company. company.
Shareholder / Income arising to Income arising to Income arising to
holder of shareholders shareholder taxable holder of specified
specified exempt under as capital gains u/s securities taxable
securities section 10(34A) 46A. as capital gains u/s
46A.

167 | P a g e #HakkसेCA by CA. Akash Sir


168
4. Income under head Capital Gains

Transaction Year of Full value of Cost of Period of


taxability consideration acquisition holding
Buyback of Buyback takes Buyback price Issue price NA
shares by place (including
domestic co. premium)

For example: Anaya Ltd. issued 1,00,000 equity shares on 01.04.2003 to its
shareholder @ ₹10 each at a premium of ₹2 per share. Miss Manavi, one of the
shareholders, subscribed to 20,000 shares. On 01.05.2023, Anaya Ltd. bought back its
10% of the shares from all its shareholder @ ₹200 per shares on proportionate basis.
In this case, no tax shall be paid by Miss Manavi on surrender of 2,000 shares.
Anaya Ltd. shall pay additional income tax @ 23.296% on buyback income. Buyback
income shall be calculated as difference between buyback price and issue price which is
₹18,80,000 (i.e. ₹20,00,000 – ₹1,20,000). Additional income tax to be paid by Anaya
Ltd. is ₹4,37,965.

TP: 16 Distribution of asset on Liquidation [Section 46]

To be discussed in chapter “income from other sources”

TP: 17 Capital gain in case of Depreciable Asset [Section 50]

Meaning of depreciable asset: It means those capital assets on which depreciation


is charged in profit and loss account for tax purpose.

Case 1: If (Full value of consideration – Transfer exp) > (Opening wdv +


Purchase during the year), then always SHORT TERM CAPITAL GAIN.

Full value of consideration 15,00,000


Less: Transfer expenses (1,00,000)
Less: Cost of acquisition being (opening wdv + purchase) (12,00,000)
Short term capital gain 2,00,000

Case 2: If (Full value of consideration – Transfer exp) < (Opening wdv +


Purchase during the year) and Block does not exist, then always SHORT TERM
CAPITAL LOSS.

Full value of consideration (Plant 1 and 2) 10,00,000


Less: Transfer expenses (1,00,000)

168 | P a g e #HakkसेCA by CA. Akash Sir


169
4. Income under head Capital Gains

Less: Cost of acquisition (opening wdv + purchase) (Plant 1 and 2) (12,00,000)


Short term capital loss 3,00,000

Case 3: If (Full value of consideration – Transfer exp) < (Opening wdv +


Purchase during the year) and Block exist, then no capital gain/ loss.

In other words, under section 50, capital gain from sale of depreciable assets can be
invoked only under following circumstances:
a) When one or more assets in block are sold for consideration more than the value
of block – STCG
b) When all assets are transferred for a consideration more than the value of block –
STCG
c) When all assets are transferred for a consideration less than value of block –
STCL

Note: Capital gain or loss shall never arise if some of assets are sold for a
consideration less than value of block. In this case, depreciation shall be calculated.

For example:

Local Ltd. has one plant and machinery on 01.04.2023 with w.d.v. ₹6,00,000 and it was
acquired by the company on 01.04.2010 and the plant was sold on 01.01.2024 for
₹11,00,000 and selling expenses are ₹30,000, in this case, capital gains shall be
computed in the manner given below:
Full value of consideration 11,00,000
Less:
(i) Written down value of the asset in the beginning of the year (6,00,000)
(ii) Selling expenses (30,000)
Short Term Capital Gains 4,70,000

Illustration: 24

Mr. Tanmay has the following Assets which are eligible for depreciation at 15% on
Written Down Value (WDV) basis:
01.04.2020 WDV of plant ‘X’ and Plant ‘Y’ ₹2,00,000
10.12.2023 Acquired a new plant ‘Z’ for ₹2,00,000
22.01.2024 Sold plant ‘Y’ for ₹4,00,000
Expenditure incurred in connection with transfer ₹10,000
Compute eligible depreciation claim/chargeable capital gain if any, for the Assessment
Year 2024-25.

169 | P a g e #HakkसेCA by CA. Akash Sir


170
4. Income under head Capital Gains

Solution:

Computation of depreciation and capital gains of Mr. Tanmay for the A.Y. 2024-
25

W.D.V. of Plant ‘X’ & Plant ‘Y’ as on 01.04.2020 2,00,000


Less: Depreciation @ 15% for the assessment year 2021-22 (30,000)
W.D.V. of Plant ‘X’ & Plant ‘Y’ as on 01.04.2021 1,70,000
Less: Depreciation @ 15% for the assessment year 2022-23 (25,500)
W.D.V. of Plant ‘X’ & Plant ‘Y’ as on 01.04.2022 1,44,500
Less: Depreciation @ 15% for the assessment year 2023-24 (21,675)
W.D.V. of Plant ‘X’ & Plant ‘Y’ as on 01.04.2023 1,22,825

Cost of new Plant ‘Z’ acquired during the previous year


ending on 31.03.2024 2,00,000

Full value of consideration (Sale proceeds of Plant ‘Y’) 4,00,000


Less: Deduction under section 50
W.D.V of the block (3,22,825)
Expenditure incurred in connection with transfer (10,000) (3,32,825)
Short term Capital Gains 67,175

No depreciation since block value ceases to exist.

TP: 18 Capital Gains in case of Specified Mutual Fund or


Market Linked Debentures [Section 50AA]

Capital gain arising from the transfer or redemption or maturity of unit of a


Specified Mutual Fund acquired on or after 01.04.2023 or Market Linked Debenture
would be deemed to be Short Term Capital Gains and chargeable at normal rate of
tax.

Computation of capital gains:

Full value of consideration being amount received as a result of


transfer or redemption xxx
Less: Cost of acquisition being expenditure incurred on purchase xxx
Less: Expenditure incurred on transfer (STT not allowed) xxx
Short term capital gains xxx

170 | P a g e #HakkसेCA by CA. Akash Sir


171
4. Income under head Capital Gains

Meaning of Specified mutual fund: For the purposes of section 50AA, “specified
mutual fund” means a mutual fund by whatever name called, where not more than 35%
of its total proceeds is invested in the equity shares of domestic companies.

Meaning of Market linked Debenture: “Market Linked Debenture" means a security


by whatever name called, which has an underlying principal component in the form of
a debt security and where the returns are linked to market returns on other
underlying securities or indices and include any security classified or regulated as a
market linked debenture by the Securities and Exchange Board of India;

TP: 19 Slump sale [Section 50B]

Meaning of slump sale [Section 2(42C)]: Slump sale means transfer of one or more
undertakings as a result of sale for a lumpsum consideration without assigning value
to the individual asset or liabilities in such sale.

Capital gain on slump sale = Full value of consideration (-) cost of acquisition (-)
transfer expenses

Full value of consideration = Fair market value of capital asset as on the date of
transfer.

Fair market value = Higher of


a) Fair market value of the assets transferred under slump sale (as per rules)
or,
b) Fair market value of the consideration (money or asset) received.

Cost of acquisition = Net-worth of undertaking or division

Net worth = Aggregate value of assets (before revaluation) (-) Aggregate


liabilities (as per books)

Aggregate value of asset means

Nature of asset Value


In case of depreciable asset WDV of the asset
Asset on which 100% deduction is claimed Nil
Self generated Goodwill Nil
Other assets Book value

Long term capital gain or Short-term capital gain?

171 | P a g e #HakkसेCA by CA. Akash Sir


172
4. Income under head Capital Gains

Period of holding of > 36 months <= 36 months


undertaking
Capital gain Long term Short term
Rate of tax 20% (without index) Normal slab rate

For example: Good Ltd. has sold one of its divisions on 01.11.2023 for ₹45,00,000
and its net worth on 01.11.2023 was ₹30,00,000 and it was setup in 2005, in this case
there is long term capital gain of ₹15,00,000.

Illustration: 25

Mr. Anil is a proprietor of ABC Enterprises having 2 units started on 01.04.2014. He


transferred on 01.04.2023 his unit 1 by way of slump sale for a total consideration of
₹45 Lacs. The expenses are incurred for this transfer were ₹65,000/-. His Balance
Sheet as on 31.03.2023 is as under:

Liabilities Total Assets Unit 1 Unit 2 Total


₹ ₹ ₹ ₹
Own Capital 21,00,000 Building 15,00,000 4,00,000 19,00,000
Revaluation Reserve 6,00,000 Machiner 5,00,000 2,00,000 7,00,000
(for building of unit 1 ) y
Bank Loan 4,00,000 Debtors 3,00,000 70,000 3,70,000
(70% for unit 1)
Trade creditors ) 3,10,000 Other 3,50,000 90,000 4,40,000
(25% for unit 1) assets
Total 34,10,000 Total 26,50,000 7,60,000 34,10,000

Other information:
(i) Revaluation reserve is created by revising upward the value of the building of unit 1.
(ii) No individual value of any asset is considered in the transfer deed.
Compute the capital gain for the assessment year 2024-25.

Solution:

Computation of capital gains on slump sale of Unit 1

Full value of consideration (Sale value) 45,00,000


Less: Expenses on transfer (65,000)
Less: Net worth (See Note (i) below) (16,92,500)
Long term capital gain 27,42,500

172 | P a g e #HakkसेCA by CA. Akash Sir


173
4. Income under head Capital Gains

Note (i): Computation of net worth of Unit 1 of ABC Enterprises


Building (excluding ₹6 lakhs on account of revaluation) 9,00,000
Machinery 5,00,000
Debtors 3,00,000
Other assets 3,50,000
Total assets 20,50,000
Less:
Bank Loan (2,80,000)
Creditors (77,500)
Net worth 16,92,500

HW Question: 20

Mr. Bansal is a proprietor of Bansal Electricals since 20.05.2021. He has transferred


his shop by way of slump sale for a total consideration of ₹40 Lakh. The professional
fees & brokerage paid for this sale are ₹80,000. His Balance Sheet as on 31.03.2024 is
as under:

Liabilities ₹ Assets ₹
Own Capital 10,50,000 Building 5,00,000
Bank Loan 5,00,000 Furniture 5,00,000
Trade Creditors 2,50,000 Debtors 2,00,000
Unsecured Loan 2,00,000 Other Assets 8,00,000
Total 20,00,000 Total 20,00,000

Other Information:
1. No individual value of any asset is considered in the transfer deed.
2. Other assets include trademarks valuing ₹2,00,000 as on 01.04.2023 on which no
depreciation has been provided.
3. Furniture of ₹1,50,000 purchased on 05.11.2023 on which no depreciation has been
provided.
4. Unsecured loan includes ₹50,000 as advance received from his wife, which she has
agreed to waive off.
Compute the capital gain for A.Y. 2024-25.

HW Question: 21

Mr. Akash is a proprietor of #HakkसेCA having 2 units. He transferred on 01.04.2023


his unit 1 by way of slump sale for a total consideration of ₹25 lakhs. The fair market
value of capital assets of unit 1 on 01.04.2023 is ₹30 lakhs. Unit 1 was started in the
year 2007-08. The expenses incurred for this transfer were ₹28,000. His balance
sheet as on 31.03.2023 is as under:

173 | P a g e #HakkसेCA by CA. Akash Sir


174
4. Income under head Capital Gains

Liabilities Total Assets Unit 1 Unit 2 Total


₹ ₹ ₹ ₹
Own Capital 15,00,000 Building 12,00,000 2,00,000 14,00,000
Revaluation 3,00,000 Machinery 3,00,000 1,00,000 4,00,000
Reserve
(for building of
unit 1) 2,00,000 Debtors 1,00,000 40,000 1,40,000
Bank Loan
(70% for unit 1) 1,50,000 Other 1,50,000 60,000 2,10,000
Trade creditors assets
(25% for unit 1
Total 21,50,000 Total 17,50,000 4,00,000 21,50,000

Other information:

(i) Revaluation reserve is created by revising upward the value of the building of Unit 1.
(ii) No individual value of any asset is considered in the transfer deed.
(iii) Other assets of Unit 1 include patents acquired on 01.04.2021 for ₹50,000 on
which no depreciation has been charged. (Rate of depreciation is 25% on wdv basis)

Compute the capital gain for assessment year 2024-25.

TP: 20 Full Value of Consideration in case of transfer of


immovable property [Section 50C]

➢ In case of transfer of capital asset being land or building or both, if


consideration is less than stamp duty value, then stamp duty value shall be full
value of consideration of such transfer.

For example: Mr. Suyash has sold his house on 01.05.2023 for ₹5,00,000. It was
purchased by him on 01.06.2021 for ₹2,00,000. Stamp duty value of the house as on
01.05.2023 was ₹8,00,000. In this case, Full value of consideration shall be
₹8,00,000 and not ₹5,00,000. Short term capital gain shall be ₹6,00,000.

➢ If date of agreement and date of registration is not same, then stamp duty value
as on date of agreement can be taken provided that the whole amount of
consideration or part therefore has been paid by A/c. payee cheque, account
payee DD or through use of electronic clearing system (ECS) or any other mode
prescribed ON or BEFORE the date of agreement.

174 | P a g e #HakkसेCA by CA. Akash Sir


175
4. Income under head Capital Gains

For example: Mr. Suyash has entered into an agreement to sell his house on
01.05.2022 for ₹5,00,000. The sale deed (registration) was made on 01.05.2023. The
house was purchased by him on 01.06.2021 for ₹2,00,000. Stamp duty value of the
house as on 01.05.2022 was ₹7,00,000 and on 01.05.2023 was ₹8,00,000. In this
case, if part payment is being made on or before 01.05.2022 by account payee
cheque, then full value of consideration shall be ₹7,00,000 and not ₹5,00,000 and
short-term capital gain shall be ₹5,00,000. If part payment is made in cash or bearer
cheque, then full value of consideration shall be ₹8,00,000 only and STCG shall be
₹6,00,000.

➢ Safe harbour rule: If stamp duty value does not exceed 110% of the
consideration, then actual sale consideration shall be deemed to be full value of
consideration.

For example: Mr. Suyash has sold his house on 01.05.2023 for ₹5,00,000. It was
purchased by him on 01.06.2021 for ₹2,00,000. Stamp duty value of the house as on
01.05.2023 was ₹5,45,000. In this case, Full value of consideration shall be
₹5,00,000 since stamp duty value (i.e. ₹5,45,000) does not exceed 110% of actual
sale consideration (i.e. 110% of ₹5,00,000). Short term capital gain shall be
₹3,00,000.

➢ If assessee claims that the stamp duty value is more than fair market value, then
assessing officer may transfer matter to valuation officer provided valuation is
not under dispute in court.
If value adopted by valuation officer is more than stamp duty value then FVC =
SDV otherwise value determined by valuation officer shall be FVC.

TP: 21 Full Value of Consideration in case of transfer of unlisted


shares [Section 50CA]

In case of transfer of unlisted shares, full value of consideration shall be FMV if


consideration is less than FMV.

Rules to determine FMV are not is syllabus.

TP: 22 Full Value of Consideration in case consideration is not


ascertainable

In case where the consideration received or receivable in not ascertainable in money


term, then full value of consideration shall be FMV of the asset transferred.

175 | P a g e #HakkसेCA by CA. Akash Sir


176
4. Income under head Capital Gains

TP: 23 Advance money forfeited [Section 51 & Section 56(2)(ix)]

It is possible for an assessee to receive some advance in regard to the transfer of


capital asset. Due to the breakdown of the negotiation, the assessee may have
retained the advance. This is called forfeiture of advance.

Advance money received on or before 01.04.2014

Such advance money forfeited shall be reduced from cost of acquisition while
calculating capital gain in the year in which asset is actually transferred.

For example: A house was purchased on 01.05.2008 for ₹4,50,000 and was used as a
residence by the owner. The owner had contracted to sell this property in June, 2013
for ₹15 lacs and had received an advance of ₹70,000 towards sale. The intending
purchaser did not proceed with the transaction and the advance was forfeited by
the owner. The property was sold in December, 2023 for ₹19,00,000. In this case,
there shall not be any tax in the year of forfeiture. During P.Y. 2023-24, COA of
house shall be reduced by ₹70,000 and COA shall be ₹3,80,000.

Advance money received after 01.04.2014

Such advance money forfeited shall be taxed under section 56(2)(ix) under head
other sources in the year of forfeiture.

For example: A house was purchased on 01.05.2008 for ₹4,50,000 and was used as a
residence by the owner. The owner had contracted to sell this property in June, 2016
for ₹15 lacs and had received an advance of ₹70,000 towards sale. The intending
purchaser did not proceed with the transaction and the advance was forfeited by
the owner. The property was sold in December, 2023 for ₹19,00,000. In this case,
advance money forfeited of ₹70,000 shall be taxed under head other sources during
P.Y. 2016-17. During P.Y. 2023-24, COA of house shall be ₹4,50,000.

HW Question: 22

Compute the net taxable capital gains and tax liability of Smt. X (under old tax regime)
on the basis of the following information :
A house was purchased on 01.05.2008 for ₹4,50,000 and was used as a residence by
the owner. The owner had contracted to sell this property in June, 2023 for ₹15 lacs
and had received an advance of ₹70,000 towards sale. The intending purchaser did not
proceed with the transaction and the advance was forfeited by the owner. The
property was sold in December, 2023 for ₹19,00,000.

176 | P a g e #HakkसेCA by CA. Akash Sir


177
4. Income under head Capital Gains

TP: 24 Exemptions from capital gain

Mann Ki Baat (simple language):

We have studied about various types of assets, rates of taxes, computation of gains
under various circumstances. Now, we need to study about the exemptions from those
capital gains provided to assessee as an incentive to make further investments from
sale proceeds. Let’s start.

1. Compulsory acquisition of urban agricultural land [Section 10(37)]


➢ Capital gain arising on compulsory acquisition of agricultural land situated in
urban area is exempt from capital gain tax.
➢ Capital gain shall arise in hands of Individual or HUF
➢ Land should have been used for agricultural purpose during preceding two
years immediately preceding the date of transfer by individual or parents or
HUF.

For example: Mr. Jagmal has urban agricultural land of ₹30,00,000 in Delhi which
has been used by him for agriculture during last 3 years. The Govt. has
compulsory acquired the land as a part of national highway project and provided
compensation of ₹70,00,000. In this case, capital gain of ₹40,00,000 shall be
exempt u/s. 10(37).

2. Exemption from capital gain on sale of residential house [section 54]

➢ Assessee: Individual or a Hindu Undivided Family.

➢ Asset: Capital asset transferred should be residential building or land


appurtenant, the income of which is chargeable under the head “Income from
house property”.

➢ Type of capital gain: Long term capital gain

➢ Investment: The assessee has within a period of one year before or two years
after (-1, +2) the date on which the transfer took place purchased, or has
within a period of three years after (+3) that date constructed, one
residential house in India (no exemption for house outside India).

Note: Exemption of two houses is allowed provided capital gains is upto ₹2


crores. Further such option is allowed only once in the life time of the
assessee i.e. afterwards benefit of only one house shall be allowed.

177 | P a g e #HakkसेCA by CA. Akash Sir


178
4. Income under head Capital Gains

➢ Amount of exemption: Exemption shall be allowed to be the extent of


investment but only upto ₹10 Cr.

Example:

S. No. LTCG Computed Cost of new residential house Exempt LTCG


(1) 7 Crore 12 Crore 7 Crore
(2) 12 Crore 14 Crore 10 Crore
(3) 11 Crore 9 Crore 9 Crore
(4) 15 Crore 13 Crore 10 Crore

➢ Withdrawal of exemption: The house so purchased/constructed must not be


transferred within a period of three years otherwise exemption given shall be
withdrawn and for this purpose while computing capital gains, its cost of
acquisition shall be reduced by the amount of the exemption earlier allowed.

➢ Capital gains account Scheme 1988: The amount of capital gain has to be
utilised till the last date of furnishing of return of income otherwise amount
should be deposited in capital gains account scheme 1988 and proof of such
deposit should be enclosed with the return of income. Subsequently the
amount should be withdrawn from this scheme and should be utilised for the
specified purpose otherwise it will be considered to be long term capital gain
of the year in which the prescribed period has expired. However, the
capital gain in excess of ₹10 Crore would not be taken into account for
the purpose of deposit in CGAS.

➢ Extension of time for acquiring new asset or depositing or investing


amount of capital gain section 54H: If the asset has been acquired
compulsorily by the Government, period of investment shall be determined
from the date of payment instead of the date of compulsory acquisition.

➢ If any person has purchased a house and has deposited some amount in capital
gain account scheme for further construction on the same house, in that case
exemption shall be allowed even for the amount deposited in CGAS account as
decided in B. B. Sarkar vs Commissioner Of Income-Tax (Calcutta HC)

Mann Ki Baat (simple language):

The above exemption is given to the Individual/ HUF who has sold their residential
house property (long term) and then purchased/ constructed another residential house
property within time limit. This exemption is provided as an incentive to invest in

178 | P a g e #HakkसेCA by CA. Akash Sir


179
4. Income under head Capital Gains

residential house property. Suppose Mr. X has long term capital gain of ₹30,00,000 on
sale of residential house property and he invested ₹28,00,000 on purchase of another
residential house property, in this case only ₹2,00,000 is taxable.

Illustration: 26

Mr. Amilal purchased one residential house on 01.07.2001 for ₹2,00,000 and it was sold
by him on 01.07.2023 for ₹100 lakhs and he purchased one house in 01.07.2024 for
₹20,00,000. He sold this house on 01.07.2025 for ₹22,00,000. Compute his Tax
Liability for A.Y. 2024-25 under old provisions of the Act and also capital gains for
various years.

Solution:

Computation of capital gain and tax liability for A. Y. 2024-25

Full value of consideration 1,00,00,000


Less: Indexed cost of acquisition
= 2,00,000 / 100 x 348 (6,96,000)
Long Term Capital Gains 93,04,000
Less: Exemption u/s 54 (20,00,000)
Long Term Capital Gains 73,04,000
Income under the head Capital Gain 73,04,000
Gross Total Income 73,04,000
Less: Deduction u/s 80C to 80U Nil
Total Income 73,04,000

Computation of Tax Liability


{Since normal income is nil, as per section 112 deficiency of ₹2,50,000 shall be allowed
from long term capital gain and balance income shall be taxed at flat rate of 20%}

Tax on ₹70,54,000 (₹73,04,000 – ₹2,50,000) @ 20% 14,10,800


Add: Surcharge @ 10% 1,41,080
Tax before health & education cess 15,51,880
Add: HEC @ 4% 62,075
Tax Liability 16,13,955
Rounded off u/s 288B 16,13,960

Computation of Capital Gain for the assessment year 2026-27

Capital gain on sale of House


Full value of consideration 22,00,000

179 | P a g e #HakkसेCA by CA. Akash Sir


180
4. Income under head Capital Gains

Less: Cost of acquisition (₹20,00,000 – ₹20,00,000) (Nil)


Short Term Capital Gain 22,00,000

HW Question: 23

Mr. Rohan purchased one residential house on 01.10.2002 for ₹5,00,000 and sold the
house on 01.07.2023 for ₹100,00,000 and purchased one house on 01.01.2024 for
₹20,00,000 and this house was sold by him on 01.01.2025 for ₹25,00,000.
Compute his income tax liability for assessment year 2024-25 and also capital gains for
all the years.
(b) Presume the house purchased on 01.01.2024 was sold on 31.01.2024
(c) Presume the house purchased on 01.01.2024 was purchased on 01.10.2024 and was
not sold upto 01.09.2027.
(d) Presume no house was purchased but the amount was deposited in capital gains
account scheme on 31.07.2024 and the amount remained unutilized.

HW Question: 24

Mr. Rasiq purchased one residential house on 01.04.2002 for ₹5,00,000. This house was
acquired compulsorily by the Government on 01.10.2013 and compensation of
₹50,00,000 was fixed by the government but the amount was paid by the Government
on 01.03.2024. The assessee has purchased one residential house on 01.02.2024 for
₹2,00,000 and the house was sold by him on 01.02.2025 for ₹4,00,000. Compute his tax
liability for the assessment year 2024-25 under old provisions of the Act and also
capital gains for the various years.
(b) Presume the house was purchased on 01.09.2024 instead of 01.02.2024.

Illustration: 27

Mr. Aman purchased one house on 01.04.2001 for ₹2,00,000 and sold the house on
01.07.2023 for ₹70,00,000 and purchased one house on 01.09.2023 for ₹12,00,000 and
it was sold by him on 01.01.2024 for ₹15,00,000. He is aged 82 years. Compute his
income and tax liability for assessment year 2024-25 assuming that he shifts out of
new tax regime.

Solution:

Computation of income under the head Capital Gains

Full value of consideration 70,00,000


Less: Indexed cost of acquisition
= 2,00,000 / Index of 01-02 x Index of 23-24

180 | P a g e #HakkसेCA by CA. Akash Sir


181
4. Income under head Capital Gains

= 2,00,000 / 100 x 348 (6,96,000)


Long Term Capital Gains 63,04,000

The assessee has the option either not to avail exemption under section 54 or to avail
exemption under section 54 and also it will be withdrawn.

Option I Exemption is not availed:

Long Term Capital Gain 63,04,000

Sale of house purchased on 01.09.2023


Full value of consideration 15,00,000
Less: Cost of acquisition (12,00,000)
Short term capital gain 3,00,000
Income under the head Capital Gains 66,04,000
Gross Total Income 66,04,000
Less: Deduction u/s 80C to 80U Nil
Total Income 66,04,000

Computation of Tax Liability

Tax on LTCG ₹61,04,000 (₹63,04,000 – ₹2,00,000) @ 20% 12,20,800


Tax on ₹3,00,000 at slab rate Nil
Add: Surcharge @10% 1,22,080
Tax before health & education cess 13,42,880
Add: HEC @ 4% 53,715
Tax Liability 13,96,595
Rounded off u/s 288B 13,96,600

Option II Exemption is availed

Long Term Capital Gain 63,04,000


Less: Exemption u/s 54 (12,00,000)
Long Term Capital Gain 51,04,000

Sale of house purchased on 01.09.2021


Full value of consideration 15,00,000
Less: Cost of acquisition (12,00,000 – 12,00,000) Nil
Short term capital gain 15,00,000
Income under the head Capital Gains 66,04,000
Gross Total Income 66,04,000
Less: Deduction u/s 80C to 80U Nil

181 | P a g e #HakkसेCA by CA. Akash Sir


182
4. Income under head Capital Gains

Total Income 66,04,000

Computation of Tax Liability

Tax on LTCG ₹51,04,000 @ 20% 10,20,800


Tax on ₹15,00,000 at slab rate 2,50,000
Tax before surcharge 12,70,800
Add: Surcharge @10% 1,27,080
Tax before health & education cess 13,97,880
Add: HEC @ 4% 55,915
Tax Liability 14,53,795
Rounded off u/s 288B 14,53,800

Hence, assessee should opt Option-I.

HW Question: 25

Mr. Manohar purchased a residential house on 31st July 2021 for ₹10,00,000 and made
some additions to the house incurring ₹2,00,000 in August 2021. He sold the house
property in April 2023 for ₹20,00,000. Out of the sale proceeds, he spent ₹5,00,000
to purchase another house property in September 2023. What is the amount of capital
gains taxable in the hands of Mr. Manohar for the A.Y. 2024-25?

3. Exemption from capital gain on sale of urban agricultural land [section


54B]

➢ Assessee: Individual or a Hindu Undivided Family.

➢ Asset: The asset transferred should be land which, in the two years
immediately preceding the date on which the transfer took place, was being
used by the assessee or a parent of his for agricultural purposes.

➢ Type of capital gain: Short-term or long-term capital gain

➢ Investment: The assessee has, within a period of two years after (+2) that
date, purchased any other land for being used for agricultural purposes.

➢ Amount of exemption: Exemption shall be allowed to be the extent of


investment.

➢ Withdrawal of exemption: The land so purchased must not be transferred


within a period of three years otherwise exemption given shall be withdrawn

182 | P a g e #HakkसेCA by CA. Akash Sir


183
4. Income under head Capital Gains

and for this purpose while computing capital gains, its cost of acquisition shall
be reduced by the amount of the exemption earlier allowed.

➢ Capital gains account Scheme 1988: The amount of capital gain has to be
utilised till the last date of furnishing of return of income otherwise amount
should be deposited in capital gains account scheme 1988 and proof of such
deposit should be enclosed with the return of income. Subsequently the
amount should be withdrawn from this scheme and should be utilised for the
specified purpose otherwise it will be considered to be capital gain of the year
in which the prescribed period has expired.

Mann Ki Baat (simple language):

The above exemption is given to the Individual/ HUF who has sold their urban
agricultural land (short term or long term) and then purchased another agricultural land
within time limit. This exemption is provided as an incentive to invest in agricultural
land. Suppose Mr. X has short term capital gain of ₹30,00,000 on sale of urban
agricultural land and he invested ₹28,00,000 on purchase of another agricultural land,
in this case only ₹2,00,000 is taxable.

Illustration: 28

Mr. Shyamlal purchased agricultural land in urban area on 01.10.2002 for ₹3,00,000 and
it was being used for agricultural purposes by him. It was sold on 01.01.2024 for
₹50,00,000. The assessee has purchased one agricultural land in the rural area on
10.01.2024 for ₹10,00,000 and this land was sold by him on 11.02.2024 for ₹11,00,000
and has invested ₹30,000 in National Saving Certificate. He is aged about 86 years.
Compute his tax liability for assessment year 2024-25.
(b) Presume the land was purchased in the urban area instead of rural area.

Solution (a):

Computation of Capital Gains

Full value of consideration 50,00,000


Less: Indexed cost of acquisition
= 3,00,000 / 105 x 348 (9,94,286)
Long Term Capital Gain 40,05,714
Less: Exemption u/s 54B (10,00,000)
Long Term Capital Gain 30,05,714
Income under the head Capital Gain 30,05,714
Gross Total Income 30,05,714

183 | P a g e #HakkसेCA by CA. Akash Sir


184
4. Income under head Capital Gains

Less: Deduction u/s 80C Nil


{Deduction u/s 80C is not allowed from LTCG}
Total Income 30,05,710

Computation of Tax Liability


{Since normal income is nil, as per section 112 deficiency of ₹5,00,000 shall be allowed
from long term capital gain and balance income shall be taxed at flat rate of 20%}

Tax on ₹25,05,710 (₹30,05,710 – ₹5,00,000) @ 20% 5,01,142


Add: HEC @ 4% 20,046
Tax Liability 5,21,188
Tax Liability (Rounded off u/s 288B) 5,21,190
Note: If land is purchased in rural area, exemption is allowed under section 54B but on
its sale exemption is not withdrawn.

Solution (b):

Computation of Capital Gains

Full value of consideration 50,00,000


Less: Indexed cost of acquisition
= 3,00,000 / 105 x 348 (9,94,286)
Long Term Capital Gain 40,05,714

The assessee has the option either not to avail exemption under section 54B or to avail
exemption under section 54B.

Option I Exemption is not availed:

Long Term Capital Gain 40,05,714

Urban agricultural land

Full value of consideration 11,00,000


Less: Cost of acquisition (10,00,000)
Short Term Capital Gain 1,00,000
Income under the head Capital Gains 41,05,714
Gross Total Income 41,05,714
Less: Deduction u/s 80C {NSC} (30,000)
Total Income 40,75,714
Total Income (Rounded Off u/s 288A) 40,75,710

184 | P a g e #HakkसेCA by CA. Akash Sir


185
4. Income under head Capital Gains

Computation of tax liability

Tax on long term capital gain (40,05,710 – 4,30,000) @ 20% 7,15,142


Tax on ₹70,000 at slab rate Nil
Add: HEC @ 4% 28,606
Tax Liability 7,43,748
Tax Liability (Rounded Off u/s 288B) 7,43,750

Option II Exemption is availed:

Long Term Capital Gain 40,05,714


Less: Exemption u/s 54B (10,00,000)
Long Term Capital Gain 30,05,714

Urban agricultural land

Full value of consideration 11,00,000


Less: Cost of acquisition (10,00,000 – 10,00,000) Nil
Short Term Capital Gain 11,00,000
Income under the head Capital Gains 41,05,714
Gross Total Income 41,05,714
Less: Deduction u/s 80C {NSC} (30,000)
Total Income 40,75,714
Total Income (Rounded Off u/s 288A) 40,75,710

Computation of tax liability

Tax on long term capital gain ₹30,05,714 @ 20% 6,01,143


Tax on ₹10,70,000 at slab rate 1,21,000
Tax before health & education cess 7,22,143
Add: HEC @ 4% 28,886
Tax Liability 7,51,029
Tax Liability (Rounded off u/s 288B) 7,51,030

Hence, the assessee should opt for option–I and his tax liability shall be 7,43,750.

HW Question: 26

Mr. Jonny purchased agricultural land in urban area for ₹3,00,000 on 01.10.2005 and
this land was transferred by him on 01.07.2023 for ₹32,00,000 (this agricultural land
is used for agricultural purpose since its purchase). Mr. Jonny purchased one
agricultural land on 30.09.2024 in the urban area for ₹6,00,000. The agricultural land

185 | P a g e #HakkसेCA by CA. Akash Sir


186
4. Income under head Capital Gains

was sold on 01.01.2025 for ₹10,00,000. He has one business also with turnover
₹105,00,000 and has income from business ₹1,10,000.
Compute capital gains for various years and also tax liability for assessment year 2024-
25 under old provisions of the Act.

HW Question: 27

Mr. Mohan has an agricultural land (costing ₹6 lakh) in Lucknow and has been using it
for agricultural purposes since 01.04.2003 till 01.08.2013 when the Government took
over compulsory acquisition of this land. A compensation of ₹ 10 lakh was settled. The
compensation was received by Mr. Mohan on 01.07.2023. Compute the amount of capital
gains taxable in the hands of Mr. Mohan.
Will your answer be any different if Mr. Mohan had by his own will sold this land to his
friend Mr. Soham? Explain.
Will your answer be different if Mr. Mohan had not used this land for agricultural
activities? Explain.
Will your answer be different if the land belonged to Mohan Ltd. and not Mr. Mohan
and compensation on compulsory acquisition was received by the company? Explain.

4. Exemption from capital gain on compulsory acquisition of land or


building [section 54D]

➢ Assessee: All

➢ Asset: The asset should be land or building forming part of an industrial


undertaking belonging to the assessee which, in the two years immediately
preceding the date on which the transfer took place, was being used by the
assessee for the purposes of the business of the said undertaking and further
there should be compulsory acquisition.

➢ Type of capital gain: Short-term or long-term capital gain

➢ Investment: The assessee can invest the amount in land or building for the
purpose of industrial undertaking within a period of three years after (+3) the
date of payment by the Govt.

➢ Amount of exemption: Exemption shall be allowed to be the extent of


investment.

➢ Withdrawal of exemption: The land or building so purchased/constructed


must not be transferred within a period of three years otherwise exemption
given shall be withdrawn and for this purpose while computing capital gains on

186 | P a g e #HakkसेCA by CA. Akash Sir


187
4. Income under head Capital Gains

the transfer of new asset, its cost of acquisition shall be reduced by the
amount of the exemption earlier allowed.

➢ Capital gains account Scheme 1988: The amount of capital gain has to be
utilised till the last date of furnishing of return of income otherwise amount
should be deposited in capital gains account scheme 1988 and proof of such
deposit should be enclosed with the return of income. Subsequently the
amount should be withdrawn from this scheme and should be utilised for the
specified purpose otherwise it will be considered to be capital gain of the year
in which the prescribed period has expired.

Mann Ki Baat (simple language):

The above exemption is given to all the assesses who have transferred their
industrial land or building under compulsory acquisition (short term or long term) and
then purchased another land or building for the purpose of industrial undertaking
within time limit. This exemption is provided as an incentive to invest in industrial
undertaking after old land/ building has been compulsory acquired. Suppose Mr. X has
short term capital gain of ₹30,00,000 on compulsory acquisition of land/ building
used in industrial undertaking and he invested ₹28,00,000 on purchase of another
land or building for the purpose of industry, in this case only ₹2,00,000 is taxable.

Illustration: 29

Mr. Naman has one industrial undertaking in Noida industrial area and the building
which is being used for industrial purposes was purchased on 01.10.2007. Since then it
was being used for industrial purpose and was purchased for ₹23,00,000 and its w.d.v.
as on 01.04.2014 is ₹10,38,000. This building was acquired by the Government on
01.01.2015 and compensation fixed was ₹25,00,000. Entire payment was released by
the Government on 01.07.2023. The assessee has purchased one building for the
purpose of industrial undertaking in Gazipur Industrial Area on 01.01.2024 for
₹6,00,000. Compute his tax liability for assessment year 2024-25 assuming that he has
shifted out of new tax provisions u/s. 115BAC.

Solution:

Computation of Capital Gains under section 45(5)


Capital gains shall be computed in the year of compulsory acquisition i.e. in the previous
year 2014-15

Full value of consideration 25,00,000


Less: w.d.v of the building (10,38,000)

187 | P a g e #HakkसेCA by CA. Akash Sir


188
4. Income under head Capital Gains

Short Term Capital Gain 14,62,000

Computation of capital gains and tax liability for the assessment year 2024-25
Capital gain shall be taxed in the year in which payment has been given by the
Government i.e. in the previous year 2023-24

Short Term Capital Gain 14,62,000


Less: Exemption u/s 54D (6,00,000)
Short Term Capital Gain 8,62,000
Income under the head Capital Gain 8,62,000
Gross Total Income 8,62,000
Less: Deduction u/s 80C to 80U Nil
Total Income 8,62,000

Computation of Tax Liability

Tax on ₹8,62,000 at slab rate 84,900


Add: HEC @ 4% 3,396
Tax Liability 88,296
Rounded off u/s 288B 88,300

5. Exemption from capital gain on making investment in certain bonds


[Section 54EC]

➢ Assessee: All

➢ Asset: Any land or building

➢ Type of capital gain: Long-term capital gain

➢ Investment: The assessee shall invest within a period of six months after
the date of such transfer, in the long-term specified asset. “Long-term
specified asset” means any bond redeemable after five years, issued by, —
(i) National Highways Authority of India
(ii) Rural Electrification Corporation Limited
(iii) Power Finance Corporation Limited.
(iv) Indian Railway Finance Corporation Limited.

➢ Amount of exemption: Maximum exemption allowed in a particular previous


year shall be 50 lakhs.

188 | P a g e #HakkसेCA by CA. Akash Sir


189
4. Income under head Capital Gains

➢ Withdrawal of exemption: If the long term specified asset is transferred or


converted into cash within a period of 5 years, exemption earlier allowed shall
be considered to be long term capital gains of the year in which such asset
was transferred or converted into cash.
Converting into cash means taking a loan on the security of the specified
asset.

➢ Capital gains account Scheme 1988: Not applicable.

➢ Extension of time for acquiring new asset or depositing or investing


amount of capital gain section 54H: If the asset has been acquired
compulsorily by the Government, period of investment shall be determined
from the date of payment instead of the date of compulsory acquisition.

Mann Ki Baat (simple language):

The above exemption is given to all the assesses who have transferred their land or
building (long term) and then invested in specified long term bonds within time limit.
This exemption is provided as an incentive to invest in specified bonds. Suppose Mr.
X has long term capital gain of ₹30,00,000 on sale of commercial land used and he
invested ₹28,00,000 in bonds of NHAI, in this case only ₹2,00,000 is taxable.

Illustration: 30

Mr. Saurav purchased agricultural land in the urban area on 01.04.2001 for ₹2,00,000.
It was being used for agricultural purposes since then and was sold by the assessee on
01.07.2023 for ₹1,23,00,000. He made following investments:
(i) Bonds of National Bank for Agriculture and Rural Development on 01.06.2023 for
₹1,50,000 which are redeemable after 5 years.
(ii) He purchased agricultural land on 01.09.2023 for ₹2,00,000.
(iii) He has invested ₹75,000 on 01.10.2023 in the bonds of National Highway Authority
of India redeemable after five years.
He sold the bonds of National Highway Authority of India on 15.04.2024 for
₹3,00,000.
Compute his capital gains for various years and also tax liability for assessment year
2024-25 under old tax provisions.

Solution:

Previous year 2023-24

Computation of Capital gains

189 | P a g e #HakkसेCA by CA. Akash Sir


190
4. Income under head Capital Gains

Full value of consideration 123,00,000


Less: Indexed cost of acquisition
= 2,00,000 / 100 x 348 (6,96,000)
Long Term Capital Gain 1,16,04,000
Less: Exemption u/s 54B (2,00,000)
Less: Exemption u/s 54EC (75,000)
Long Term Capital Gain 1,13,29,000
Income under the head Capital Gain 1,13,29,000
Gross Total Income 1,13,29,000
Less: Deduction u/s 80C to 80U Nil
Total Income 1,13,29,000

Computation of Tax Liability


{Since normal income is nil, as per section 112 deficiency of ₹2,50,000 shall be allowed
from long term capital gain and balance income shall be taxed at flat rate of 20%}

Tax on ₹1,10,79,000 (₹1,13,29,000 – ₹2,50,000) @ 20% 22,15,800


Tax before Surcharge 22,15,800
Add: Surcharge @ 15% 3,32,370
Tax before health & education cess 25,48,170
Add: HEC @ 4% 1,01,927
Tax Liability 26,50,097
Rounded off u/s 288B 26,50,100

Previous year 2024-25

Full value of consideration 3,00,000


Less: Cost of acquisition (75,000)
Short Term Capital Gain 2,25,000
Long Term Capital Gain (withdrawal of exemption) 75,000

HW Question: 28

Softech Ltd. purchased one commercial building on 01.07.1995 for ₹2,00,000 and paid
brokerage of ₹20,000 and its market value as on 01.04.2001 is ₹2,10,000. The company
sold the building on 01.07.2023 for ₹5,00,00,000 and invested ₹60,00,000 in bond of
NHAI redeemable after five years. Compute tax liability of the company for
Assessment Year 2024-25.
(b) Presume building was sold for ₹11,72,00,000.

190 | P a g e #HakkसेCA by CA. Akash Sir


191
4. Income under head Capital Gains

HW Question: 29

Mr. Vijendra a senior citizen (aged 65 years) sold residential building at Alwar for
₹40,00,000 on October 1st, 2023. This building was acquired by his father on
01.01.1999 for ₹1,00,000. On the death of his father on July 5th, 2006, he inherited
this building. Fair market value of this property on 01.04.2001 was ₹1,50,000. He paid
brokerage @ 1% to the real estate agent at the time of sale of the building. He
purchased a residential building at Bangalore on March 7th, 2024 for ₹8,00,000 and
deposited ₹3,00,000 on April 20th, 2024 in the bonds of National Highways authority
of India redeemable after one year. His other incomes are ₹ 50,000. He deposited
₹10,000 in public provident fund. Compute total income and tax liability of Mr. Vijendra
for the assessment year 2024-25 under old tax regime.

HW Question: 30

Mr. Bhuvam sold his residential house property on 08.06.2023 for ₹70 lakhs which was
purchased by him for ₹20 lakhs on 05.05.2006.
He paid ₹1 lakh as brokerage for the sale of said property. The stamp duty valuation
assessed by sub-registrar was ₹90 lakhs.
He bought another house property on 25.12.2023 for ₹11 lakhs.
He deposited ₹8 lakhs on 10.11.2023 in the capital gain bond of National Highway
Authority of India (NHAI).
He deposited another ₹10 lakhs on 10.07.2024 in the capital gain deposit scheme with
SBI for construction of additional floor of house property.
Compute income under the head “Capital Gains” for A.Y. 2024-25 as per Income Tax
Act, 1961 and also Income tax payable on the assumption that he has no other income
chargeable to tax.

6. Exemption from capital gain on making investment in units of mutual


fund [section 54EE]

➢ Assessee: All

➢ Asset: Any capital asset

➢ Type of capital gain: Long-term capital gain

➢ Investment: The assessee shall invest within a period of six months after
the date of such transfer, in the long-term specified asset. “Long-term
specified asset” means units of such fund as may be notified by the Central
Government.

191 | P a g e #HakkसेCA by CA. Akash Sir


192
4. Income under head Capital Gains

➢ Amount of exemption: Maximum exemption allowed in a particular previous


year shall be 50 lakhs.

➢ Withdrawal of exemption: If the long term specified asset is transferred or


converted into cash within a period of 3 years, exemption earlier allowed shall
be considered to be long term capital gains of the year in which such asset
was transferred or converted into cash.
Converting into cash means taking a loan on the security of the specified
asset.

➢ Capital gains account Scheme 1988: Not applicable.

7. Exemption from capital gain on transfer of any capital asset (other


than residential house property) [section 54F]

➢ Assessee: Individual or HUF

➢ Asset: Any capital asset other than residential house property

➢ Type of capital gain: Long-term capital gain

➢ Investment: The assessee has within a period of one year before or two years
after (-1, +2) the date on which the transfer took place purchased, or has
within a period of three years after (+3) that date constructed, one
residential house and further the assessee should either purchase or
construct only one house and also assessee should not have more than one
house in his name at the time of transfer of the asset besides the house
which is being purchased or constructed for availing exemption.

➢ Amount of exemption: Exemption allowed shall be that percentage of the


capital gain as the amount invested bears to net consideration. i.e.

Exemption = (capital gain x investment / net consideration). However, if


amount of investment i.e. cost of new residential house is more than 10
Crore, then, only upto ₹10 Cr shall be considered.
Net consideration is equal to full value of consideration less selling expenses.

Example:

S. Net LTCG Cost of new Exempt


No. Consideration Computed residential house LTCG
(1) 15 Crore 7.5 Crore 12 Crore 5 Crore

192 | P a g e #HakkसेCA by CA. Akash Sir


193
4. Income under head Capital Gains

(2) 20 Crore 12 Crore 15 Crore 6 Crore


(3) 16 Crore 12 Crore 8 Crore 6 Crore
(4) 10 Crore 6 Crore 10 Crore 6 Crore
(5) 12 Crore 6 Crore 12 Crore 5 Crore

➢ Withdrawal of exemption: The house so purchased or constructed must not


be transferred for a minimum period of three years otherwise exemption
earlier allowed shall be considered to be the long-term capital gain of the year
in which the asset has been transferred (i.e. exemption shall be withdrawn in
the similar manner as given under section 54EC). Similarly if the assessee has
purchased any other house within one year before or two years after or the
assessee has constructed any other house within three years after the date
of transfer of original asset, exemption given shall be withdrawn in that case
also.

➢ Capital gains account Scheme 1988: The amount of capital gain has to be
utilised till the last date of furnishing of return of income otherwise amount
should be deposited in capital gains account scheme 1988 and proof of such
deposit should be enclosed with the return of income. Subsequently the
amount should be withdrawn from this scheme and should be utilised for the
specified purpose otherwise it will be considered to be capital gain of the year
in which the prescribed period has expired. However, the net
consideration in excess of ₹10 Crore would not be taken into account for
the purpose of deposit in CGAS.

➢ Extension of time for acquiring new asset or depositing or investing


amount of capital gain section 54H: If the asset has been acquired
compulsorily by the Government, period of investment shall be determined
from the date of payment instead of the date of compulsory acquisition.

Illustration: 31

Mr. Ashok purchased gold on 01.04.1991 for ₹3,00,000 and its market value on
01.04.2001 is ₹2,00,000. This gold was sold by him on 01.01.2024 for ₹35,00,000 and
selling expenses are ₹37,000. He has purchased one house on 01.05.2024 for ₹4,00,000
because he did not have any house in his name and he deposited ₹3,00,000 in capital
gain account scheme on 30.09.2024.
Mr. Ashok is also engaged in a business and he has turnover of his business
₹105,00,000 and cost of goods sold ₹100,00,000 and other expenses ₹5,10,000.
He has withdrawn ₹2,00,000 from capital gain account scheme on 01.01.2025 and
constructed 1st floor of the house which was purchased by him on 01.05.2024.
Remaining amount in the capital gain account scheme was unutilized.

193 | P a g e #HakkसेCA by CA. Akash Sir


194
4. Income under head Capital Gains

Compute Assessee’s tax liability for assessment year 2024-25 and capital gains for
various years.

Solution:

Previous year 2023-24

Computation of capital gain


Full value of consideration 35,00,000
Less: Indexed cost of acquisition
= 3,00,000 / 100 x 348 (10,44,000)
Less: Selling expenses (37,000)
Long Term Capital Gain 24,19,000
Less: Exemption u/s. 54F
= 24,19,000 / 34,63,000 x 7,00,000 (4,88,969)
Long Term Capital Gain 19,30,031
Income under the head Capital Gain 19,30,031
Loss under the head Business/Profession (10,000)
Gross Total Income 19,20,031
Less: Deduction u/s 80C to 80U Nil
Total Income (rounded off u/s 288A) 19,20,030

Computation of tax liability


{Since normal income is nil, as per section 112 deficiency of ₹2,50,000 shall be allowed
from long term capital gain and balance income shall be taxed at flat rate of 20%}

Tax on ₹16,70,030 (19,20,030 – 2,50,000) @ 20% 3,34,006


Add: HEC @ 4% 13,360
Tax Liability 3,47,366
Tax Liability (rounded off u/s 288B) 3,47,370

Previous year 2026-27


Amount deposited in capital gain a/c scheme 3,00,000
Less: Amount withdrawn (2,00,000)
Balance amount 1,00,000
Long Term Capital Gain
= 24,19,000 / 34,63,000 x 1,00,000 = ₹69,853 69,853
(Proportionate exemption with regard to the unutilized amount lying in the capital gain
account scheme is chargeable to tax after expiry of period of three years.)

194 | P a g e #HakkसेCA by CA. Akash Sir


195
4. Income under head Capital Gains

HW Question: 31

On 25.04.2023, Mr. Vijay sold an urban agricultural land for ₹60,00,000 which he had
been using for agricultural purposes for several years. He acquired that land in 2000
for ₹2,50,000. The market value of such land as on 01.04.2001 was ₹5,00,000. He
purchased rural agricultural land for ₹8,00,000 on 25.06.2023 which was sold for
₹12,50,000 on 18.01.2024. A sum of ₹12,50,000 was also invested by him in purchase of
residential property on 25.07.2023. He did not own any house property before this
date. The new house property was sold on 28.03.2024 for ₹15,00,000.
Compute tax liability for assessment year 2024-25 under old tax provision.

HW Question: 32

Mr. Akshay sold gold for ₹5,50,000 on 01.10.2023 which had been acquired by him in
October, 2004 for ₹55,000. He wants to utilize the said amount of sale consideration
for purchase or construction of a new residential house. He already owns one
residential house at the time of sale of the gold on 01.10.2023.
He has deposited ₹4,00,000 under the capital gains deposit scheme with a specified
bank on 30.04.2024.
Ascertain the capital gains taxable in Mr. Akshay’s hands for assessment year 2024-25
and advise him as to what further action he has to take to avail the exemption.

Illustration: 32

Mr. Manit purchased 500 debentures on 01.07.2001 of Empire Ltd. @ ₹390 per
debenture and paid brokerage @ 1.5%. The debentures were converted into share @ 3
share for each debenture on 01.07.2011. Market value on the date of conversion was
₹170 per share. All the shares were sold on 01.07.2023 @ ₹900 per share and no
securities transaction tax has been paid and paid brokerage @ 1.5%.
A sum of ₹1,00,000 was invested in purchasing a house on 28.06.2024 because the
assessee did not have any house and ₹1,00,000 was deposited in capital gain account
scheme on 30.06.2024 for availing exemption under section 54F and ₹ 50,000 was
withdrawn on 02.07.2024 to construct first floor of the house purchased on
28.06.2024.
Compute total income and tax liability for assessment year 2024-25 and capital gains
for various years under old tax provisions.

Solution:

Computation of Capital Gains for Previous Year 2023-24 Assessment Year 2024-25

Full value of consideration (1,500 x 900) 13,50,000

195 | P a g e #HakkसेCA by CA. Akash Sir


196
4. Income under head Capital Gains

Less: Indexed cost of acquisition


= (500 x 390) / Index of 01-02 x Index of 23-24
= 1,95,000 / 100 x 348 (6,78,600)
Less: Brokerage @ 1.5%
= 1.5 % of ₹13,50,000 = ₹20,250 (20,250)
Long Term Capital Gains 6,51,150
Less: Exemption u/s 54F
= 6,51,150 / 13,29,750 x 2,00,000 (97,936)
Long Term Capital Gain 5,53,214
Gross Total Income 5,53,214
Less: Deduction u/s 80C to 80U Nil
Total Income 5,53,214
Rounded off u/s 288A 5,53,210

Computation of Tax Liability

Tax on ₹3,03,210 (₹5,53,210 – ₹2,50,000) @ 20% 60,642


Add: HEC @ 4% 2,426
Tax Liability 63,068
Rounded off u/s 288B 63,070

Assessment Year 2027-28


Proportionate capital gains for unutilized amount shall be as given below:
Long Term Capital Gain = 6,51,150 / 13,29,750 x 50,000 = ₹24,484
(Proportionate exemption with regard to the unutilised amount lying in the capital gain
account scheme is Chargeable to tax after expiry of period of three years.)

TP: 25 Reference to Valuation Officer [Section 55A read with


rule 111AA]

If the Assessing Officer is of the view that the fair market value of a capital asset
computed by the assessee is not correct, Assessing Officer may refer the valuation
to the Valuation Officer in the following circumstances:

(i) If the value in the opinion of the Assessing Officer is exceeding by


- more than 15% of the value computed by the assessee or
- more than ₹25,000 of the value computed by the assessee.

For example: Mr. X has converted one capital asset into stock in trade and its
market value computed by the assessee is ₹1,00,000 but in the opinion of the
Assessing Officer, value should be ₹1,10,000, in this case valuation cannot be
referred to the Valuation Officer. But if the value in the opinion of the Assessing

196 | P a g e #HakkसेCA by CA. Akash Sir


197
4. Income under head Capital Gains

Officer is ₹1,20,000, in this case matter can be referred to the Valuation Officer.
Similarly, if the value computed by the assessee is ₹2,00,000 but in the opinion of
the Assessing Officer value should be ₹2,27,000, matter can be referred to the
Valuation Officer.

(ii) in a case where the value of the asset has been estimated by a registered valuer,
if the Assessing Officer is of opinion that the value so claimed is at variance with its
fair market value.
“Valuation Officer” means an expert employed by Income Tax Department to
determine the value of various assets.

197 | P a g e #HakkसेCA by CA. Akash Sir


198
4. Income under head Capital Gains

Comprehensive Questions

Question: 1

Mr. Rohit (aged 56 years) sold the following assets during the previous year 2023-24:
1. He purchased one house in rural area on 01.10.1991 for ₹2,00,000 and incurred
₹50,000 on its improvement on 01.07.2000. Its market value on 01.04.2001 is
₹2,30,000. It was sold on 01.04.2023 for ₹5,00,000.
2. He purchased agricultural land in the rural area for ₹2,00,000 on 01.07.2002 and
sold it on 01.07.2023 for ₹3,00,000.
3. He purchased one T.V. for his personal use on 01.01.2003 for ₹25,000 and sold it on
30.12.2023 for ₹20,000.
4. He purchased gold on 01.07.2018 for ₹3,00,000 and sold it on 01.04.2023 for
₹4,50,000.
5. He has one motor car in his business with written down value as on 01.04.2023 of
₹2,00,000 and it was sold by him on 01.07.2023 for ₹2,50,000.
6. He purchased one house on 01.10.2002 for ₹7,00,000 and incurred ₹4,50,000 on
01.10.2012 to construct its first floor and subsequently the house was sold on
01.01.2024 for ₹90,00,000 and selling expenses were 2% of the sale price.
Compute tax liability of Mr. Rohit for the assessment year 2024-25 under regular
provisions of the Act.

Question: 2 [IFHP + Capital gain + TDS]

Mr. Mohit purchased one house property on 01.07.1992 for ₹3,00,000 and incurred
₹1,00,000 on its improvement in 1995-96 and its market value as on 01.04.2001 was
₹32,00,000 and he incurred ₹5,00,000 on its improvement in 2014-15 and sold the
house on 01.11.2023 for ₹2,00,00,000.
He purchased one commercial building on 01.04.2020 for ₹50,00,000 and it was let out
@ ₹2,00,000 p.m. to XYZ Ltd. and XYZ Ltd. has deducted tax at source. Mr. Mohit has
paid Municipal Tax of ₹20,000 p.m. Compute Income Tax Payable for A.Y. 2024-25
under old tax provisions and also amount of tax deducted at source by XYZ Ltd. and
also tax deducted at source by the person who has purchased the house property.

Question: 3 [Capital Gains]

Mr. Lokesh is the owner of the following assets:


1. He purchased gold in 1997-98 for ₹90,000 and its market value as on 01.04.2001 is
₹1,01,000.
2. He purchased equity shares in A Ltd. (listed) in 1993-94 for ₹1,92,000 and its
market value on 01.04.2001 is ₹2,00,000 and market value as on 31.01.2018 is
₹2,83,000.

198 | P a g e #HakkसेCA by CA. Akash Sir


199
4. Income under head Capital Gains

Mr. Lokesh died on 16.08.2008 and as per his will these assets were transferred to his
son Mukesh. Mr. Mukesh now sells these assets on 10.06.2023 for ₹20,00,000 and
₹3,00,000 respectively and securities transaction tax has been paid on sale of equity
shares.
Find out the amount of capital gains chargeable to tax and also tax liability for the
assessment year 2024-25 assuming that he opts out of the provisions of section
115BAC.

Question: 4 [Capital Gains]

Mr. Mihir has submitted information regarding sale of certain assets as given below:
1. He purchased one house on 01.10.1998 for ₹5,00,000 and paid brokerage ₹25,000. He
entered into an agreement to sell this house on 01.04.2001 for ₹5,10,000 but the buyer
backed out. He constructed its first floor on 01.01.2014 by incurring ₹4,00,000 and
subsequently this house was sold on 01.01.2024 for ₹1,60,00,000 and selling expenses
were ₹85,000.
2. He purchased Preference shares in ABC Ltd. on 01.07.2013 for ₹1,50,000 and sold
these shares on 31.03.2024 for ₹1,00,000.
3. He purchased one motor car for personal use on 28.02.2003 for ₹2,00,000 and sold
it on 01.04.2023 for ₹2,10,000.
4. He purchased gold ornaments on 01.10.2000 for ₹2,10,000. Its market value on
01.04.2001 is ₹2,00,000 and it was sold by him on 01.07.2023 for ₹8,00,000.
5. He purchased silver utensils on 01.07.2002 for ₹30,000 and these utensils were sold
by him on 01.01.2024 for ₹23,000.
6. He has invested ₹35,000 in the units of UTI.
Compute his income tax liability for assessment year 2024-25 under regular provisions
of the Act.

Question: 5 [Capital gains] [July – 2021 (New Course)]

Examine the taxability of Capital gains in the following scenarios for the Assessment
Year 2024-25, determine the taxable amount and rate of tax applicable:
(i) On 28th February, 2024, 10,000 shares of Ram Ltd., a listed company are sold by
Mr. Bhishwa @ 550 per share and STT was paid at the time of sale of shares. These
shares were acquired by him on 5th April, 2017 @ 395 per share by paying STT at the
time of purchase. On 31st January, 2018, the shares of Ram Ltd. were traded on a
recognized stock exchange at the Fair Market Value of ₹ 390 per Share.
(ii) Mr. Ashok is the owner of residential house which was purchased on 1st September,
2016 for ₹9,00,000. He sold the said house on 4th September, 2023 for ₹ 19,00,000.
Valuation as per stamp valuation authorities was ₹ 45,00,000. He invested ₹ 19,00,000
in NHAI Bonds on 21st March 2024.

199 | P a g e #HakkसेCA by CA. Akash Sir


200
4. Income under head Capital Gains

Question: 6 [JULY – 2021 (New Course)]

Mr. Patel is a proprietor of Star Stores since 20.05.2020. He has transferred his shop
by way of slump sale for a total consideration of ₹ 40 Lakh. The professional fees &
brokerage paid for this sale are ₹80,000. His Balance Sheet as on 31.03.2024 is as
under:
Liabilities Assets
Own Capital 10,50,000 Building 5,00,000
Bank Loan 5,00,000 Furniture 5,00,000
Trade Creditors 2,50,000 Debtors 2,00,000
Unsecured Loan 2,00,000 Other Assets 8,00,000
Total 20,00,000 Total 20,00,000

Other Information:
1. No individual value of any asset is considered in the transfer deed.
2. Other assets include trademarks valuing ₹ 2,00,000 as on 01.04.2023 on which no
depreciation has been provided.
3. Furniture of ₹ 1,50,000 purchased on 05.11.2023 on which no depreciation has been
provided.
4. Unsecured loan includes ₹ 50,000 as advance received from his wife, which she has
agreed to waive off.
Computer the capital gain for A.Y. 2024-25.

Question: 7 [Capital Gains] [Nov – 2019 (New Course)]

Mr. Rajan provides you the following details with regard to sale of certain securities by
him during F.Y. 2023-24:
(i) Sold 10,000 shares of A Ltd. on 05.04.2023 @ ₹650 per share
A Ltd. is a listed company. These shares were acquired by Mr. Rajan on 05.04.2016 @
₹100 per share. STT was paid both at the time of acquisition as well as at the time of
transfer of such shares which was affected through a recognized stock exchange. On
31.01.2018, the shares of A Ltd. were traded on a recognized stock exchange as under:
Highest price - ₹300 per share
Average price - ₹290 per share
Lowest price - ₹280 per share

(ii) Sold 1,000 units of B Mutual Fund on 20.04.2023 @ ₹50 unit


B Mutual Fund is an equity-oriented fund. These units were acquired by Mr. Rajan on
15.04.2017 @ ₹10 per unit. STT was paid only at the time of transfer of such units. On
31.01.2018, the Net Asset Value of the units of B Mutual Fund was ₹55 per unit.

(iii) Sold 100 shares of C Ltd. on 25.04.2023 @ ₹200 per share

200 | P a g e #HakkसेCA by CA. Akash Sir


201
4. Income under head Capital Gains

C Ltd. is an un-listed company. These shares were issued by the company as bonus
shares on 30.09.1997. The Fair Value of these shares as on 01.04.2001 was ₹50 per
share.
Calculate the amount chargeable to tax under the head ‘Capital Gains’ and also calculate
tax on such gains for A.Y. 2024-25 assuming that the other incomes of Mr. Rajan
exceed the maximum amount not chargeable to tax. (Ignore surcharge and cess).

Question: 8 [Capital Gains] [May – 2019 (New Course)]

Mr. Joy owned a residential house in Noida. It was acquired on 09.09.2009 for
₹30,00,000. He sold it for ₹1,57,00,000 on 07.01.2020. Mr. Joy utilized the sale
proceeds of the above property to acquire a residential house in Panchkula for
₹2,05,00,000 on 20.07.2020. The said house property was sold on 01.06.2023 and he
purchased another residential house in Delhi for ₹2,57,00,000 on 02.03.2024. The
property at Panchkula was sold for ₹3,25,00,000.
Calculate capital gains chargeable to tax for the assessment year 2020-21 and 2024-
25. All workings should form part of your answer.

Question: 9 [Capital Gains/ Reverse Mortgage] [May – 2018 (Old Course)]

Mrs. Mohini an individual, aged 68 years, mortgaged her Residential Property,


purchased for ₹3 lakhs on 01.10.2002, with a bank, under a notified reverse mortgage
scheme and was sanctioned a loan of ₹20 lakh. As per the said scheme, she was
receiving the loan amount in equal monthly instalments of ₹30,000 per month from the
bank. Mrs. Mohini was not able to repay the loan on maturity and in lieu of settlement
of the loan surrenders the residential property to the bank. Bank sold the property for
₹25 lakhs on 22.02.2024. She had no other income during the year.

Discuss the tax consequences and compute tax for the Asst. Year 2024-25.

Question: 10 [Capital Gains] [Nov – 2017]

Mr. Anil entered into an agreement with Mr. Dhawan to sell his residential house
located at Navi Mumbai on 16.08.2023 for ₹80,00,000. The sale proceeds were to be
paid in the following manner;
(i) 20% through account payee bank draft on the date of agreement.
(ii) 60% on the date of the possession of the property.
(iii) Balance after the completion of the registration of the title of the property.
Mr. Dhawan was handed over the possession of the property on 15.12.2023 and the
registration process was completed on 14.01.2024. He paid the sale proceeds as per the
sale agreement.

201 | P a g e #HakkसेCA by CA. Akash Sir


202
4. Income under head Capital Gains

The value determined by the Stamp Duty Authority on 16.08.2023 was ₹90,00,000
whereas, on 14.01.2024 it was ₹91,50,000.
Mr. Anil had acquired the property on 01.04.2001 for ₹10,00,000. After recovering the
sale proceeds from Dhawan, he purchased another residential house property for
₹35,00,000.
Compute the income under the head “Capital Gains” for the Assessment Year 2024-25
and also compute tax liability under old tax provisions.

Question: 11 [Capital Gains] [May– 2017]

Mr. Ramawatar bought a vacant Land for ₹80 lakhs in May 2004. Registration and other
expenses were 10% of the cost of land. He constructed a residential building on the
said land for ₹100 lakhs during the financial year 2006-07.
He entered into an agreement for sale of the above said residential house with Mr.
John (not a relative) in April 2016. The sale consideration was fixed at ₹700 lakhs and
on 23.4.2016, Mr. Ramawatar received ₹20 lakhs as advance in cash by executing an
agreement. The sale deed was executed and registered on 14.01.2024 for the agreed
consideration. However, the State stamp valuation authority had revised the values;
hence the value of property for stamp duty purposes was ₹780 lakhs. Mr. Ramawatar,
paid 1% as brokerage on sale consideration received.
Subsequent to sale, Mr. Ramawatar made following investments:
(i) Acquired a residential house at Delhi for ₹110 lakhs.
(ii) Acquired a residential house at London for ₹190 lakhs.
(iii) Subscribed to NHAI capital gains bond (approved under section 54EC) for ₹45
lakhs on 29.03.2024 and for 50 lakhs on 12.05.2024.
Compute the income chargeable under the head 'Capital Gains'. The choice of
exemption must beneficial to the assessee.

Question: 12 [Capital Gains] [Nov– 2016]

Mr. Piyush, a resident individual, aged 55 years, purchased 10 Plots in the financial year
2003-04 for ₹12 Lakh. On 1stApril 2004, he started a business of property dealing and
converted all 10 plots as stock in trade of his business and recorded the cost at ₹40
lakhs in his books being the fair market value on 1stApril 2004.
On 31st March 2011, he sold all 10 Plots for ₹55 Lakh and purchased a residential house
property for ₹50 Lakh. He has constructed 2 rooms in this residential house in June
2011 and has spent ₹8 Lakh.
He sold the above residential house on 5th Feb 2024, for ₹85 Lakh. The valuation
adopted by Stamp valuation authority for the payment of stamp duty was ₹128 Lakh.
On the request of Mr. Piyush, A.O. made a reference to the valuation officer. The
Valuation Officer determined the value at ₹130 Lakh. Mr. Piyush paid brokerage 1% of
sale consideration.

202 | P a g e #HakkसेCA by CA. Akash Sir


203
4. Income under head Capital Gains

Compute the total Income and total Tax liability of Mr. Piyush for the Assessment year
2024-25 under normal provisions of the Act.

Question: 13 [Capital Gains] [Nov – 2013]

Mr. Sahil sold a house, held as a capital asset, to his friend Mr. Soheb on 1st December,
2023 for a consideration of ₹25,00,000. The Sub-Registrar refused to register the
document for the said value, as according to him, stamp duty valuation based on State
Government guidelines was ₹45,00,000. Mr. Sahil preferred an appeal to the Revenue
Divisional Officer, who fixed the value of the house as ₹35,00,000 (₹22,00,000 for
land and the balance for building portion). The differential stamp duty was paid,
accepting the said value determined. Mr. Sahil had purchased the land on 1st June,
2006 for ₹5,19,000 and completed the construction of the house on January, 2021 for
₹14,00,000.
Briefly discuss the tax implications in the hands of Mr. Sahil for the assessment year
2024-25 and compute the capital gains chargeable to tax.

Question: 14 [MAY – 2013]

Mrs. X had purchased 500 equity shares in A Ltd. At a cost of ₹30 per share
(brokerage 1%) in February 1999.
She got 50 bonus shares in September 2000.
She again got 550 bonus shares by virtue of her holding on March, 2005. Fair market
value of the shares of A Ltd. on April, 2001 is ₹50. Market value as on 31.01.2018 ₹200
per share.
In January 2024, she transferred all her shares @ 300 per share (brokerage 2%).
Compute the capital gains taxable in the hands of Mrs. X for the Assessment Year
2024-25 assuming.
(a) A Ltd. is an unlisted company and securities transaction tax was not applicable at
the time of sale.
(b) A Ltd. is a listed company and the shares are sold in a recognized stock exchange
and securities transaction tax was paid at the time of sale.

203 | P a g e #HakkसेCA by CA. Akash Sir


204
5. Income from Other Sources

Income from Other Sources [Section 56 & 57]


Mann Ki Baat (simple language):

We have studied that various types of Income are taxable under respective head of
Income. However, we should understand that it is not practically possible to bifurcate all
the Income under 4 heads of income (studied already). Therefore, it is very important to
have one residuary head of Income where all the other income can be taxed and that is
why we are going to study Income from Other Sources.

TP: 1 What all incomes are taxed under the head income from
other sources?

1. Interest income
2. Dividend income
3. Casual income
4. Gift
5. Family pension
6. Payment received under keyman insurance policy to a person who is not an employee
7. Income from owning and maintaining of race horses
8. Forfeiture of advance money
9. Any other income which is not taxable under first four heads.

Mann Ki Baat (simple language):

Let’s start the Chapter with most the important topic i.e. Dividend Income. Most of us are
aware about meaning of Dividend as per our general understanding. We believe that
dividend refers to distribution of profits by a company to its shareholders. However,
Income tax has its own definition of dividend which is much wider than our general
understanding. You might wonder that why Income tax law has defined dividend in such a
manner, the answer to that question is very simple, Income tax law wants to cover all those
situations where a shareholder can take cash/ other benefits out of the company without
payment of dividend tax.

204 | P a g e #HakkसेCA by CA. Akash Sir


205
5. Income from Other Sources

TP: 2 What is Dividend? [Section 2(22)]

The term dividend has a very limited meaning under Companies Act 2013 but it has a
very wide meaning under Income Tax Act, 1961 and is called Deemed Dividend. It is
divided into 5 parts:

(i) Distribution in cash or as Assets. Section 2(22)(a)


(ii) Issue of Bonus shares etc. Section 2(22)(b)
(iii) Distribution on liquidation (Closing down). Section 2(22)(c)
(iv) Distribution on reduction of share capital. Section 2(22)(d)
(v) Loan and advance by a closely held company. Section 2(22)(e)

2(22)(a): Distribution of “Accumulated Profits, whether capitalized or not” in


form of release of assets to shareholders including other than cash

If any company has distributed any amount to its shareholders either in cash or in kind,
it will be considered to be dividend but only to the extent of accumulated profits
including capitalized profit.

For example: Balance sheet of Easy Limited as on 31.03.2023 is:

Liabilities Amount Assets Amount


Share Capital [including 12,00,000 Total assets [including 20,00,000
bonus shares of ₹5 Lacs] cash]
Reserves and surplus 4,00,000
Loans and other liabilities 4,00,000
20,00,000 20,00,000

Company distributes its assets having Book value of ₹6,00,000 to its shareholders on
01.04.2023. Market value of same is:
Case a) ₹8,00,000
Case b) ₹10,00,000

In these cases, deemed dividend shall be:


a) Deemed dividend = ₹8,00,000 (since total of accumulated profits is ₹9,00,000 i.e.
₹500,000 (bonus shares) + ₹4,00,000 (reserves and surplus))
b) Deemed dividend = ₹9,00,000 (dividend cannot exceed accumulated profits)

Note: Book value of asset shall be ignored.

205 | P a g e #HakkसेCA by CA. Akash Sir


206
5. Income from Other Sources

Mann Ki Baat (Simple Language):

From above example, please note that the shareholders of the company were planning to
take out assets of the company out of the business without payment of tax on dividend.
However, Income tax law has covered such distribution of asset as deemed dividend and
therefore, now shareholders have to pay income tax on such distribution of assets by the
company. One more point to be noted is that the deemed dividend cannot exceed the
accumulated profits available in the company. Accumulated profits include capitalized
profits which is bonus shares in our example.

2(22)(b): Distribution of “debentures”, “deposit certificates” to all


shareholders and “bonus shares” to preference shareholders

If any company has distributed debentures, debenture stocks or deposit certificates to


its shareholders or bonus shares to its preference shareholders then, it will be
considered to be dividend but only to the extent of accumulated profits including
capitalized profit. The Market value of debentures, deposit certificates or bonus
shares shall be considered as deemed dividend.

Note: Bonus shares given to equity shareholders are NOT treated as dividend.

For example: Balance sheet of Hard Limited as on 31.03.2023 is:

Liabilities Amount Assets Amount


Share Capital [including 20,00,000 Total assets [including cash] 30,00,000
preference shares of ₹8 Lacs]
Reserves and surplus 6,00,000
Loans and other liabilities 4,00,000
30,00,000 30,00,000

Company distributes 1,00,000 bonus shares to its shareholders having market value of ₹8
each.
Case a) Preference shareholders
Case b) Equity shareholders

In these cases, deemed dividend shall be:


a) Deemed dividend = ₹600,000 (since total accumulated profits is ₹600,000)
b) Deemed dividend = Zero (Issue of bonus shares to equity shareholders is not deemed
dividend)

206 | P a g e #HakkसेCA by CA. Akash Sir


207
5. Income from Other Sources

Mann Ki Baat (Simple Language):

From above example we can understand that issue of bonus shares to preference
shareholders is treated as deemed dividend but issue of bonus shares to equity
shareholders is not treated as deemed dividend. Simple logic behind this is that issue of
bonus shares to equity shareholders does not impact their total capital (equity capital +
surplus) in the company and therefore there is not any additional benefit on such issue.
However, issue of bonus shares to preference shareholders has an impact of increase in
their total capital (preference capital + bonus equity capital) and therefore it is treated as
deemed dividend in their hands.

2(22)(c): Distribution on Liquidation

If any company has distributed any amount to its shareholders in connection with its
liquidation, it will be considered to be dividend but only to the extent of accumulated
profits and any excess over it shall be considered to be full value of consideration as per
section 46 and capital gains shall be computed accordingly.

For example: Balance sheet of Important Limited as on 31.03.2023:

Liabilities Amount Assets Amount


Share Capital [1,00,000 10,00,000 Total assets [including 34,00,000
equity shares of ₹10 each] cash]
Reserves and surplus 20,00,000
Loans and other liabilities 4,00,000
34,00,000 34,00,000

Important Ltd. has 1,00,000 equity shares of ₹10 each and the company goes into
liquidation on 01.04.2023. Company has net distributable amount of ₹60 lakhs (Cash + FMV
of other assets) after discharging all the liabilities including income tax. It has
accumulated profits of ₹20 lakhs. The entire amount was distributed among the
shareholders. Mr. Good is holding 10,000 equity shares which were purchased by him on
01.03.2023 for ₹1,10,000. In this case, deemed dividend and capital gain shall be computed
as below:
Net Distributable Amount 60,00,000
Total accumulated profits 20,00,000

Share of Mr. Good in total distribution (10%) 6,00,000


Share of Mr. Good out of accumulated profits

207 | P a g e #HakkसेCA by CA. Akash Sir


208
5. Income from Other Sources

which is considered dividend u/s. 2(22)(c) (2,00,000)


Balance to be considered full value of consideration 4,00,000
Less: Cost of acquisition of shares (1,10,000)
Short term Capital Gain 2,90,000
Dividend u/s 2(22)(c) 2,00,000

Mann Ki Baat (simple language):

When a company gets dissolved, it distributes 2 things to shareholders:


1. Dividend (upto accumulated profits),
2. Capital invested (total distribution – deemed dividend).
We simply need to identify the dividend portion and capital portion. Dividend portion is
taxed as deemed dividend and capital portion is considered as “full value of consideration”
for the purpose of capital gain.

Illustration: 1

Aruna Ltd. has 1,00,000 equity shares of ₹10 each and Mr. Arun purchased 10,000 equity
shares on 01.01.2023 of ₹10 each and the company goes into liquidation on 31.07.2023 and
company has net distributable amount of ₹60 lakhs after discharging all the liabilities and
it includes accumulated profits of ₹20 lakhs and the entire amount was distributed among
the shareholders. Calculate his total income.

Solution:

Computation of total income of Mr. Arun for A.Y. 2024-25:

Income under head other sources


Deemed dividend on liquidation of Aruna Ltd.
[₹20 lakhs * 10%] 2,00,000

Income under head capital gains


Gain/ Loss on liquidation of Aruna Ltd.
Full value of consideration [₹40 Lakhs * 10%] 4,00,000
Less: Cost of acquisition 1,00,000
Short term capital Gain [Since POH is 7 months] 3,00,000

Gross Total Income/ Net Total Income 5,00,000

208 | P a g e #HakkसेCA by CA. Akash Sir


209
5. Income from Other Sources

Illustration: 2 [MAY 2008]

Mr. Anil purchased 10,000 equity shares of Anila Pvt. ltd. on 28.02.2023 for ₹1,20,000.
The company was wound up on 31.07.2023. The following is the summarized financial
position of the company as on 31.07.2023:

Liabilities Assets
1,00,000 Equity Shares 10,00,000 Land 42,00,000
General Reserve 40,00,000 Cash at bank 10,50,000
Provision for Taxation 2,50,000
52,50,000 52,50,000

The tax liability was ascertained at ₹3,00,000. The remaining assets were distributed to
the shareholders in the proportion of their shareholding. The market value of land as on
31.07.2023 is ₹1,00,00,000. The land received above was sold by Mr. Anil on 28.02.2024
for ₹15,00,000. Discuss the tax consequences in the hands of the company and Mr. Anil.

Solution:

Tax implication in the hands of the company


As per section 46, if any company is in liquidation and it has distributed its assets to its
shareholders, no capital gains shall be computed in the hands of the company hence in the
given case there is no capital gains for distribution of land to shareholders.

Tax implications in the hands of Mr. Anil (shareholder)

Mr. Anil holds 1/10th of the shareholding of the company


Market value of land received (1/10 * 1,00,00,000) 10,00,000
Cash at bank [1/10th of ₹(10,50,000 – ₹3,00,000)] 75,000
10,75,000
Less: Deemed Dividend u/s 2(22)(c) 1/10th of (₹40,00,000 - ₹50,000) (3,95,000)
Consideration for computing Capital Gain 6,80,000
Cost of acquisition of Shares 1,20,000
Short term capital gains 5,60,000

Dividend u/s 2(22)(c) of ₹3,95,000 will be Taxable in the hands of Mr. Anil (Shareholder)
during the F.Y. 2023- 24.

209 | P a g e #HakkसेCA by CA. Akash Sir


210
5. Income from Other Sources

Mr. Anil has received the land from the company for ₹10 lakh but it has been sold by him
for ₹15 lakh, in this case capital gains shall be computed in the manner given below:

Sale consideration 15,00,000


Less: Fair market value of the land on the date of distribution (10,00,000)
Short term capital gain 5,00,000

2(22)(d): Distribution on reduction of share capital

Any distribution to its shareholders by a company on the reduction of its capital, to the
extent to which the company possesses accumulated profits is considered as deemed
dividend.

For example: Balance sheet of Rohan Limited as on 31.03.2023:

Liabilities Amount Assets Amount


Share Capital [1,00,000 10,00,000 Total assets [including 22,00,000
equity shares of ₹10 each] cash]
Reserves and surplus 2,00,000
Loans and other liabilities 10,00,000
22,00,000 22,00,000

Case 1) Mr. Rohan is holding 10,000 shares in Rohan Ltd. of ₹10 each and company has paid
₹5 per share in connection with reduction of share capital, in this case ₹50,000 shall be
considered to be dividend.
Case 2) Mr. Rohan is holding 50,000 shares in Rohan Ltd. of ₹10 each and company has paid
₹5 per share in connection with reduction of share capital, in this case ₹2,00,000 shall be
considered to be dividend (to the extent of accumulated profits including capitalized
profits).

Mann Ki Baat (simple language):

Generally, company reduces its share capital as a part of restructuring or to distribute


excess cash to the shareholders. Capital reduction could be a better way to distribute
cash without payment of income tax on dividend. Therefore, Income tax Act has defined
capital reduction as deemed dividend to curb the opportunity of tax evasion.

210 | P a g e #HakkसेCA by CA. Akash Sir


211
5. Income from Other Sources

2(22)(e): Loan and advance by a closely held company

(i) If any closely held company (also called company in which public are not
substantially interested) has given any loan or advance to an equity shareholder who is
holding not less than 10% of the voting power of the company, in such cases such loan or
advance shall be considered to be dividend in the hands of such shareholder but only to
the extent of accumulated profits excluding capitalized profits.

For example: Mahadev Pvt. Ltd. a closely held company has general reserves of
₹7,00,000 and current profits of ₹2,00,000. The company has given a loan of ₹3,00,000
to one such shareholder, Mr. Ganesh who is holding 12% of equity shares of the company.
In this case, it will be considered to be dividend in the hands of Mr. Ganesh. If loan
given by the company is ₹10,00,000, the amount of dividend shall be ₹9,00,000.

(ii) If the loan or advance has been given to any concern (Partnership firm, company,
AOP, BOI etc.) in which such a shareholder has substantial interest (20% or more), such
loan or advance shall also be considered to be dividend in the hands of such concern but
only to the extent of accumulated profits excluding capitalized profits.

For example: Mr. Ganesh is the beneficial owner of 10% equity shares in Mahadev Pvt.
Ltd. (A closely held company) and the company has general reserve of ₹10,00,000 and
has given a loan of ₹6,00,000 to a partnership firm Sitaram in which Mr. Ganesh is
holding 20% shares. In this case, the loan so given shall be considered to be dividend in
the hands of partnership firm.

Illustration: 3 [NOV 2016]

Mr. Mihit has 15% share-holding in IPL (P) Ltd. and has also 50% share in Mihit & Sons, a
partnership firm. The accumulated profit of IPL (P) Ltd. is ₹20 Lakh. Mihit & Sons had
taken a loan of ₹25 Lakh, from IPL (P) Ltd. Explain, whether the above loan is treated as
dividend, as per the provision of Income Tax Act, 1961.

Solution:

As per Section 2(22)(e), if the loan or advance has been given to any concern in which
shareholder has substantial interest, such loan or advance shall be considered to be
dividend in the hands of such concern but only to the extent of accumulated profits
excluding capitalized profits.
In this case dividend in the hands of the shareholder is nil and in hands of the firm are
₹20 lakhs.

211 | P a g e #HakkसेCA by CA. Akash Sir


212
5. Income from Other Sources

(iii) Where a loan had been treated as dividend and subsequently, the company
declares and distributes dividend to all its shareholders including the borrowing
shareholder, and the dividend so paid is set off by the company against the previous
borrowing, the adjusted amount will not be again treated as a dividend.

For example: Mr. Ganesh is holding 10% shares in Mahadev private limited a closely held
company and has taken a loan of 10,00,000 and it was considered to be dividend under
section 2(22)(e) and in subsequent year the company has declared dividend of
₹12,00,000 which was deposited in the loan account of Mr. Ganesh. In this case, only
₹2,00,000 will be considered as dividend.

(iv) If any such company has the business of lending as substantial part of its
business, in such cases the above provisions shall not apply.

For example: Mahadev Pvt. Ltd. is a closely held company and is engaged in banking
business (lending of money), in this case section 2(22)(e) is not applicable for Mahadev
Pvt. Ltd.

(v) As per section 2(22)(e), if any trade advance is given to the shareholder covered
under section 2(22)(e), it will not be considered to be dividend.

For example: Mr. Ganesh is holding 10% share in XYZ private limited a closely held
company and Mr. Ganesh is supplying certain goods to the company and has received
some trade advance, it will not be considered as dividend.

Mann Ki Baat (simple language):

Companies may advance amount to shareholders and do not declare dividend even if surplus
is available with the company. It could be best way to take out cash out of business
without payment of income tax on dividend. To curb this opportunity of tax evasion,
Income tax Act has defined such loans as deemed dividend.

Illustration: 4

Sohil, a resident Indian, holding 28% of equity shares in a company, took a loan of
₹5,00,000 from the same company. On the date of granting the loan, the company had
accumulated profit of ₹4,00,000. The company is engaged in some manufacturing activity.

212 | P a g e #HakkसेCA by CA. Akash Sir


213
5. Income from Other Sources

(i) Is the amount of loan taxable as deemed dividend, if the company is a company in which
the public are substantially interested?
(ii) What would be your answer, if the company is a private limited company (i.e. which is
not a company in which the public are substantially interested)?

Solution:

Any payment by a company, other than a company in which the public are substantially
interested, of any sum by way of advance or loan to an equity shareholder who is the
beneficial owner of shares holding not less than 10% of the voting power, is deemed as
dividend under section 2(22)(e), to the extent the company possesses accumulated profits.

(i) The provisions of section 2(22)(e), however, will not apply where the loan is given by a
company in which public are substantially interested. In such a case, the loan would not be
taxable as deemed dividend.

(ii) However, if the loan is taken from a private company (i.e. a company in which the public
are not substantially interested), which is a manufacturing company and not a company
where lending of money is a substantial part of the business of the company, then, the
provisions of section 2(22)(e) would be attracted, since Sohil holds more than 10% of the
equity shares in the company. The amount chargeable as deemed dividend cannot, however,
exceed the accumulated profits held by the company on the date of giving the loan.
Therefore, the amount taxable as deemed dividend would be limited to the accumulated
profit i.e., ₹4,00,000 and not the amount of loan which is ₹5,00,000.

TP: 3 Casual Income [Section 56(2)(ib)]

Casual income means income in the nature of winnings from lotteries, crossword puzzles,
races including horse races, card games and other games of any sort, gambling, betting
etc. Casual income is taxable under head other sources as per section 56(2)(ib).

Such winnings are chargeable to tax at a flat rate of 30% under section 115BB and tax is
deductible at source (TDS) @ 30% on such income in case it exceeds ₹10,000 as per
section 194B/ 194BB (covered under TDS chapter).

However, income by way of winnings from any online game would not be taxed under
section 115BB with effect from A.Y. 2024-25. A new section has been inserted (i.e.
Section 115BBJ) which is similar to Section 115BB which provides taxation of online
game income.

213 | P a g e #HakkसेCA by CA. Akash Sir


214
5. Income from Other Sources

For example: Mr. Manmohan won ₹2,00,000 from camel races. It shall be casual income in
his hands taxable @ 30%.

Additional points to remember:

1. No deduction/ expense shall be allowed from casual income.


2. Rebate u/s. 87A shall be allowed from casual income.
3. Benefit of unexhausted basic exemption limit shall not be allowed.
4. Casual loss is not allowed to be calculated and therefore, there is no question of set-
off or carry forward and set-off.

For example: Mr. Ashish purchased one lottery ticket of ₹10,000 and there was a
winning of ₹1,20,000. Deductions allowed under section 80C to 80U ₹1,00,000. His
total income and tax liability under old tax provisions shall be:

Winning from lottery tickets 1,20,000


Less: Expense on purchase 0
Taxable income 1,20,000

Tax on ₹1,20,000 @ 30% 36,000


Less: Rebate u/s. 87A (12,500)
Tax before HEC 23,500
Add: HEC @ 4% 940
Tax Liability 24,440

Note: No expenditure or deduction or loss is allowed to be adjusted from casual


income, however, rebate is allowed from tax of casual income.

TP: 4 Income from Owning and Maintaining of Race Horses [Section


56]

Income from owning and maintaining of Race Horses:

If any person has income from owning and maintaining of race horses, such income shall
be taxable under the head other sources and income shall be computed in the normal
manner and will be taxed at the normal rates.

214 | P a g e #HakkसेCA by CA. Akash Sir


215
5. Income from Other Sources

Income from owning and maintaining of any other animal:

If the assessee is engaged in the business of owning and maintaining any other animal,
his income shall be computed under the head business/ profession because section 56
includes only income from owning and maintaining race horses.

Additional points to remember (detailed discussion in set-off chapter):

1. Loss from one unit of “owning and maintaining of race horses” can only be set-off
against income from another unit of “owning and maintaining of race horses” in the
same year.
2. Un-adjusted loss from “owning and maintaining of race horses” can be carried
forward and only be set-off against income from “owing and maintaining of race
horses” in subsequent years.

Illustration: 5

Mr. Sahil has earned the following incomes during P.Y. 2023-24:
• Income from business of owning and maintaining race camels of ₹60,000
• Income from owning and maintaining of race horses of ₹10,000; and
• Income from horse races of ₹7,000.
Compute his tax liability for A.Y. 2024-25 under old tax provisions.

Solution: Computation of Total Income and tax liability for A.Y. 2024-25:

Income u/h Business/Profession:


Income from owing & maintaining race camels 60,000

Income u/h Other Sources:


Income from owning and maintaining of race horses 10,000
Income from horse races (casual income) 7,000
17,000
Total Income 77,000

Computation of Tax Liability:


Tax on casual income of ₹7,000 @ 30% 2,100
Tax on Rs. 70,000 at slab rates Nil
Less: Rebate u/s 87A (2,100)
Tax Liability Nil

215 | P a g e #HakkसेCA by CA. Akash Sir


216
5. Income from Other Sources

TP: 5 Taxability of interest income [Section 56]

Interest income shall be taxable under the head Other Sources. Interest shall be
included in the hands of the person who receives such interest (other than the
provisions of clubbing of Income).

For example: Mr. Upesh has invested ₹5,00,000 in 10% debentures issued by Super Ltd
on 01.04.2022. Interest is payable annually on 31st January every year. Due to some
urgency, Mr. Upesh has sold the debentures to Mr. Ramesh on 25.01.2024. Interest of
₹50,000 is paid by Super Ltd on 31.01.2024. In this case, the entire ₹50,000 shall be
treated as income of Mr. Ramesh.

However, some of the interest incomes shall be exempt from income tax under section
10(15) and are as given below:

1. Interest on Capital Investment Bonds issued by the Government.


2. Interest on Relief Bonds issued by RBI.
3. Interest on Post Office Savings Bank Account to the extent of ₹3,500 per year and
in the case of joint account, exemption shall be allowed up-to ₹7,000 per year.
4. Interest on Public Provident Fund Account.
5. Any other interest income notified under section 10(15).

Illustration: 6 [Other sources + TDS] Important

Mr. Akshay holds the following securities on April 1,2023:


• ₹4,30,000, 7% MP Government Bonds (interest is paid on July 15 every year);
• ₹3,80,000, 11% debentures of Ashok Ltd (non-listed) (interest is paid on June 30 every
year).
Apart from the aforesaid securities, Mr. Akshay has invested in some other securities and
received the following interest net of 10% TDS, wherever applicable:
• ₹36,000 on securities issued by UP Government;
• ₹90,000 on securities issued by Central Government of India; and
• ₹1,08,000 on debentures of Sumit Ltd.
His business income is ₹24,32,000. He pays ₹6,000 as commission to his bank for
collecting interest on securities. Determine the taxable income of Mr. Akshay for A.Y.
2024-25.

Solution:

Computation of Total Income of Mr. Akshay for A.Y. 2024-25:

216 | P a g e #HakkसेCA by CA. Akash Sir


217
5. Income from Other Sources

Particulars Amount Amount


Income u/h ‘business/profession’ 24,32,000
Income u/h ‘other sources’:
• Interest on MP govt Bonds (₹4,30,000 * 7%) 30,100
• Interest on debenture of Ashok Ltd (₹3,80,000 * 11%) 41,800
• Interest on securities issued by UP Govt 36,000
• Interest on securities issued by Central Government of
India 90,000
• Interest on debentures of Sumit Ltd (1,08,000/90%) 1,20,000
Total 3,17,900
Less: Commission paid by bank for collecting interest (6,000) 3,11,900
Total Income 21,43,900

Note: No Tax is Deducted at Source under section 193 where the interest is paid by
central government/ state government on securities issued by them.

TP: 6 Books of Accounts [Section 145(1) read with ICDS]

For capital gain, house property and salary: A person is not required to maintain any
books of accounts under the head salary or house property or capital gains and income
has to be computed as per the procedure given in the relevant head.

For PGBP and other sources: Books of accounts are required under the head
Business/Profession and under the head Other Sources. Every assessee has to prepare
their accounts on mercantile system of accounting.
However, an individual whom tax audit under section 44AB is not applicable, has the
option to maintain books of accounts either on the basis of mercantile system of
accounting or on cash basis. Any system of accounting once adopted has to be followed
consistently, however it can be changed with the permission of Assessing Officer.

For example: Mr. Gyani has deposited ₹10,00,000 in ABC Ltd. @ 10% p.a. and interest
income is due on yearly basis on 31st March every year. Interest income which was due on
31.03.2024 was received on 01.04.2024. In this case, if the assessee is maintaining books
of account on the basis of mercantile system of accounting, income is taxable in previous
year 2023-24, and if the books are maintained on cash basis, income is taxable in the
previous year 2024-25.

217 | P a g e #HakkसेCA by CA. Akash Sir


218
5. Income from Other Sources

TP: 7 Payment under Keyman Insurance Policy

PGBP: Sometimes employer may take a life policy in the name of any of his employees
who are considered to be very important for business or profession and such policy is
called keyman insurance policy and premium is paid by employer and employer is allowed
to debit it to profit and loss account and amount received on maturity shall be
considered to be income of employer as per section 28.

Salary: If any payment has been received by the employee, it will be considered to be
income under the head salary.

Other sources: Similarly, a policy may be taken in the name of any other person who is
considered to be very important for the business of the employer, such policy is also
called keyman insurance policy. If payment has been received by such other person, it
will be considered to be his income under the head other sources as per section 56.

TP: 8 Forfeiture of advance money (covered in capital gains)

If any person has entered into an agreement to sell any capital asset and some advance
money was received but the buyer refused to purchase the capital asset and advance
money was forfeited, in such cases the amount so forfeited shall be considered to be
income under the head Other Sources.

For example: Mr. Dev has entered into agreement to sell a house property for ₹50 lakh
to Miss Diwani and advance money of ₹5,00,000 was received but Miss Diwani refused to
purchase the property and advance money was forfeited, in this case ₹5,00,000 shall be
considered to be income of Mr. Dev under the head Other Sources.

TP: 9 Income of closely held company by issue of Shares [Section


56(2)(viib)]

As per section 56(2)(viib), where a private company (company not being a company in
which the public are substantially interested) receives, in any previous year, from any
person (resident or non-resident), any consideration for issue of shares that exceeds
the face value of such shares, the aggregate consideration received for such shares as
exceeds the fair market value of the shares shall be income of company under head
other sources.

218 | P a g e #HakkसेCA by CA. Akash Sir


219
5. Income from Other Sources

Mann Ki Baat (simple language):

Simply speaking, if shares are issued by a private company at a price which is higher than
the market value and also higher than the face value, in that case taxable amount shall be
the issue price less market price.

For example: Gangaram Pvt. Ltd. a closely held company has submitted information as
given below:
1. Face value ₹100 per share, Market value ₹120 per share and issue price ₹150 per share,
in this case taxable amount shall be ₹30 per share.
2. Face value ₹100 per share, Market value ₹80 per share and issue price ₹95 per share, in
this case taxable amount shall be Nil because issue price is not exceeding the face value.
3. Face value ₹100 per share, Market value ₹80 per share and issue price ₹110 per share,
in this case taxable amount shall be 110 – 80 = ₹30 per share because issue price is
exceeding the face value and also market value.

Illustration: 7 [NOV 2019]

OLX Investments (P) Ltd. was incorporated during P.Y. 2021-22 having a paid-up capital of
₹10 Lakhs. In order to increase its capital, the company further issues, 1,00,000 shares
(having face value of ₹100 each) during the year at par as on 01.08.2023. The FMV of such
share as on 01.08.2023 was ₹85.
(i) Determine the tax implications of the above transaction in the hands of company,
assuming it is the only transaction made during the year.
(ii) Will your answer change, if shares were issued at ₹105 each?
(iii) What will be your answer, if shares were issued at ₹105 and FMV of the share was
₹120 as on 01.08.2023?

Solution:

As per section 56(2)(viib), where a company, not being a company in which public are
substantially interested, receives in any previous year, from any resident person, any
consideration for issue of shares that exceeds the face value of the shares, aggregate
consideration received for such shares as exceeds the Fair market value of the shares
shall be taxable under the head other sources.

219 | P a g e #HakkसेCA by CA. Akash Sir


220
5. Income from Other Sources

(i) In the given case, shares are issued at face value and does not issue in price exceeding
face value of the shares. Hence no amount is taxable.

(ii) In the given case, shares are issued exceeding face value hence taxable amount shall
be (Issue Price – FMV of Shares) * No. of Shares = (105-85) x 1,00,000 = ₹20,00,000.

(iii) In the given case, shares are issued exceeding face value but shares are issued lower
than FMV, hence no amount shall be taxable in hands of company.

TP: 10 Bond Washing Transactions [Section 94]

If any person has transferred any security in the name of any other person sometimes
before the due date and has reacquired it sometimes after the due date in order to
evade tax, it will be considered to be a bond washing transaction and income shall be
considered to be of the person who has manipulated in this manner.

For example: Mr. Yuvraj Arora has purchased security of ₹10,00,000 in Maharaj Ltd. on
01.04.2023 @ 10% and interest is due on half yearly basis i.e. on 30th Sept and 31st
March of every year. If Mr. Yuvraj Arora has transferred this security just before the
due date in the name of any other person through a fictitious sale transaction and has
re-transferred it in his name after the due date through a fictitious purchase
transaction so that he can evade tax, it will be called bond washing transaction and in
such cases interest income is taxable in the hands of Mr. Yuvraj Arora.

[This practice is generally adopted by high-income class assessees to evade the tax by
transferring securities to low-income class assessee on the eve of due date of payment
of interest.]

TP: 11 Taxability of compensation or other payment in connection


with employment.

As per section 56(2)(xi), any compensation or other payment, due to or received by any
person, by whatever name called, in connection with the termination of his employment
or the modification of the terms and conditions relating thereto, shall be taxable under
the head other sources.

220 | P a g e #HakkसेCA by CA. Akash Sir


221
5. Income from Other Sources

Mann Ki Baat (simple language):

Any payment received in connection with termination of employment (not in lieu of services
offered as employee), it will be taxable under head other sources.

TP: 12 Taxability of sum received under a LIP which is not exempt


u/s. 10(10D)

Taxability of sum received under a LIP which is not exempt u/s 10(10D)
Where any sum is received (including the amount allocated by way of bonus) at any time
during a previous year, under a life insurance policy, other than the sum
(i) Received under ULIP
(ii) Received under a Keyman insurance policy
Which is not exempt under section 10(10D), the sum so received as exceeds the
aggregate of the premium paid during the term of such life insurance policy, and not
claimed as deduction under any other provision of the Act, computed in the prescribed
manner, would be chargeable to tax under the head “ Income from other sources”.

TP: 13 Deductions allowable under [Section 57]

While computing income under the head other sources, expenses incurred in connection
with earning of such income shall be allowed to be deducted.

However, in case of dividend income or income in respect of units of mutual fund


specified u/s 10(23D) or units of UTI, deduction shall be allowed only for the interest
expenses and that too shall also be restricted to 20% of the such income.

For example: Mr. X has taken a loan of ₹10 Lakh and paid interest ₹1 lakh and ₹5,000
as bank fees. The amount was invested in shares of a company and dividend received is
₹2 lakh, in this case interest of ₹1 lakh shall not be allowed to be deducted rather
amount allowed to be deducted shall be ₹40,000 (20% of dividend) and income shall be
considered to be ₹1,60,000.

Illustration: 8

Mr. RAM has taken a loan of ₹1,00,000 @ 10%. The amount was invested by him in the
securities of one company. During the year he has received gross interest of ₹18,000 and
has paid collection charges to the bank ₹500. He has paid interest ₹10,000 on the loan
taken by him for investment and has long term capital gain under section 112A of

221 | P a g e #HakkसेCA by CA. Akash Sir


222
5. Income from Other Sources

₹4,00,000 and casual income ₹10,000. Deductions allowed under section 80C to 80U
₹10,000. Compute his tax liability for assessment year 2024-25 under old tax provisions.

Solution:

Gross Interest 18,000


Less:
(i) Bank Charges u/s 57 (500)
(ii) Interest paid for borrowing the amount u/s 57 (10,000)
7,500
Casual Income 10,000
Income under the head Other Sources 17,500
Income under the head Capital Gains (LTCG 112A) 4,00,000
Gross Total Income 4,17,500
Less: Deductions under section 80C to 80U (7,500)
Total Income 4,10,000

Computation of Tax Liability

Tax on Casual Income 10,000 X 30% 3,000


Tax on LTCG 112A (4,00,000 – 2,50,000 – 1,00,000) X 10% 5,000
Less: Rebate u/s 87A (3,000)
Tax before HEC 5,000
Add: HEC @ 4% 200
Tax Liability 5,200
Note: Rebate is not allowed from tax on LTCG 112A

TP: 14 Profits chargeable to tax [Section 59]

If the assessee has claimed any expenditure while computing income and subsequently,
he has recovered the same amount, the amount so recovered shall be considered to be
income of the year in which amount has been recovered.

For example: Mr. Shyam has received a cheque of ₹1,00,000 being interest from ABC
Ltd. and cheque was deposited in his bank account and bank has deducted ₹1,000 being
collection charge. His income was considered to be ₹99,000. In the next year bank has
refunded ₹500 being excess charges collected, in this case ₹500 shall be considered to
be income of the year in which the amount has been received.

222 | P a g e #HakkसेCA by CA. Akash Sir


223
5. Income from Other Sources

TP: 15 Family Pension

(i) Regular payments given by the employer to the employee after retirement is called
pension and it is taxable under the head salary and standard deduction is allowed under
section 16(ia).

For example: Mr. Mukesh is retired from Reliance Ltd. and is getting pension ₹40,000
per month, in this case taxable amount under the head salary shall be ₹40,000 X 12 –
₹50,000 = ₹4,30,000.

(ii) After the death of the employee, employer may pay some pension to the family
member of the employee and it is called family pension. It is taxable under the head
Other Sources but as per section 57 deduction is allowed equal to 1/3rd of such pension
but maximum ₹15,000.

For example: Mrs. Neeta is getting family pension of ₹5,000 p.m., in this case taxable
amount shall be (5,000 x 12) minus 1/3 of ₹60,000 or ₹15,000 whichever is less i.e.
taxable amount shall be ₹45,000. If family pension is ₹3,000 per month, taxable amount
shall be ₹36,000 – ₹12,000 = ₹24,000.

Illustration: 9 [Other sources + Capital gains]

Mrs. Sita is getting family pension of ₹7,000 p.m. She also has dividend income from
domestic company of ₹31,000. She has long term capital gain under section 112A
₹3,00,000. Compute her tax liability for assessment year 2024-25 under regular tax
provisions.

Solution:

Income under the head Other Sources


Family Pension (7,000 x 12) 84,000
Less: Deduction u/s 57 (15,000)
1/3 of ₹84,000 or ₹15,000 whichever is less
69,000
Dividend income 31,000
1,00,000
Income under the head Capital Gains
223 | P a g e #HakkसेCA by CA. Akash Sir
224
5. Income from Other Sources

Long term capital gain 112A 3,00,000

Gross Total Income 4,00,000


Less: Deduction u/s 80C to 80U Nil
Total Income 4,00,000

Computation of Tax Liability

Tax on LTCG 112A [(3,00,000 – 1,50,000 – 1,00,000) @ 10%] 5,000


Tax on ₹1,00,000 at slab rate Nil
Tax before health & education cess 5,000
Add: HEC @ 4% 200
Tax Liability 5,200

Note: Rebate not allowed from LTCG 112A

TP: 16 Taxability of interest received on payment of compensation


from the government

As per section 145B, interest received for payment of compensation from the
Government or other similar agency in connection with compulsory acquisition of land or
building shall be taxable in the year in which it has been received and it will be taxable
under the head other sources however, as per section 57 deduction shall be allowed
@50% of such interest.

For example: Government has acquired one land of Mr. Dukhilal in Noida in 2013 and
payment was given by the Government in the year 2023-24 and has also paid interest of
₹1,00,000, in this case, taxable amount shall be ₹1,00,000 – ₹50,000 = ₹50,000.

Illustration: 10

Interest on enhanced compensation received by Mr. Sukhilal during the previous year
2023-24 is ₹6,50,000. Out of this interest, ₹2,00,000 relates to the previous year 2020-
21, ₹2,15,000 relates to previous year 2021-22 and ₹2,35,000 relates to previous year
2022-23. Discuss the tax implication, if any, of such interest income for A.Y. 2024-25.

Solution:

224 | P a g e #HakkसेCA by CA. Akash Sir


225
5. Income from Other Sources

The entire interest of ₹6,50,000 would be taxable in the year of receipt, namely, P.Y.
2023-24.
Interest on enhanced compensation taxable u/s 56 6,50,000
Less: Deduction under section 57 @ 50% (3,25,000)
Interest chargeable under the head “Income from other sources” 3,25,000

Illustration: 11

On 10.10.2023, Mr. Govind (a bank employee) received ₹5,00,000 towards interest on


enhanced compensation from State Government in respect of compulsory acquisition of his
land effected during the financial year 2015-16. Out of this interest, ₹1,50,000 relates to
the financial year 2016-17; ₹1,65,000 to the financial year 2017-18; and ₹1,85,000 to the
financial year 2018-19. He incurred ₹50,000 by way of legal expenses to receive the
interest on such enhanced compensation. How much of interest on enhanced compensation
would be chargeable to tax for the assessment year 2024-25?

Solution:

Section 145B provides that interest received by the assessee on enhanced compensation
shall be deemed to be the income of the assessee of the year in which it is received,
irrespective of the method of accounting followed by the assessee and irrespective of the
financial year to which it relates.

Section 56(2)(viii) states that such income shall be taxable as ‘Income from other
sources’. 50% of such income shall be allowed as deduction by virtue of section 57(iv) and
no other deduction shall be permissible from such Income. Therefore, legal expenses
incurred to receive the interest on enhanced compensation would not be allowed as
deduction from such income.

Computation of interest on enhanced compensation taxable as “Income from other


sources” for the A.Y. 2024-25:
Interest on enhanced compensation taxable u/s 56(2)(viii) 5,00,000
Less: Deduction under section 57(iv) (50% x ₹5,00,000) 2,50,000
Taxable interest on enhanced compensation 2,50,000

225 | P a g e #HakkसेCA by CA. Akash Sir


226
5. Income from Other Sources

Any sum of money or value of property received


without consideration or for inadequate
consideration [GIFT] [Section 56(2)(x)]
Gift received by any person can be divided into 3 parts:
1. Gift of sum of money [cash, bank, highly liquid instruments]
2. Gift of any property other than immovable property [movable property]
3. Gift of immovable property

TP: 17 Gift of sum of money

If any person has received any sum of money from one or more persons without
consideration and the aggregate value of all such gifts received during the year exceeds
₹50,000, the whole of the aggregate value of such sum shall be taxable under the head
other sources. But if the aggregate value is upto ₹50,000, entire amount shall be
exempt from income tax.

For example: Mr. Vijendra has received 3 gifts of ₹15,000 each from his 3 friends,
entire amount of ₹45,000 is exempt from income tax but if he has received 3 gifts of
₹20,000 each, entire amount of ₹60,000 shall be taxable. Further it will be considered
to be normal income and taxable under head income from other sources.

Mann Ki Baat (simple language):

Where aggregate value of cash gift exceeds ₹50,000, entire value of gift shall be taxable.
Where value of gifts is upto ₹50,000, no amount shall be taxable.

TP: 18 Meaning of Property

"Property" means the following capital asset of the assessee, namely —


(i) immovable property being land or building or both;
(ii) shares and securities;
(iii) Jewellery;
(iv) archaeological collections (relating to past/ ancient)
(v) drawings (a picture or diagram made with a pencil, pen, or crayon without paint.)
(vi) paintings;
(vii) sculptures;
(viii) any work of art; or

226 | P a g e #HakkसेCA by CA. Akash Sir


227
5. Income from Other Sources

(ix) bullion (Gold and silver in the form of biscuits / bricks / bars)

Mann Ki Baat (simple language):

From above, we understand that the definition of property is very narrow and does not
include many other movable assets like car, mobile phone, laptop etc.

TP: 19 Gift of any property other than immovable property

Without consideration: If any person has received gift of any property other than
immovable property without consideration and the aggregate fair market value of such
properties received during a particular year exceeds ₹50,000, it will be taxable under
the head other sources. But if aggregate value of all such properties is upto ₹50,000, it
will be exempt from income tax.

Inadequate consideration: If any person has received gift of any property other than
immovable property and the consideration paid is less than the aggregate fair market
value of such properties by an amount exceeding ₹50,000, aggregate fair market value
as exceeds such consideration (i.e. aggregate FMV - consideration) shall be taxable
under the head income from other sources.

For example: Mr. Amar has gifted a necklace to Miss Charu. If the FMV of the necklace
does not exceed ₹50,000, no amount would be taxable in the hands of Miss Charu in
respect of such gift. However, if its FMV is ₹1,00,000, the entire amount of ₹1,00,000
would be taxable in the hands of Miss Charu.

Important Note: If any person has received gift of any other movable property (other
than defined under TP: 18), it will not be taxable e.g. motor car or plant and machinery
or a watch or a mobile phone etc.
For example: Mr. Jagmal received a mobile phone valued ₹70,000 from his friend, in
this case, it will be not be chargeable to income tax as Gift.

Mann Ki Baat (simple language):

From above, please note some very important points. First point is that the gift of only
“property” is taxable. If any asset is not a property as per definition, then gift is not
taxable. Second point is that if any person has purchased property for inadequate
consideration, then shortfall is taxable as gift. For example, Mr. Rajveer has purchased

227 | P a g e #HakkसेCA by CA. Akash Sir


228
5. Income from Other Sources

jewellery worth ₹2,00,000 for ₹1,40,000, in this case, ₹60,000 is chargeable to tax in
hands of Mr. Rajveer as gift.

TP: 20 Meaning of stamp duty value

If any person is selling immovable property, its Conveyance Deed (sale deed) shall be
prepared in the office of Registrar and some tax has to be paid to the State
Government for transferring the property and it is called stamp duty and the value on
which such duty is charged is called stamp duty value (also called circle rate). A person
may not disclose the right value hence the value is determined by State Government.

TP: 21 Gift of immovable property

Without consideration: If any person has received any immovable property without
consideration and the stamp duty value of such property exceeds ₹50,000, entire
stamp duty value shall be taxable under the head other sources. But if SDV of such
property is upto ₹50,000, it will be exempt from income tax. Value of individual
immovable property shall be taken into consideration instead of aggregate value of all
such properties.

Inadequate consideration: If any immovable property has been received for a


consideration which is less than the stamp duty value of the property by an amount
exceeding the higher of the following:
a) ₹50,000
b) 110% of the actual consideration,
in such cases taxable amount shall be the stamp duty value of such property as exceeds
such consideration (i.e. SDV - consideration).

For example:
(i) Mr. Naveen purchased immovable property for ₹3,00,000 but stamp duty value is
₹5,00,000, taxable amount shall be ₹2,00,000.
(ii) Mr. Karan has sold immovable property to Mr. Mohan for ₹1,00,00,000 but stamp duty
value is ₹1,10,00,000, in this case, the taxable amount shall be Nil because the stamp duty
value is not exceeding 110% of the actual consideration but if stamp duty value is ₹
1,11,00,000, taxable amount shall be ₹ 11,00,000 because stamp duty is exceeding 110% of
actual consideration.

228 | P a g e #HakkसेCA by CA. Akash Sir


229
5. Income from Other Sources

Mann Ki Baat (simple language):

From above, it is important to note that gift taxation on immovable property is applicable
where gift value of individual property exceed ₹50,000 and not aggregate. (In other cases,
aggregate value was seen instead of individual property value). One more important point is
that there is an additional limit of 110% of consideration. Where immovable property has
been received at less than SDV but SDV does not exceed 110% of actual consideration, no
amount is taxable as gift.

Now, let’s focus on one more important point about SDV. We need to understand that
there are two types of agreement in case of sale-purchase of immovable property.
1. Agreement to sale – An agreement where parties agree to sale-purchase the
immovable property.
2. Registration – An agreement where actual transfer of property takes place.
Now, question arises as to which date is to be considered for SDV if SDV is different on
both days.

How to determine SDV where date of agreement and date of registration is


different?

If the date of the agreement fixing the amount of consideration for the transfer of
immovable property and the date of registration are not the same, in such cases, the
stamp duty value on the date of the agreement may be taken into consideration
provided part of the consideration should have been paid by (i) account payee cheque, (ii)
an account payee bank draft or (iii) by use of electronic clearing system through a bank
account or (iv) through such other electronic modes as may be prescribed on or before
the date of agreement.

[Other electronic mode means Credit Card, Debit Card, Net Banking, IMPS (Immediate
Payment Service), UPI (Unified Payment Interface), RTGS (Real Time Gross
Settlement), NEFT (National Electronic Funds Transfer), and BHIM (Bharat Interface
for Money) Aadhaar Pay]

For example: Mr. Ajay has entered into agreement with a builder DLF Limited on
01.07.2017 for purchase of one building for ₹20,00,000 but stamp duty value was
₹27,00,000 and advance of ₹3,00,000 was given by account payee cheque.
Property was transferred in his name on 01.07.2023 and on that date stamp duty value was
₹35,00,000.

229 | P a g e #HakkसेCA by CA. Akash Sir


230
5. Income from Other Sources

In this case amount of gift shall be ₹7,00,000 (₹27,00,000 – ₹20,00,000). (Difference


amount is more than ₹50,000 and SDV is more than 110% of the consideration). Similarly,
it will also be considered to be normal income.

Mann Ki Baat (simple language):

From above, it is important to note that there is an option with the Assessee to consider
the SDV of agreement date or registration date. However, in order to consider SDV of
date of agreement, the Assessee has to make part payment on or before date of
agreement. Payment has to be made in prescribed modes only. This condition is kept to
avoid fake back dated agreements.

Now, let’s study about exceptions to the above gift provisions.

TP: 22 The gift is exempt in the following cases

1) If any individual has received any gift from any of his relative, it will be exempt from
income tax. The term relative shall include;
(a) spouse of the individual;
(b) brother or sister of the individual;
(c) brother or sister of the spouse of the individual;
(d) brother or sister of either of the parents of the individual;
(e) any lineal ascendant or descendant of the individual; (ascendant means mother/
father/ grandmother / grandfather and so on: Descendant means son / daughter /
grandson / granddaughter etc.)
(f) any lineal ascendant or descendant of the spouse of the individual;
(g) spouse of the person referred to in items (b) to (f).

Note: Brother/ Sister of either of the parents of spouse of individual in not


covered.

For example: Mr. Ahmed has received ₹5,00,000 from Mr. Ali without any
consideration.
Case i) Mr. Ahmed is brother of Mr. Ali – In this case, amount received by Mr. Ahmed is
not chargeable to tax.
Case ii) Mr. Ali is uncle (father’s brother) of Mr. Ahmed – In this case, amount received
by Mr. Ahmed is not chargeable to tax since brother of father is covered under
definition of relative.

230 | P a g e #HakkसेCA by CA. Akash Sir


231
5. Income from Other Sources

Case iii) Mr. Ahmed is uncle (father’s brother) of Mr. Ali – In this case, amount received
by Mr. Ahmed is taxable as gift since nephew (son of brother) is not covered under
definition of relative.

2) If any individual has received any gift from any person of any amount on the occasion
of his/her marriage, it will be exempt from income tax.
If gift is received by the parents of such individual, in that case it will be taxable. If
any individual has received gift on the occasion of marriage anniversary, it will be
taxable.

For example: Mr. Ram has received gifts (cash, property) worth ₹50,00,000 from
relatives and friends on the occasion of his marriage with Sita. In this case, no amount
shall be taxable in hands of Mr. Ram.

Illustration: 12

A sum of ₹2,00,000 is received as gift from non-relatives by Mr. Rajpal on the occasion of
marriage of his son. Explain the taxability as gift.

Solution:

The gift of ₹2,00,000 shall be taxable in hands of Mr. Rajpal.

3) If any person has received any gift under a will/ inheritance, it will be exempt from
income tax.

For example: Mr. Rakesh Ambani has received property of worth ₹3,000 Cr under will
from his father Herubhai Ambani. In this case, no amount shall be taxable as gift.

4) In contemplation of death of the payer or donor (Contemplation of Death means the


apprehension of an individual that his life will end in the immediate future by a
particular illness etc.)

5) from any local authority or charitable hospital or charitable educational institution


or charitable trust or other similar organization.

Illustration: 13

231 | P a g e #HakkसेCA by CA. Akash Sir


232
5. Income from Other Sources

Mr. Chezian is employed in a company with taxable salary income of ₹5,00,000. He


received a cash gift of ₹1,00,000 from Atma Charitable Trust (registered under section
12AB) in December 2023 for meeting his medical expenses.
Is the cash gift so received from the trust chargeable to tax in the hands of Mr. Chezian?

Solution:

The provisions of section 56(2)(x) would not apply to any sum of money or any property
received from any trust or institution registered under section 12AB. Therefore, the cash
gift of ₹1 lakh received from Atma Charitable Trust, being a trust registered under
section 12AB, for meeting medical expenses would not be chargeable to tax under section
56(2)(x) in the hands of Mr. Chezian.

6) If an individual has received from any person in respect of any expenditure actually
incurred by him on his medical treatment or treatment of any member of his
family, for any illness related to COVID-19 subject to such conditions, as the Central
Government may, by notification in the Official Gazette, specify in this behalf.

7) If a member of the family of a deceased person has received—


(a) from the employer of the deceased person; (any amount) or
(b) from any other person or persons to the extent that such sum or aggregate of such
sums does not exceed ten lakh rupees, where the cause of death of such person is
illness related to COVID-19 and the payment is—
(i) received within twelve months from the date of death of such person; and
(ii) subject to such other conditions, as the Central Government may, by notification
in the Official Gazette, specify in this behalf.

Meaning of family (for point (6) and (7)):

For the purposes of this clause, "family", in relation to an individual, means -


(i) the spouse and children of the individual; and
(ii) the parents, brothers and sisters of the individual or any of them, wholly or mainly
dependent on the individual.

Mann Ki Baat (simple language):

From the above, we understand that gift is not taxable if it is received from certain
person or on certain occasions (as explained above).

232 | P a g e #HakkसेCA by CA. Akash Sir


233
5. Income from Other Sources

TP: 23 Special Cases

1) Taxability of gift received by HUF from its members: If any Hindu


undivided family has received any gift from any of its members, it will be exempt from
income tax.

For example: One HUF has received cash gift of ₹10,00,000 from one of its members,
it will be exempt from income tax. If HUF has given gift to its member, it will be
taxable.

2) Taxability of stock-in-trade: If any person has received any asset as stock-in-


trade, it will not be taxable as gift.

For example: Mr. Aman is a dealer in gold and he has purchased gold for ₹20 lakhs but
market value is ₹ 27 lakhs, in this case it will not be taxable as gift (because cost will be
shown in the books as ₹20 lakhs and entire profit on sale shall be taxable under the
head business/profession).

Other example: Mr. Arman has a business of buying and selling of shares. Mr. Arman
has purchased shares from Mr. Vishal for ₹10 lakhs whereas their FMV is ₹15 lakhs. In
this case, the amount of ₹5 lakhs shall not be treated as gift in the hands of Mr. Arman.
Instead, the cost of shares in the book of accounts would be shown as ₹10 lakhs and the
entire profit on the sale in excess of ₹10 lakhs shall be taxed u/h PGBP.

3) Gifts to the Employees [Section 17(2)(viii) Rule 3(7)(iv)]: Gift (including gift
vouchers/ token) given by the employer in kind up-to ₹5,000 in aggregate during a
particular year is exempt and excess over it is taxable under head Salary (as per
circular no. 8/2012 dated 05th October, 2012).

Gifts in cash or gifts convertible into cash i.e. gift cheques etc. shall be fully chargeable
to tax under head Salary.

For example: Mrs. Sheela is employed in Adani Ltd. and he has received a cash gift of
₹11,000 from her employer, in this case taxable amount shall be ₹11,000 and it will be
income under the head salary and shall be taxable at the normal rate but if Mrs. Sheela
has received one wrist watch of ₹11,000 from his employer, taxable amount shall be
₹6,000.

233 | P a g e #HakkसेCA by CA. Akash Sir


234
5. Income from Other Sources

4) Taxability of gift received in connection with business/profession [Section


28]: The value of any benefit or perquisite, whether convertible into money or not,
arising from business or the exercise of a profession, shall be taxable under the head
business profession. If any person has received any gift or perquisite or benefit either
in cash or in kind from any of his clients, it will be considered to be business receipt and
shall be taken into consideration while computing income under the head profits and
gains from business or profession.

For example: A Doctor has received a gift of ₹40,000 from one of his clients, in this
case it will be considered to be income under the head business/profession.

Other example: A client is extremely happy with the work of his CA and apart from the
agreed upon fees, the client gives a wrist watch worth ₹50,000 to the CA. The value of
such watch shall be included in the income of the CA u/h PGBP.

5) Taxability of scholarship/ award / reward:

Scholarship [Section 10(16)]: Any scholarship received by a person for meeting the
cost of education shall be exempt from income tax.

Award/ Reward Section 10(17A): Any award or reward whether in cash or in kind
instituted by the Central Government or the State Government shall be exempt from
income tax. Similarly, any private award or reward shall be exempt from income tax if
approved by the Central Government.

234 | P a g e #HakkसेCA by CA. Akash Sir


235
5. Income from Other Sources

Comprehensive Questions

Question: 1 [Other sources + Gift] [May 2016 Exam]

Discuss the taxability or otherwise in the hands of the recipient as per the provision of
the Income Tax Act:
(i) Healthy Pvt. Ltd., a closely held company, issued 10,000 shares at ₹130 per share.
(The face value of the share is ₹100 per share and the fair market value of the
share is ₹120 per share).
(ii) Mr. Akash received an advance of ₹50,000 on 01.09.2023 against the sale of his
house. However, due to non-payment of installment in time, the contract has
cancelled and the amount of ₹50,000 was forfeited.
(iii) Mr. Naveen, a member of his father’s HUF, transferred a house property to the
HUF without consideration. The value of the house is ₹10 lacks as per the Registrar
of Stamp Duty.
(iv) Mr. Kumar gifted a car to his sister’s son (Sunil) for achieving good marks in CA
Final Exam. The fair market value of the car is ₹5,00,000.

Question: 2 [Other sources + Gift + Capital gain + Deduction]

Find the tax liability of Mrs. Abhey (age 40 years), a resident individual, for the
assessment year 2024-25 from the following particulars of her incomes and spending for
the previous year ending March 31, 2024.
Income from house property (Computed) 90,000
Dividend from UTI 35,000
Family pension (gross) 90,000
Interest on bank FD (gross) 14,000
Dividend from foreign company 36,000
Gift received from her sister 26,000
Winnings from lotteries (gross) 70,000
Long-term capital gain 1,20,000
Payment for purchase of National Savings Certificates 35,000
Assume that he has shifted out from the provisions of section 115BAC.

Question: 3 [Other sources + Deductions]

Mr. Nitin has submitted information given below:


i) Income from owning and maintaining of race horse of ₹2,00,000.

235 | P a g e #HakkसेCA by CA. Akash Sir


236
5. Income from Other Sources

ii) Income from owning and maintaining of race camels of ₹1,00,000.


iii) He had winning of ₹1,60,000 from horse race on 01.12.2023 and winning from camel
race of ₹1,80,000 on 07.12.2023.
iv) He purchased lottery tickets of ₹10,000 on 01.02.2024 and had winning of ₹2,00,000
on 12.02.2024.
v) He has received Royalty of book of literary nature @ 50% of print price of ₹600 and
total copies sold are 2,000.
Compute tax liability for the A.Y. 2024-25 under normal provisions of the Act.

Question: 4 [Other sources + Capital Gain + Clubbing]

Aruna Ltd. has 1,00,000 equity shares of ₹10 each and Mr. Arun purchased 10,000 equity
shares on 01.01.2022 of ₹10 each and the company goes into liquidation on 31.07.2022 and
company has net distributable amount of ₹60 lakhs after discharging all the liabilities and
it includes accumulated profits of ₹20 lakhs and the entire amount was distributed among
the shareholders.
Minor son of Mr. Arun has interest income of ₹2 lakhs from one bank deposit which was
gifted to him by his grand-father.
Mrs. Arun has one business and income from business is ₹1 lakh entire capital was gifted
by Mr. Arun.
Mr. Arun is growing flowers and has income of ₹7 lakh from sale of flowers.
Compute his tax liability for Assessment Year 2024-25 assuming that the Assessee has
not opted for section 115BAC.

Question: 5 [Residential status & scope of total income + House property + Other
sources] [NOV 2019 (New Course)]

Mr. Jagdish, aged 61 years, has set-up his business in Thailand and is residing in Thailand
since last 20 years. He owns a house property in Bangkok, half of which is used as his
residence and half is given on rent (such rent received, converted in INR is ₹6,00,000).
The annual value of the house in Thailand is ₹50,00,000 i.e. converted value in INR.

He purchased a flat in Pune during F.Y. 2019-20, which has been given on monthly rent of
₹27,500 since 01.07.2022. The annual property tax of Pune flat is ₹40,000 which is paid
by Mr. Jagdish whenever he comes to India. Mr. Jagdish last visited India in July 2021. He
has taken a loan Union Bank of India for purchase of the Pune flat amounting to
₹15,00,000. The interest on such loan for the F.Y. 2023-24 was ₹84,000. However,
interest for March 2024 quarter has not yet been paid by Mr. Jagdish.

236 | P a g e #HakkसेCA by CA. Akash Sir


237
5. Income from Other Sources

He had a house in Jaipur which was sold in May 2019. In respect of this house, he received
arear of rent of ₹96,000 in Feb 2024 (not taxed earlier).

He also derived some other incomes during F.Y. 2023-24 which are as follows:

1. Profit from business in Thailand of ₹2,75,000


2. Interest on bonds of a Japanese Co. of ₹45,000 out of which 50% was received in India.
3. Income from Apple Orchid in Nepal given on contract and the yearly contract fee of
₹5,00,000, for F.Y. 2023-24 was deposited directly by the contractor in Kathmandu
branch of Union Bank of India in Mr. Jagdish’s bank account maintained with Union Bank of
India’s Pune Branch.

Compute the total income of Mr. Jagdish for Assessment Year 2024-25 chargeable to
income tax in India.

Question: 6 [Clubbing, Gift and Other sources]

Mr. Mahadev, a noted bhajan singer of Rajasthan and his wife Mrs. Dariya furnish the
following information relating to the Assessment Year 2024-25.

1 Income of Mr. Mahadev- professional bhajan singer (computed) 5,65,000


2 Income under the head salary of Mrs. Dariya (computed) 3,80,000
3 Loan received by Mrs. Dariya from Ramu & Jay (Pvt) Ltd. (Mrs. 2,50,000
Dariya holds 35% shares of the Co. Co. and has incurred losses since
its inception 2 years back)
4 Income of their minor son Golu winning signing reality show on T.V. 2,50,000
5 Cash gift received by Golu from friend of Mr. Mahadev on winning 21,000
the show
6 Interest income received by minor married daughter Gudia from 40,000
deposit with Ramu & Jay Pvt Ltd.

Compute total taxable income of Mr. Mahadev & Mrs. Dariya for the Assessment Year
2024-25.

Question: 7 [Other Sources] [Nov 2016]

State with reasons whether the following receipts are taxable or not under the provisions
of Income tax Act, 1961?

237 | P a g e #HakkसेCA by CA. Akash Sir


238
5. Income from Other Sources

Mr. Suman received an advance of ₹3 lakhs on 06.06.2023 to transfer his residential


house property. Since the transfer was not made during the previous year due to failure in
negotiations, he deducted the advance money forfeited from the cost of acquisition of the
property.

Question: 8 [Other Sources] [May 2016]

Discuss the taxability or otherwise in the hands of the recipients, as per the provisions of
the Income-tax Act, 1961:
(i) Abhishek Private Limited, a closely held company, issued 10,000 shares at ₹130 per
share. (The face value of the share is ₹100 per share and the fair market value of the
share is ₹120 per share).
(ii) Mr. Abhijeet received an advance of ₹50,000 on 01.09.2023 against the sale of his
house. However, due to non-payment of instalment in time, the contract has cancelled and
the amount of ₹50,000 was forfeited.

Question: 9 [Other Sources]

Discuss with reason, whether the following transactions are true or false, as per the
provisions of Income Tax Act, 1961:
Dividend received by a dealer in shares or one engaged in buying/selling of shares, is
chargeable under the head “Income from other sources”.

Question: 10 [Other Sources] [NOV 2020]

Ms. Anima received following amounts during the previous year 2023-24.

(1) Received loan of ₹5,00,000 from the ABC Private Limited, a closely held company
engaged in bicycle business. She is holding 10% of the equity share capital in the said
company. The accumulated profit of the company was ₹2,00,000 on the date of the loan.
(2) Received Interest on enhanced compensation of ₹5,00,000. Out of this interest,
₹1,50,000 relates to the previous year 2019-20, ₹1,90,000 relates to previous year 2020-
21 and ₹1,60,000 relates to previous year 2021-22. She paid 1 lakh to her advocate for his
efforts in the matter.
Discuss the tax implications, if any, arising from these transactions in her hand with
reference to Assessment Year 2024-25.

238 | P a g e #HakkसेCA by CA. Akash Sir


239
5. Income from Other Sources

Question: 11 [Other Sources + Agricultural Income]

State with brief reasoning whether the following receipts are chargeable to income-tax or
are exempt (if chargeable, the amount taxable is to be mentioned) for the assessment
year 2024-25:
(i) Interest on enhanced compensation received on 12.03.2024 for
acquisition of urban land, of which 40% relates to the earlier year. 96,000
(ii) Rent received for letting out agricultural land for a movie shooting. 72,000
Computation is NOT required.

Question: 12 [Gift]

From the following transactions relating to Mrs. Natasha, dealer in shares, determine the
amount chargeable to tax in her hands for the A.Y. 2024-25. Your answer should be
supported by the reasons:
(i) On 01.01.2024, being her birthday, she received a gift of ₹40,000 by means of cheque
from her father's maternal uncle.
(ii) On 12.02.2024, she acquired a vacant site from her friend for ₹1,32,000. The State
stamp valuation authority fixed the value of site at ₹ 2,00,000 for stamp duty purpose.
(iii) She bought 50 equity shares of a private company from another friend for ₹ 75,000.
The fair market value of such shares on the date of purchase was ₹ 1,33,000.

Question: 13 [Gift]

Mr. Sumit, a dealer in shares, received the following without consideration during the P.Y.
2023-24 from his friend Mr. Rohan, -
(1) Cash gift of ₹75,000 on his anniversary, 15th April, 2023.
(2) Bullion, the fair market value of which was ₹60,000, on his birthday, 19th June, 2023.
(3) A plot of land at Faridabad on 1st July, 2023, the stamp value of which is ₹5 lakh on
that date. Mr. Rohan had purchased the land in April, 2016.
(4) Mr. Sumit purchased from his friend Miss Roshni, who is also a dealer in shares, 1000
shares of Adani Ltd. @ ₹400 each on 19th June, 2023, the fair market value of which was
₹600 each on that date. Mr. Sumit sold these shares in the course of his business on 23rd
June, 2023.
(5) On 1st November, 2023, Mr. Sumit took possession of property (building) booked by
him two years back at ₹20 lakh. The stamp duty value of the property as on 1st November,

239 | P a g e #HakkसेCA by CA. Akash Sir


240
5. Income from Other Sources

2023 was ₹32 lakh and on the date of booking was ₹ 23 lakh. He had paid ₹ 1 lakh by
account payee cheque as down payment on the date of booking.
Compute the income of Mr. Sumit chargeable under the head “Income from other sources”
for A.Y. 2024-25.

Question: 14 [Gift]

Discuss the taxability or otherwise of the following in the hands of the recipient under
section 56(2)(x) the Income-tax Act, 1961 –
(i) X HUF received ₹75,000 in cash from niece of Mr. X (i.e., daughter of Mr. X’s sister).
Mr. X is the Karta of the HUF.
(ii) Miss. X, a member of her father’s HUF, transferred a house property to the HUF
without consideration. The stamp duty value of the house property is ₹9,00,000.
(iii) Mr. X received 100 shares of A Ltd. from his friend as a gift on occasion of his 25th
marriage anniversary. The fair market value on that date was ₹100 per share. He also
received jewellery worth ₹45,000 (FMV) from his nephew on the same day.
(iv) X HUF gifted a car to son of Karta for achieving good marks in XII board examination.
The fair market value of the car is ₹5,25,000.

Question: 15 [Gift + Other sources]

Examine whether the following are chargeable to tax and the amount liable to tax:
(i) A sum of ₹1,20,000 was received as gift from non-relatives by Raj on the occasion of
the marriage of his son Pravin.
(ii) Interest on enhanced compensation of ₹96,000 received on 12.03.2024 for acquisition
of urban land, of which 40% relates to P.Y. 2022-23.

Question: 16 [Gift]

The following details have been furnished by Mrs. Hemali pertaining to the year ended
31.03.2024:
(i) Cash gift of ₹51,000 received from her friend on the occasion of her “Shastiaptha
Poorthi”, a wedding function celebrated on her husband completing 60 years of age. This
was also her 25th wedding anniversary.
(ii) On the above occasion, a diamond necklace worth ₹2 lacs presented by her sister living
in Dubai.
(iii) When she celebrated her daughter's wedding on 21.02.2024, her friend assigned in
Mrs. Hemali's favour, a fixed deposit held by the said friend in a scheduled bank; the value
of the fixed deposit and the accrued interest on the said date was ₹52,000.

240 | P a g e #HakkसेCA by CA. Akash Sir


241
5. Income from Other Sources

Compute the income, if any, assessable as income from other sources.

Question: 17 [Gift]

Discuss taxability in the following cases:


(i) Mr. X has received three gifts from his three friends:
(a) ₹55,000 in cash
(b) Land with market value of ₹5,00,000 but the value for the purpose of charging
stamp duty of ₹4,00,000.
(c) Jewellery with market value of ₹3,00,000
(ii) Mr. X has received gift of ₹50,000 in cash from his friend.
(iii) Mr. X has received gift of ₹1,50,000 in cash from his brother.
(iv) Mr. X has received gift of ₹1,50,000 in cash from his mother’s sister.
(v) Mr. X has received gift of ₹1,50,000 in cash from his father’s brother.
(vi) Mr. X has received gift of ₹1,50,000 in cash from his cousin.
(vii) Mr. X has received gift of ₹1,50,000 in cash from brother of his spouse.
(viii) Mr. X has received gift of ₹1,50,000 in cash from his grandfather.
(ix) Mr. X has received gift of ₹1,50,000 in cash from spouse of his brother.
(x) Mr. X has received gift of ₹1,50,000 in cash from husband of his sister.
(xi) Mr. X has received gift of ₹1,50,000 in cash from sister of his brother’s wife.
(xii) Mr. X has received gift of ₹1,50,000 in cash from the sister of his spouse.
(xiii) Mr. X has received gift of ₹5,000 in cash on his birthday from each of his eleven
friends.
(xiv) Mr. X has received gift of property valued ₹1,50,000 from his friend.
(xv) Mr. X has received gift of ₹1,50,000 in cash from his friend on the occasion of his
marriage.
(xvi) Mr. X has received gift of ₹75,000 in cash and property ₹75,000 from his fiancée.

Question: 18 [Gift] [RTP]

Mr. X reports the following transactions to you:


(i) Received cash gifts on the occasion of his marriage on 15.06.2023 of ₹1,08,000. It
includes gift of ₹28,000 received from non-relatives.
(ii) On 15.08.2023, being his birthday, he received a gift by means of cheque from his
mother's maternal aunt for an amount of ₹49,000.
(iii) On 25.12.2023, he acquired a vacant site from his friend for ₹1,50,000. The State
stamp valuation authority fixed the value of site at ₹2,25,000 for stamp duty purpose.
(iv) He bought 200 equity shares of a listed company from another friend for ₹75,000.
The value of shares in the stock exchange on the date of purchase was ₹1,75,000.

241 | P a g e #HakkसेCA by CA. Akash Sir


242
5. Income from Other Sources

(v) A cell phone worth ₹21,000 is gifted by his friend on 16.8.2023.


Determine the amount chargeable to tax in the hands of Mr. X for the Assessment Year
2024-25. Your answer should be supported with reasons.

Question: 19 [Capital gain and Gift]

Mr. Subramani sold a house plot to Mrs. Vimala for ₹45 lakhs on 12.05.2023. The valuation
determined by the stamp valuation authority was ₹53 lakhs. Discuss the tax consequences
of above, in the hands of each one of them, viz, Mr. Subramani & Mrs. Vimala. Mrs. Vimala
has sold this plot to Ms. Padmaja on 21.03.2023 for ₹55 lakhs. The valuation as per stamp
valuation authority remains the same at ₹53 lakhs. Compute the capital gains arising on
sale of the house plot by Mrs. Vimala.
Note: None of the parties viz Mr. Subramani, Mrs. Vimala & Ms. Padmaja are related to
each other; the transactions are between outsiders.

Question: 20 [Residential status, Scope of total income, house property, IFOS] [MAY
– 2007 (6 Marks)]

Miss Charlie, an American national, got married to Mr. Radhey of India in USA on
02.03.2023 and came to India for the first time on 16.03.2023. She left for USA on
23.09.2023. She returned to India again on 27.03.2024. While in India, she had purchased
a show room in Mumbai on 22.04.2023, which was leased out to a company on a rent of
₹25,000 p.m. from 01.05.2023. She had taken loan from a bank for purchase of this show
room on which bank had charged interest of ₹97,500 up to 31.03.2024.
She had received the following gifts from her relatives and friends during 01.04.2023 to
30.06.2023:
- From parents of husband 51,000
- From married sister of husband 11,000
- From two very close friends of her husband, ₹1,51,000 and ₹21,000 1,72,000
Determine her residential status and compute the total income chargeable to tax along
with the amount of tax payable on such income for the Assessment Year 2024-25 under
old tax provisions. .

242 | P a g e #HakkसेCA by CA. Akash Sir


Profits and Gains of Business or Profession
(PGBP) [Section 28 to 44DB]
TP: 1 Income chargeable to tax under head PGBP [Section 28]

As per section 28, income from any business/ profession shall be taxable under the
head profits and gains of business or profession (PGBP). Such income shall be
computed in the similar manner as in the case of general practice of accountancy.
However, incomes and expenditures shall be such as are given under Income Tax Act,
1961.

Some of the incomes which are taxable under head PGBP are:

i) Income from speculation business


ii) Value of benefits or perquisites including gifts in connection with business/
profession
iii) Non-compete fees
iv) Payments received under keyman insurance policy
v) Export incentives i.e. profits on sale of import entitlements, duty drawbacks, etc.
vi) Sum received by partner of firm
vii) FMV of inventory on its conversion into capital asset
viii) etc.

Mann Ki Baat (simple language):

Section 28 is the charging section of PGBP which talks about various incomes which are
to be charged to tax under head PGBP and expenses which are allowed to be debited
against such income. Now, we should note that there are no special accounts which are
to be made for PGBP, however, general accounts are used with some adjustments
(income or expenses) to compute income under head PGBP. We will study about these
incomes and expenses in the later part of this chapter.

TP: 2 Computation of Income from PGBP [Section 29]

The income referred to in section 28 shall be computed in accordance with the


provisions contained in sections 30 to 43D.

243 | P a g e #HakkसेCA by CA. Akash Sir


Mann Ki Baat (simple language):

Section 29 basically talks about the manner of computation of income under head PGBP.
Section 29 refers to section 30 to 43D for computation of income. Now we are going to
study section 30 to 43D in detail.

TP: 3 Rent, Rates, Taxes, Repairs and Insurance for Buildings


[Section 30]

Section 30 allows deduction of all the expenses relating to buildings used by the
assessee for the purposes of his business or profession and such expenses may be:
(i) Revenue nature repairs expenses (incurred by owner or tenant)
(ii) Municipal tax or local tax or land revenue (but on payment basis as per section
43B)
(iii) Premium for insurance of building etc.

Note: No notional rent is allowed as deduction where building is owned by the


assessee himself. This means that no rent shall be allowed where property is self-
owned.

Mann Ki Baat (simple language):

From above, we understand that all the expenses related to building used in business or
profession shall be allowed to be debited in profit and loss account.

Now, let’s brainstorm some of the interesting aspects of the above provision.

244 | P a g e #HakkसेCA by CA. Akash Sir


1. What if the premise is partly used for business/ profession & partly for other/
personal purpose?
Answer: Where premises is partly used in business and partly for other purpose
then only proportionate expenses (relating to business purpose) shall be allowed to
be debited in profit and loss account. [section 38(1)]

2. What if owner uses own premises for business/ profession?


Answer: Where building is owned by the business man, then NO notional rent shall
be allowed to be debited in profit and loss account. However, if partnership firm
uses premise which is owned by one of the partners, then reasonable rent payable
to partner shall be allowed as expense.
For example: If Mr. Ramu uses his own building in business, then notional rent
cannot be allowed in profit and loss account. However, if Ramu & Sons, his firm pays
rent of ₹10,000 per month to Mr. Ramu (Partner), then Ramu & Sons shall be
allowed deduction of rent expense. It is assumed that rent is reasonable.

3. Are capital expenses allowed as deduction?


Answer: No, only revenue expenses are allowed as deduction.

TP: 4 Repairs and insurance of machinery, plant and furniture


[section 31]

As per section 31, if any assessee has any plant and machinery or furniture/ fixture
in his business/ profession, the assessee is allowed to debit all the expenses to the
profit and loss account and such expenses may be like repairs or insurance or rent of
plant etc.

Note: If plant & machinery etc. are owned by the assessee, its notional rent is not
allowed to be debited.

Mann Ki Baat (simple language):

Above expenses are allowed as deduction only where machinery, plant and furniture are
actually used by the assessee in business or profession. Please note that usage include
both active use & passive use.
For example, stand by equipment and fire extinguishers can be capitalized even if they
are ‘ready for use’ and not used actually in business and thereby, repair and insurance
expense of these assets shall be allowed.

245 | P a g e #HakkसेCA by CA. Akash Sir


TP: 5 Depreciation Allowance [section 32]

(i) Charge of depreciation is Mandatory:

Explanation 5 to section 32 provides that the deduction on account of


depreciation shall be made compulsorily, whether or not the assessee has claimed
the deduction in computing his total income.

It means that even if the assessee has not claimed the depreciation in particular
year, it shall be assumed to be claimed and accordingly depreciation shall be
calculated for next year.

(ii) Basis:

Depreciation under Income Tax Act is allowed on the basis of written down value
(WDV) method but only in case of power generating unit, the assessee has the option
to compute depreciation either on the basis of WDV method or on the basis of
Straight-Line Method (SLM).

(iii) Some special points:

a) No depreciation is allowed on land.


b) Plant does not include tea bushes, livestock, animal, human body, or stock in trade
and accordingly no depreciation shall be allowed on these assets.
c) The asset should be actually used for the purpose of business or profession
(active or passive usage). It means that the depreciation shall not be allowed if
the asset is only purchased and not used in the business/ profession.

(iv) Rate of depreciation shall be as given below:

PART A TANGIBLE ASSETS

I Buildings

Block 1. Buildings which are used mainly for residential purposes 5%


except hotels and boarding houses
Block 2. Buildings which are not used mainly for residential purposes 10%
and not covered by Block (1) above and (3) below
Block 3. Buildings acquired on or after 1st September, 2002 for 40%
installing machinery and plant forming part of water supply

246 | P a g e #HakkसेCA by CA. Akash Sir


project or water treatment system and which is put to use for
the purpose of business of providing infrastructure facilities
Block 4. Purely temporary erections such as wooden structures 40%
II FURNITURE AND FITTINGS

Block 1. Furniture and fittings including electrical fittings [“Electrical 10%


fittings” include electrical wiring, switches, sockets, other
fittings and fans, etc.]
III PLANT AND MACHINERY

Block 1. Motor cars other than those used in a business of running 30%
them on hire, acquired during the period from 23.08.2019 to
31.03.2020 and put to use on or before 31.03.2020
Block 2. Motor cars other than those used in a business of running 15%
them on hire, acquired or put to use on or after 01.04.1990
[Other than motor cars mentioned in (1) above]
Block 3. Motors buses, motor lorries, motor taxis used in a business of 45%
running them on hire, acquired during the period from
23.08.2019 to 31.03.2020 and put to use on or before
31.03.2020
Block 4. Motors buses, motor lorries, motor taxis used in the business 30%
of running them on hire [Other than mentioned in (3) above]
Block 5. Moulds used in rubber and plastic goods factories 30%
Block 6. Aeroplanes, Aeroengines 40%
Block 7. Specified air pollution control equipments, water pollution 40%
control equipments, solid waste control equipment and solid
waste recycling and resource recovery systems
Block 8. Plant & Machinery used in semi-conductor industry covering all 30%
Integrated Circuits (ICs)
Block 9. Life saving medical equipment 40%
Block 10. Machinery and plant, acquired and installed on or after the 1st 40%
September, 2002 in a water supply project or a water
treatment system and which is put to use for the purpose of
business of providing infrastructure facility
Block 11. Containers made of glass or plastic used as re-fills 40%
Block 12. Energy saving devices (as specified) 40%
Block 13. Renewable Energy Saving Devices (as specified) 40%
(i) Electrically operated vehicles including battery powered or 40%
fuel-cell powered vehicles
(ii) Windmills and any specially designed devices which run on 40%
windmills installed on or after 01.04.2014
(ii) Any special devices including electric generators and 40%
pumps running on wind energy installed on or after 01.04.2014
Block 14. Windmills and any specially designed devices running on 15%
windmills installed on or before 31.03.2014 and any special

247 | P a g e #HakkसेCA by CA. Akash Sir


devices including electric generators and pumps running on
wind energy installed on or before 31.03.2014
Block 15. Computers including computer software 40%
Block 16. Books (annual publications or other than annual publications) 40%
owned by assessees carrying on a profession
Block 17. Books owned by assesses carrying on business in running 40%
lending libraries
Block 18. Plant & machinery (General rate) 15%
IV SHIPS
Block 1. Ocean-going ships 20%
Block 2. Vessels ordinarily operating on inland waters not covered by 20%
Block (3) below
Block 3. Speed boats operating on inland water 20%
PART B INTANGIBLE ASSETS

Know-how, patents, copyrights, trademarks, licenses, franchises or any 25%


other business or commercial rights of similar nature, not being goodwill
of a business or profession

(v) Computation of Depreciation:

a) Asset put to use for 180 days or more: If any particular asset is purchased
during the year and it has been put to use for 180 days or more during the year,
in that case, depreciation is allowed at the normal rate (rates discussed above).

For example: Mr. X purchased one asset on 01.05.2023 and put to use on same
date. In this case, depreciation shall be allowed at full rate.

b) Asset put to use for less than 180 days: If any particular asset is purchased
during the year and it has been put to use for less than 180 days during the year,
in that case, depreciation is allowed at half the normal rate.

For example: Mr. X purchased one asset on 01.11.2023 and put to use on same
date. In this case, depreciation shall be allowed at half rate.

c) Asset put to use for less than 180 days: If any particular asset is purchased
during the year and it has been put to use for less than 180 days during the year,
in that case, depreciation is allowed at half the normal rate.

For example: Mr. X purchased one asset on 01.11.2023 and put to use on same
date. In this case, depreciation shall be allowed at half rate.

248 | P a g e #HakkसेCA by CA. Akash Sir


d) Asset put to use for less than 180 days: If any particular asset is purchased
during the year and it has been put to use for less than 180 days during the year,
in that case, depreciation is allowed at half the normal rate.

For example: Mr. X purchased one asset on 01.11.2023 and put to use on same
date. In this case, depreciation shall be allowed at half rate.

e) Asset not put to use during the year of purchase: If it is purchased during
the year and is not at all put to use, depreciation shall not be allowed. But in the
subsequent year whenever the asset is put to use, full depreciation shall be
allowed irrespective of period of use.

For example: Mr. purchased one asset on 01.06.2022 and it is not put to use
during P.Y. 2022-23. In this case, depreciation shall not be allowed during P.Y.
2022-23. Now, the asset is put to use on 28.02.2024. In this case, depreciation
shall be allowed at full rate even if it is put to use for less than 180 days.

“Put to use” do not mean putting the asset to actual use rather it means making
an asset ready for use.

(vi) Block of Asset:

Under the provisions of income tax law, depreciation is not computed on the
basis of individual assets rather it is computed on the basis of a group of
assets called Block of Assets which means a group of similar type of assets
having same rate of depreciation and shall be computed in the manner given
below:

Step 1: Opening Value as on first day of previous year (PY) i.e. 01.04.2023
Step 2: Add:
a) Asset purchased and put to use for less than 180 days
b) Asset purchased and put to use for 180 days or more
Step 3: less:
Sale value of asset sold during the year (both “less than 180 days” or
“180 days or more”)
Step 4: wdv before depreciation (Step 1+Step 2-Step 3)
Step 5: Calculation of depreciation allowance on amount calculated in Step 4:
a) Asset purchased and put to use for less than 180 days at half
the applicable rate.
b) Asset purchased and put to use for 180 or more at the applicable
rate.

249 | P a g e #HakkसेCA by CA. Akash Sir


Illustration: 1

Mr. Anshu, a proprietor engaged in manufacturing business, furnishes the following


particulars:

Particulars ₹
(1) Opening balance of plant and machinery as on 40,00,000
01.04.2023 (rate 15%)
(2) New plant and machinery purchased and put to 10,00,000
use on 08.06.2023
(3) New plant and machinery acquired and put to 5,00,000
use on 15.12.2023
(4) Computer acquired and installed in the office 7,00,000
premises on 02.01.2024 (rate 40%)

Compute the amount of depreciation as per the Income tax Act, 1961 for the A.Y.
2024-25. Assume that all the assets were purchased by way of account payee
cheque.

Solution:

Computation of depreciation allowance for A.Y. 2024-25 in case of Mr. Anshu

Particulars Block A Block B


(15%) (40%)
Opening balance of plant and 40,00,000 0
machinery as on 01.04.2023
Assets purchased and put to use
for less than 180 days
- New plant and machinery 5,00,000 0
purchased and put to use for less
than 180 days i.e. on 15.12.2023
- Computer acquired and installed 0 7,00,000
in the office premises for less
than 180 days i.e. on 02.01.2024
Assets purchased and put to use
for 180 days or more
- - New plant and machinery 10,00,000 0
purchased and put to use for 180
days or more i.e. on 08.06.2023
55,00,000 7,00,000
Less: Sold during the year 0 0
Wdv before depreciation 55,00,000 7,00,000
Depreciation allowance

250 | P a g e #HakkसेCA by CA. Akash Sir


- - Assets purchased and put to use 37,500 1,40,000
for less than 180 days [5,00,000*15%/2] [7,00,000*40%/2]
- - Assets purchased and put to use 7,50,000 0
for 180 days or more [50,00,000*15%]
Total depreciation allowance 7,87,500 1,40,000
under section 32

Step 5A: If amount calculated in Step 4 i.e. “wdv before depreciation” is more
than “Assets purchased and put to use for less than 180 days” but is less than
“Total addition” made during the year, then calculate depreciation as below:
a) On asset purchased and put to use for less than 180 days at half the
applicable rate
b) On (wdv before depreciation calculated in step 4 – above amount) at the
applicable rate

Illustration: 2

Opening w.d.v. as on 01.04.2023 (Building A) ₹3,00,000


Purchased and put to use on 01.06.2023 (Building B) ₹4,00,000
Purchased and put to use on 01.11.2023 (Building C) ₹5,00,000
Sold during the year (Building B) ₹4,00,000
Rate of depreciation is 10%
Calculate Depreciation allowance for A.Y. 2024-25.

Solution:

Particulars Amount
Step 1 Opening w.d.v. as on 01.04.2023 3,00,000
Step 2 - Purchased and put to use for less than 180 days 5,00,000
- Purchased and put to use for 180 days or more 4,00,000
12,00,000
Step 3 Sold during the year (4,00,000)
Step 4 W.D.V. before depreciation 8,00,000
Step 5 Depreciation allowance
- Assets purchased and put to use for less than 180 25,000
days [5,00,000 * 10%/2]
- Assets purchased and put to use for 180 days or 30,000
more [(8,00,000-5,00,000) * 10%]
Total depreciation allowance 55,000

What if, building C, which is “purchased and put to use for less than 180 days” is
sold instead of building B in above example?

251 | P a g e #HakkसेCA by CA. Akash Sir


Solution: Please note that our answer in above case will remain same even if we
have sold the asset which was purchased on 01.11.2023 i.e. purchased and put to
use for less than 180 days.

(Now, you may think that since we have sold the asset which was purchased and put
to use for less than 180 days, therefore, depreciation on the remaining assets shall
be allowed at full rate on balance amount calculated in step 4. Please keep this in
mind that we NEVER give importance to individual asset (i.e. building A, B or C)
while calculating depreciation under “Block of Assets” method. Therefore, we shall
NOT consider the individual asset and IGNORE the names of assets if given in the
questions.)

Step 5B: If amount calculated in Step 4 i.e. “wdv before depreciation” is


less than “Assets purchased and put to use for less than 180 days”, then
calculate depreciation as below:
a) On “Balance amount” at half the applicable rate

Illustration: 3

Opening w.d.v. as on 01.04.2023 (Building A) ₹3,00,000


Purchased and put to use on 01.06.2023 (Building B) ₹4,00,000
Purchased and put to use on 01.11.2023 (Building C) ₹5,00,000
Sold during the year (Building B) ₹8,00,000
Rate of depreciation is 10%
Calculate Depreciation allowance for A.Y. 2024-25.

Solution:

Particulars Amount
Step 1 Opening w.d.v. as on 01.04.2023 3,00,000
Step 2 - Purchased and put to use for less than 180 days 5,00,000
- Purchased and put to use for 180 days or more 4,00,000
12,00,000
Step 3 Sold during the year (8,00,000)
Step 4 Wdv before depreciation 4,00,000
Step 5 Depreciation allowance
- Assets purchased and put to use for less than 180 20,000
days [4,00,000 * 10%/2]
- Assets purchased and put to use for 180 days or 0
more
Total depreciation allowance 20,000

252 | P a g e #HakkसेCA by CA. Akash Sir


What if, building C, which is “purchased and put to use for less than 180 days” is
sold instead of building B in above example?

Solution: Please note that our answer in above case will remain same even if we
have sold the asset which was purchased on 01.11.2023 i.e. purchased and put to
use for less than 180 days.

(Now, you may think that since we have sold the asset which was purchased and put
to use for less than 180 days, therefore, depreciation on the remaining assets shall
be allowed at full rate on balance amount calculated in step 4. Please keep this in
mind that we NEVER give importance to individual asset (i.e. building A, B or C)
while calculating depreciation under “Block of Assets” method. Therefore, we shall
NOT consider the individual asset and IGNORE the names of assets if given in the
questions.)

Step 6: If any block of asset has negative balance after deducting sales value,
then such negative balance is Short-Term Capital Gain under section 50 of the
Act (re-discussed in capital gain chapter) and no depreciation allowance is
allowed.

Illustration: 4

Opening w.d.v. as on 01.04.2023 (Building A) ₹3,00,000


Purchased and put to use on 01.06.2023 (Building B) ₹4,00,000
Purchased and put to use on 01.11.2023 (Building C) ₹5,00,000
Sold during the year (Building B) ₹14,00,000
Rate of depreciation is 10%
Calculate Depreciation allowance for A.Y. 2024-25.

Solution:

Particulars Amount
Step 1 Opening w.d.v. as on 01.04.2023 3,00,000
Step 2 - Purchased and put to use for less than 180 5,00,000
days
- Purchased and put to use for 180 days or 4,00,000
more
12,00,000
Step 3 Sold during the year (14,00,000)
Step 4 Wdv before depreciation (2,00,000)
Step 5 Depreciation allowance
- Assets purchased and put to use for less 0
than 180 days

253 | P a g e #HakkसेCA by CA. Akash Sir


- Assets purchased and put to use for 180 days 0
or more
Total depreciation allowance 0

DO NOT WRITE ABOVE TABLE IN EXAM. Presentation of exam is as below:

Full value of consideration (FVC) 14,00,000


Less: Cost of acquisition (COA) i.e. w.d.v plus addition 12,00,000
Short term capital gain 2,00,000

Step 7: If any block of asset has positive balance after deducting sales value,
but there is no asset left in the block i.e. block cease to exist, then such positive
balance is Short Term Capital Loss under section 50 of the Act (re-discussed in
capital gain chapter) and no depreciation allowance is allowed.

Illustration: 5

Opening w.d.v. as on 01.04.2023 (Building A) ₹3,00,000


Purchased and put to use on 01.06.2023 (Building B) ₹4,00,000
Purchased and put to use on 01.11.2023 (Building C) ₹5,00,000
Sold during the year (Building A, B & C) ₹10,00,000
Rate of depreciation is 10%
Calculate Depreciation allowance for A.Y. 2024-25.

Solution:

Particulars Amount
Step 1 Opening w.d.v. as on 01.04.2023 3,00,000
Step 2 - Purchased and put to use for less than 180 5,00,000
days
- Purchased and put to use for 180 days or more 4,00,000
12,00,000
Step 3 Sold during the year (10,00,000)
Step 4 Wdv before depreciation 2,00,000
Step 5 Depreciation allowance
- Assets purchased and put to use for less than 0
180 days
- Assets purchased and put to use for 180 days 0
or more
Total depreciation allowance 0

Do not write above table in exam. Presentation of exam is as below:

254 | P a g e #HakkसेCA by CA. Akash Sir


Full value of consideration (FVC) 10,00,000
Less: Cost of acquisition (COA) i.e. w.d.v plus addition 12,00,000
Short term capital loss 2,00,000

Illustration: 6

Mr. Ravin started his business on 01.04.2020 and purchased various plants and
machinery as given below:
He has purchased plant P1 on 01.04.2020 which was put to use on 01.06.2020 for
₹20,00,000.
He has purchased plant P2 on 01.05.2020 which was put to use on 01.07.2020 for
₹25,00,000.
He has purchased plant P3 on 01.06.2020 which was put to use on 01.09.2020 for
₹25,00,000.
He has purchased plant P4 on 01.07.2020 which was put to use on 01.09.2020 for
₹35,00,000.
He sold plant P1 on 01.01.2021 for ₹11,00,000.
He purchased plant P5 on 01.05.2021 and was put to use on 01.11.2021 for ₹26,00,000.
He purchased plant P6 on 01.12.2021 and was put to use on 31.03.2022 for ₹20,00,000.
He purchased plant P7 on 01.06.2022 and put to use on 10.12.2022 for ₹10,00,000.
He sold plant P2 on 31.03.2023 for ₹9,00,000.
He purchased plant P8 on 01.07.2023 and was put to use on 01.01.2024 for ₹27,00,000.
Determine depreciation for various years. (Ignore additional depreciation)

Solution:

Computation of depreciation allowance for P.Y. 2020-21

Particulars Amount
Opening written down value as on 01.04.2020 0
Add: Assets purchased and put to use for less than 0
180 days
Add: Assets purchased and put to use for 180 days
or more
- Purchased P1 on 01.04.2020, put to use on 20,00,000
01.06.2020
- Purchased P2 on 01.05.2020, put to use on 25,00,000
01.07.2020
- Purchased P3 on 01.06.2020, put to use on 25,00,000
01.09.2020
- Purchased P4 on 01.07.2020, put to use on 35,00,000 1,05,00,000
01.09.2020
Less: Assets sold during the year

255 | P a g e #HakkसेCA by CA. Akash Sir


- Sale P1 on 01.01.2021 11,00,000
Written down value P2, P3 and P4 on 31.03.2021 94,00,000

Depreciation allowance @15%:


- Assets purchased and put to use for 180 days 14,10,000
or more [94,00,000 *
15%]

Computation of depreciation allowance for P.Y. 2021-22

Particulars Amount
Written down value P2, P3 and P4 on 31.03.2021 79,90,000
Add: Assets purchased and put to use for less than
180 days
- Purchased P5 on 01.05.2021, put to use on 26,00,000
01.11.2021
- Purchased P6 on 01.12.2021, put to use on 20,00,000 46,00,000
31.03.2022
Add: Assets purchased and put to use for 180 days 0
or more
Less: Assets sold during the year 0
Written down value P2, P3, P4, P5 and P6 on 1,25,90,000
31.03.2022

Depreciation allowance @15%:


- Assets purchased and put to use for less than 3,45,000
180 days [1,25,90,000 *
7.5%]
- Assets purchased and put to use for 180 days 11,98,500
or more [79,90,000 *
15%]

Computation of depreciation allowance for P.Y. 2022-23

Particulars Amount
Written down value P2, P3, P4, P5 and P6 on 1,10,46,500
01.04.2022
Add: Assets purchased and put to use for less than
180 days
- Purchased P7 on 01.06.2022, put to use on 10,00,000 10,00,000
10.12.2022
Add: Assets purchased and put to use for 180 days 0
or more
Less: Assets sold during the year

256 | P a g e #HakkसेCA by CA. Akash Sir


- Sale P2 on 31.03.2023 9,00,000 9,00,000
Written down value P3, P4, P5, P6 and P7 on 1,11,46,500
31.03.2023

Depreciation allowance @15%:


- Assets purchased and put to use for less than 75,000
180 days [10,00,000 *
7.5%]
- Assets purchased and put to use for 180 days 15,21,975
or more [1,01,46,500 *
15%]

Computation of depreciation allowance for P.Y. 2023-24

Particulars Amount
Written down value P3, P4, P5, P6 and P7 on 95,49,525
01.04.2023
Add: Assets purchased and put to use for less than
180 days
- Purchased P8 on 01.07.2023, put to use on 27,00,000 27,00,000
01.01.2024
Add: Assets purchased and put to use for 180 days 0
or more
Less: Assets sold during the year 0
Written down value P3, P4, P5, P6, P7 and P8 on 1,22,49,525
31.03.2024

Depreciation allowance @15%:


- Assets purchased and put to use for less than 2,02,500
180 days [27,00,000 *
7.5%]
- Assets purchased and put to use for 180 days 14,32,429
or more [95,49,525 *
15%]

Illustration: 7

Arman Ltd. is a manufacturing company. On April 1st, 2023, it owns plant A and plant B
(depreciation rate: 15 per cent; depreciated value of block being ₹2,40,000). Plant C
(depreciation rate: 15 per cent) is purchased by the company on June 10th, 2023 for
₹60,000 and it was used in the office premises. It is put to use on the same day.
Find out the tax consequences in the following different situations:
1. Plant B is destroyed by fire on January 25th, 2024. ₹10,000, being the compensation,
is paid by the insurance company on February 10th, 2024;

257 | P a g e #HakkसेCA by CA. Akash Sir


2. If the insurance compensation in situation (1) is ₹3,70,000;
3. Plant A, B And C is destroyed by fire on January 25th, 2024. Compensation paid by
insurance company on February 10th, 2024 is ₹20,000;
4. If the insurance compensation in situation (3) is ₹4 lakhs.

Solution:

Situation 1
Written down value of Plant A and Plant B as on 01.04.2023 2,40,000
Add: Plant C purchased on 10.06.2023 and
put to use on the same date 60,000
Less: Insurance claim of plant B (10,000)
Written down value as on 31.03.2024 2,90,000
Depreciation @ 15% on ₹2,90,000 43,500

Situation 2
Full value of consideration (Insurance claim) 3,70,000
Less: Written down value of Plant A and Plant B as on 01.04.2023 (2,40,000)
Less: Plant C purchased and put to use on 10.06.2023 (60,000)
Short term capital gain as per section 50 70,000
No depreciation is allowed

Situation 3
Full value of consideration (Insurance claim) 20,000
Less: Written down value of Plant A and Plant B as on 01.04.2023 (2,40,000)
Less: Plant C purchased and put to use on 10.06.2023 (60,000)
Short term capital loss as per section 50 2,80,000
No depreciation is allowed

Situation 4
Full value of consideration (Insurance claim) 4,00,000
Less: Written down value of Plant A and Plant B as on 01.04.2023 (2,40,000)
Less: Plant C purchased and put to use on 10.06.2023 (60,000)
Short term capital gain as per section 50 1,00,000
No depreciation is allowed

HW Question: 1

A newly qualified Chartered Accountant Miss Manavi, commenced practice and has
acquired the following assets in her office during F.Y. 2023-24 at the cost shown
against each item. Calculate the amount of depreciation that can be claimed from her
professional income for A.Y. 2024-25:

258 | P a g e #HakkसेCA by CA. Akash Sir


Sr. Description Date of Date when Amount
No. Acquisition put to use (₹)
1 Computer 27 Sep.,2023 02 Oct., 2023 35,000
2 Computer software 01 Oct., 2023 01 Oct., 2023 8,500
3 Computer printer 01 Oct., 2023 03 Oct., 2023 12,500
4 Books 01 Apr., 2023 01 Apr., 2023 13,000
5 Office furniture (Acquired from 01 Apr., 2023 01 Apr., 2023 3,00,000
practicing C.A.)
6 Laptop 26 Sept.,2023 01 Oct., 2023 43,000
7 Fire extinguisher 01 Apr., 2023 01 Apr., 2023 2,500
(No instance
arose to use
during F.Y.
2023-24)
8 Purchased practicing CA’s office
in April’ 2023 who had run it for
4 years, for ₹5 lacs which
includes ₹2 lacs for goodwill and
₹3 lacs for cost of furniture
(included in 5 above)

HW Question: 2

Mr. Babu has the following Assets which are eligible for depreciation at 15% on
Written Down Value (WDV) basis:
01.04.2020 WDV of plant ‘X’ and Plant ‘Y’ ₹2,00,000
10.12.2023 Acquired a new plant ‘Z’ for ₹2,00,000
22.01.2024 Sold plant ‘Y’ for ₹4,00,000
Expenditure incurred in connection with transfer ₹10,000
Compute eligible depreciation claim/ chargeable capital gain if any, for the Assessment
Year 2024-25.

HW Question: 3

Goyal Ltd. furnishes you the following information:


Block I: Plant and machinery (consisting of 3 plants), rate of depreciation 15%, w.d.v. on
April 1, 2023 of ₹2,70,000.
Block II: Buildings (two buildings), rate of depreciation 10%, w.d.v. on April 1st, 2023 of
₹6,50,000.
Acquired on June 2, 2023, 2 plants for ₹2,10,000 and put to use on the same date.
Sold on November 30, 2023 all the five plants for ₹5,00,000.
Acquired on December 15, 2023 two plants for ₹1,60,000 and put to use on the same
date.

259 | P a g e #HakkसेCA by CA. Akash Sir


Admissible rate of depreciation in relation to all acquired plants is 15%.
Compute the amount of depreciation admissible to Goyal Ltd. for the Assessment Year
2024-25.

TP: 6 Additional Depreciation on new P&M [section 32(1)(iia)]

➢ Additional depreciation shall be allowed @ 20% of Actual cost of P&M and


➢ only on plant and machinery for the purpose of manufacturing (including
printing or printing and publishing)
➢ to the assessee engaged in generation, transmission or distribution of
electricity.
➢ Allowed only once i.e. it is not allowed every year.
➢ No additional depreciation under New Tax Regime u/s. 115BAC.

Additional depreciation is not allowed in the following cases:


(i) Second hand plant and machinery (whether used in India or outside India) i.e.
plant and machinery should be brand new.
(ii) Any machinery or plant (including computers etc.) installed in any office
premises or any residential accommodation, including accommodation in the
nature of a guest-house or
(iii) Any office appliances or road transport vehicles or ships and aircraft
Any machinery or plant, the actual cost of which has been debited to profit and loss
account. It means that if 100% cost of such P&M is already allowed, then no
additional depreciation shall be allowed.

Special Note: If the asset is purchased and put to use for less than 180 days,
additional depreciation shall be allowed at 10% in the PY and remaining additional
depreciation shall be allowed in the subsequent year if the assessee exercises
the option of shifting out of default tax regime u/s. 115BAC.

Mann Ki Baat (Simple Language):

This provision has come into existence to promote the manufacturing and power
generation businesses by reducing tax burden in the year of P&M purchase. However, if
you deep dive into concept then you will realize that there is no additional benefit as
such during the entire life of asset because if additional depreciation is allowed in 1
year, then, it will reduce the w.d.v. of asset for subsequent year which will reduce
depreciation of subsequent years. However, please note that taxable income in the
initial year will reduce due to additional depreciation which will help business by
lessening the burden of income tax in 1st year of acquisition of New P&M.

260 | P a g e #HakkसेCA by CA. Akash Sir


One most important point to note that the additional depreciation is allowed only if the
Assessee has shifted out of the provisions of section 115BAC (in other words, he opted
for old tax regime).

Illustration: 8

Opening w.d.v. as on 01.04.2023 (Plant and Machinery “A”) - ₹30,00,000


New Plant and Machinery purchased and put to use on 01.06.2023 (“B”) – ₹10,00,000
New Plant and Machinery Purchased and put to use on 01.11.2023 (”C”) – ₹15,00,000
Rate of depreciation is 15%
Calculate Depreciation allowance and additional depreciation for A.Y. 2024-25.

Solution:

Opening w.d.v. as on 01.04.2023 30,00,000


- Purchased and put to use for less than 180 days 15,00,000
- Purchased and put to use for 180 days or more 10,00,000
55,00,000
Sold during the year 0
Balance Amount 55,00,000
Depreciation allowance
- Assets purchased and put to use for less than 180 1,12,500
days [15,00,000 * 15%/2]
- Assets purchased and put to use for 180 days or 6,00,000
more [40,00,000 * 15%]
Total depreciation allowance 7,12,500
Additional depreciation allowance
- New plant and machinery purchased and put to use 1,50,000
for less than 180 days [15,00,000 * 20%/2]
- New plant and machinery purchased and put to use 2,00,000
for 180 days or more [10,00,000 * 20%]
Total additional depreciation allowance 3,50,000

Please note that remaining 10% additional depreciation on new plant and machinery of
₹15,00,000 which is purchased and put to use for less than 180 days will be allowed in
subsequent P.Y. i.e. P.Y. 2024-25.

Illustration: 9

Opening w.d.v. as on 01.04.2023 (Plant and Machinery “A”) - ₹30,00,000


New Plant and Machinery purchased and put to use on 01.06.2023 (“B”) – 10,00,000
New Plant and Machinery Purchased and put to use in office on 01.11.2023 (”C”) –
15,00,000

261 | P a g e #HakkसेCA by CA. Akash Sir


Rate of depreciation is 15%
Calculate Depreciation allowance and additional depreciation for A.Y. 2024-25.

Solution:

Opening w.d.v. as on 01.04.2023 30,00,000


- Purchased and put to use for less than 180 days 15,00,000
- Purchased and put to use for 180 days or more 10,00,000
55,00,000
Sold during the year 0
Balance Amount 55,00,000
Depreciation allowance
- Assets purchased and put to use for less than 180 1,12,500
days [15,00,000 * 15%/2]
- Assets purchased and put to use for 180 days or 6,00,000
more [40,00,000 * 15%]
Total depreciation allowance 7,12,500
Additional depreciation allowance
- New plant and machinery purchased and put to use 0
for less than 180 days
- New plant and machinery purchased and put to use 2,00,000
for 180 days or more [10,00,000 * 20%]
Total additional depreciation allowance 2,00,000

Please note that no additional depreciation shall be allowed on new plant and machinery
of ₹15,00,000 which is purchased and installed in office.

Illustration: 10

Opening w.d.v. as on 01.04.2023 (Plant and Machinery “A”) - ₹30,00,000


Second hand Plant and Machinery purchased and put to use on 01.06.2023 (“B”) –
₹10,00,000
New Plant and Machinery Purchased and put to use on 01.11.2023 (”C”) – ₹15,00,000
Rate of depreciation is 15%
Calculate Depreciation allowance and additional depreciation for A.Y. 2024-25.

Solution:

Opening w.d.v. as on 01.04.2023 30,00,000


- Purchased and put to use for less than 180 days 15,00,000
- Purchased and put to use for 180 days or more 10,00,000
55,00,000
Sold during the year 0

262 | P a g e #HakkसेCA by CA. Akash Sir


Balance Amount 55,00,000
Depreciation allowance
- Assets purchased and put to use for less than 180 1,12,500
days [15,00,000 * 15%/2]
- Assets purchased and put to use for 180 days or 6,00,000
more [40,00,000 * 15%]
Total depreciation allowance 7,12,500
Additional depreciation allowance
- New plant and machinery purchased and put to use 1,50,000
for less than 180 days [15,00,000 * 20%/2]
- New plant and machinery purchased and put to use 0
for 180 days or more
Total additional depreciation allowance 1,50,000

Please note that no additional depreciation shall be allowed on second hand plant and
machinery of ₹10,00,000.

Illustration: 11

Opening w.d.v. as on 01.04.2023 (Plant and Machinery “A”) - ₹30,00,000


New Plant and Machinery purchased and put to use in guest house on 01.06.2023 (“B”) –
₹10,00,000
New car purchased and put to use on 01.11.2023 (”C”) – ₹15,00,000
Rate of depreciation is 15%
Calculate Depreciation allowance and additional depreciation for A.Y. 2024-25.

Solution:

Particulars Amount
Opening w.d.v. as on 01.04.2023 30,00,000
- Purchased and put to use for less than 180 days 15,00,000
- Purchased and put to use for 180 days or more 10,00,000
55,00,000
Sold during the year 0
Balance Amount 55,00,000
Depreciation allowance
- Assets purchased and put to use for less than 180 1,12,500
days [15,00,000 * 15%/2]
- Assets purchased and put to use for 180 days or 6,00,000
more [40,00,000 * 15%]
Total depreciation allowance 7,12,500
Additional depreciation allowance

263 | P a g e #HakkसेCA by CA. Akash Sir


- New plant and machinery purchased and put to use 0
for less than 180 days
- New plant and machinery purchased and put to use 0
for 180 days or more
Total additional depreciation allowance 0

Note: Please note that no additional depreciation shall be allowed on P&M installed in
guest house and on motor vehicle.

Mann Ki Baat (Simple Language):

From above examples, we understand that there can be numerous questions which can
be formed basis on conditions explained above. You have to remember every condition
thoroughly and practice different questions.

Illustration: 12

Ram Ltd. is engaged in manufacturing and has submitted information as given below:
1. Factory Building - Written down value on 01.04.2023 was ₹12,00,000.
2. Plant and Machinery (Rate 15%) – Written down value on 01.04.2023 is ₹8,70,000.
3. Purchase of new plant (eligible for additional depreciation) on 30.06.2023 (Put to use
on 01.07.2023) for ₹1,20,000.
4. Purchase of new plant (eligible for additional depreciation) on 31.12.2023 (Put to use
on 01.01.2024) for ₹1,10,000.
5. Sale of old Plant on 01.12.2023 for ₹6,40,000.
6. Motor Car (Rate 15%) – Written down value on 01.04.2023 was ₹1,20,000.
7. Sale of Car on 30.09.2023 for ₹1,50,000.
Compute depreciation allowance for A.Y. 2024-25.

Solution:

Factory Building, Depreciation @ 10%


Written down value on 01.04.2023 12,00,000
Depreciation @ 10% 1,20,000

Plant and Machinery, Depreciation @ 15%


Written down value on 01.04.2023 8,70,000
Add: Purchased and put to use for less than 180 days
- Purchase on 31.12.2023, put to use on 01.01.2024 1,10,000
Add: Purchased and put to use for 180 days or more
- Purchase on 30.06.2023, put to use on 01.07.2023 1,20,000
Less: Sale of old plant on 01.12.2023 (6,40,000)
Written down value on 31.03.2024 4,60,000
Depreciation allowance:

264 | P a g e #HakkसेCA by CA. Akash Sir


- Depreciation @ 7.5% on ₹1,10,000 8,250
- Depreciation @ 15% on ₹3,50,000 52,500
Additional depreciation
- New plant and machinery purchased on 30.06.2023
(1,20,000 x 20%) 24,000
- New plant and machinery purchased on 31.12.2023
(1,10,000 x 20%/2) 11,000

Motor Car, Depreciation @ 15%


Sale consideration (sale on 30.09.2023) 1,50,000
Less: Written down value on 01.04.2023 1,20,000
Short term capital gain 30,000

HW Question: 4

Bharat Ltd. is engaged in manufacturing and company has purchased plant and
machinery during the previous year 2023-24 for ₹26 crores (purchased and put to use
on 10.11.2023) and it includes second hand plant and machinery for ₹5 crores.
Compute depreciation/ additional depreciation for A.Y. 2024-25 and also w.d.v as on
01.04.2024

HW Question: 5

Mohan Ltd., a manufacturing company purchased the following new Plant and Machinery.
Date of Acquisition and Installation Actual Cost (in Crores)
25.05.2023 10.00
31.10.2023 22.00
From the above information compute the amount of depreciation available u/s 32,
additional depreciation, if any for the Assessment Years 2024-25

HW Question: 6

Laxman Ltd. is engaged in manufacturing and company has purchased new plant and
machinery during the previous year 2023-24:
1. ₹20.00 crore (purchased and put to use on 01.07.2023)
2. ₹30.00 crore (purchased and put to use on 01.11.2023)
Compute depreciation/ additional depreciation and also w.d.v as on 01.04.2024.

HW Question: 7

Rahul Ltd. an industrial undertaking has started manufacturing on 01.05.2023 and the
company has purchased the following assets:

265 | P a g e #HakkसेCA by CA. Akash Sir


1. Plant and machinery for use in the factory ₹30 lakhs, purchased on 01.07.2023 and
put to use on 15.07.2023.
2. Air-conditioner and generator for ₹2,00,000, purchased on 01.08.2023 and put to
use on 10.08.2023 for use in office premises.
3. One motor car for ₹10 lakhs for use of business, purchased on 01.09.2023 and put to
use on 10.09.2023.
4. One T.V. and one fridge for ₹50,000, purchased and put to use on 01.05.2023.
5. Furniture and fixture for use in factory ₹5,00,000, purchased and put to use on
01.06.2023.
Compute depreciation and additional depreciation allowance for A.Y. 2024-25.

HW Question: 8

Mr. Shyam is engaged in the business of generation and distribution of electric power.
He always claims depreciation on written down value. From the following details,
compute the depreciation allowable as per the provisions of the Income-tax Act, 1961
for the assessment year 2024-25:
(₹ in lacs)
(i) Opening WDV of block (15% rate) 42
(ii) New machinery purchased on 12.10.2023 10
(iii) Machinery imported from Colombo on 12.04.2023.
This machine had been used only in Colombo earlier and
the assessee is the first user in India. 9
(iv) New computer installed in generation wing of the unit
on 15.07.2023 2
Compute depreciation allowance for A.Y. 2024-25.

HW Question: 9

Mr. Manish, a proprietor engaged in manufacturing business, furnishes the following


particulars:

Sr. no. Particulars ₹


1 Opening balance of plant and machinery as on 01.04.2023 30,00,000
(i.e., WDV as on 31.3.2023 after reducing depreciation
for P.Y. 2022-23)
2 New plant and machinery purchased and put to use on 20,00,000
08.06.2023
3 New plant and machinery acquired and put to use on 8,00,000
15.12.2023
4 Computer acquired and installed in the office premises 3,00,000
on 02.01.2024

266 | P a g e #HakkसेCA by CA. Akash Sir


Compute the amount of depreciation and additional depreciation as per the Income-tax
Act, 1961 for the A.Y. 2024-25. Assume that all the assets were purchased by way of
account payee cheque.

TP: 7 Depreciation allowance for power generating undertaking

A power generating unit shall have the option to claim depreciation either on the
basis of SLM or WDV method and any option taken cannot be changed subsequently.

If the assessee has claimed depreciation on the basis of WDV, depreciation shall be
allowed on the basis of block of the assets (already studies) and if depreciation is
claimed on SLM, depreciation shall be computed on the basis of individual asset
however concept of 180 days shall be applicable. Rate of depreciation for SLM
shall be as prescribed under Income Tax Act.

Treatment on sale of asset on which depreciation is claimed on SLM


method:

Terminal depreciation

If the asset is sold, any loss on their sale shall be considered to be terminal
depreciation and shall be allowed to be debited to the profit and loss account.

Balancing Charge Section 41(2)

If any asset has been sold or destroyed etc. and depreciation was claimed on SLM
basis, any profit on sale shall be considered to be income under the head
business/profession and shall be called balancing charge but only to the extent
depreciation was debited to the profit and loss account. The excess over it shall be
taxable as capital gains under section 50A.

If the amount is received after closing down of the business, still it will be
considered to be income under the head business/profession i.e. it will be a case of
having income under the head business/profession but without any business/
profession.

Mann Ki Baat (simple language):

Option has been given to the assessee to calculate depreciation on SLM or WDV
Method. Now, in case WDV method is chosen, then all the concepts shall remain same

267 | P a g e #HakkसेCA by CA. Akash Sir


as Block of Asset method. If SLM is chosen, then the concept of gain/ loss on sale shall
change and balancing charge/ terminal depreciation shall be calculated as above.

Illustration: 13

Lights and Power Ltd. engaged in the business of generation of power, furnishes the
following particulars pertaining to P.Y. 2023-24. Compute the depreciation allowable
under section 32 for A.Y. 2024-25. The company has opted for the depreciation
allowance on the basis of written down value.

Particulars ₹
1. Opening Written down value of Plant and Machinery (15% block)
as on 01.04.2023 5,78,000
2. Purchase of second hand machinery (15% block) on 29.12.2023
for business purpose and put to use on the same date 2,00,000
3. Machinery Y (15% block) purchased and installed on 12.07.2023
for the purpose of power generation 8,00,000
4. Acquired and installed for use a new air pollution control equipment
(40% block) on 31.07.2023 2,50,000
5. New air conditioner purchased and installed in office premises
on 08.09.2023 3,00,000
6. New machinery Z (15% block) acquired and installed on 23.11.2023
for the purpose of generation of power 3,25,000
7. Sale value of an old machinery X, sold during the year 3,10,000

Solution:

Computation of depreciation allowance under section 32 for the A.Y. 2024-25

Particulars P&M P&M


(15%) (40%)
Opening WDV as on 01.04.2023 5,78,000
Add: Plant and Machinery acquired during
the year
- Second hand machinery 2,00,000
- Machinery Y 8,00,000
- Air conditioner for office 3,00,000
- Machinery Z 3,25,000
- Air pollution control equipment 2,50,000
Less: Asset sold during the year (3,10,000) Nil
Written down value before depreciation 18,93,000 2,50,000

268 | P a g e #HakkसेCA by CA. Akash Sir


Normal depreciation
40% on air pollution control equipment 1,00,000
Depreciation on plant and machinery put to use
For less than 180 days@ 7.5%
(2,00,000 + 3,25,000) 39,375
15% on ₹ 13,68,000 2,05,200
Additional depreciation
- Machinery Y (₹ 8,00,000 × 20%) 1,60,000
- Machinery Z (₹ 3,25,000 × 10%) 32,500
Total depreciation 4,37,075 1,00,000

TP: 8 Actual Cost [Section 43(1)]

The expression “actual cost” means the actual cost of the asset to the assessee as
reduced by that portion of the cost thereof, if any, as has been met directly or
indirectly by any other person or authority. In simple words, actual cost means cost
incurred by the assessee to purchase the asset. However, if any portion of the cost
is met directly by some other person, then it should be included in actual cost.

Moreover, if the assessee has received any subsidy from the Government or other
similar agency, the subsidy so received shall be deducted and only the balance amount
shall be considered to be the actual cost. For example: If the assessee has received
a subsidy of ₹2 lakh in connection with plant and machinery of ₹20,00,000 because it
was a non-polluting plant, then, actual cost of asset shall be ₹18,00,000.

IMPORTANT: However, if any payment is made otherwise than by an account payee


cheque drawn on a bank or account payee bank draft or use of electronic clearing
system through a bank account or through such other prescribed electronic mode, in
excess of ₹10,000 to a person in a single day, then such expenditure shall not form
part of actual cost of such asset [Second proviso to section 43(1)]. For example, Mr.
X purchased one machinery for ₹10,00,000 and made payment of ₹20,000 in cash and
balance through NEFT, then Actual cost of the asset shall be ₹9,80,000.

The prescribed electronic modes include credit card, debit card, net banking, IMPS
(Immediate payment Service), UPI (Unified Payment Interface), RTGS (Real Time
Gross Settlement), NEFT (National Electronic Funds Transfer), and BHIM (Bharat
Interface for Money) Aadhar Pay.

Actual cost in certain special situations [Explanations to section 43(1)]

1. Building previously the property of the assessee: Where a building which was
previously the property of the assessee is brought into use for the purposes of the

269 | P a g e #HakkसेCA by CA. Akash Sir


business or profession, its actual cost to the assessee shall be the actual cost of the
building to the assessee, as reduced by an amount equal to the depreciation
calculated at the rates in force on that date that would have been allowable had the
building been used for the purposes of the business or profession since the date of
its acquisition by the assessee. Note: This provision is only applicable to building
and not to any other capital asset.

Illustration: 14

Dr. Sagar purchased a residential building on 01.12.2020 for ₹12,00,000 and it was put
to use on the same date. Till 01.12.2022 the same was self-occupied as residence. On
this date, the building was brought into use for the purpose of his medical profession
(it was used as residential building). What would be the depreciation allowable for the
Assessment Year 2023-24?

Solution:

Computation of depreciation for the Assessment Year 2023-24

In this case notional depreciation shall be allowed as per section 43(1) and depreciation
allowable for the Assessment Year 2023-24 shall be computed in the manner given
below:

Cost of building as on December 1st, 2020 12,00,000


Less: depreciation for the previous year 2020-21 (60,000)
(5% of ₹ 12,00,000)
(As building purchased during the year 2020-21 is put to use for less than 180 days
during the year)
Written down value as on 01.04.2021 11,40,000
Less: depreciation for previous year 2021-22 @ 10% (1,14,000)
Written down value as on 01.04.2022 10,26,000
Depreciation for the previous year 2022-23 @ 10% 1,02,600

Illustration: 15

A car purchased by Dr. Soman on 10.08.2019 for ₹5,25,000 for personal use is brought
into professional use on 1.07.2022 by him, when its market value was ₹2,50,000.
Compute the actual cost of the car and the amount of depreciation for the assessment
year 2023-24 assuming the rate of depreciation to be 15%.

270 | P a g e #HakkसेCA by CA. Akash Sir


Solution:

As per section 43(1), the expression “actual cost” would mean the actual cost of asset
to the assessee. The purchase price of ₹5,25,000 is, therefore, the actual cost of the
car to Dr. Soman. Market value (i.e. ₹2,50,000) on the date when the asset is brought
into professional use is not relevant. Therefore, amount of depreciation on car as per
section 32 for the A.Y.2023-24 would be ₹78,750, being ₹5,25,000 x 15%.

Mann Ki Baat (simple language):

From the above concept, we understood 2 things. First is that the actual cost of the
asset shall be purchase price (not in cash exceeding ₹10,000) and second is that in
case of building, notional depreciation should be reduced for the period building is
used for personal purpose.

2. Where a stock in trade is converted into capital assets: In such a case, the
fair market value of the stock shall be treated as actual cost of the asset.

3. Asset is acquired by way of Gift or inheritance: In such case, actual cost of


asset shall be actual cost of the previous owner minus depreciation allowable to the
assessee as if the asset was only asset in the relevant

Further, any expenditure incurred by the Assessee on freight, installation etc. shall
be added in actual cost.

4. Second hand asset: Where the asset has been previously used by any other
person for the purpose of business/ profession and subsequently the assessee
acquired such asset and the AO is satisfied that the main purpose of such transfer is
to claim higher depreciation, then the Actual cost shall be cost taken by AO with
prior approval of JCIT.

5. Re-acquisition of asset: Where the asset which had belonged to the assessee is
re-acquired by him, then actual cost shall be-
(a) actual cost when he first acquired the asset minus depreciation allowance
(b) the actual price at which the asset is re-acquired
Whichever is less.

271 | P a g e #HakkसेCA by CA. Akash Sir


TP: 9 Depreciation in case of succession or business
reorganization or amalgamation or demerger of companies

Where succession or reorganization or amalgamation or demerger of business takes


place during the year, the asset is partly used in old company and partly in new
company.
In such a case, depreciation shall be calculated as if no succession or business
reorganization or amalgamation or demerger has taken place.
Then amount of depreciation shall be apportioned between entities in the ratio of the
number of days for which the asset was used by them.

Illustration: 16

Mr. Lokesh carrying on business as proprietor converted the same into a limited
company by name Lokesh Cycles (P) Ltd. from 01.07.2023. The details of the assets are
given below:

Block - I WDV of plant & machinery (rate of depreciation @ 15%) 12,00,000
Block - II WDV of building (rate of depreciation @ 10%) 25,00,000

The company Lokesh Cycles (P) Ltd. acquired plant and machinery in December 2023 for
₹ 10,00,000. It has been doing the business from 01.07.2023.

Compute the quantum of depreciation to be claimed by Mr. Lokesh and successor


Lokesh Cycles (P) Ltd. for the assessment year 2024-25. Note: Ignore additional
depreciation.

Solution:

Calculation of depreciation allowable u/s. 32 as if there is no succession:

Particulars Block I Block II


(15%) (10%)
Opening w.d.v. as on 01.04.2023 12,00,000 25,00,000
Add: Purchased and put to use for less than 180 days
- Machinery acquired during Dec 2023 10,00,000
W.D.V as on 31.03.2024 22,00,000 25,00,000
Depreciation allowance:
- Assets purchased and put to use for less than 180 days 75,000
[10,00,000
* 15%/2]
- Assets purchased and put to use for 180 days or more 1,80,000 2,50,000

272 | P a g e #HakkसेCA by CA. Akash Sir


[12,00,000 [25,00,000 *
* 15%] 10%]
Total depreciation allowance 2,55,000 2,50,000

Number of days when assets are used by


Sole Proprietors 91 days
Company 274 days
Depreciation available to the sole proprietary Concern
(i.e. 91/ 366 of ₹4,30,000) 1,06,913

Depreciation available to the company


(i.e. 275/ 366 of ₹4,30,000) 3,23,087
Depreciation to the company on plant purchased 75,000
Total depreciation allowed to the company 3,98,087

Note: The depreciation on the plant which was purchased after conversion shall be
allowed to the company and further it is presumed that the plant was put to use on the
date of purchase.

HW Question: 10

M/s Hari & Co., a sole proprietary concern is converted into a company, Hari Co. Ltd.
With effect from December 29, 2023. The written down value of assets as on April 1st,
2023 is as follows:

Items Rate of Dep. WDV as on 01.04.2023


Building 10% ₹ 3,50,000
Furniture 10% ₹ 50,000
Plant & Machinery 15% ₹ 2,00,000

Further, on October 15, 2023, M/s Hari & Co. purchased a plant for ₹1,00,000 (rate of
depreciation 15%) and it was put to use on the same date. After conversion, the
company added another plant worth ₹50,000 (rate of depreciation 15%) on 01.01.2024
and put to use on the same date.
Compute the depreciation available to (i) M/s Hari & Co. and (ii) Hari Co. Ltd. for A.Y.
2024-25.

HW Question: 11

Rohan Ltd. has a block of assets carrying 15% rate of depreciation, whose written down
value on 01.04.2023 was ₹40 lacs. It purchased another asset of the same block on
01.11.2023 for ₹14.40 lacs and put to use on the same day. Rohan Ltd. was amalgamated
with Mohan Ltd. with effect from 01.01.2024. You are required to compute the

273 | P a g e #HakkसेCA by CA. Akash Sir


depreciation allowable to Rohan Ltd. & Mohan Ltd. for the previous year ended on
31.03.2024.

TP: 10 Expenditure on Scientific Research [Section 35(1)(i)/(iv)]

This section allows a deduction in respect of any expenditure on scientific research


incurred in relation to the business of the Assessee or contribution (donation) by the
assessee for scientific research etc.

274 | P a g e #HakkसेCA by CA. Akash Sir


(a) In house scientific research - Incurred after commencement of business:

(b) In house scientific research - Incurred before commencement of business:

For prior 3 years

For prior 3 years

275 | P a g e #HakkसेCA by CA. Akash Sir


(c) Contribution to outsiders:

Illustration: 17

Mr. Praveen Kumar has furnished the following particulars relating to payments made
towards scientific research for the year ended 31.03.2024:
(i) Payments made to K Research Ltd. 20
(ii) Payment made to LMN College 15
(iii) Payment made to OPQ College 10
Note: K Research Ltd. And LMN College are approved research institutions and
these payments are to be used for the purpose of scientific research
(iv) Payment made to national laboratory 8
(v) Machinery purchased for in-house scientific research 25
(vi) Salaried to research staff engaged in in-house scientific research 12
Compute the amount of deduction available under section 35 of the Income Tax Act,
1961 while arriving at the business income of the assessee assuming that he has opted
out of default tax regime.

Solution:

Computation of deduction allowable under section 35

276 | P a g e #HakkसेCA by CA. Akash Sir


Particulars Amount % of Amount of
(₹in lacs) weighted deduction
deduction (in lacs)
Payment for scientific research
K Research Ltd. 20 100% 20
LMN College 15 100% 15
OPQ College 10 Nil Nil
National Laboratory 8 100% 8
In-house research
Capital expenditure 25 100% 25
Revenue expenditure 12 100% 12
Deduction allowable under section 35 80

Sale of assets used for scientific research [Section 41(3)]

If any assessee has acquired any capital asset for scientific research and amount was
debited to profit and loss account and subsequently the asset was sold, amount
received shall be considered to be income under the head business/profession but
only to the extent amount was debited to profit and loss account. If the assessee
has closed down his business/profession at that time, still it is income under the
head business/profession.

For example: ABC Ltd. purchased one plant and machinery for ₹ 20 lakhs on
01.10.2021 for scientific research and entire amount was debited to the Profit and
loss account, subsequently the asset was sold for ₹ 23 lakhs in the year 2023-24, in
this case deemed income under section 41(3), shall be ₹20 lakhs i.e. the amount
recovered on sale maximum to the extent of the amount debited and excess over it
shall be capital gain.

Transfer of asset to the normal business

If any asset was used for scientific research and subsequently it was transferred to
the normal business, in such cases, it will be entered in the respective block of assets
and its w.d.v shall be taken to be nil.

Illustration: 18

Crocin Bio-medicals Ltd. is engaged in the business of manufacture of bio-medical


items. The following expenses were incurred in respect of activities connected with
scientific research:

277 | P a g e #HakkसेCA by CA. Akash Sir


Year ended Item Amount (₹)
31.03.2020 Land 10,00,000
(Incurred after 01.09.2019) Building 25,00,000
31.03.2021 Plant and machinery 5,00,000
31.03.2022 Raw materials 2,20,000

The business was commenced on 01.09.2023 and expenditure incurred on raw materials
and salaries is ₹1,80,000. In view of availability of better model of plant and machinery,
the existing plant and machinery were sold for ₹8,00,000 on 01.03.2024.
Discuss the implications of the above for the Assessment Year 2024-25 along with
brief computation of deduction permissible under section 35 assuming that necessary
conditions have been fulfilled.

Solution:

The eligible expenditure and quantum of deduction will be:


(a) Current year capital or revenue expenditure incurred for scientific research.
(b) Any expenditure incurred during earlier 3 years immediately preceding the date of
commencement of business on payment of salary or purchase of materials, or capital
expenditure incurred other than expenditure on acquisition of land (actual expenditure
qualifies for deduction).

The deduction available under section 35 for scientific research will, therefore, be:
Particulars ₹
(a) Land Nil
(b) Building 25,00,000
(c) Revenue expenses of last 3 years 2,20,000
(d) Capital expenditure of last 3 years: Plant and machinery 5,00,000
(e) Current year revenue expenditure 1,80,000
Deduction under section 35 34,00,000

Carry forward of unadjusted capital expenditure of scientific research

Un-adjusted capital expenditure of scientific research shall be allowed to be carried


forward just like unabsorbed depreciation i.e. carry forward shall be allowed for
unlimited period and brought forward expenditure can be adjusted from any income
under any head except casual income.

Illustration: 19

Kundan Ltd. was incorporated on 01.01.2023 for manufacture of tyres and tubes for
motor vehicles. The manufacturing unit was set up on 01.05.2023. The company

278 | P a g e #HakkसेCA by CA. Akash Sir


commenced its manufacturing operations on 01.06.2023. The total cost of the plant and
machinery installed in the unit is ₹55 crore. The said plant and machinery included
second hand plant and machinery bought for ₹20 crore and new plant and machinery for
scientific research relating to the business of the assessee acquired at a cost of ₹15
crore.
Compute the amount of depreciation allowable under section 32 of the Income-tax Act,
1961 in respect of the assessment year 2024-25.

Solution:

Particulars ₹ in crore
Total cost of plant and machinery 55.00
Less: Used for Scientific Research (15.00)
40.00
Normal Depreciation at 15% on ₹ 40 crore 6.00
Additional Depreciation: 20.00 x 20% 4.00
No additional depreciation allowed on second hand machinery and also machinery used
for scientific research.

Illustration: 20

Mr. Gamma, a proprietor started a business of manufacture of tyres and tubes for
motor vehicles on 01.01.2023. The manufacturing unit was set up on 01.05.2023. He
commenced his manufacturing operations on 01.06.2023. The total cost of the plant and
machinery installed in the unit is ₹120 crores. The said plant and machinery included
second hand plant and machinery bought for ₹20 crores and new plant and machinery
for scientific research relating to the business of the assessee acquired at a cost of
₹15 crores.
Compute the amount of depreciation allowable under section 32 of the Income-tax Act,
1961 in respect of the assessment year 2024-25. Assume that all the assets were
purchased by way of account payee cheque and Mr. Gamma has not opted for the
provisions of section 115BAC.

Solution:

Computation of depreciation allowable for the A.Y. 2024-25 in the hands of Mr.
Gamma

Particular Amount (cr.)


Total cost of plant and machinery 120
Less: Used for Scientific Research (since allowed as deduction 15
u/s. 35)

279 | P a g e #HakkसेCA by CA. Akash Sir


105
Normal Depreciation at 15% on 105 crores 15.75
Additional Depreciation:
Cost of plant and machinery 120
Less: Second hand plant and machinery 20
Less: Plant and machinery used for scientific research,
the whole of the actual cost of which is allowable 15 35
as deduction under section 35(1)(iv) read with
section 35(2)(ia)
85
Additional Depreciation at 20% on 85 crores 17
Depreciation allowable for A.Y. 2024-25 32.75

HW Question: 12

On 01.04.2023, ABC Ltd. owns plants A, B, C and D (rate of depreciation 15%)


depreciated value of the block as on 01.04.2023 is ₹5,40,000. On 14.06.2023 plant E
which was initially purchased for ₹96,000 for conducting scientific research is
transferred from laboratory to the factory. No other asset is purchased or sold. Find
out depreciation allowed for the previous year 2023-24.

HW Question: 13

A Ltd. which is engaged in manufacturing, furnishes the following particulars for the
P.Y. 2023-24. Compute the deduction allowable under section 35 for A.Y. 2024-25,
while computing its income under the head “Profits and gains of business or profession”.

Particular ₹
1 Amount paid to Indian Institute of Science, Bangalore, a 1,00,000
notified research organization for scientific research
2 Amount paid to IIT, Delhi for an approved scientific research 2,50,000
Programme
3 Amount paid to X Ltd, a company registered in India which has 4,00,000
as its main object
4 Expenditure incurred on in-house research and development
facility as approved by the prescribed authority
(a) Revenue expenditure on scientific research 3,00,000
(b) Capital expenditure (including cost of acquisition of land 7,50,000
₹5,00,000) on scientific research

280 | P a g e #HakkसेCA by CA. Akash Sir


HW Question: 14

Mr. Abhimanyu has furnished the following particulars relating to payments made and
expenditure incurred towards scientific research for the year ended 31.3.2024:

S.no. Particulars ₹ (in lakhs)


(i) Payment made to AB University, an approved University 15
(ii) Payment made to Siya College 17
(iii) Payment made to IIT, Madras (under an approved program for
Scientific research) 12
(iv) Machinery purchased for in-house scientific research 25
Compute the deduction available under section 35 of the Income-tax Act, 1961 for A.Y.
2024-25, while computing his income under the head “Profits and gains of business or
profession” assuming that he has not opted for section 115BAC.

TP: 11 Deduction in respect of expenditure on Specified Business


[section 35AD]

Specified business notified by CBDT are as follows:

# Particulars Date of
commencement
1. Cold chain facility for storing agricultural produce, 01.04.2009 onwards
meat and meat products, poultry and dairy products etc.
2. Warehousing facility for storage of agricultural 01.04.2009 onwards
produce.
3. Hospitals with at least one hundred beds for patients. 01.04.2010 onwards
4. Housing project under a scheme for affordable 01.04.2011 onwards
housing.
5. Production of fertilizer including increase in installed 01.04.2011 onwards
capacity of an existing plant.
6. Pipeline network for cross country distribution of 01.04.2007 onwards
natural gas or petroleum products.
7. Pipeline network for the transportation of iron ore. 01.04.2014 onwards
8. Hotel of two star or above category. 01.04.2010 onwards
9. Housing project for slum development. 01.04.2010 onwards
10. Inland container depot or a container freight station. 01.04.2012 onwards
11. Bee-keeping and production of honey. 01.04.2012 onwards
12. Warehousing facility for storage of sugar. 01.04.2012 onwards
13. Semi-conductor wafer fabrication manufacturing unit. 01.04.2014 onwards
14. Developing or maintaining or operating a new 01.04.2017 onwards
infrastructure facility.

281 | P a g e #HakkसेCA by CA. Akash Sir


a) Deduction of capital expenditure:

Incurred after commencement of business: The assessee shall be allowed to debit


the capital expenditure to the profit and loss account and further amount allowed to
be debited shall be equal to the capital expenditure incurred.

Incurred before commencement of business: The capital expenditure incurred


before commencement of business shall also be allowed to be debited in the year in
which the business has commenced.

The following capital expenditure shall not be allowed


• Acquisition of any land; or
• Goodwill; or
• Financial instrument

Capital Expenditure shall not include any expenditure in respect of which the
payment or aggregate of payments made to a person in a day, otherwise than by an
account payee cheque drawn on a bank or an account payee bank draft or use of
electronic clearing system through a bank account or Credit Card, Debit Card, Net
Banking, IMPS (Immediate Payment Service), UPI (Unified Payment Interface),
RTGS (Real Time Gross Settlement), NEFT (National Electronic Funds Transfer), and
BHIM (Bharat Interface for Money) Aadhaar Pay, exceeds ₹10,000.

b) No deduction under new/ special tax regime [Important]:

In case if an individual, HUF, AOP, BOI carrying on above specified business opts for
default tax regime u/s. 115BAC, deduction u/s. 35AD shall not be allowed.

A company would not be eligible for deduction u/s. 35AD if it opts for special
provisions of section 115BAA/ 115BAB.

c) Treatment on subsequent sell/ transfer etc. of such capital asset at anytime


[Section 28(vii)]:

If any capital asset which was debited to profit and loss account u/s. 35AD, has been
transferred/ sold/ discarded etc., amount received on sale shall be considered to be
income under the head business/profession.

282 | P a g e #HakkसेCA by CA. Akash Sir


d) Treatment on subsequent transfer of such capital asset to other normal
business within a period of 8 years [Section 35AD(7B)]:

If any capital asset was acquired for the said business and amount was debited to
profit and loss account, it must be used for the said business for a period of at least
8 years otherwise the amount debited shall be considered to be income of the
assessee of the year in which the asset has been used for other purpose, however
normal depreciation shall be deducted and only balance amount shall be considered to
be income.

e) Treatment on subsequent transfer of such capital asset to other normal


business after a period of 8 years:

No treatment is required.

f) Treatment of loss from specified business [Section 73A] (detailed discussion


under set-off chapter):

As per section 73A, loss of specified business can be set off only from profits and
gains of any other specified business and carried forward is allowed for unlimited
periods and in the subsequent years also, the loss can be set off only from income of
specified business.

g) Other conditions:

i) Business should not be set-up by splitting up, or the reconstruction, of a business


already in existence
ii) Business should not be set-up by transfer of P&M used for any other purpose.
However, 20% of value of P&M can be transferred from other business.

Mann Ki Baat (simple language):

The above provision was introduced to give tax benefits to those business houses who
were engaged in specified businesses to motivate those business houses to invest in
such businesses. This provision has the effect of reducing tax liability in initial years
and thus, promoting investment in specified businesses.

Illustration: 21

An Assessee starts business of setting up and operating a warehousing facility for


agricultural produce on 01.06.2023. Following information is given to you:

283 | P a g e #HakkसेCA by CA. Akash Sir


(i) Profits from operating warehousing facility ₹50,00,000
The following assets have been purchased for warehousing facility and the profit of
₹50,00,000 is computed without giving effect to the following:
(ii) Machinery purchased on 30.06.2022 ₹9,00,000
(iii) Land purchased on 30.06.2022 ₹15,00,000
(iv) Machinery purchased on 31.12.2023 ₹12,00,000
(v) Building purchased on 31.12.2022 ₹10,00,000
(vi) Building Constructed on 31.05.2023 ₹29,00,000
Compute income or loss for the assessment year 2024-25 assuming that the Assessee
has opted out of provisions of section 115BAC.

Solution:

Profit of specified business 50,00,000


Less: Deduction allowable under section 35AD
Machinery purchased on 30.06.2022 (9,00,000)
Machinery purchased on 31.12.2023 (12,00,000)
Building purchased on 31.12.2022 (10,00,000)
Building constructed on 31.05.2023 (29,00,000)
Loss (10,00,000)
The loss of ₹10,00,000 can be set-off only against the profits of specified business in
current year and next years. Such loss can be carried forward indefinitely.

HW Question: 15

Mr. Manav commenced operations of the businesses of setting up a warehousing facility


for storage of food grains, sugar and edible oil on 01.04.2023. He incurred capital
expenditure of ₹80 lakh, ₹60 lakh and ₹50 lakh, respectively, on purchase of land and
building during the period January, 2023 to March, 2023 exclusively for the above
businesses, and capitalized the same in its books of account as on 1st April, 2023. The
cost of land included in the above figures are ₹50 lakh, ₹40 lakh and ₹30 lakh,
respectively. Further, during the P.Y. 2023-24, it incurred capital expenditure of ₹20
lakh, ₹15 lakh & ₹10 lakh, respectively, for extension/ reconstruction of the building
purchased and used exclusively for the above businesses.
Compute the income under the head “Profits and gains of business or profession” for
the A.Y. 2024-25 and the loss to be carried forward, assuming that Mr. Manav has
fulfilled all the conditions specified for claim of deduction under section 35AD. The
profits from the business of setting up a warehousing facility (before claiming
deduction under section 35AD and section 32) for the A.Y. 2024-25 is ₹16 lakhs, ₹14
lakhs and ₹31 lakhs, respectively. Assume that the Assessee has not opted for section
115BAC

284 | P a g e #HakkसेCA by CA. Akash Sir


HW Question: 16

XYZ Ltd. commenced operations of the business of a new three-star hotel in Madurai,
Tamil Nadu on 01.04.2023. The company incurred capital expenditure of ₹50 lakh
during the period January, 2023 to March, 2023 exclusively for the above business,
and capitalized the same in its books of account as on 1st April, 2023. Further, during
the P.Y. 2023-24, it incurred capital expenditure of ₹2 crore (out of which ₹1.50 crore
was for acquisition of land) exclusively for the above business.
Compute the income under the head “Profits and gains of business or profession” for
the A.Y. 2024-25, assuming that XYZ Ltd. has fulfilled all the conditions specified for
claim of deduction under section 35AD. The profits from the business of running this
hotel (before claiming deduction under section 35AD) for the A.Y. 2024-25 is ₹25
lakhs.
Assume that the company also has another existing business (specified business) of
running a four-star hotel in Coimbatore, which commenced operations 2 years back, the
profits from which are ₹120 lakhs for the A.Y. 2024-25.

TP: 12 Amortisation of certain Preliminary Expenses [Section


35D]

Preliminary expenses mean expenses incurred prior to commencement of business.

➢ Allowed to be debited in 5 annual equal installments after commencement of


business.
➢ Allowed to an Indian company and also to resident assessee i.e. it is not allowed
to non-residents and to foreign company.
➢ Only the notified expenditure incurred before commencement of business shall
be allowed and such expenses may be:

1. Expenditure in connection with—


(i) preparation of feasibility report.
(ii) preparation of project report.
(iii) conducting market survey or any other survey necessary for the business
of the assessee.
(iv) engineering services relating to the business of the assessee.
Provided that the work in connection with the above is carried out by the
assessee himself or by a concern which is approved by the Central Board of
Direct Taxes.

2. Legal charges for drafting any agreement between the assessee and any other
person for purpose of the business of the assessee.

285 | P a g e #HakkसेCA by CA. Akash Sir


3. Where the assessee is a company, also expenditure—
(i) by way of legal charges for drafting the Memorandum and Articles of
Association of the company.
(ii) on printing of the Memorandum and Articles of Association.
(iii) by way of fees for registering the company under the provisions of the
Companies Act.
(iv) in connection with the issue of shares or debentures of the company,
being underwriting commission, brokerage and charges for drafting, typing,
printing and advertisement of the prospectus.

➢ Maximum expenditure allowed shall be upto 5% of the project cost but an


Indian company has the option to take 5% of the capital employed.

“Cost of the project” means in a case of new business, the actual cost of the fixed
assets, being land, buildings, plant, machinery, furniture, fittings etc. as on the last
day of the year in which the assessee has commenced the business.

“Capital employed” means in a case of new business, the aggregate of the issued
share capital, debentures and long-term borrowings as on the last day of the previous
year in which the business of the company commences.

In case of an existing business if there is extension of such business, expenses


incurred in connection with such extension shall also be allowed in the similar manner
as in case of new business and project cost and capital employed shall be taken into
consideration relating to extension of business.

➢ Audit of accounts: Accounts shall be audited by a CA in practice u/s.


44AB on or before 30th Sept of relevant A.Y.

Mann Ki Baat (simple language):

Above provision is inserted to benefit the assessee who has incurred specified
expenditures before commencement of business and now wants to claim such expenses
during the year of commencement. Such expenditure can be claimed in 5 equal annual
installments.

Illustration: 22

Lake Ltd. an Indian company has incurred expenditure before the commencement of
business as under:
1. Expenditure on advertisements ₹3,00,000
2. Expenditure on preparation of project report and the report was

286 | P a g e #HakkसेCA by CA. Akash Sir


prepared by a concern which is approved by the Board ₹85,000
3. Expenditure in connection with travelling and stay in hotels ₹45,000
4. Expenditure on drafting and printing of memorandum and articles
of associations ₹4,00,000
All the above expenditures have been debited to the profit and loss account and the
company has computed income to be ₹7 lakhs.
The company has commenced its business on 01.06.2023.
Company’s project cost is ₹50 lakhs and capital employed is Rs. 57 lakhs.
Compute company’s Tax Liability for Assessment Year 2024-25 under old provisions.

Solution:

Net profit as per profit and loss account 7,00,000


Add:
• Expenditure on advertisement 3,00,000
• Expenditure in connection with travelling and stay in hotels 45,000
• Excessive expenditure under section 35D [WN: 1] 4,28,000
Income under the head Business/Profession 14,73,000
Gross Total Income 14,73,000
Less: Deduction u/s 80C to 80U Nil
Total Income 14,73,000

Computation of Tax Liability


Tax on ₹14,73,000 @ 30% 4,41,900
Add: HEC @ 4% 17,636
Tax Liability 4,59,576
Rounded off u/s 288B 4,59,580

WN: 1
Eligible expenditure under section 35D
1. Expenditure on preparation of project report 85,000
2. Expenditure on drafting and printing of memorandum and articles
of associations 4,00,000
Total = 4,85,000
Expenditure allowed under section 35D cannot exceed 5% of the
capital employed 57,00,000 x 5% = 2,85,000
Instalment allowed 2,85,000/5 = 57,000
Expenditure disallowed = 4,85,000 – 57,000 = 4,28,000

287 | P a g e #HakkसेCA by CA. Akash Sir


TP: 13 Amortisation of expenditure incurred under voluntary
retirement scheme [Section 35DDA]

(1) Nature of expenditure: This section applies to an assessee who has incurred
expenditure in any previous year in the form of payment to any employee in
connection with his voluntary retirement, in accordance with any scheme or schemes
of voluntary retirement.

(2) Amount of deduction: The amount of deduction allowable is one-fifth of the


amount paid for that previous year, and the balance in four equal installments in the
four immediately succeeding previous years.

(3) No deduction under any provisions of the Act: No deduction shall be allowed in
respect of the above expenditure under any other provision of the Act.

Mann Ki Baat (simple language):

If any employer has given voluntary retirement to the employees and has paid any
amount in connection with such voluntary retirement, it will be allowed to be debited in
5 annual equal installments.

Illustration: 23

Aman Co. Ltd. paid ₹120 lakhs as compensation as per approved Voluntary Retirement
Scheme (VRS) during the financial year 2023-24.
How much is deductible under section 35DDA for the assessment year 2024-25?

Solution:

It is deductible in 5 equal annual instalments commencing from the previous year of


payment. Therefore, ₹24 lakhs, being 1/5th of ₹120 lakhs, is deductible under section
35DDA for the A.Y. 2024-25.

TP: 14 Other Deductions [Section 36]

This section authorizes deduction of certain specific expenses. The items of


expenditure and the conditions under which such expenditures are deductible are:

288 | P a g e #HakkसेCA by CA. Akash Sir


(1) Deductibility of insurance premium taken on stock [Section 36(1)(i)]

If insurance policy has been taken out against risk, damage or destruction of the
stock or stores of the business or profession, the premium paid is deductible.
But the premium in respect of any insurance undertaken for any other purpose is not
allowable under the clause.

(2) Premium paid by employer for health insurance of employees [Section


36(1)(ib)]

This clause seeks to allow a deduction to an employer in respect of premium paid by


him by any mode of payment other than cash to effect or to keep in force an
insurance on the health of his employees in accordance with a scheme framed by:
(i) the General Insurance Corporation of India and approved by the Central
Government; or
(ii) any other insurer and approved by the IRDA

(3) Bonus and Commission [Section 36(1)(ii)]

These are deductible in full provided the sum paid to the employees as bonus or
commission shall not be payable to them as profits or dividends if it had not been
paid as bonus or commission.
It is a provision intended to safeguard against a private company or an association
escaping tax by distributing a part of its profits by way of bonus amongst the
members, or employees of their own concern instead of distributing the money as
dividends or profits.

(4) Interest on borrowed capital [Section 36(1)(iii)]

Deduction of interest is allowed in respect of capital borrowed for the purposes of


business or profession in the computation of income under the head "Profits and
gains of business or profession".

Capital may be borrowed for several purposes like for acquiring a capital asset, or to
pay off a trading debt or loss etc. The scope of the expression ‘for the purposes of
business’ is very wide. Capital may be borrowed in the course of the existing business
as well as for acquiring assets for extension of existing business.

As per proviso to section 36(1)(iii), deduction in respect of any amount of interest


paid, in respect of capital borrowed for acquisition of new asset (whether capitalised
in the books of account or not) for any period beginning from the date on which the

289 | P a g e #HakkसेCA by CA. Akash Sir


capital was borrowed for acquisition of the asset till the date on which such asset
was first put to use shall not be allowed.

Mann Ki Baat (simple language):

If any assessee has taken a loan for the purpose of business/ profession, interest on
such loan is allowed. No interest is allowed to the proprietor on his capital (similarly
no salary or any other payment is allowed to the proprietor).

(5) Discount on Zero Coupon Bonds (ZCBs) [Section 36(1)(iiia)]

Section 36(1)(iiia) provides deduction for the discount on ZCB on pro rata basis
having regard to the period of life of the bond to be calculated in the manner
prescribed

Term Meaning
Discount Difference of the amount received or receivable by an
infrastructure capital company/ infrastructure capital fund/ public
sector company/ scheduled bank on issue of the bond and the
amount payable by such company or fund or bank on maturity or
redemption of the bond
Period of life The period commencing from the date of issue of the bond and
of the bond ending on the date of the maturity or redemption.

(6) Contributions to provident and other funds [Section 36(1)(iv) and (v)]

Contribution to the employees’ recognized provident fund/ approved superannuation


fund/ approved gratuity fund are allowable subject to the following conditions:
(a) The gratuity fund should be settled upon a trust.
(b) The amount contributed should be periodic payment and not an adhoc payment to
start the fund.
(c) The gratuity fund should be for exclusive benefit of the employees

(7) Employer’s contribution to the account of the employee under a Pension


Scheme referred to in section 80CCD [Section 36(1)(iva)]

(i) Section 36(1)(iva) to provide that the employer’s contribution to the account of an
employee under a Pension Scheme as referred to in section 80CCD would be allowed
as deduction while computing business income.
(ii) However, deduction would be restricted to 10% of salary of the employee in the
previous year.

290 | P a g e #HakkसेCA by CA. Akash Sir


(iii) Salary, for this purpose, includes dearness allowance, if the terms of employment
so provide, but excludes all other allowances and perquisites.

Illustration: 24

Manit Ltd. contributes 20% of basic salary to the account of each employee under a
pension scheme referred to in section 80CCD. Dearness Allowance is 40% of basic
salary and it forms part of pay of the employees.
Compute the amount of deduction allowable under section 36(1)(iva), if the basic salary
of the employees aggregate to ₹10 lakhs. Would disallowance under section 40A(9) be
attracted, and if so, to what extent?

Solution:

Computation of deduction u/s 36(1)(iva) and disallowance u/s 40A(9)

Particulars
Basic Salary 10,00,000
Dearness Allowance@40% of basic salary [DA forms part of pay] 4,00,000
Salary for the purpose of section 36(1)(iva) (Basic Salary + DA) 14,00,000
Actual contribution (20% of basic salary i.e., 20% of ₹10 lakh) 2,00,000
Less: Permissible deduction under section 36(1)(iva) (10% of basic
salary plus dearness pay = 10% of ₹14,00,000 = ₹1,40,000) 1,40,000
Excess contribution disallowed under section 40A(9) 60,000

(8) Deductibility of employee’s contribution received by the employer [Section


36(1)(va)].

As per section 36(1)(va), employees contribution shall be allowed to be debited only


if employer has deposited the amount in the relevant account upto the time allowed
in the relevant Act.

As per paragraph 38 of The Employees’ Provident Funds Scheme, 1952, the employer
should pay within 15 days of the subsequent month.

As per section 31 of Employees' State Insurance (General) Regulations, 1950, ESI


contribution should be deposited maximum upto 15th of subsequent month.

291 | P a g e #HakkसेCA by CA. Akash Sir


TP: 15 Deduction for Bad Debts of a Business [Section
36(1)(vii)]

If any assessee has written off bad debts as irrecoverable in the books of accounts,
he will be allowed to debit such bad debts to the profit and loss account. However,
the provision for bad debts is not allowed (in general provision or reserve for any
purpose is not allowed.)

Recovery of bad debts Section 41(4)

If any amount was debited as bad debts and subsequently it was recovered by the
assessee, it will be considered to the income of the assessee under the head business
/profession of the year in which it has been recovered and if the assessee does not
have business or profession in that particular year, even than it will be considered to
be income under the head business/profession. If debt was incurred by a person but
it was recovered by his successor, in that case it will not be considered to be income
of successor.

For example: Mr. Ram debited bad debts ₹4,00,000 in previous year 2018-19 but
recovered ₹1,00,000 in previous year 2023-24, in this case as per section 41(4) it will
be considered to be income under the head business profession of previous year
2023-24. If amount was debited by his father and after his death, his son has
inherited the business and has recovered ₹1,00,000, it will not be considered to be
income of son i.e. successor.

For example: Mr. Ram debited bad debts ₹4,00,000 in previous year 2018-19,
however, AO allowed only Rs. 3,00,000 as bad debt. Now, he has recovered ₹1,50,000
in previous year 2023-24, in this case as per section 41(4), Rs. 50,000 (being
difference between bad debt recovered and bad debt not allowed by AO) will be
considered to be income under the head business profession of previous year 2023-
24. If amount was debited by his father and after his death, his son has inherited
the business and has recovered ₹1,50,000, it will not be considered to be income of
son i.e. successor.

TP: 16 Expenses on family planning by a company [Section


36(1)(ix)]

Any expenditure of revenue nature bona fide incurred by a company for the purpose
of promoting family planning amongst its employees will be allowed as a deduction in
computing the company’s business income

292 | P a g e #HakkसेCA by CA. Akash Sir


• Where the expenditure is of a capital nature, one-fifth of such expenditure will
be deducted in the previous year in which it was incurred and in each of the four
immediately succeeding previous years.
• This deduction is allowable only to companies and not to other assessees.
• The assessee would be entitled to carry forward and set off the unabsorbed
part of the allowance in the same way as unabsorbed depreciation.

The capital expenditure on promoting family planning will be treated in the same way
as capital expenditure for scientific research for purposes of dealing with the profit
or loss on the sale or transfer of the asset including a transfer on amalgamation.

TP: 17 Deduction of securities/ commodities transaction tax paid


[Section 36(1)(xv)/ 36(1)(xvi)]

The amount of securities transaction tax paid by the assessee during the year in
respect of taxable securities transactions entered into in the course of business
shall be allowed as deduction under section 36 subject to the condition that such
income from taxable securities transactions is included under the head ‘Profits and
gains of business or profession’.

Thus, securities transaction tax paid would be allowed as a deduction like any other
business expenditure. If it is a case of capital gain, it will not be allowed to be
deducted

For example: Mr. Shyam purchased shares of ₹4 lakh and sold for ₹10 lakh after 6
months and paid STT ₹ 1000 in this case capital gains u/s 111A shall be ₹6 lakh and
shall be taxable @ 15% and if shares were sold after 1 year it will be long term
capital gain u/s. 112A and shall be taxable @10% and amount of capital gains shall be
₹6 lakh. If he has business of sale purchase of shares, STT shall be deducted and
capital gain shall be ₹5,99,000 and shall be taxable at the normal rate.

Deduction for commodities transaction tax paid in respect of taxable


commodities transactions [Section 36(1)(xvi)]

If the assessee has paid commodities transaction tax in connection with taxable
commodities transaction which are part of his business, CTT shall be allowed to be
debited to the profit and loss account. Such business is considered to be speculative
business.

293 | P a g e #HakkसेCA by CA. Akash Sir


TP: 18 Residuary Expenses [Section 37]

As per section 37(1), if any expenditure is neither allowed nor disallowed


specifically under any particular section, such expenditure is allowed to be debited if
it is related to business or profession and is revenue in nature.

▪ If it is capital expenditure, depreciation is allowed.


▪ Personal expenditure is never allowed.
▪ Illegal expense is not allowed.
▪ Any fine or penalty for an offence is not allowed.
▪ It should have been incurred by the assessee in the accounting year.
▪ It must have been incurred after the business was set up
▪ The expenditure should not have been incurred by the assessee for any purpose
which is an offence or is prohibited by law.

This section is thus limited in scope. It does not permit an assessee to make all
deductions which a prudent trader would make in ascertaining his own profit. It
might be observed that the section requires that the expenditure should be wholly
and exclusively laid out for purpose of the business but not that it should have been
necessarily laid out for such purpose. Therefore, expenses wholly and exclusively laid
out for the purpose of trade are, subject to the fulfilment of other conditions,
allowed under this section even though the outlay is unnecessary.

Note: If any person has paid any fine or penalty in connection with income tax, GST
etc., it will not be allowed.

Any expenditure incurred by an assessee on the activities relating to corporate


social responsibility referred to in section 135 of the Companies Act, 2013 shall
not be deemed to be an expenditure incurred by the assessee for the purposes
of the business or profession.

Advertisements in souvenirs of political parties:

Section 37(2B) disallows any deduction on account of advertisement expenses


representing contributions made by any person carrying on business or profession in
computing the profits and gains of the business or profession. It has specifically
been provided that this provision for disallowance would apply notwithstanding
anything to the contrary contained in section 37(1).

In other words, the expenditure representing contribution for political purposes


would become disallowable even in those cases where the expenditure is otherwise

294 | P a g e #HakkसेCA by CA. Akash Sir


incurred by the assessee in his character as a trader and the amount is wholly and
exclusively incurred for the purpose of the business.

Accordingly, a taxpayer would not be entitled to any deduction in respect of


expenses incurred by him on advertisement in any souvenir, brochure, tract or the
like published by any political party, whether it is registered with the Election
Commission of India or not.

Mann Ki Baat (simple language):

No allowance shall be made in respect of expenditure incurred by an assessee on


advertisement in any souvenir, brochure, tract, pamphlet or the like published by a
political party.

Illustration: 25

ABC Limited is a company engaged in the business of biotechnology. The net profit of
the company for the financial year ended 31.03.2024 is ₹15,25,890 after debiting the
following items:
1. Purchase price of raw material used for the purpose of in-house
research and development 1,80,000
2. Purchase price of asset used for in-house research and
development
(1) Land 5,00,000
(2) Building 3,00,000
3. Expenditure incurred on advertisement in the souvenir published
by a political party 75,000
Compute the income under the head “Profits and gains of business or profession” for
the A.Y. 2024-25 of ABC Ltd.

Solution:

Computation of income under the head “Profits and gains of business or profession”
for the A.Y. 2024-25

Net profit as per profit and loss account 15,25,890


Add: Purchase price of Land used in in-house research and development 5,00,000
Add: Expenditure incurred on advertisement in the souvenir published by
a political party not allowed as deduction as per section 37(2B) 75,000
Profits and gains of business/profession 21,00,890

295 | P a g e #HakkसेCA by CA. Akash Sir


Note: The expenditure incurred on advertisement in the souvenir published by a
political party is disallowed as per section 37(2B) while computing income under the
head “Profit and Gains of Business or Profession”.

TP: 19 Expenditure in connection with assets which are partly in


business use and partly in personal use [Section 38]

If any person has any asset in business or profession as well as in personal use,
expenditure is allowed only to the extent the asset is in the use of the business or
profession.

For example: If Mr. X has one motor car which is used to the extent of 60% in
business and 40% for personal use, all expenditures shall be allowed to be debited to
the extent of 60%.

Depreciation in such case:

Proportionate depreciation shall be allowed which relates to business use. Such


depreciation shall be reduced to calculate closing written down value.

For example: If Mr. X has one motor car (depreciated @15%) of ₹10,00,000 which
is used to the extent of 60% in business and 40% for personal use, in this case
depreciation shall be ₹90,000 (i.e. 1,50,000 * 60%). Closing WDV shall be ₹9,10,000.

TP: 20 Inadmissible deductions [Section 40]

Section 40(a)(i): TDS not made on payment of interest, royalty etc outside
India:

Any interest, royalty, fees for technical services or any other sum payable “outside
India” or “in India to a NR or foreign co.” not allowed to be deducted as expense if:
i) Tax has not been deducted at source during the previous year; or
ii) Tax has been deducted at source but has not been deposited with the govt. on
before due date of return filing.
However, deduction shall be allowed in the year in which tax has been deducted and
deposited with the govt.

Q1. What if TDS has been made during the P.Y. 2023-24, however deposited with the
govt during P.Y. 2025-26?
Ans: Expense shall be disallowed during P.Y. 2023-24 and shall be allowed during P.Y.
2025-26.

296 | P a g e #HakkसेCA by CA. Akash Sir


Q2. What if TDS has not been made during P.Y. 2023-24, however, TDS has been made
during P.Y. 2024-25 and deposited during P.Y. 2025-26?
Ans: Expense shall be disallowed during P.Y. 2023-24 and shall be allowed during P.Y.
2025-26.

Q3. What if TDS has been made during P.Y. 2023-24 and deposited during P.Y. 2024-
25 before due date of return filing?
Ans: Expense shall be allowed during P.Y. 2023-24.

Q4. What is payment is made during P.Y. 2023-24, however, TDS has been made during
P.Y. 2024-25 and deposited during P.Y. 2024-25 before due date of return filing?
Ans: Expense shall be disallowed during P.Y. 2023-24 and shall be allowed during P.Y.
2024-25.

Q5. What if payer failed to deduct tax at source during P.Y. 2023-24, however, payee
has submitted his return and has taken into account such income and paid tax on the
same during P.Y. 2024-25?
Ans: In this case, it will be assumed that TDS has been made during the year in which
return has been filed i.e. P.Y. 2024-25 and hence deduction shall be allowed during P.Y.
2024-25.

For example: Mr. X, resident individual, has made payment of ₹10,00,000 towards
interest expenses to Mr. Y, non-resident without tax deduction at source during P.Y.
2023-24. In this case such expenses shall not be allowed to be deducted during P.Y.
2023-24. However, Mr. Y himself filed his return of income and has shown such income
of ₹10,00,000 in his return and has paid tax on 30.07.2024. In this case, it shall be
assumed that Mr. X has made TDS and deposited with Govt. on 30.07.2024 and
therefore, expense shall be allowed during P.Y. 2024-25.

Section 40(a)(ia): TDS not made on payment of interest, royalty etc to a


resident:
30% of any sum payable to resident shall not allowed to be deducted as expense if:
i) Tax has not been deducted at source during the previous year; or
ii) Tax has been deducted at source but has not been deposited with the govt. on
before due date of return filing.
However, deduction shall be allowed in the year in which tax has been deducted and
deposited with the govt.

Q1. What if TDS has been made during the P.Y. 2023-24, however deposited with the
govt during P.Y. 2025-26?

297 | P a g e #HakkसेCA by CA. Akash Sir


Ans: 30% of expense shall be disallowed during P.Y. 2023-24 and shall be allowed during
P.Y. 2025-26.

Q2. What if TDS has not been made during P.Y. 2023-24, however, TDS has been made
during P.Y. 2024-25 and deposited during P.Y. 2025-26?
Ans: 30% of expense shall be disallowed during P.Y. 2023-24 and shall be allowed during
P.Y. 2025-26.

Q3. What if TDS has been made during P.Y. 2023-24 and deposited during P.Y. 2024-
25 before due date of return filing?
Ans: Whole expense shall be allowed during P.Y. 2023-24.

Q4. What is payment is made during P.Y. 2023-24, however, TDS has been made during
P.Y. 2024-25 and deposited during P.Y. 2024-25 before due date of return filing?
Ans: 30% of expense shall be disallowed during P.Y. 2023-24 and shall be allowed during
P.Y. 2024-25.

Q5. What if payer failed to deduct tax at source during P.Y. 2023-24, however, payee
has submitted his return and has taken into account such income and paid tax on the
same during P.Y. 2024-25?
Ans: In this case, it will be assumed that TDS has been made during the year in which
return has been filed i.e. P.Y. 2024-25 and hence deduction of 30% shall be allowed
during P.Y. 2024-25.

For example: Mr. X, resident individual, has made payment of ₹10,00,000 towards
interest expenses to Mr. Y, non-resident without tax deduction at source during P.Y.
2023-24. In this case ₹3,00,000 shall not be allowed to be deducted during P.Y. 2023-
24. However, Mr. Y himself filed his return of income and has shown such income of
₹10,00,000 in his return and has paid tax on 30.07.2024. In this case, it shall be
assumed that Mr. X has made TDS and deposited with Govt. on 30.07.2024 and
therefore, ₹3,00,000 shall be allowed during P.Y. 2024-25.

Illustration: 26 [PGBP, TDS]

During the financial year 2023-24, the following payments/ expenditure were made/
incurred by Mr. Vishal, a resident individual (whose turnover during the year ended
31.03.2022 was ₹99 lacs):
(i) Interest of ₹12,000 was paid to ABC & Co., a resident partnership firm, without
deduction of tax at source;
(ii) Interest of ₹4,000 was paid as interest to Mr. John, a non-resident, without
deduction of tax at source;

298 | P a g e #HakkसेCA by CA. Akash Sir


(iii) ₹3,00,000 was paid as salary to a resident individual without deduction of tax at
source;
(iv) He had sold goods worth ₹5 lacs to Mr. Anand. He paid Commission of ₹15,000 to
Mr. Sohil on 02.07.2022. In none of these transactions, tax was deducted at source.
Briefly discuss whether any disallowance arises under the provisions of section 40(a) of
the Income tax Act, 1961.

Solution:

(i) Since, turnover of Mr. Vishal is less than 100 lakhs in the preceding year hence
expenditure is allowed even if tax has not been deducted at source.
(ii) In the case of interest paid to a non-resident, there is obligation to deduct tax at
source under section 195, hence non-deduction of tax at source will attract
disallowance.
(iii) Disallowance under section 40(a) is attracted for failure to deduct tax at source
under section 192 from salaries. Therefore, 70% of salary i.e. ₹3,00,000 x 70% =
₹2,10,000 shall be allowed in previous year 2023-24 and balance i.e. ₹90,000 is
disallowed.
(iv) Since, turnover of Mr. Vishal is less than 100 lakhs in the preceding year hence
expenditure is allowed even if tax has not been deducted at source.

HW Question: 17 [PGBP, TDS]

M/s ABC Ltd., submits the following details of expenditures pertaining to the financial
year 2023-24:
(i) Payment of professional fees to Mr. M of ₹50,000. Tax not deducted at source.
(ii) Interior works done by Mr. H for ₹2,00,000 on a contract basis. Payment made in
the month of March 2024. Tax deducted in March 2024, was paid on 30.06.2024.
(iii) Factory Rent paid to Mrs. R of ₹15,00,000. Tax deducted at source and paid on
01.11.2024.
(iv) Interest paid on Fixed Deposits of ₹2,00,000. Tax deducted on 31.12.2023 and paid
on 28.09.2024.
(v) Payment made to M/s G & Co. towards import of Raw Materials of ₹25,00,000. No
tax was deducted at source. The supplier G & Co. is located in London.
Examine the above with reference to allowability of the same in the Assessment Year
2024-25 under the Income Tax Act, 1961. Your answer must be with reference to
Section 40(a) read with relevant tax deduction at source provisions.

Section 40(a)(ii): Tax and cess levied on profits not allowed as deduction

Payment of taxes on income shall not be allowed as deduction.

299 | P a g e #HakkसेCA by CA. Akash Sir


For the removal of doubts, it is hereby clarified that for the purposes of this sub-
clause, the term “tax” shall include and shall be deemed to have always included any
surcharge or cess, by whatever name called, on such tax.

Section 40(a)(iii): TDS not made on payment of salary to a non-resident/ outside


India

Any sum which is chargeable under head salaries if it is payable outside India or to a
NR on which tax has been deducted or paid to the Govt. then such sum is disallowed.

Section 40(a)(v): Tax paid on perquisites on behalf of employees

In case an employer provides perquisites to employee and also pays tax on such
perquisites, the such tax shall not be allowed as deduction to employer. Also, such
tax shall be exempt in hands of employee under section 10(10CC).

For example: Mr. Dinesh pays tax of ₹5,00,00,000 on perquisites provided to Miss
Yogita, then in that case, Mr. Dinesh is not allowed to claim deduction of ₹5,00,00,000
and also Miss Yogita need not to consider such tax as her income.

Section 40(b): Interest on capital and remuneration to partners

Interest on capital to partner:

➢ Interest to partners is allowed but maximum 12% p.a. simple interest and excess
shall be disallowed.
➢ Interest shall be allowed only if authorized by partnership deed. If deed has
been amended from back date to provide for interest, then such interest shall
not be allowed.

300 | P a g e #HakkसेCA by CA. Akash Sir


Remuneration to partners:

➢ Remuneration to working partners is allowed. It means that any remuneration to


non-working partners shall not be allowed.
➢ Remuneration shall be authorized by partnership deed. If deed has been
amended from back date to provide for remuneration, such remuneration shall
not be allowed.
➢ Remuneration shall be allowed only upto the prescribed limit which is as follows:

Book profit Deduction allowed


First 3,00,000 or loss 1,50,000 or 90% of book profit
whichever is higher
Above 3,00,000 60% of book profit

➢ Meaning of book profit: Book profit means profit calculated as per profit and
loss account as reduced/ increased by any adjustments made under section 30 to
43D but before charging remuneration.

Note: Share received by a partner from income of Partnership Firm [Section


10(2A)] - If any partner has received share out of the profits of the partnership
firm, such share shall be exempt from income tax.

Mann Ki Baat (simple language):

We understand that the income of partnership firm is taxable at flat rate of 30%
whereas the income of partners is taxable at slab rate. Therefore, it is beneficial for
the partnership firm to distribute the entire income to partners and make firm’s
income ZERO. In order to curb this tax evasion, income tax has inserted above
provision wherein the act has restricted the payment of interest and remuneration to
partners.

For example: If a firm has paid ₹7,50,000 as remuneration to its partners for the P.Y.
2023-24, in accordance with its partnership deed, and it has a book profit of ₹10 lakh,
then, the allowable remuneration calculated as per the limits specified in section 40(b)
would be –

Particulars ₹
On first ₹ 3 lakh of book profit [₹ 3,00,000 × 90%] 2,70,000
On balance ₹ 7 lakh of book profit [₹ 7,00,000 × 60%] 4,20,000
The excess amount of ₹60,000 (i.e., ₹7,50,000 – ₹6,90,000) would be disallowed as per
section 40(b).

301 | P a g e #HakkसेCA by CA. Akash Sir


Illustration: 27

X & Y, a partnership firm consisting of two partners, reports a net profit of ₹7,00,000
before deduction of the following items:
(1) Salary of ₹20,000 each per month payable to two working partners of the firm (as
authorized by the deed of partnership).
(2) Depreciation on plant and machinery under section 32 (computed) of ₹1,50,000.
(3) Interest on capital at 15% per annum (as per the deed of partnership). The amount
of capital eligible for interest of ₹5,00,000
Compute:
(i) Book-profit of the firm under section 40(b) of the Income-Tax Act 1961.
(ii) Allowable working partner salary for the Assessment Year 2024-25 as per section
40(b) of the Income tax Act, 1961.

Solution:

(i) Computation of Book profit under section 40(b) of Income Tax Act 1961

Net Profit 7,00,000.00


Less: Depreciation u/s 32 (1,50,000.00)
Less: Interest on capital (5,00,000 x 12%) (60,000.00)
Book Profit as per section 40(b) 4,90,000.00

(ii) Calculation of allowable salary to partners

Book Profit 4,90,000.00


Allowable Salary
On first 3,00,000 of book profit
90% of book profits or 1,50,000 whichever is higher 2,70,000.00
On balance book profit
60% on balance book profit (1,90,000 x 60/100) 1,14,000.00
Hence, salary as per section 40(b) would be 3,84,000.00

HW Question: 18

XYZ are the partners in a firm with profit sharing ratio 5:3:2 and profit and loss
account of the partnership firm is as given below:

Particulars Amount (₹) Particulars Amount (₹)


Purchases 90,00,000 Sales 1,02,00,000
Salary and bonus to partners Discount 10,000
X 3,00,000

302 | P a g e #HakkसेCA by CA. Akash Sir


Y 2,50,000
Z 1,50,000
Municipal tax payable 30,000 Interest from Indian 60,000
company
General expenses 1,00,000 Interest on drawings 10,000
Expenditure on technical 40,000 Income tax refund 5,000
know-how (purchased and put
to use on 01.01.2024)
Advance Income Tax 70,000
Expenses on GST 10,000
proceedings
Expenses on income tax 8,000
proceedings
Advertisements 50,000
Interest on capital to
partners @ 13% p.a.
X 65,000
Y 39,000
Z 26,000
Rent of building owned by 1,20,000
partnership
Firm
Net Profit 27,000
102,85,000 102,85,000

Additional information:
1. Capital contributed by Mr. X is ₹5,00,000 and by Mr. Y ₹3,00,000 and by Mr. Z
₹2,00,000.
2. Salary paid to Mr. X is ₹3,00,000 and to Mr. Y is ₹2,50,000 and to Mr. Z is
₹1,50,000.
3. The partnership firm has brought forward business loss for assessment year 2021-
22 amounting to ₹1,00,000.
4. Municipal tax was paid on 01.11.2024

Personal incomes of partners:


(i) Mr. X has income from house property of ₹5,00,000 and amount invested in National
Saving Certificate of ₹80,000.
(ii) Mr. Y has income from house property of ₹2,00,000 and amount invested in National
Saving Certificate of ₹1,00,000.
(iii) Mr. Z has loss from house property of ₹2,00,000.
Compute Tax Liability of the partnership firm and also that of its partners for the
Assessment Year 2024-25 under old tax provisions.

303 | P a g e #HakkसेCA by CA. Akash Sir


TP: 21 Excessive payments to Relative/ Related Person [Section
40A(2)]

If the assessee incurs any expenditure and payment has been given to any person
who is his relative or a related person and such expenditure is excessive or
unreasonable having regard to the fair market value of the goods, services or
facilities, then, the excess expenditure shall not be allowed as a deduction.

For example: Mr. X has purchased raw material for his business from his brother
and has paid ₹5,00,000 but market value is ₹3,00,000, in this case expenditure
disallowed shall be ₹2,00,000.

The persons covered in this category are –


1. If any individual has made any payment to his relative. As per section 2(41)
Relative, in relation to an individual, means the husband, wife, brother or sister or
any lineal ascendant or descendant of that individual.

2. If the assessee is a company, firm, association of persons or Hindu Undivided


Family etc. and it has made payment to any director of the company, partner of the
firm, or member of the association or family, or any relative of such director,
partner or member etc.

3. If any person made payment to any other person who has substantial interest (i.e.
20% or more) in the business of the assessee
For example: ABC Ltd. has paid ₹5,00,000 to XYZ Ltd. and XYZ Ltd. is holding 20%
shares of ABC Ltd., in this case excessive payment is disallowed.

Mann Ki Baat (simple language):

It is very simple concept. If anyone has made expense payment to the relative and such
payment is in excess than the market value, then such excess expenditure shall be
disallowed.

TP: 22 Payment in excess of ₹10,000 in cash [Section 40A(3)


Rule 6DD]

If an assessee has incurred any expenditure and the payment or the aggregate of
the payments made to a person with regard to such expenditure on any single day
exceeds ₹10,000 and payment was made otherwise than through Account payee
cheque or

304 | P a g e #HakkसेCA by CA. Akash Sir


account payee bank draft or use of electronic clearing system through a bank
account or Credit Card, Debit Card, Net Banking, IMPS (Immediate Payment
Service), UPI (Unified Payment Interface), RTGS (Real Time Gross Settlement),
NEFT (National Electronic Funds Transfer), and BHIM (Bharat Interface for
Money) Aadhaar Pay, in such cases entire expenditure is disallowed.

In case of payment made for plying, hiring or leasing goods carriages, the ceiling
of ten thousand rupees shall be enhanced to ₹35,000.

For example:
(i) If ABC Ltd. has paid ₹65,000 in cash, expenditure disallowed shall be ₹65,000.
(ii) If Mr. X has paid ₹11,000 by a bearer cheque, amount disallowed shall be ₹11,000.
(iii) If ABC Ltd. has paid ₹10,050 by a crossed cheque, amount disallowed shall be
₹10,050.
(iv) ABC Ltd. has paid ₹35,000 by an account payee cheque; entire amount is allowed.
(v) Mr. X pays a salary to his employee ₹15,000 by crossed cheque, in this case entire
expenditure is disallowed.
(vi) ABC Ltd. has paid ₹32,000 in cash to a goods transport agency for transportation
of goods, expenditure is allowed.
(vii) Mr. X purchases goods worth ₹75,000 on 01.01.2023 and payment was made
₹60,000 on 03.01.2023 by account payee cheque and ₹8,000 in cash on 03.01.2023
and ₹7,000 in cash on 05.01.2023, in this case expenditure is allowed.
(viii) Mr. X purchases goods worth ₹8,000 and ₹5,000 against two bills from Mr. Y
and makes the payment ₹13,000 in cash in a single day, in that case entire
expenditure is allowed.
(ix) Mr. X purchases goods worth ₹15,000 from Mr. Y against one bill but makes
payment of ₹7,500 and ₹7,500 at different times on the same date, in that case
entire expenditure is disallowed.

Exceptions under rule 6DD

As per rule 6DD the above provisions are not applicable with regard to following
payments:

1. Payment made to Reserve Bank of India, State Bank of India or other banking
institutions, LIC, UTI/ Central/ State Government etc.

2. If the payment is made in a village or town and there is no bank at such place on
the date of making the payment and payment is being given to any person who
ordinarily resides at that place or has his business or profession at that place.

3. Where the payment is made for the purchase of


(i) agricultural or forest produce; or
(ii) the produce of animal husbandry or dairy or poultry farming; or
(iii) fish or fish products; or

305 | P a g e #HakkसेCA by CA. Akash Sir


(iv) the products of horticulture or apiculture (Honey making),
to the cultivator, grower or producer of such articles, produce or products.

4. Where the payment is made for the purchase of the products manufactured in a
cottage industry, to the producer of such products.

5. Where the payment is made by transferring funds from one bank account to the
other or payment is being made by any credit card/ a debit card/ letter of credit
etc., payment is allowed.

6. If payment is being made to an employee after retirement or to his family


member after the death of the employee and payment is in connection with gratuity
etc. and payment is not exceeding ₹50,000.

7. Any other situation given under Rule 6DD.

TP: 23 Provision for Gratuity Fund [Section 40A(7)]

No deduction shall be allowed for provision of gratuity. However, actual payment of


gratuity shall be allowed as deduction as and when becomes payable.

Moreover, the assessee can make provision for contribution towards approved
gratuity fund but such provision should be actuarial provision i.e. it should not be
hypothetical and such provision shall be allowed.

TP: 24 Employer’s contribution to various funds [Section 40A(9)]

Employer’s contribution to various funds is allowed only if such funds are notified
under any Act. If the employer has contributed to the recognized provident fund,
approved superannuation fund, approved gratuity fund or any other similar fund
required under any other Act, such contribution is allowed (but payment has to be
made upto the last date of filing of return of income as per section 43B).

If the employer has contributed to any other fund like unrecognized provident fund,
unapproved gratuity fund, unapproved superannuation fund etc., expenditure shall not
be allowed.

306 | P a g e #HakkसेCA by CA. Akash Sir


TP: 25 Profits chargeable to tax [Section 41]

Section 41(1): Remission or cessation of trading liability

As per section 41(1), any expenditure/ allowance allowed in any previous year in form
of loss, expenditure or trading liability and subsequently such liability cease to exist,
then it shall be considered as Income under head PGBP.

For example: Mr. X has debited ₹20,000 to profit and loss account being the
municipal tax paid but in the subsequent year there was refund of ₹3,000, in this
case it will be considered to be income under the head business/profession in the
year of recovery. If amount has been recovered by the successor of business, in that
case, it will be considered to be income of such successor.

Another example: Mr. X has purchased goods of ₹30,00,000 from a supplier and
debited such purchase in profit and loss account. At the time of making payment, Mr.
X made a payment of ₹22,00,000 instead of ₹30,00,000 as full and final settlement.
In this case ₹8,00,000 shall be treated as income under head PGBP.

Section 41(2): Balancing charge

Already covered under depreciation concept.

Section 41(3): Asset acquired for scientific research

Already covered under section 35.

Section 41(4): Recovery of bad debt

Already covered under section 36(1)(vii)

TP: 26 Certain deductions allowed only on actual payment


[Section 43B]

➢ If any assessee has maintained books of accounts on the basis of mercantile


system of accounting, all the expenditures are allowed on due basis. But certain
expenditures listed under section 43B are allowed only on actual payment basis.

These expenditures are:

(a) any sum payable by the assessee by way of tax, duty, cess or fee, by
whatever name called, under any law like Municipal Tax, Professional Tax,
Composition Tax etc.

307 | P a g e #HakkसेCA by CA. Akash Sir


(b) Employer’s contribution to any provident fund or superannuation fund or
gratuity fund, Employees State Insurance (ESI) or any other fund for the
welfare of employees.

(c) Bonus or commission or leave salary to the employee.

(d) Interest on any loan or borrowing from any Public Financial Institution or a
State Financial Corporation or a State Industrial Investment Corporation or
scheduled bank.

(e) Any sum payable by the assessee to the Indian Railways for the use of railway
assets.

(f) any sum payable by the assessee as interest on any loan or borrowing from a
deposit taking non-banking financial company or systemically important non-
deposit taking non-banking financial company, in accordance with the terms and
conditions of the agreement governing such loan or borrowing.

➢ The assessee is allowed to make the payment till the last date of filing of return
of income relating to the previous year in which the expenditure was incurred.

➢ If the payment is made after the last date of filing of return of income,
expenditure is allowed in the year in which the assessee has made the payment.

For example: ABC Ltd has debited bonus of ₹3,00,000 to the profit/ loss A/c for
the previous year 2023-24 and the company paid the bonus on 07.12.2024, in this
case expenditure is not allowed in the previous year 2023-24. Rather expenditure is
allowed in the previous year 2024-25. Similarly, if the payment is made by the
company on 07.05.2025, expenditure shall be allowed in the previous year 2025-26.

Illustration: 28

Mr. Shyamsundar (age 82 years) has one house which is 50% in business/ profession
and 50% is let out @ 10,000 p.m. and municipal taxes for the entire house are ₹7,000
which were paid on 10.04.2024 and business income of Mr. Shyamsundar before
debiting any expense of house property is ₹7,80,000. Compute tax liability for the
Assessment Year 2024-25 under old provisions.

Solution:

Income under the head business/profession


Business income ₹7,80,000
Less: Municipal taxes (₹3,500)
Net Profit as per P/L accounts ₹7,76,500

308 | P a g e #HakkसेCA by CA. Akash Sir


(Municipal tax allowed because payment has been made upto the last date of filing of
return of income)

Income under the head House Property


Gross annual value 1,20,000
Less: Municipal taxes (allowed only on actual payment basis) Nil
Net annual value 1,20,000
Less: 30% of NAV u/s 24(a) (36,000)
Less: Interest on capital borrowed u/s 24(b) Nil
Income under the head House Property 84,000
(Municipal tax not allowed because payment has not been made upto the end of previous
year)

Gross Total Income 8,60,500


Less: Deduction u/s 80C to 80U Nil
Total Income 8,60,500

Computation of Tax Liability


Tax on ₹8,60,500 at slab rate 72,100
Add: HEC @ 4% 2,884
Tax Liability 74,984
Rounded off u/s 288B 74,980

➢ Conversion of interest into loan/ borrowings/ debentures etc.: Where any sum
payable as interest on any loan is converted into loan/ borrowing/ debenture etc.,
the interest so converted (not actually paid) shall not be allowed as deduction.
Such converted interest shall be allowed as deduction when such portion of loan
has been repaid.

Illustration: 29

Hari, an individual, carried on the business of purchase and sale of agricultural


commodities like paddy, wheat, etc. He borrowed loans from Andhra Pradesh State
Financial Corporation (APSFC) and Indian Bank and has not paid interest as detailed
hereunder:


(i) Andhra Pradesh State Financial Corporation (P.Y. 2022-23 & 2023- 15,00,000
(ii) 24 30,00,000
Indian Bank (P.Y. 2023-24) 45,00,000

Both APSFC and Indian Bank, while restructuring the loan facilities of Hari during the
year 2023-24, converted the above interest payable by Hari to them as a loan
repayable in 60 equal installments. During the year ended 31.3.2024, Hari paid 5
installments to APSFC and 3 installments to Indian Bank.

309 | P a g e #HakkसेCA by CA. Akash Sir


Hari claimed the entire interest of ₹ 45,00,000 as an expenditure while computing the
income from business of purchase and sale of agricultural commodities. Examine
whether his claim is valid and if not what is the amount of interest, if any, allowable.

Solution:

According to section 43B, any interest payable on the term loans to specified financial
institutions and any interest payable on any loans and advances to, interalia, scheduled
banks shall be allowed only in the year of payment of such interest irrespective of the
method of accounting followed by the assessee. Where there is default in the payment
of interest by the assessee, such unpaid interest may be converted into loan. Such
conversion of unpaid interest into loan shall not be construed as payment of interest
for the purpose of section 43B. The amount of unpaid interest so converted as loan
shall be allowed as deduction only in the year in which the converted loan is actually
paid.
In the given case of Hari, the unpaid interest of ₹15,00,000 due to APSFC and of
₹30,00,000 due to Indian Bank was converted into loan. Such conversion would not
amount to payment of interest and would not, therefore, be eligible for deduction in
the year of such conversion. Hence, claim of Hari that the entire interest of
₹45,00,000 is to be allowed as deduction in the year of conversion is not tenable. The
deduction in the year of conversion is not tenable. The deduction shall be allowed only
to the extent of repayment made during the financial year. Accordingly, the amount of
interest eligible for deduction for A.Y. 2024- 25 shall be calculated as follows:

Interest Number of Amount per Instalments Interest


outstanding Instalments instalment paid allowable (₹)
APSFC 15 lakh 60 25,000 5 1,25,000
Indian Bank 30 lakh 60 50,000 3 1,50,000
Total amount eligible for deduction 2,75,000

➢ Sum payable to micro or small enterprises: Section 15 of the Micro, Small


and Medium Enterprises Development Act, 2006 mandates payment of goods or
services to supplier being Mirco or Small Enterprises, on or before the date
agreed (can not be more than 45 days) or within 15 days (if no date is agreed).

If payment is made within the due date, expense shall be allowed on accrual
basis. However, if payment is not made within due date, expense shall be allowed
in the year in which payment is actually made for the purpose of calculating
income u/s. PGBP.

For example: Mr. A has purchased goods of Rs. 10,000 from A & Co., a micro
enterprise on 01.03.2024. As per written agreement between them, the payment
has to be made by 05.04.2024. Mr. A follows mercantile method of accounting.

310 | P a g e #HakkसेCA by CA. Akash Sir


(i) If Mr. A paid the sum on 02.04.2024: Since Mr. A Paid the sum on or
before 05.04.2024, the deduction would be allowed in P.Y. 2023-24.
(ii) If Mr. A paid the sum on 20.04.2024: Since Mr. A paid the sum beyond
the time limit, the deduction would be allowed in the year of actual payment i.e.
P.Y. 2024-25.

S. No. Meaning
Manufacturing enterprises and enterprises rendering services
(1) Micro Enterprise
Investment in Plant and Machinery or AND Turnover <= ₹ 5
Equipment <= ₹ 1 crore
(2) Small Enterprise
Investment in Plant and Machinery or AND Turnover <= ₹ 50
Equipment <= ₹ 10 crore

HW Question: 19

Mr. Naman has computed his income of ₹3,50,000 and some of the amounts debited to
the profit and loss account are as given below:
1. Household expense of ₹5,000
2. Rent for own building of ₹1,20,000 (half of the building is self-occupied and balance
half in business use).
3. Municipal tax of the building of ₹3,000 (amount was paid on 01.04.2024)
4. Expenditure on repairs of the building of ₹4,000.
5. Premium paid for insurance of the building of ₹2,000.
6. Mr. X has purchased one motor car for ₹3,00,000 on 01.01.2024 and it was put to
use on the same date. The car was used for personal purpose as well as official use
(50% official and 50% personal). Assessee has also debited petrol expenses of ₹5,000.
7. He has debited ₹20,000 being the amount invested in public provident fund.
Compute his tax liability for the Assessment Year 2024-25 under normal provisions of
the tax.

TP: 27 Stamp duty value to be the sales value for immovable


property being stock in trade [Section 43CA]

➢ In case of transfer of immovable asset being land or building or both held as


stock-in-trade, if the sale value is less than stamp duty value, then stamp duty
value shall be sale value of such transfer.

For example: Mr. Suyash has sold a house (held as stock) on 01.05.2023 for Rs.
5,00,000. It was purchased by him on 01.06.2021 for Rs. 2,00,000. Stamp duty value
of the house as on 01.05.2023 was Rs. 8,00,000. In this case, sale value shall be Rs.
8,00,000 and not Rs. 5,00,000.

311 | P a g e #HakkसेCA by CA. Akash Sir


➢ If date of agreement and date of registration is not same, then stamp duty value
as on date of agreement can be taken provided that the whole amount of
consideration or part therefore has been paid by A/c. payee cheque, account
payee DD or through use of electronic clearing system (ECS) or any other mode
prescribed ON or BEFORE the date of agreement.

For example: Mr. Suyash has entered into an agreement to sell a house (held as
stock) on 01.05.2022 for Rs. 5,00,000. The sale deed (registration) was made on
01.05.2023. The house was purchased by him on 01.06.2021 for Rs. 2,00,000. Stamp
duty value of the house as on 01.05.2022 was Rs. 7,00,000 and on 01.05.2023 was Rs.
8,00,000. In this case, if part payment is being made on or before 01.05.2022 by
account payee cheque, then sale value shall be Rs. 7,00,000 and not Rs. 5,00,000. If
part payment is made in cash or bearer cheque, then sale value shall be Rs. 8,00,000
only.

➢ Safe harbour rule: If stamp duty value does not exceed 110% of the sale value,
then actual consideration shall be deemed to be the sale value.

For example: Mr. Suyash has sold a house (held as stock) on 01.05.2023 for Rs.
5,00,000. It was purchased by him on 01.06.2021 for Rs. 2,00,000. Stamp duty value
of the house as on 01.05.2023 was Rs. 5,45,000. In this case, sale value shall be Rs.
5,00,000 since stamp duty value (i.e. Rs. 5,45,000) does not exceed 110% of actual
sale consideration (i.e. 110% of Rs. 5,00,000).

➢ If assessee claims that the stamp duty value is more than fair market value, then
assessing officer may transfer matter to valuation officer provided valuation is
not under dispute in court.
If value adopted by valuation officer is more than stamp duty value then sale
value = SDV otherwise value determined by valuation officer shall be sale value.

TP: 28 Compulsory maintenance of accounts [Section 44AA read


with Rule 6F]

As per section 44AA, books of accounts shall be compulsory maintained by following


persons:

1. Person having specified profession:

➢ Specified Profession shall include:


1. Legal profession, 2. Medical profession, 3. Engineering profession,
4. Architectural profession, 5. Profession of accountancy,

312 | P a g e #HakkसेCA by CA. Akash Sir


6. Technical consultancy, 7. Interior decoration, 8. Authorised representatives,
9. Film artists, 10. Company Secretary, 11. Information Technology.

➢ Every person having specified profession have to maintain any books of accounts
as may enable the Assessing Officer to compute his total income.

➢ If gross receipt exceeds ₹1,50,000 in all the three years immediately preceding
the previous year, then, they have to maintain prescribed books of accounts (as
per Rule 6F).
For Example:
Mr. X is engaged in medical profession and his gross receipt during the various
years is as under:
1. 2022-23 1,40,000
2. 2021-22 1,70,000
3. 2020-21 1,25,000
In this case, during the previous year 2023-24, Mr. X is not required to maintain
prescribed books of accounts because gross receipt has not exceeded ₹1,50,000
during all the three years immediately preceding the relevant previous year. But
if receipt during 2022-23 is ₹1,75,000 and during 2020-21 is ₹1,55,000, he has
to maintain prescribed books of accounts during P.Y. 2023-24.

➢ If profession has been newly setup in the previous year and gross receipt are
likely to exceed ₹1,50,000, he should maintain prescribed books of accounts.

2. Persons (other than Individual/ HUF) carrying on business or any


profession, not specified above

➢ If their income from business or profession exceeds ₹1,20,000 or their total


sales turnover or gross receipts as the case may be, in business or profession
exceeds ₹10,00,000 in any one of the 3 years immediately preceding the previous
year, they will be required to maintain any books of accounts.

➢ In case of business or profession newly set up in any previous year, obligation to


maintain accounts will arise if the income is likely to exceed ₹1,20,000 or total
sales turnover or gross receipts as the case may be in business or profession are
likely to exceed ₹10,00,000 during such previous year.

3. Persons (Individual/ HUF) carrying on business or any profession, not


specified under point 1

➢ If their income from business or profession exceeds ₹2,50,000 or their total


sales turnover or gross receipts as the case may be, in business or profession

313 | P a g e #HakkसेCA by CA. Akash Sir


exceeds ₹25,00,000 in any one of the 3 years immediately preceding the
previous year, they will be required to maintain any books of accounts.

➢ In case of business or profession newly set up in any previous year, obligation to


maintain accounts will arise if the income is likely to exceed ₹2,50,000 or total
sales turnover or gross receipts as the case may be in business or profession are
likely to exceed ₹25,00,000 during such previous year.

4. Persons whose business/ profession income is to be computed on


presumptive basis under section 44AD/ 44ADA/ 44AE

➢ If income of any person is to be computed under section 44AD or 44ADA or


44AE on presumptive basis but such person has rejected presumptive income and
his income is exceeding the maximum amount which is exempt from income tax, in
such cases such person shall be required to maintain any books of accounts (also
audit is required as per section 44AB).
For example: Mr. X has turnover of his business ₹20,00,000 but he has rejected
presumptive income, books of account is required.

The books of accounts are to be kept and maintained for the period of atleast 6
years from the end of the relevant assessment year.

Mann Ki Baat (simple language):

Above section deals with maintenance of books of accounts that enable the Assessing
Officer to verify return of income. The above provision can be asked in MCQ.

TP: 29 Compulsory Tax Audit [Section 44AB]

As per section 44AB, following persons have to get their accounts audited:

1. Business + T/O > 100 Lakhs: Every person carrying on business, if his total
sales turnover or gross receipts, in business exceeds ₹100 lakhs during the previous
year.

Important Note: This section shall not apply to the person, who declares profits and
gains for the previous year in accordance with the provisions of section 44AD and his
total sales, turnover or gross receipts, as the case may be, in business does not
exceed ₹200 Lakhs/ ₹300 Lakhs in such previous year.

314 | P a g e #HakkसेCA by CA. Akash Sir


2. Profession + G/R > 50 Lakhs: Every person carrying on profession if his gross
receipts in profession exceed ₹50 lakh during the previous year.

Important Note: This section shall not apply to the person, who declares profits and
gains for the previous year in accordance with the provisions of section 44ADA and
his total gross receipts in profession does not exceed ₹50 Lakhs/ ₹75 Lakhs in such
previous year.

3. Business u/s 44AE + rejected section 44AE: If income of any person is to be


computed under section 44AE on presumptive basis but such person has rejected
presumptive income and has claimed profit or gains lower than the profits under
section 44AE, in such cases such person shall be required to get the accounts
audited.

4. Profession u/s 44ADA + rejected section 44ADA: If income of any person is to


be computed under section 44ADA on presumptive basis but such person has
rejected presumptive income and has claimed profit or gains lower than the profits
under section 44ADA and his income exceeds the basic exemption limit, in such cases
such person shall be required to get the accounts audited.

5. Business u/s 44AD + opts out of section 44AD: If income of any person has
computed under section 44AD on presumptive basis during any prior previous year
and he opts out of section 44AD during any of the 5 assessment years succeeding
the AY in which he opted 44AD, and his income exceeds the basic exemption limit, in
such cases such person shall be required to get the accounts audited.

6. Business + T/O > ₹1,000 Lakhs + Receipt/ payment > 5% in cash: Every
person carrying on business, if his total sales turnover or gross receipts, in business
exceeds ₹1,000 lakh during the previous year provided that in the case of a person
whose––
(a) aggregate of all amounts received including amount received for sales, turnover or
gross receipts during the previous year, in cash, does not exceed 5% of the said
amount. and
(b) aggregate of all payments made including amount incurred for expenditure, in
cash, during the previous year does not exceed 5% of the said payment.

Provided further that for the purposes of this clause, the payment or receipt, as the
case may be, by a cheque drawn on a bank or by a bank draft, which is not account
payee, shall be deemed to be the payment or receipt, as the case may be, in cash.

Due date of audit report: The accounts should be audited by a Chartered


Accountant and audit report should be submitted latest by one month prior to the

315 | P a g e #HakkसेCA by CA. Akash Sir


last date of filing of return of income. That is one month prior to 31st Oct of A.Y. i.e.
30th Sept 2024 for our P.Y.

Penalty for violating provisions of Section 44AB [Section 271B]: If any person
fails to get his accounts audited or fails to submit audit report in time, penalties may
be imposed under section 271B equal to ½% of total turnover or gross receipt subject
to a maximum of ₹1,50,000.
For example: Mr. X has turnover of his business ₹105 lakhs but he has failed to get
his accounts audited, in this case penalties may be imposed amounting to ₹52,500 but
if his turnover was ₹400 lakhs, penalties imposable shall be ₹2,00,000 but maximum
₹1,50,000.

Mann Ki Baat (simple language):

Section 44AB is the most important section of PGBP since it makes sure that business/
profession with higher turnover do not escape income tax by filing wrong ITR. It
imposes an obligation on the assessee to get their accounts audited by a CA in practice.
This is important section for exam.

TP: 30 Presumptive taxation for businesses [Section 44AD]

➢ Business T/O upto ₹200 Lakhs + Income shall be 8%/ 6% of T/O: If any
assessee has turnover of his business upto ₹200 lakhs, such assessee is allowed
to compute income on presumptive basis and income under PGBP shall be
presumed to be minimum 8% of the turnover and no further deduction is allowed
under section 30 to 38.

Rate of 6% shall be applied instead of 8% if the amount of total turnover or


gross receipts which is received by an account payee cheque or an account payee
bank draft or use of electronic clearing system through a bank account or Credit
Card, Debit Card, Net Banking, IMPS (Immediate Payment Service), UPI (Unified
Payment Interface), RTGS (Real Time Gross Settlement), NEFT (National
Electronic Funds Transfer), and BHIM (Bharat Interface for Money) Aadhaar
Pay during the previous year or before the due date specified in subsection (1) of
section 139 in respect of that previous year.

Resident Individual/ HUF/ Firm: Such option is allowed only to an Individual/


HUF/ Firm who are resident but not to LLP or Company.

➢ Business T/O upto ₹300 Lakhs + Cash receipt <= 5% of Total turnover +
Income shall be 8%/ 6% of T/O: If any assessee has turnover of his business

316 | P a g e #HakkसेCA by CA. Akash Sir


upto ₹300 lakhs and the aggregate amount received during the previous year, in
cash, does not exceed 5% of the total turnover or gross receipts of such
previous year, such assessee is allowed to compute income on presumptive basis
and income under PGBP shall be presumed to be minimum 8%/ 6% of the turnover
and no further deduction is allowed under section 30 to 38.

For the above purposes, the receipt of amount or aggregate of amounts by a


cheque drawn on a bank or by a bank draft, which is not account payee, shall be
deemed to be the receipt in cash.

➢ Business u/s 44AD + opts out of section 44AD: If an assessee has opted for
presumptive income under section 44AD and in the subsequent 5 assessment
years he has opts out of presumptive income, in that case he will not be allowed
to opt for presumptive income for 5 assessment years subsequent to assessment
year in which he opted out of section 44AD.
If assessee has rejected the presumptive income, he will be required to maintain
any books of accounts and also audit is required.
For example: Mr. X has opted for presumptive income under section 44AD in the
previous year 2022-23 and if he opts out of 44AD during P.Y. 2023-24, he will
not be allowed to opt for 44AD in subsequent 5 years i.e P.Y. 2024-25, P.Y. 2025-
26, P.Y. 2026-27, P.Y. 2027-28 and P.Y. 2028-29.

➢ Other points:

1. Section 44AD is applicable only to business and not to specified profession and
also it is not applicable for the persons having earning as commission or
brokerage.
2. Such assessee shall be required to pay advance tax to the extent of 100% of tax
liability on or before 15th march of the relevant previous year otherwise interest
shall be charged @ 1% for one month on the amount of default.
3. Brought forward business loss is allowed to be adjusted from such income but
brought forward depreciation is not allowed to be adjusted from such income.
4. The assessee shall be exempt from maintaining books of accounts or audit.

Mann Ki Baat (simple language):

Let’s imagine a situation where a small business has to apply the provision contained
under section 30 to 43D, it will be very difficult for him to calculate PGBP income since
he is not financially well versed to maintain books of accounts and apply such provisions.
Therefore, the Government has launched the scheme of presumptive income wherein no

317 | P a g e #HakkसेCA by CA. Akash Sir


books of accounts are required. The assessee only needs to consider 8%/ 6% of total
turnover as his income.

Illustration: 30

Mr. Ashu is engaged in a business with turnover ₹1,70,00,000 (all payments received by
account payee cheque, bank draft or through electronic clearing) and expenses incurred
in connection with earning of income are ₹1,60,00,000. He has LTCG ₹5,00,000. He has
brought forward business loss of ₹1,00,000 of P.Y. 2019-20. Compute his Income and
Tax Liability for previous year 2023-24, in two situations -
(i) He has opted for section 44AD.
(ii) He has not opted for section 44AD.

Solution:

(i) He opts for section 44AD:

Income under head profits and gains of business or profession


Presumptive Income shall be (170,00,000 x 6%) 10,20,000
Less: Brought forward loss of P.Y. 2019-20 (1,00,000)
Income under the head Business or Profession 9,20,000

Income under the head Capital Gains


Long Term Capital Gains 5,00,000

Gross Total Income 14,20,000


Less: Deduction under chapter VIA Nil
Total Income 14,20,000

Computation of Tax Liability


Tax on ₹9,20,000 at slab rate 96,500
Tax on LTCG ₹5,00,000 @ 20% u/s 112 1,00,000
1,96,500
Add: HEC @ 4% 7,860
Tax Liability 2,04,360

Note: The Assessee shall be exempt from maintaining books of accounts and also from
Audit.

(ii) He does not opt section 44AD:

Income under the head profits and gains of business or profession

318 | P a g e #HakkसेCA by CA. Akash Sir


Gross Receipts 170,00,000
Less: Expenses Incurred (160,00,000)
Less: Brought forward loss of P.Y. 2019-20 (1,00,000)
Income under the head Business or Profession 9,00,000

Income under the head Capital Gains


Long Term Capital Gains 5,00,000

Gross Total Income 14,00,000


Less: Deduction under chapter VIA Nil
Total Income 14,00,000

Computation of Tax Liability


Tax on ₹9,00,000 at slab rate 92,500
Tax on LTCG ₹5,00,000 @ 20% u/s 112 1,00,000
1,92,500
Add: HEC @ 4% 7,700
Tax Liability 2,00,200

Note: The Assessee shall be liable to maintain books of accounts and also liable to
Audit.

HW Question: 20

Mr. Mohan engaged in Retails Trade, reports a turnover of ₹58,50,000 (all payments
received in account payee cheque) for the financial year 2023-24. His income from the
said business as per books of account is computed at ₹2,90,000. Retail trade is the only
source of income for Mr. Mohan.
(i) Is Mr. Mohan eligible to opt for presumptive determination of his income chargeable
to tax for the Assessment Year 2024-25?
(ii) Is so, determine his income from retail trade as per the applicable presumptive
provision.
(iii) In case, Mr. Mohan has not opted for presumptive taxation of income from retail
trade, what are his obligations under the Income-tax Act, 1961?
(iv) What is the ‘due date’ for filing his return of income, under both the options?

HW Question: 21

Mr. Naman is engaged in the business of producing and selling TV. During the previous
year 2023-24, his turnover was ₹1.75 crores. He opted for paying tax as per
presumptive taxation scheme laid down in section 44AD. He has no other income during
the previous year. Is he liable to pay advance tax and if so, what is the minimum amount

319 | P a g e #HakkसेCA by CA. Akash Sir


of advance tax to be paid and the due date for payment of such advance tax assuming
that whole of the turnover represents cash receipts assuming that he has not opted
for section 115BAC?

TP: 31 Presumptive taxation for profession [Section 44ADA]

➢ Profession G/R upto ₹50 Lakhs + Income shall be 50% of G/R: If any
assessee has gross receipt of his profession upto ₹50 lakhs, such assessee is
allowed to compute income on presumptive basis and income under PGBP shall be
presumed to be minimum 50% of the gross receipt and no further deduction is
allowed under section 30 to 38.

Resident Individual/ Firm: Such option is allowed only to an Individual/ Firm


who are resident but not to LLP or Company.

➢ Profession G/R upto ₹75 Lakhs + Income shall be 50% of G/R: If any
assessee has gross receipt of his profession upto ₹75 lakhs and the aggregate
amount received during the previous year, in cash, does not exceed 5% of the
total turnover or gross receipts of such previous year, such assessee is allowed
to compute income on presumptive basis and income under PGBP shall be
presumed to be minimum 50% of the gross receipt and no further deduction is
allowed under section 30 to 38.

For the above purposes, the receipt of amount or aggregate of amounts by a


cheque drawn on a bank or by a bank draft, which is not account payee, shall be
deemed to be the receipt in cash.

➢ Other points:

1. Such assessee shall be required to pay advance tax to the extent of 100% of tax
liability on or before 15th march of the relevant previous year otherwise interest
shall be charged @ 1% for one month on the amount of default.
2. Brought forward business loss is allowed to be adjusted from such income but
brought forward depreciation is not allowed to be adjusted from such income.
3. The assessee shall be exempt from maintaining books of accounts or audit.
4. Such Assessee has the option to reject presumptive income but in that case the
assessee shall be required to maintain any books of accounts and also audit is
required.
5. Assessee can change the option on year-to-year basis.

320 | P a g e #HakkसेCA by CA. Akash Sir


Illustration: 31

Mr. Aman is engaged in specified profession and has gross receipt ₹42,00,000. He has
Long term Capital Gain ₹7,00,000 and brought forward business loss ₹30,000 of A.Y.
2020-21. He invested ₹20,000 in LIC in his own name. Compute his Tax Liability for the
Assessment Year 2024-25. He has opted for Section 44ADA.

Solution:

Income under head profits and gains business or profession


Gross Receipt 42,00,000
Presumptive Income u/s 44ADA (50% of Rs. 42,00,000) 21,00,000
Income under the head Business Profession 21,00,000
Less: B/F business loss (30,000)
Income under the head Business Profession 20,70,000
Income under the head Capital Gains 7,00,000
Gross Total Income 27,70,000
Less: Deduction u/s 80C (20,000)
Total Income 27,50,000

Computation of Tax Liability


Tax on ₹20,50,000 at slab rate 4,27,500
Tax on LTCG ₹7,00,000 @ 20% 1,40,000
Tax before health & education cess 5,67,500
Add: HEC @ 4% 22,700
Tax Liability 5,90,200

TP: 32 Presumptive taxation for Plying, Hiring or Leasing Goods


Carriages [Section 44AE]

If any person is engaged in the business of plying, hiring or leasing goods carriages,
he will have the option to compute income under the head business/profession on
presumptive basis.

➢ Heavy goods vehicle (> 12MT) + Income shall be ₹1,000 per MT per month:
If it is a heavy goods vehicle income shall be presumed to be ₹1,000 per ton of
gross weight per month or part of the month. Heavy goods vehicle means goods
vehicle having gross weight more than 12 ton (12000 kg.). For example, if weight
of vehicle is 14 ton (14000 kg), income shall be ₹ 14,000 per month.

321 | P a g e #HakkसेCA by CA. Akash Sir


No further deduction is allowed under section 30 to 38 but in case of a firm
interest and salary to partners is allowed as per section 40(b).

➢ Other than heavy goods vehicle (<= 12MT) + Income shall be ₹7,500 per
month: If it is not a heavy goods vehicle income shall be presumed to be ₹7,500
per month or part of the month. For example, if weight of vehicle is 11 ton
(11,000 kg), income shall be ₹ 7,500 per month.

No further deduction is allowed under section 30 to 38 but in case of a firm


interest and salary to partners is allowed as per section 40(b).

➢ Not owner of more than 10 vehicles: Assessee should not have more than 10
goods carriages at any time during the year otherwise such option is not allowed.

➢ Income shall be calculated based on the months the assessee owned the vehicles
and not put to use. For example: Mr. X purchased vehicle in April 2023 but put
to use during Aug 2023, in this case, income shall be presumed for 12 months and
not 8 months.

➢ Other points:

1. If actual income is more than the presumptive income, actual income shall be
taken into consideration.
2. The assessee shall be exempt from maintaining books of accounts or audit.
3. The assessee has the option to reject presumptive income but in that case
assessee should maintain any books of accounts and also audit is required.
4. An assessee, who is in possession of a goods carriage, whether taken on hire
purchase or on instalments, shall be deemed to be the owner of such goods
carriage.
5. Assessee can change the option on year-to-year basis.
6. Brought forward depreciation shall not be allowed to be adjusted but brought
forward business loss shall be allowed to be adjusted.

Illustration: 32

Mr. Mohan retired from Govt. service in March 2023. He got ₹20,00,000 on account of
retirement benefits. Out of the aforesaid sum, he purchased on 23rd April 2023 a few
motor vehicles and got their delivery on that date.
The particulars of the vehicles are given below–
Vehicle Number Cost of the vehicle
Heavy goods vehicle (15 ton) 2 ₹9,00,000
Medium goods vehicle (8 ton) 4 ₹4,50,000

322 | P a g e #HakkसेCA by CA. Akash Sir


Light commercial Vehicle (4 ton) 3 ₹3,20,000

He started plying the vehicles from 04.06.2023. On an average every vehicle remains
off the road for about a week for repairs and maintenance. He maintains a rough
record of the receipts and outgoings which is given below –
Receipts ₹3,70,000
Less: Expenses (Excluding depreciation and salaries to Mr. Soham) (₹60,000)
₹3,10,000
You are required to compute the total income of Mr. Mohan from the business of goods
carriage for the previous year 2023-24.

Solution:

Computation of Business Income


As per section 44AE
Other than heavy goods vehicle (₹7,500 x 7 x 12) 6,30,000
Heavy goods vehicle (₹1,000 x 15 x 12 x 2) 3,60,000
Business Income 9,90,000
Gross Total Income 9,90,000
Less: Deduction u/s 80C to 80U Nil
Total Income 9,90,000

HW Question: 22

An assessee owns a heavy commercial vehicle having gross vehicle weight of 15 ton each
for 9 months 15 days, a medium goods vehicle having gross vehicle weight of 8 ton for 9
months and a light goods vehicle having gross vehicle weight of 5 ton for 12 months
during the previous year. Compute his income applying the provisions of section 44AE.

HW Question: 23 [PGBP + Deductions]

Mr. Jatin (aged 38) owned 6 heavy goods vehicles having gross vehicle weight of 16 ton
(16,000 Kg) each as on 01.04.2023. He acquired 2 more light goods vehicles having
gross vehicle weight of 8 ton (8,000 Kg) each on 01.07.2023. He is solely engaged in the
business of plying goods vehicles on hire since financial year 2019-20.
He did not opt for presumptive provision contained in section 44AE for the financial
year 2022-23. His books were audited under section 44AB and the return of income
was filed on 05.08.2023. He has unabsorbed depreciation of ₹70,000 and Business loss
of ₹1,00,000 for the financial year 2022-23. Following further information is provided
to you:
(i) Paid medical insurance premium of ₹23,000 for his parents (both aged above 70) by
means of bank demand draft.

323 | P a g e #HakkसेCA by CA. Akash Sir


(ii) Paid premium on life insurance policy of his married daughter of ₹25,000.
(iii) Repaid principal of ₹40,000 and interest of ₹15,000 to Canara Bank towards
education loan of his daughter, who completed B.E. two years ago. She is employed
after completion of her studies.
Assuming that Mr. Jatin has opted for presumptive provision contained in section 44AE
of the Income-tax Act, 1961, compute the total income of Mr. Jatin for the A.Y. 2024-
25.

HW Question: 24

Mr. Akshay is engaged in the business of plying goods carriages. On 1st April, 2023, he
owns 10 trucks (out of which 6 are heavy goods vehicles having capacity of 18 ton and
balance 4 trucks having capacity of 8 ton). On 2nd May, 2023, he sold two of the heavy
goods vehicles and purchased two light goods vehicles having capacity of 8 ton on 6th
May, 2023. Those new vehicles could however be put to use only on 15th June, 2023.
Compute the Total Income and Tax Liability of Mr. Akshay for the Assessment Year
2024-25, taking note of the following data in two situations i.e. presumptive basis and
normal basis.
Freight charges collected 9,90,000
Less: operational expenses 5,25,000
Depreciation as per sec 32 1,85,000
Other office expenses 15,000
(7,25,000)
Net Profit 2,65,000
Other business and non-business income 1,00,000

TP: 33 Order of set-off of losses/ depreciation

Order of set of losses and depreciation shall be as given below:


(a) Current year expenses
(b) Current year depreciation/ Current year capital expenditure on scientific
research and current year expenditure on family planning, to the extent allowed.
(c) Brought forward loss from business/profession [Section 72(1)]
(d) Unabsorbed depreciation [Section 32(2)]
(e) Unabsorbed capital expenditure on scientific research [Section 35(4)].
(f) Unabsorbed expenditure on family planning [Section 36(1)(ix)]

Illustration: 33

The Profit & Loss account of Mr. X for the previous year ending 31.03.2024 is as given
below:

324 | P a g e #HakkसेCA by CA. Akash Sir


Particulars Amount Particulars Amount
(Debits) ₹ (Credits) ₹
To Purchases 90,00,000 By Sales 102,00,000
To Business expenses 6,00,000 By Stock 1,50,000
To Depreciation 50,000 By Dividend from foreign 30,000
company (gross)
To Salary to Mr. X 3,60,000 By Sundry receipts 20,000
To Interest on capital 1,89,000 By Long term capital gain 1,00,000
To Sundry expenses 1,01,000
To Net profit 2,00,000
105,00,000 105,00,000

You are further informed that –


1. Purchases include cash purchases of ₹1,00,000 (payment made on a particular date to
a particular person)
2. Bonus of ₹1,07,000 for the previous year 2022-23 was paid on 31.12.2023 but not
included in the profit and loss account.
3. Recovery of bad debts during the year from a discontinued business of ₹1,00,000
but not included in the profit and loss account. Deduction was allowed in respect of bad
debts.
4. Written down value of machinery as on 01.04.2023 was ₹5,00,000. Rate of
depreciation being 15%.
a. Machinery sold during the year for ₹1,00,000
b. Machinery acquired and put to use in December 2023 for ₹4,00,000
5. Loss and allowances carried forward
• Business loss – Assessment Year 2020-21 = ₹3,00,000
• Depreciation – Assessment Year 2021-22 = ₹2,00,000
Mr. X has not opted for presumptive taxation of Income u/s 44AD. Compute Total
Income and Tax Liability for the Assessment Year 2024-25.

Solution:

Computation of Income under the head business/profession


Net Profit as per profit and loss account 2,00,000
Add:
Cash purchases {u/s 40A(3)} 1,00,000
Recovery of bad debts {as per sec 41(4)} 1,00,000
Salary of Mr. X 3,60,000
Interest on capital 1,89,000
Less:
Bonus paid (1,07,000)
Depreciation on machinery (40,000)

325 | P a g e #HakkसेCA by CA. Akash Sir


Working Note:
Written down value 5,00,000
Less: Sale (1,00,000)
Add: Purchase 4,00,000
8,00,000
Depreciation
7.5% on ₹4,00,000 30,000
15% on ₹4,00,000 60,000
Total 90,000
Already provided in profit & loss A/c 50,000
Balance 40,000
Dividend from foreign company (30,000)
Long term capital gains (1,00,000)
Income under the head Business/Profession 6,72,000
Less: b/f Business Loss (3,00,000)
Less: Unabsorbed depreciation (2,00,000)
Income under the head Business/Profession 1,72,000

Income under the head Capital Gains (LTCG) 1,00,000

Income under the head Other Sources 30,000


{Dividend from foreign company}

Gross Total Income 3,02,000


Less: Deduction u/s 80C to 80U Nil
Total Income 3,02,000

Computation of Tax Liability


Tax on LTCG ₹52,000 (1,00,000 – 48,000) @ 20% u/s 112 10,400
Tax on ₹2,02,000 at slab rate Nil
Less: Rebate u/s 87A (10,400)
Tax Liability Nil

TP: 34 Speculation business [Explanation 2 to section 28]

Meaning of Speculative transaction: It means a transaction in which a contract for


the purchase or sale of any commodity, including stocks and shares, is periodically or
ultimately settled otherwise than by the actual delivery or transfer of the
commodity or scrips
For example: Mr. X entered into a contact for purchase of one plot from Mr. A and
same plot was sold by him to Mr. Y at a higher rate and he has directed Mr. Y to pay
the amount directly to Mr. A and surplus amount to Mr. X and he directed Mr. A to

326 | P a g e #HakkसेCA by CA. Akash Sir


transfer the plot directly in the name of Mr. Y, it will be called speculative
transaction but if Mr. X has transferred the plot in his name and after that plot was
transferred in the name of Mr. Y, it will be called normal business/ capital transfer.

The following shall not be deemed to be a speculative transaction:


(a) Hedging contract in respect of raw materials/ merchandise
(b) Hedging contract in respect of stock/ shares
(c) Forward contract
(d) Trading in securities derivatives
(e) Trading in commodities derivatives

TP: 35 Expenditure by way of payment to associations and


institutions for carrying out rural development programs [Section
35CCA]

100% deduction shall be allowed where payment is made to:


(a) to an association or institution, which has as its object the undertaking of any
program of rural development, to be used for carrying out any program of rural
development approved by the prescribed authority; or
(b) to an association or institution, which has as its object the training of persons for
implementing programs of rural development; or
(c) to a rural development fund set up and notified by the Central Government in this
behalf; or
(d) to the National Urban Poverty Eradication Fund set up and notified by the Central
Government in this behalf.

327 | P a g e #HakkसेCA by CA. Akash Sir


Comprehensive Questions

Question: 1 [PGBP]

The following is the receipts and payments account of a medical practitioner for the
year ending 31.03.2024

Receipts Amount Payments Amount


Balance b/d 1,30,000 Clinic expenses 1,24,000
Visiting fees 45,75,000 Medical books purchased and
put to use on 01.07.2023 15,000
Consultation fees 9,15,000 Surgical equipment 90,000
Sale of medicines 28,000 Motor car expenses 36,000
Payment received for using Indian Medical Association
Operation Theatre 18,000 membership fees 7,000
Dividend from domestic Payment to C.A. firm for filing
company 22,000 return of income 4,000
Bank loan for purchasing
a flat 2,00,000 Entertainment expenses 24,000
Life insurance policy
(maturity proceeds) 1,00,000 Medical purchases 33,000
Rental income from flat 60,500 Purchase of flats 2,80,000
Bank interest on loan 30,000 Balance c/d 54,05,500
60,48,500 60,48,500

Additional information:
1. A cash payment of ₹75,000 was given to him by a patient in appreciation of his
medical services but was not recorded in books.
2. Flat was purchased on 01.04.2023 and was self occupied for residence for a month
from the date of its purchase. Thereafter it was let out @ ₹5,500 p.m., the municipal
value of the flats is ₹66,000 p.a. and municipal taxes assessed, though not paid, is
₹4,500.
3. One–third of motor car expenses relate to his personal use. Depreciation on car
allowable under Income Tax Act for professional use is ₹12,000.
4. The rate of depreciation on surgical equipment is 15%. The written down value of
equipment on 01.04.2023 is ₹60,000. He sold some of the equipment for ₹30,000
during the year. New equipment was purchased on 01.11.2023 for ₹90,000 and was put
to use on the same date.
Compute his Total Income and Tax Liability for the Assessment Year 2024-25 under
old tax provisions.

Question: 2 [PGBP, Salary]

Mr. Jonny is an advocate in Delhi High Court. He keeps his books on cash basis. His
receipts and payments account for the financial year 2023-24 is given below:

328 | P a g e #HakkसेCA by CA. Akash Sir


Receipts Amount Payments Amount
₹ ₹
Balance b/d 49,200 Rent Paid for Building 1,44,000
Consultancy fee 55,35,500 Office expenses 46,000
Remuneration from 7,000 New car purchased and put to 3,00,000
university as evaluator of use on 01.05.2023
LLB exams
Sale proceeds of residential 5,00,000 Computer purchased and put 50,000
house (it was purchased on to use on 01.04.2023
01.07.2022 for ₹3,00,000)
Salary from law faculty for 45,000 Legal books purchased 30,000
working as part time
lecturer
Car expenses 42,000
Advance Income tax paid 22,000
Electricity and water charges 16,000
for the entire house
Son’s college tuition fee paid 54,000
Gift to daughter 25,000
Life insurance premium paid 12,000
on own life (sum assured
₹50,000)
Balance c/f 53,95.700
61,36,700 61,36,700

Additional information:
1. On 31.03.2024 legal fees outstanding amounted to ₹22,000
2. Rent is payable @ ₹12,000 p.m.
3. 70% of the use of the car is for official purpose and 30% for personal purpose.
4. Legal books for ₹12,000 was purchased on 01.05.2023 and put to use on the same
date and for ₹18,000 on 01.11.2023 and put to use on the same date.
5. Half of the house taken on rent is being used for residential purposes.
Compute the Total Income and Tax Payable of Mr. Jonny for the Assessment Year
2024-25 under old tax provisions.

Question: 3

ABC Ltd. has net profits of ₹7,00,000 after debiting municipal taxes of ₹12,000
relating to the previous year 2023-24, which were paid on 20.09.2024. Municipal taxes
are related to a building which is owned by the company, the ground floor and first
floor (which is 2/3rd of the complete building) was being used by company. The
company has debited market rent of ₹20,000 p.m. to the profit & loss account for using
the building and credited rent of ₹10,000 p.m. to the profit & loss account for the
second floor of the building which has been let out to some person during the previous

329 | P a g e #HakkसेCA by CA. Akash Sir


year 2023-24.
Compute Tax Liability of the company for the Assessment Year 2024-25 under old tax
provisions.

Question: 4

Mr. Akash is a Chartered Accountant and has prepared the following income and
expenditure account as on 31.03.2024.

Income and Expenditure Account

Expenditure Amount Income Amount


₹ ₹
Office expenses 12,000 Professional fee 65,00,000
Employee’s salary 20,000 Consultancy Fee 55,000
Magazines and newspapers 800 Dividend from Indian 8,500
co.
Entertainment Expenses 17,500 Profit on sale of 8,450
(Personal) debentures (STCG)
Donation for a charity show 600 Gift from father in-law 6,050
Interest on loan for 8,00
professional purpose
Income Tax (advance tax) 5,000
Car Expenses 2,500
Purchase of books 2,000
Stationery 21,000
Diwali gift to employees 1,000
Rent of own building 60,000
Municipal tax 1,000
White washing and Painting of 2,000
building
Expenses incurred on the 3,000
Opening
ceremony (refreshments)
Net profit 64,28,800
65,78,000 65,78,800

You are required to compute his Total Income and Tax Liability (old provisions) for the
Assessment Year 2024-25 considering the following points –
1. The car is used equally for official and personal purposes.
2. ₹1,500 for domestic servant’s salary is included in employee’s salary.
3. Books were purchased on 01.09.2023 and were put to use on the same date.
4. Payment of stationery ₹20,500 was made by a bearer cheque and ₹ 500 was paid in
cash.

330 | P a g e #HakkसेCA by CA. Akash Sir


5. Mr. Akash is owner of a building. Its written down value is ₹ 90,000 on 01.04.2023.
The building is used for official purposes. No depreciation is claimed.
6. Furniture having written down value of ₹30,000 as on 01.04.2023 is also used for
profession. Office chairs and tables were purchased and put to use on 30.03.2024 for
the purpose of a new office which has been inaugurated on 31.03.2024. No depreciation
has been debited to the profit and loss account. Actual cost ₹20,000.
7. Employee’s salary includes bonus of ₹5,000 which was paid to one of the employees
on 01.07.2024.

Question: 5

State with reasons, the deductibility or otherwise of the following expenses/payments


under the Income-tax Act, 1961, while computing income under the head “Profits and
gains of business or profession” for the Assessment Year 2024-25:
(i) Mr. Mohan paid ₹75,000 as commodity transaction tax in respect of sale of
commodity during the previous year 2023-24.
(ii) Dinesh & Co. has set up a warehousing facility for storage of food grains. It
commenced operations on 01.04.2023. For this purpose, Dinesh & Co. incurred capital
expenditure of ₹50 lakhs on purchase of building in March 2022. (Through an account
payee cheque.)

Question: 6

Mr. Ashok (age 79 years) is running a shop at Chandni Chowk and has submitted the
following profit and loss account for the Assessment Year 2024-25.

Particulars Amount Particulars Amount


(Debits) ₹ (Credits) ₹
Opening stock 10,00,000 Sales 110,00,000

Purchases 95,70,000 Closing Stock 4,00,000


Salaries 1,50,000
Market rent (building is owned by the 1,00,000
assessee himself)
Municipal taxes of the building (due) 5,000
Loss by theft 19,000
Donation for Ram Lila celebration 1,500
Provision for bad debts 11,000
Gifts to relatives 400
Presents to clients for 300
advertisements
Public provident fund 12,000
Interest on loan for business 13,000
Interest on capital 4,000
Addition to business premises 2,00,000

331 | P a g e #HakkसेCA by CA. Akash Sir


Repairs of business premises 600
Income tax 2,000
Fine for violation of traffic rules 100
Net Profit 3,11,100
114,00,000 114,00,000

Additional information:
1. Purchases includes purchase of ₹1,00,000 from a relative and it is excessive by
₹20,000 and payment was made in cash.
2. Salary includes ₹14,000 paid outside India without deducting tax at source and
₹7,000 were paid to one of the relatives which is more than the market rate by ₹1,000.
3. Business is being run in a commercial building which is owned by the assessee and its
written down value on 01.04.2023 is ₹10 lakhs and addition was made to the building on
01.01.2024 and brought into immediate use and no depreciation has been debited to
profit and loss account.
Mr. Ashok has not opted for presumptive taxation of Income u/s 44AD. Compute his
Total Income and Tax Liability for the Assessment Year 2024-25 under old tax
provisions.

Question: 7

Mrs. Verma submitted the following profit & loss account for the Assessment Year
2024-25.

Particulars Amount Particulars Amount


(Debits) ₹ (Credits) ₹
Salary (including proprietor’s 46,000 Gross Profit 1,22,000
salary of ₹15,000)
General Expenses 6,000 Bad debts recovered (not allowed 2,000
earlier due to lack of evidence)
Advertisements 39,000 Interest on company deposit 5,000
Interest on proprietor’s 2,000 Long term capital gains 5,00,000
capital
Provision for bad debts 2,000
Depreciation 4,000
Reserve for GST 10,000
Advance income tax 9,500
Donation to scientific 1,000
research
institution
Motor car expenses 1,000
Stationery 1,900
Net Profit 5,06,600
6,29,000 6,29,000

332 | P a g e #HakkसेCA by CA. Akash Sir


Other information:
1. General expenses include ₹ 300 given to a poor student to enable him to pursue his
studies.
2. Motor car expenses include ₹300 for personal purposes.
3. Scientific institution is an approved institution.
Compute her Tax Liability and Tax Payable for the Assessment Year 2024-25 under old
tax provisions.

Question: 8

ABC Ltd. submits the profit & loss account for the year ending 31st March 2024.

Particulars Amount Particulars Amount


(Debits) ₹ (Credits) ₹
Salary to staff 3,00,000 Gross Profit 5,27,000
Capital expenditure for promotion of 14,000 Rent of flats given 24,000
family planning amongst employee to staff
GST (paid on 01.11.2024) 24,000 Sundry receipts 7,000
Gratuity paid to staff 24,000 Short term Capital 60,000
gains on sale of land
Reserve for future losses 30,000
Reserve for bad debts 14,000
Payment of advance income-tax 17,000
Car expenses 20,000
Depreciation 30,000
Office expenses 12,000
Repair of flats given to staff 24,000
Sundry expenses 46,000
Net Profit 63,000
6,18,000 6,18,000

Determine the Total Income and Tax Liability of company for the Assessment year
2024-25 under old tax provisions.

Question: 9

From the following profit and loss account of Mr. Rahman for the year ending March
31st, 2024, compute his Total Income and Tax Liability for the Assessment Year 2024-
25.

Particulars Amount Particulars Amount


(Debits) ₹ (Credits) ₹
Opening stock 4,62,000 Sales 103,00,000

333 | P a g e #HakkसेCA by CA. Akash Sir


Purchases 90,35,000 Closing stock 4,97,000
Salaries 8,50,000 Rental income from house 84,000
property
Rent rate & taxes 1,25,000 Dividends from an Indian 12,000
company
Legal charges 45,000 Income from owning and 20,000
maintaining of race camels
Miscellaneous expenses 22,000
Provision for gratuity 22,000
Provision for Income Tax 53,000
Salary to Mrs. X 36,000
Depreciation 40,000
Net Profit 2,23,000
109,13,000 109,13,000

Additional information:
(i) Purchases include ₹1,10,000 paid in cash to a cultivator for purchase of an
agricultural produce.
(ii) Purchases also include ₹15,000 paid by way of compensation to a supplier as the
assessee was unable to take the delivery of goods due to lack of storage space and
finance.
(iii) Opening stock was overvalued by 25% and closing stock was undervalued by 25%.
(iv) Salary includes ₹ 25,000 paid as customary bonus on the occasion of Diwali over and
above the bonus payable under the Payment of Bonus Act, 1965.
(v) Rent, rates and taxes include:
(a) ₹3,000 on account of municipal taxes for property let out and payment was made on
31.03.2024.
(b) Penalty imposed by GST department of ₹25,000.
(vi) Provision for Gratuity is on actuarial basis.
(vii) Mrs. Rahman is a law graduate and actively working in the assessee firm and salary
paid is reasonable.
(viii) He has invested ₹1,00,000 in equity shares of infrastructure development
companies notified u/s 80C.
(ix) He has loss from owning and maintaining of race horses ₹20,000.
Mr. Rahman has not opted for presumptive taxation of Income u/s 44AD.

Question: 10 [PGBP]

Net profit as per the profit and loss account of Mr. Rakesh is ₹7,70,000 for the year
ending 31st March, 2024.
The following information is noted from the accounts:
(a) Advertisement expenditure debited to profit and loss account includes the
following:
(i) Expenditure incurred outside India: ₹56,000 (Tax has been deducted at source and
paid during the year)

334 | P a g e #HakkसेCA by CA. Akash Sir


(ii) Articles presented by way of advertisement (60 articles cost of each being ₹700,
and 36 articles cost of each being ₹1,500);
(iii) ₹20,000 being the cost of advertisement which appeared in a newspaper owned by
a political party;
(iv) ₹14,400 being capital expenditure on advertisement; (eligible for dep. @ 25%)
(v) ₹9,000 paid in cash
(vi) ₹9,000 paid to a concern in which Rakesh has substantial interest (amount is
excessive to the extent of ₹1,800)

(b) Out of salary to the employees debited to the profit and loss account:
(i) ₹60,000 is employee’s contribution to the recognized provident fund, ₹47,500 of
which is credited in the employee’s account in the relevant fund before the due date
for provident fund;
(ii) ₹58,000 is bonus which is paid on 13th November, 2024;
(iii)₹44,000 is commission which is paid on 1st December, 2024;
(iv) ₹25,000 is incentive to workers, which is paid on 10th December, 2024.
(v) ₹46,000 is paid outside India in respect of which tax is not deducted at source;
(vi) ₹6,000 being capital expenditure for promoting family planning amongst employees;
and
(vii) ₹55,000 being entertainment allowance given to employees.
(c) Entertainment expenses debited to profit and loss account is ₹ 12,000.
Determine the Total Income and Tax Liability of Mr. Rakesh for the Assessment Year
2024-25 under old tax regime.

Question: 11 [PGBP]

The profit and loss account of Mr. Mahesh for the year ending 31st March, 2024
discloses net profit of ₹3,90,000. Travelling expenses debited to the profit and loss
account include the following:
(i) ₹1,80,000 being expenditure incurred on a foreign tour, out of which ₹15,000 is
incurred in Indian currency and ₹1,65,000 in foreign currency for a visit of 8 days to
Germany; out of 8 days, 2 days are utilized by Mr. Mahesh for attending personal work.
(ii) ₹45,000 being expenditure on air–fare in India by a sales manager.
(iii) ₹6,500 incurred for purchasing a machine for factory. (Put to use for more than
180 days)
(iv) ₹66,000 being hotel expenses as follows:
(a) 4 days visit to Madras: ₹18,000
(b) 3 days visit to Bombay: ₹8,000
(c) 17 days visit to Bangalore: ₹40,000
Salary to employees include the following:
(1) Own salary of Mr. Mahesh: ₹ 26,000
(2) Commission on purchases to employees
(which is actually paid on 1st November, 2024) ₹42,000
Find out the Total Income and Tax Liability of Mr. Mahesh for the Assessment Year
2024-25 under old tax provisions.

335 | P a g e #HakkसेCA by CA. Akash Sir


Question: 12 [PGBP + Other sources + Capital gain + Deduction]

From the following profit and loss account of Mr. X for the year ended 31st March,
2024, compute his Total Income and Tax Liability for the Assessment Year 2024-25
under old tax provisions:

Particulars Amount Particulars Amount


(Debits) ₹ (Credits) ₹
Opening Stock 9,50,000 Sales 101,06,000
Purchases 80,50,000 Closing Stock 3,60,000
Salaries 7,00,000 Long term capital gain on 36,000
sale of house property
Rent, rates and taxes 1,25,000 Dividends from foreign 12,000
company
Deposit in National Saving 42,000 Winnings of a lottery 5,00,000
Certificate (gross)
Miscellaneous Expenses 21,000
Provision for Income Tax 31,000
Provision for gratuity 24,000
Provision for GST 45,000
Salary to Mrs. X 48,000
Purchased one computer on 40,000
01.11.2023 and put to use on
the same date
Net Profit 9,38,000
110,14,000 110,14,000

Additional information:
(i) Purchases include:
(a) Purchase of ₹ 1,00,000 from a relative (market price ₹80,000) and payment was
made in cash.
(b) Purchase of ₹25,000 being the products manufactured without aid of power in a
cottage industry and the payment was made to its producer and payment was made in
cash.
(c) Purchases of ₹35,000 from a person who is residing in a village having no bank and
payment was made in cash.

(ii) Opening and closing stock were overvalued by 10%.


(iii) Salary includes ₹25,000 being bonus paid to the staff on 01.11.2024 on the occasion
of Diwali.
(iv) Rent, rates and taxes include Municipal tax paid on 01.11.2024 of ₹30,000
(v) Provision for Gratuity is on actuarial basis.

336 | P a g e #HakkसेCA by CA. Akash Sir


(vi) Mrs. X is a housewife and payment are excessive by ₹48,000.
Mr. X has not opted for presumptive taxation of Income u/s 44AD.

Question: 13 [PGBP + Other Sources]

The profit and loss account of ABC Ltd. for the year ended 31st March, 2024 showed a
net profit of ₹8,00,000 and some of the debits and credits are as given below:
(A) Debit side of profit and loss account included the following:
(i) The depreciation provided in the books of ₹60,000, however the amount computed
under the Income Tax Act of ₹1,20,000.
(ii) ₹30,000 was paid to the company’s lawyer for arguing appeals of the company
before the Income Tax Appellate Tribunal against levy of penalty for some earlier
cases where appeals have been dismissed by the tribunal.
(iii) ₹2,000 being fine imposed by the municipality for violating their regulations.
(iv) Provision for Income Tax of ₹35,000.
(B) The credit side of the profit and loss account included the following:
(i) Income from units of UTI of ₹35,000
(ii) Dividend from Indian company of ₹20,000
(C) It is also observed that both the opening stock of ₹90,000 and closing stock of
₹1,08,000 are undervalued by 10% on cost.
Compute the Total Income and Tax Liability of the company for the Assessment Year
2024-25 under old tax provisions.

Question: 14 [PGBP]

ABC Ltd., a manufacturing company, which maintains accounts under mercantile system
has disclosed a net profit of ₹12.50 lakhs for the year ending 31st March, 2024. You
are required to compute the Total Income and Tax Liability of the company for the
Assessment Year 2024-25, after considering the following information, duly explaining
the reasons for each item of adjustment:
(i) Advertisement expenditure includes the sum of ₹60,000 paid in cash to the sister
concern of a director, the market value of which is ₹52,000.
(ii) Repairs of plant and machinery includes ₹1.80 lakhs towards replacement of worn-
out parts of machineries.
(iii) A sum of ₹6,000 on account of liability foregone by a creditor has been taken to
general reserve. The same was charged to the revenue account in the Assessment Year
2021-22.
(iv) Sale proceeds of import entitlements amounting to ₹1 lakh has been credited to
profit and loss account, which the company claims as capital receipt not chargeable to
income tax.
(v) The company has donated ₹2,00,000 to National Urban Poverty Eradication Fund.
The amount has been debited to the profit and loss account.
(vi) Being also engaged in the biotechnology business, the company incurred the
following expenditure on in-house research and development as approved by the
prescribed authority:

337 | P a g e #HakkसेCA by CA. Akash Sir


(a) Research equipment purchased for ₹1,50,000.
(b) Remuneration paid to scientists for ₹50,000.
The total amount of ₹2,00,000 is debited to the profit and loss account.

Question: 15 [PGBP + House Property + Other Sources + Set-off]

Mr. Sunil is a leading lawyer of Mumbai. He deposits in the bank all the receipts and
always pays all the expenses by cheque. The analysis of his bank account for the year
ended 31st March, 2024 is as under:

Receipts Amount Payments Amount


₹ ₹
Balance b/f 15,000 Salaries 5,00,000
Professional Fees 16,75,000 Rent of chamber 2,55,000
Dividend from Indian Co. 8,000 Telephone Expenses 26,000
Rent from house property 60,000 Magazine Subscription 3,000
which is let out
Dividend from UTI 10,000 Motor car expenses 10,000
Interest from a company 8,000 Motor car (purchased and 3,00,000
(gross) put to use on 01.12.2023)
Gift from his son from 6,000 Misc. office expenses 5,500
outside India
Honorarium for delivering 5,000 Advance payment of income- 38,000
lectures in C.A. institute tax
Honorarium for writing 1,000 Personal expenses 48,500
articles in Hindustan Times
House property expenses: 10,000
Municipal taxes 6,000
Repairs 1,000
Insurance 2,000
Collection Charges 1,000
Subscription to Bar 1,500
Association
Balance c/f 5,90,500
17,88,000 17,88,000

Mr. Sunil has not opted for presumptive taxation of Income u/s 44ADA. Compute his
Total Income, Tax Liability and Tax Payable (old tax regime) after taking into account
the following information:
(i) 10% of the motor car expenses relate to personal use.
(ii) Salaries include employer’s contribution to Recognised Provident Fund of ₹18,000
which was credited on 01.07.2024.
(iii) Mr. Sunil stays in his house, the gross annual value of which is ₹16,800.
Following are the expenses which have been included in the above account in respect of
this house:

338 | P a g e #HakkसेCA by CA. Akash Sir


(a) Municipal taxes: ₹2,000.
(b) Repairs: ₹500
(c) Insurance premium: ₹500
(iv) He has loss under the head house property ₹31,200 and the loss can be set off as
per section 71B.

Question: 16 [PGBP]

(i) Gross total income of Mrs. Bansal, aged 60, a resident of Delhi for the financial year
2023-24 is ₹4,00,000. It includes an income of ₹20,000 from the business of dealing in
shares on which she has paid securities transaction tax of ₹1,800 and it has not been
debited to the profit and loss account. She has also deposited ₹10,000 in her public
provident fund account with the State Bank of India.
Compute her Tax Liability for the Assessment Year 2024-25 under old provisions.

(ii) ABC Ltd., a domestic company, is engaged in the business of sale/purchase of shares
and the company has computed its income ₹11,00,000 after debiting securities
transaction tax of ₹1,85,000.
Compute Tax Payable by the company for the Assessment Year 2024-25.

(iii) Mr. Rohan is engaged in the business of sale/purchase of shares and he has
computed its income ₹18,00,000 after debiting securities transaction tax of ₹2,10,000.
Compute Tax Payable by Mr. Rohan under old provisions.

Question: 17 [PGBP, TDS]

Determine the previous year in which the expenditure is allowable in the following cases
(TDS is supposed to be deducted with regard to all the payments and all the payments
are in India):
(i) ABC Ltd. has made payment of interest on 10th, June 2023 and has deducted tax at
source on the same date and has deposited the amount on 08.07.2023.
(ii) The company has paid commission on 10.03.2024 and has deducted tax on the same
date but it was paid on 05.04.2024.
(iii) The company has paid fees for professional services on 31.03.2024 and deducted
tax at source on the same date but the tax was paid on 07.04.2024.
(iv) The company has paid to a contractor on 31.03.2024 and tax was deducted on the
same date but it was paid on 01.06.2024.
(v) The company has paid technical fees on 01.01.2024 and no tax has been deducted at
source.
(vi) The company has paid brokerage on 01.04.2024 and has deducted the tax on the
same date and has paid it on 07.04.2024.

Question: 18 [PGBP + Other Sources]

Following is the profit & Loss account of Mr. Aman, a dealer in shares and securities

339 | P a g e #HakkसेCA by CA. Akash Sir


for the year ended on 31st March, 2024:

Particulars Amount Particulars Amount


(Debits) ₹ (Credits) ₹
To Trading Expenses 103,60,000 By Sales 113,54,000
To Administrative 1,15,000By Interest on fixed 18,500
Expenses deposit with bank
To Financial Expenses 50,265 By Dividend from Indian 66,360
company
To Demat and Delivery 5,350 By Interest on GST refund 330
charges (Assessment Year 2022-
23)
To Securities Transaction 6,500
Tax
To Net profit before 9,02,075
depreciation
114,39,190 114,39,190

Compute Total Income and Tax Liability of Mr. Aman for Assessment Year 2024-25
under old tax provisions.

Question: 19 [PGBP + Capital gain]

Profit and loss account of Mr. Kishan for the previous year 2023-24 is as under:

Particulars Amount Particulars Amount


(Debits) ₹ (Credits) ₹
Salaries & Wages 6,00,000 Gross Profit 14,50,000
Advertisement 1,00,000 Long term capital gains 4,00,000
Travelling Expenses 2,00,000 Recovery of bad debts 50,000
(earlier it was allowed )
Depreciation on business 50,000
assets
Donation to an approved 70,000
institution for rural
development
Interest paid 2,30,000
General Expenses 3,50,000
Net Profit 3,00,000
19,00,000 19,00,000

Additional information:
(i) Salaries and wages include the sum of ₹1,60,000 paid to Mr. Kishan

340 | P a g e #HakkसेCA by CA. Akash Sir


(ii) Payment of interest includes:
(a) Interest to his major son ‘Ram’ amounting to ₹30,000 @ 15% on a deposit of
₹2,00,000
(b) Interest to Mr. Kishan amounting to ₹30,000 @ 12% p.a.
(c) Interest of ₹20,000 paid on loan taken for the payment of income tax liability.
(iii) The amount of depreciation allowable is ₹40,000.
(iv) Mr. Kishan has purchased National Saving Certificate VIII issue on 31.03.2024 for
₹40,000 and has deposited ₹60,000 in public provident fund account during the year
2023-24.
Compute Tax Liability of Mr. Kishan for the Assessment Year 2024-25 under old tax
provisions.

Question: 20 [PGBP]

Mr. X has computed his income under the head business/profession ₹10,00,000 and he
has debited the following amount.
(1) Cost of goods sold ₹7,00,000, out of which ₹4,00,000 paid to a relative for
purchasing stock and its market value is ₹3,00,000 and Mr. X has paid ₹2,00,000 by
account payee cheque and ₹2,00,000 in cash.
(2) He has debited ₹45,000 in connection with purchase of a computer which was
purchased on 27.10.2023 and was put to use on the same date and payment was made in
cash.
(3) He has purchased one generator from his relative for ₹45,000 and payment was
made in cash and market value was ₹40,000 and it was purchased on 01.10.2023 and was
put to use on 07.10.2023.
(4) He has paid advance tax being income tax of ₹45,000 on 01.10.2023.
(5) He has paid ₹21,000 to a Chartered Accountant for filing a return of income, out of
which ₹9,500 was paid in cash and balance by an account payee cheque.
(6) He has donated ₹ 20,000 to an approved research association and research work
taken up by such association is not related to the business/ profession of assessee.
(7) He has purchased household furniture for ₹12,000 for personal use.
(8) He has paid ₹20,000 in cash in connection with his medical treatment.
(9) Salary paid to the proprietor is ₹36,000.
(10) Interest on capital is ₹9,000.
(11) He has invested ₹25,000 in National Saving Certificate.
(12) He has invested ₹10,000 in public provident fund in the name of his minor child.
(13) He has debited rent of ₹35,000 in connection with his own building which is being
used in his business/profession.
(14) Opening stock debited is ₹4,50,000 which is overvalued by 10%.
(15) He has incurred ₹7,000 on printing and distribution of diaries and calendars.
Compute his Tax Liability for the Assessment Year 2024-25 under old tax regime.

Question: 21 [PGBP + House Property + Deduction]

Mr. Raju, aged 75 years, has submitted his profit and loss account for the year ending

341 | P a g e #HakkसेCA by CA. Akash Sir


31.03.2024 as given below:

Particulars Amount Particulars Amount


(₹) (₹)
Opening Stock 13,50,000 Sales 105,00,000
Purchases 75,00,000 Gift from friend 1,200
Franchises 1,00,000 Bad debts recovered 2,900
Advertisement 9,000 Rental income from House 1,40,000
Property
Income Tax of previous 7,000 Income tax refund 700
year 2022-23
Income tax (advance) 1,200 Dividends from a foreign 3,000
company
Addition to the office 45,000 Closing stock 1,80,000
building
Investment in public 70,000
provident fund
Net Profit 17,45,600
108,27,800 108,27,800

Additional information:
1. Opening and closing stocks are undervalued by 10%.
2. Franchises were purchased on 01.07.2023 and were put to use on 03.10.2023.
3. Advertisement expenditure relates to a neon sign board which was purchased and put
to use on 01.08.2023.
4. Office building has written down value of ₹56,00,000 as on 01.04.2023 and addition
was made to the building by constructing additional room on the roof. Construction was
completed on 01.11.2023 and it was put to use on the same date. The expenditure of
₹45,000 includes cost of wiring and switches of ₹4,500. No depreciation has been
debited with regard to the building.
5. Sale includes sale of ₹1,20,000 to the proprietor and the cost of these goods was
₹1,00,000 and market price ₹1,25,000.
6. Bad debts recovered were allowed earlier.
Mr. Raju has not opted for presumptive taxation of Income u/s 44AD. Compute his Tax
Liability for the Assessment Year 2024-25 under old tax provisions.

Question: 22 [PGBP + Other sources]

Mr. Bablu furnishes the following trading, profit and loss account for the previous year
ending on 31.03.2024.

Particulars Amount Particulars Amount


(Debits) ₹ (Credits) ₹

342 | P a g e #HakkसेCA by CA. Akash Sir


To Stocks 14,000 By Sales 100,84,500
To Purchases 94,80,000 By Maturity proceeds of 19,500
National Saving
Certificate
To Freight and duty 5,000 By Maturity proceeds of 24,000
Bank Fixed Deposit
To Manufacturing wages 25,000 By Maturity proceeds of 13,000
Public provident fund
To Factory, rent, rates and 30,000 By Rent of staff quarters 19,000
taxes built in 2015
To Office salaries 27,000 By Refund of income tax 1,100
penalty
To Establishment expenses 6,100 By Sale of an old 25,000
machinery
To Cost of computer 24,000 By Recovery of bad debts 6,000
(Not allowed earlier)
To Interest on capital 3,300 By Income tax refund 2,400
(it includes interest-
₹400)
To Donation to an orphan 1,000 By Gift from friends and 3,600
relatives
To Fire insurance 200 By Sundry receipts 5,000
To Bad debts 6,000 By Maturity proceeds of 24,000
LIC policy
To Income Tax 6,000 By Refund of deposit from 1,00,000
a supplier who could not
supply the machine in time
To National Urban Poverty 2,000 By Closing stocks 26,400
Eradication Fund
To Employer’s contribution to 8,000
Recognised provident fund
To Service charge for air- 11,000
conditioner
To Expenses on GST 12,000
proceedings
To Expenses on income tax 3,000
proceedings
To Diwali expenses 4,000
To Legal Expenses 4,000
To Medical expenses of 3,000
proprietor
To Staff welfare fund 2,000
To Repairs of staff quarters 4,000
To Bonus payable to 5,000

343 | P a g e #HakkसेCA by CA. Akash Sir


employees
To Provision for GST 25,000
To Municipal taxes for staff 4,000
quarters
To General reserve 5,000
To Entertainment expenses 6,000
To Net Profit 6,27,900
103,53,500 103,53,500

Mr. Bablu has not opted for presumptive taxation of Income u/s 44AD. You are
required to compute Tax Liability (old provision) after taking the following into
consideration:
1. Purchases include a purchase of ₹20,100. Its payment was made by a bearer cheque
and also includes a purchase from a relative of ₹23,000 and the payment was made in
cash and market price of the purchases is ₹22,000.
2. Factory rent, rates and taxes includes municipal tax of the factory building, which
was paid on 31.07.2024.
3. Assessee has always valued the stocks at cost price but since 2023-24 he has valued
it at market price, which was in excess of the cost price by 10%.
4. Office salaries paid include ₹12,400 to the proprietor of the business.
5. Diwali expenses include gifts of ₹1,000 made to the relatives.
6. The written down value of the block consisting of machinery as on 01.04.2023 is
₹59,000
7. The written down value of the block consisting of factory building as on 01.04.2023
is ₹85,000. An addition was made to building on 01.08.2023 at a cost of ₹12,000.
8. Service charge for air-conditioner were paid in two instalment of ₹10,000 and
₹1,000 on 10.01.2024 and 11.01.2024 in cash.
9. Employer’s contribution was made through an account payee cheque on 10.04.2024
and the cheque realized on 20.04.2024 and the due date for the purpose of provident
fund was 15.04.2024.
10. Computer was purchased on 31.03.2023 and it was put to use on 31.03.2024.

344 | P a g e #HakkसेCA by CA. Akash Sir

You might also like